The Kelalis-King-Belman Textbook of Clinical Pediatric Study Guide

The Kelalis-King-Belman Textbook of Clinical Pediatric Urology Study Guide

Editor-in-Chief Aseem R Shukla MD Director of Pediatric Urology Assistant Professor of Urologic Surgery and Pediatrics University of Minnesota Minneapolis, MN, USA Associate Editors Paul F Austin MD Associate Professor of Surgery St Louis Children’s Hospital Division of Urologic Surgery Washington University School of Medicine St Louis, MO, USA

CD Anthony Herndon MD Associate Professor Department of Surgery/Pediatrics Division of Urology/Section Pediatric Urology University of Alabama – Birmingham Birmingham, AL, USA Informa Healthcare USA, Inc. 52 Vanderbilt Avenue New York, NY 10017

© 2009 by Informa Healthcare USA, Inc. Informa Healthcare is an Informa business

No claim to original U.S. Government works Printed in the United States of America on acid-free paper 10 9 8 7 6 5 4 3 2 1

ISBN-13: 978 0 415 46016 3

This book contains information obtained from authentic and highly regarded sources. Reprinted material is quoted with permission, and sources are indicated. A wide variety of references are listed. Reasonable efforts have been made to publish reliable data and information, but the author and the publisher cannot assume responsibility for the validity of all materials or for the consequence of their use.

No part of this book may be reprinted, reproduced, transmitted, or utilized in any form by any electronic, mechanical, or other means, now known or hereafter invented, including photocopying, microfi lming, and recording, or in any information storage or retrieval system, without written permission from the publishers.

For permission to photocopy or use material electronically from this work, please access www.copyright.com (http://www.copyright.com/) or contact the Copyright Clearance Center, Inc. (CCC) 222 Rosewood Drive, Danvers, MA 01923, 978-750-8400. CCC is a not-for-profi t organization that provides licenses and registration for a variety of users. For organizations that have been granted a photocopy license by the CCC, a separate system of payment has been arranged.

Trademark Notice: Product or corporate names may be trademarks or registered trademarks, and are used only for identifi cation and explanation without intent to infringe.

For Corporate Sales and Reprint Permissions call 212-520-2700 or write to: Sales Department, 52 Vanderbilt Avenue, 16th fl oor, New York, NY 10017.

Visit the Informa Web site at www.informa.com and the Informa Healthcare Web site at www.informahealthcare.com

Composition by Exeter Premedia Services Pvt Ltd, Chennai, India Printed and bound in the United States of America Contents

Contributors xi

Preface xix

1 History and physical examination of the child 1 T Ernesto Figueroa and Aseem R Shukla 2 Laboratory assessment of the pediatric urologic patient 3 Paul F Austin 3 Fetal urology and prenatal diagnosis 5 CD Anthony Herndon 4 In-office ultrasonography 7 Dennis Liu and Max Maizels 5 Pediatric renal nuclear medicine 11 Aseem R Shukla 6 Prenatal and postnatal urologic emergencies 13 Sarah Marietti and Fernando Ferrer 7 Urinary tract infections in children 17 Hans Pohl and H Gil Rushton 8 Fungal, parasitic, and other inflammatory diseases of the genitourinary tract 23 Ilene Wong and William A Kennedy II 9 Pain management for the pediatric patient 27 Jason W Anast and Paul F Austin 10 Office pediatric urology 29 Kara Saperston , Patrick Cartwright , and Brent Snow 11 Principles of minimally invasive surgery 33 Walid Farhat and Pasquale Casale vi Contents

12 Pediatric fluid management 37 Paul F Austin 13–14 Embryology of adrenal, kidney, ureteral development, renal vasculature, and gonads 41 Steven E Lerman , Irene M McAleer , and George W Kaplan 15 Radiologic assessment of the adrenal 47 Karoush Afshar , Douglas H Jamieson , and Andrew MacNeily 16 Adrenal tumors and functional consequences 49 Julie Franc-Guimond , Anne-Marie Houle , and Aseem R Shukla 17 Surgery of the adrenal 53 CD Anthony Herndon 18 Basic science of the kidney 55 John C Thomas and John C Pope IV 20 Anomalies of the kidney 57 Michael Nguyen 21 Fetal and neonatal renal function 59 Paul F Austin 22 Cystic kidney disease 63 Larry A Greenbaum 23 Acute renal failure 67 Lawrence Copelovitch , Bernard S Kaplan , and Kevin EC Meyers 24 Renal transplantation 71 Nafisa Dharamsi , Curtis Sheldon , and Jens Gobel 25 Renal calculus disease 75 Parmod Reddy and Eugene Minevich 26 Endourology for stone disease 79 Steven Lukasewycz and Aseem R Shukla 27 Renal parenchymal imaging in children 83 Jason W Anast and Paul F Austin 28 Assessment of renal obstructive disorders: ultrasound, nuclear medicine, and magnetic resonance imaging 87 James Elmore and Andrew J Kirsch 29 Assessment of renal obstructive disorders: urodynamics of the upper tract 89 Thomas E Novak and Yegappan Lakshmanan 30 Ureteropelvic junction obstruction and multicystic dysplastic kidney: surgical management 91 Michael C Carr 31 Laparoscopic nephrectomy and pyeloplasty 93 CD Anthony Herndon Contents vii

32 Wilms' tumor 95 Sarah Marietti and Fernando Ferrer 33 Surgical approaches for renal tumors 99 João L Pippi Salle , Armando J Lorenzo , and Elizabeth R Williams 34 Pediatric upper tract trauma 105 Douglas A Husmann 35 The ureter 111 Martin Kaefer 37 The imaging of reflux and ureteral disease 115 Jeanne S Chow and David A Diamond 38 Ureteral anomalies and their surgical management 117 Christopher Cooper 39 Megaureter 119 Douglass Clayton 40 Ureteral duplication anomalies: ectopic ureters and ureteroceles 123 Jorge R Caso and Michael A Keating 41 Laparoscopic management of duplication anomalies 127 Stephen Lukasewycz and Aseem R Shukla 42 Vesicoureteral reflux: anatomic and functional basis of etiology 129 John M Park 43 Nonsurgical management of vesicoureteral reflux 131 Jack S Elder 44 Surgery for vesicoureteral reflux 135 John W Brock III and Romano T DeMarco 46 Injection therapy for vesicoureteral reflux 137 Michael A Poch and Anthony A Caldamone 47 Basic science of the urinary bladder 141 Scott R Manson 48 Basic science of prostatic development 143 Ellen S Shapiro 49 Embryology of the anterior abdominal wall, bladder, and proximal urethra 147 Steven E Lerman , Irene M McAleer , and George W Kaplan 50 Radiologic assessment of bladder disorders 149 Douglas E Coplen 51 Urodynamics of the upper and lower urinary tract 151 Jennifer J Mickelson and William E Kaplan viii Contents

52 Neurologic control of storage and voiding 155 Julian Wan and John M Park 53 Neurogenic voiding dysfunction and nonsurgical management 159 Richard N Yu and Stuart Bauer 54 Diurnal and nocturnal enuresis 163 Rosalia Misseri 55 Operations for the weak bladder outlet 165 CD Anthony Herndon 56 Bladder augmentation: current and future techniques 167 Jennifer J Mickelson , Bradley P Kropp, and Earl Y Cheng 57 Urinary diversion 171 Richard Rink and Mark P Cain 58 The Malone Antegrade Continence Enema procedure 175 Martin A Koyle 59 Minimally invasive approaches to lower urinary tract reconstruction 179 Christina Kim and Steven G Docimo 60 Genitourinary rhabdomyosarcoma and other bladder tumors 181 Paul A Merguerian and Antoine E Khoury 61 Exstrophy and epispadias 185 Linda A Baker and Richard W Grady 62 Unusual conditions of the bladder, including bladder trauma, urachal anomalies, and bladder diverticula 189 Marc Cendron 63 Posterior urethral valves 193 Stephen A Zderic and Douglas A Canning 64 Prune belly syndrome 199 R Guy Hudson and Steven J Skoog 65 Basic science of genitalia 203 CD Anthony Herndon 66 Basic science of the testis 205 Stephen Lukasewycz and Aseem R Shukla 68 Intersex 207 Hsi-Yang Wu and Howard M Snyder III 69 Imperforate anus and cloaca 211 Curtis Sheldon and William R DeFoor Jr 70 Surgery for intersex disorders/urogenital sinus 215 Mark C Adams and Romano T DeMarco Contents ix

71 Hypospadias 219 Warren T Snodgrass , Aseem R Shukla , and Douglas A Canning 72 Abnormalities of the penis and 223 Stephen Lukasewycz and Aseem R Shukla 73 Hernia, hydroceles, testicular torsion, and varicocele 225 Hiep T Nguyen 74 Cryptorchidism 229 Thomas F Kolon 75 Surgical management of the undescended testis 233 Israel Franco 76 Testicular tumors 237 Jonathan H Ross 77 Tumors of the retroperitoneum 241 Gordon A McLorie 78 Tissue engineering applications in pediatric urology 243 Anthony Atala

Index 245

Contributors

Mark C Adams MD FAAP Paul F Austin MD FAAP Professor of Urology and Pediatrics Associate Professor of Surgery Monroe Carell Jr Children’s Hospital at Vanderbilt St Louis Children’s Hospital Nashville, TN Division of Urologic Surgery USA Washington University School of Medicine St Louis, MO USA Kourosh Afshar MD FRCSC Assistant Professor of Surgery (Urology) Linda A Baker MD University of British Columbia Associate Professor of Urology Pediatric Urologist, BC Children’s Hospital Director of Pediatric Urology Unit Vancouver, BC University of Texas Southwestern Canada Children’s Medical Center at Dallas Dallas, TX USA Jason W Anast MD Resident in Urology Stuart Bauer MD FAAP Department of Urology Professor of Surgery Washington University School of Medicine Director, Neuro-Urology St Louis, MO Surgery/Urology USA Children’s Hospital Boston, MA USA Anthony Atala MD The William Boyce Professor and Chair John W Brock III MD Department of Urology Director of Pediatric Urology Wake Forest University School of Medicine Vanderbilt University Winston-Salem, NC Nashville, TN USA USA xii List of contributors

Mark P Cain MD FAAP Marc Cendron MD Associate Professor Associate Professor of Surgery (Urology) James Whitcomb Riley Hospital for Children Harvard School of Medicine Indiana University School of Medicine Department of Urology Indianapolis, IN Children’s Hospital Boston USA Boston, MA USA Anthony A Caldamone MD Chief, Pediatric Urology Earl Y Cheng MD Hasbro Children’s Hospital Associate Professor of Urology Professor of Surgery, Urology Northwestern University’s Feinberg Brown University School of Medicine School of Medicine Providence, RI Department of Pediatric Urology USA Children’s Memorial Hospital Douglas A Canning MD Chicago, IL Director USA Division of Pediatric Urology Children’s Hospital of Philadelphia Jeanne S Chow MD Philadelphia, PA Assistant Professor in Radiology USA Harvard Medical School Children’s Hospital Michael C Carr MD Radiology Assistant Professor of Surgery and Urology Boston, MA Division of Pediatric Urology USA Children’s Hospital of Pennsylvania Philadelphia, PA Douglass Clayton MD USA Resident in Urology Patrick Cartwright MD FAAP University of Alabama – Birmingham Chief of Urology Birmingham, AL Professor of Surgery and Pediatrics USA University of Utah Health Sciences Center Primary Children’s Medical Center Christopher S Cooper MD Salt Lake City, UT Associate Professor of Urology USA Director Pediatric Urology Pasquale Casale MD University of Iowa and Division of Pediatric Urology Children’s Hospital of Iowa Children’s Hospital of Pennsylvania Iowa City, IA Philadelphia, PA USA USA

Jorge R Caso MD Lawrence Copelovitch MD Resident in Urology Division of Pediatric Nephrology University of South Florida Children’s Hospital of Philadelphia Tampa, FL Philadelphia, PA USA USA List of contributors xiii

Douglas E Coplen MD James M Elmore MD Director of Pediatric Urology Clinical Assistant Professor of Urology St Louis Children’s Hospital Emory University School of Medicine St Louis, MO Atlanta, GA USA USA

William R DeFoor Jr MD Assistant Professor Walid Farhat MD Division of Pediatric Urology Assistant Professor Cincinnati Children’s Hospital Medical Center Department of Surgery Cincinnati, OH University of Toronto USA Division of Urology Hospital for Sick Children Romano T DeMarco MD FAAP Toronto, ON Pediatric Urology Canada Assistant Professor of Surgery University of Missouri

Kansas City School of Medicine Fernando Ferrer MD Kansas City, MO Director USA Pediatric Urologic Surgery Surgeon-in-Chief Nafisa Dharamsi MD FRCSC Pediatric Urology Connecticut Children’s Medical Center Winnipeg Children’s Hospital Hartford, CT Winnipeg, MB USA Canada

David A Diamond MD T Ernesto Figueroa MD Associate Professor of Surgery Chief, Division of Pediatric Urology Harvard Medical School New York Medical College Boston, MA Valhalla, NY USA USA

Steven G Docimo MD Professor and Director Israel Franco MD Pediatric Urology Associate Professor of Urology University of Pittsburgh School of Medicine New York Medical College Children’s Hospital of Pennsylvania Valhalla, NY Pittsburgh, PA USA USA

Jack S Elder MD Jens Goebel MD Chief, Department of Urology Associate Professor of Pediatrics Henry Ford Health System Medical Director of Kidney Transplantation Associate Director, Vattikuti Urology Institute Division of Nephrology and Hypertension Children’s Hospital of Michigan Cincinnati Children’s Hospital Medical Center Detroit, MI Cincinnati, OH USA USA xiv List of contributors

Richard W Grady MD Douglas A Husmann MD Associate Professor of Urology Chief and Professor of Urology Director Mayo Clinic Clinical Research Rochester, MN University of Washington School of Medicine USA Children’s Hospital and Regional Medical Center Seattle, WA USA Douglas H Jamieson MBChB FRCPC Clinical Assistant Professor of Radiology

Larry A Greenbaum MD PhD British Columbia Children’s Hospital Division Director Vancouver, BC Pediatric Nephrology Canada Marcus Professor Emory University and Children’s Healthcare Martin Kaefer MD of Atlanta Associate Professor of Urology Atlanta, GA Department of Pediatric Urology USA Indiana University Indianapolis, IN Julie Franc-Guimond MD USA Pediatric Urology CHU Sainte-Justine Montréal, PQ Bernard S Kaplan MD Canada Director of Pediatric Nephrology Children’s Hospital of Philadelphia CD Anthony Herndon MD FAAP FAS Philadelphia, PA Associate Professor USA Department of Surgery/Pediatrics Division of Urology/Section Pediatric Urology University of Alabama – Birmingham George W Kaplan MD Birmingham, AL Chief of Surgery USA Children’s Hospital and Health Center San Diego, CA Anne-Marie Houle MD USA CHU Sainte-Justine Montréal, PQ Canada William E Kaplan MD Head, Pediatric Urology R Guy Hudson MD Associate Professor of Urology Assistant Professor Northwestern University’s Feinberg School Oregon Health and Science University of Medicine Pediatric Urology Division of Urology Doembecher Children’s Hospital Children’s Memorial Hospital Portland, OR Chicago, IL USA USA List of contributors xv

Michael A Keating MD Martin A Koyle MD FAAP FACS Medical Director Chief, Division of Urology Spina Bifi da Clinic Children’s Hospital and Regional Medical Center Department of Surgery Seattle, WA Division of Urology USA Nemours Children’s Clinic Bradley P Kropp MD Orlando, FL USA Chief and Professor of Pediatric Urology Department of Pediatric Urology University of Oklahoma Health Sciences Center William A Kennedy II MD Oklahoma City, OK Associate Professor USA Department of Urology Stanford University School of Medicine Yegappan Lakshmanan MD FAAP Stanford, CA Assistant Professor, Pediatric Urology USA Brady Urological Institute Johns Hopkins Medical Institutions Baltimore, MD Antoine E Khoury MD USA Professor of Urology Chief, Division of Urology Steven E Lerman MD Hospital for Sick Children Assistant Professor of Urology University of Toronto UCLA Urology Toronto, ON Los Angeles, CA Canada USA

Dennis B Liu MD Christina Kim MD Assistant Professor of Urology Assistant Professor of Surgery University of Toledo College of Medicine Connecticut Children’s Medical Center Toledo, OH Hartford, CT USA USA Armando J Lorenzo MD

Andrew J Kirsch MD FAAP FACS Assistant Professor, Department of Surgery Professor of Urology (Urology) Emory University School of Medicine Hospital for Sick Children Children’s Healthcare of Atlanta University of Toronto Atlanta, GA Toronto, ON USA Canada

Stephen Lukasewycz MD Thomas F Kolon MD FAAP Resident in Urology Assistant Professor of Urology Department of Urologic Surgery Children’s Hospital of Philadelphia University of Minnesota Philadelphia, PA Minneapolis, MN USA USA xvi List of contributors

Irene M McAleer MD MBA FAAP FACS Kevin EC Meyers MD Pediatric Urology Division of Pediatric Nephrology Children’s Hospital Central California Children’s Hospital of Philadelphia Madeira, CA Philadelphia, PA USA USA

Jennifer J Mickelson MD Gordon A McLorie MD Fellow in Pediatric Urology Chief, Pediatric Urology Department of Pediatric Urology Department of Urology Children’s Memorial Hospital Wake Forest University School of Medicine Chicago, IL Winston-Salem, NC USA USA Eugene Minevich MD FAAP FACS Andrew MacNeily MD FRSCS FAAP Associate Professor of Surgery Associate Professor of Surgery (Urology) Cincinnati Children’s Hospital Medical Center University of British Columbia Cincinnati, OH Vancouver, BC USA Canada Rosalia Misseri MD Assistant Professor of Pediatric Urology Max Maizels MD Riley Children’s Hospital Director of Perinatal Urology Indianapolis, IN Feinberg School of Medicine at Northwestern USA University Chicago, IL Hiep T Nguyen MD USA Co-Director Center for Robotic Surgery Scott R Manson PhD Director Division of Urology Robotic Surgery Research and Training Washington University School of Medicine Assistant Professor of Surgery (Urology) St Louis, MO Harvard Medical School USA Boston, MA USA Sarah Marietti MD Michael Nguyen MD Resident in Urology Adjunct Clinical Instructor in Urology University of Connecticut Affi lliated Program in Mayo Clinic College of Medicine Urology Pediatric Urology Associates Hartford, CT Phoenix, AZ USA USA

Paul A Merguerian MD FRCSC FAAP Thomas E Novak MD Professor of Surgery (Urology) and Pediatrics Fellow, Pediatric Urology Dartmouth-Hitchcock Medical Center Brady Urological Institute Dartmouth Medical School Johns Hopkins Medical Institutions Hanover, NH Baltimore, MD USA USA List of contributors xvii

John M Park MD Richard C Rink MD FAAP Associate Professor, Urology Chief Pediatric Urology Director of Pediatric Urology James Whitcomb Riley Hospital for Children University of Michigan Medical School Indiana University School of Medicine Ann Arbor, MI Indianapolis, IN USA USA

Jonathan H Ross MD João Luiz Pippi Salle MD PhD FAAP FRCSC Head of Section of Pediatric Urology Professor, Division of Urology Glickman Urological Institute Hospital for Sick Children Cleveland Clinic Childrens Hospital University of Toronto Cleveland, OH Toronto, ON USA Canada

H Gil Rushton MD FAAP

Michael A Poch MD Chairman, Department of Pediatric Urology Resident in Urology Children’s National Medical Center The Warren Alpert Medical School of Brown Washington, DC University USA Providence, RI USA Kara Sapperston MD Resident in Urology University of Utah College of Medicine Hans Pohl MD FAAP Salt Lake City, UT Assistant Professor of Urology and Pediatrics USA Department of Pediatric Urology Children’s National Medical Center Ellen S Shapiro MD George Washington University School of Director Pediatric Urology Medicine New York University School of Medicine Washington, DC New York, NY USA USA

Curtis A Sheldon MD John C Pope IV MD Professor of Surgery Associate Professor Urologic Surgery Division of Pediatric Urology Vanderbilt Children’s Hospital Cincinnati Children’s Hospital Medical Center Nashville, TN Cincinnati, OH USA USA

Pramod P Reddy MD Aseem R Shukla MD Program Director Director of Pediatric Urology Pediatric Urology Fellowship Assistant Professor of Urologic Surgery Division of Pediatric Urology and Pediatrics Cincinnati Children’s Hospital Medical Center University of Minnesota Cincinnati, OH Minneapolis, MN USA USA xviii List of contributors

Steven J Skoog MD Julian Wan MD Professor and Director Clinical Associate Professor of Urology Pediatric Urology Pediatric Urology Division Oregon Health and Science University University of Michigan Medical School Doembecher Children’s Hospital Ann Arbor, MI Portland, OR USA USA Elizabeth R Williams MD Resident in Urology Warren T Snodgrass MD FAAP Department of Urology Professor of Urology Washington University School of Medicine University of Southwestern Medical Center at St Louis, MO Dallas USA Dallas, TX USA Ilene Wong MD Resident Department of Urology Brent Snow MD FAAP Stanford University School of Medicine Professor of Surgery and Pediatrics Stanford, CA University of Utah Health Sciences Center USA Primary Children’s Medical Center Salt Lake City, UT Hsi-Yang Wu MD USA Assistant Professor of Urology Children’s Hospital of Pittsburgh Pittsburgh, PA Howard M Snyder III MD USA Associate Director Pediatric Urology Richard N Yu MD PhD Children’s Hospital of Philadelphia Clinical Fellow in Urology Philadelphia, PA Children’s Hospital USA Boston, MA USA

John C Thomas MD Stephen A Zderic MD Adjunct Professor of Pharmacology Professor of Surgery in Urology Vanderbilt University University of Pennsylvania School of Medicine Nashville, TN Philadelphia, PA USA USA Preface

The Kelalis-King-Belman Textbook of Clinical textbook. The embryology questions relevant to Pediatric Urology is renowned as an invaluable, Chapters 19, 36, and 67 were included in Chapters comprehensive resource for practicing pediatric 13–14 by the authors and are not separately listed urologists, fellows, and residents in training. The here. No questions were submitted for Chapter fifth edition of this classic textbook was published 45, but relevant information is included in other in 2007 and edited for the first time by Drs. Steven related chapters. Docimo, Douglas Canning, and Antoine Khoury. Wherever possible, every effort was made to This edition under new leadership continues request the original authors of each chapter in the to uphold the vaunted ideals of the previous textbook to prepare questions that they felt encap- textbook editions and firmly establishes itself as sulated the key elements of their areas of expertise. the definitive resource for the specialty. The vast majority of the original authors partici- Pediatric urology became the first subspecialty pated in this exercise and we, the co-editors of this recognized by the American Board of Urology in effort, are enormously grateful. the spring of 2008, as the first Certificate of Added We express our admiration for the colossal Qualification was granted following a rigorous endeavor of Steven Docimo, Douglas Canning, qualification process and written examination. and Antoine Khoury to edit the textbook upon That there is a paucity of sample questions and which we base our study guide. Our sincere study guides available for the busy practitioner appreciation, especially, to Steven Docimo for preparing for the certification exam, or for investing us with his confidence and guiding urology residents and medical students sitting our efforts; and to Informa Healthcare for for annual in-service exams, became apparent bringing forth what we hope will be a practical and prompted the publication of this text: and valuable contribution to our discipline: The Kelalis-King-Belman Textbook of Clinical pediatric urology Pediatric Urology Study Guide. This study guide is intended as a compendium Aseem R Shukla to the authoritative textbook, and the entirety of Paul F Austin the multiple choice questions presented herein CD Anthony Herndon are based on information relayed in the actual July 2008

History and physical examination 1 of the child

T Ernesto Figueroa and Aseem R Shukla (Based on chapter written by T Ernesto Figueroa)

1. The most useful tool in the diagnosis of federal oversight is established by the HIPAA medical problems is: guidelines. In the context of pediatric urology, this requires the physician to identify early (a) Physical examination on who will be the recipient of the medical (b) Medical history information related to the child. (c) Reviews of system (d) Radiographic studies 3. Common and effective distraction techniques (e) Serum analysis include all of the following except:

Answer: (b) A properly obtained medical (a) The use of toys % history accounts for 80 of the diagnostic (b) Offering snacks process. The history taken initiates all further (c) Talking to the child about their favorite investigations and is critical to understanding activities and appreciating a medical condition. (d) Timeout (e) Coloring books 2. Health Insurance Portability and Accountability Act (HIPAA) guidelines ensure that: Answer: (d) The physician should be willing to listen and demonstrate empathy during a (a) The patient receive comprehensive medical patient visit. If the child is disruptive and unco- care despite the inability to pay for those operative, then distractive toys, coloring books services or, at most, a gentle removal from the room (b) Medical information regarding a pediatric may assist with history taking. Disciplining patient be communicated to both the a child during a medical exam may set a nega- parents regardless of marital status or state tive precedent for future interactions. of legal custody (c) Confidentiality and patient privacy are 4. The first sign of puberty in boys is usually: protected (d) Documentation in the medical record (a) The change of the voice should indicate next of kin (b) Axillary hair (e) Patients can designate power of attorney (c) The appearance of fine pubic hair in case they are unable to make medical (d) Growth of the penis and scrotum decisions for themselves (e) Testicular growth Answer: (c) It is obligatory that the patient' s Answer: (e) The first sign of puberty, usually confidentiality and privacy are protected, a around 11– 12 years of age, is testicular growth. 2 Clinical Pediatric Urology Study Guide

Secondary sexual development, described by (b) A 2-year-old child in acute urinary choices (a)– (d), follows after the onset of retention puberty. In females, the first sign of puberty is (c) A spinal cord injured patient usually breast development, followed by the (d) A chronically constipated child growth of pubic hair. (e) A 12-year-old boy with spina bifida and urinary incontinence 5. Menses usually occurs in which Tanner stage: Answer: (a) The digital rectal examination is not routinely performed in children and is (a) Stage 1 reserved for patients with pertinent complaints (b) Stage 2 described in choices (b)– (e). This exam is (c) Stage 3 performed using a well-lubricated, gloved fifth (d) Stage 4 finger. (e) Stage 5

Answer: (d) Menses heralds the onset of 7. The greatest fear of a 5-year-old boy during Tanner stage 4, and is associated with the examination is: development of coarse pubic hair and breast (a) Separation from the parents areola and papilla forming a secondary mound. (b) Loss of privacy Male children at Tanner stage 4 correspond (c) Undressing in front of strangers to the same change in pubic hair, growth of (d) Fear of needles axillary hair and voice changes. (e) Bodily injury or Answer: (e) While separation anxiety pre- 6. Digital rectal examinations in children should dominates in children less than 3 years of age, routinely be performed in all of the following fear of bodily injury or castration is most except: common in children between the ages of 4 (a) A rectal exam should always be part of a and 6. Privacy becomes a preeminent issue in complete physical examination older children and adolescents. Laboratory assessment of the 2 pediatric urologic patient

Paul F Austin (Based on chapter written by Paul F Austin and Erica J Traxel)

1. A 6-year-old girl is referred to you by her urologic causes. This panel of serum tests is pediatrician for microscopic hematuria found selectively performed if renal and bladder on urine dipstick during a work-up for an upper sonography are negative and the urine micros- respiratory infection. Your next step is to: copy suggests a nephrologic origin. These may subsequently indicate hematologic- or (a) Obtain a renal and bladder ultrasound immunologic-mediated diseases affecting the (b) Perform cystoscopy kidney. (c) Obtain a microscopic urinalysis (d) Obtain a urine culture (e) Obtain a serum basic metabolic panel 3. An asymptomatic newborn is noted to have an enlarged left hemi-scrotum with a transil- Answer: (c) Urine dipstick results will often luminable, nonreducible mass. A serum alpha- have false-positive results. Although the other fetoprotein (AFP) level was drawn and tests may be necessary depending on the measured 48,000 ng/ml. Your next step is: history and further work-up, a microscopic urinalysis will verify that red blood cells (a) Reassurance and observation (RBCs) are indeed present. (b) Scrotal exploration with possible left radical 2. With the presence of dysmorphic RBCs on (c) CT scan microscopic urinalysis, the next step would be: (d) CBC, beta human chorionic gonadotro- pin (HCG), lactate dehydrogenase (LDH) (a) Obtain a renal and bladder ultrasound and placental alkaline phosphatase (b) Perform cystoscopy (PLAP) (c) Repeat the microscopic urinalysis in (e) Scrotal U/S 1 week (d) Obtain a urine culture Answer: (a) AFP at birth is relatively high (e) Obtain a complete blood count (CBC), and will remain so for the first several months basic metabolic panel, C3 level, antistrep- of life, due to the yolk sac elements present tolysin-O (ASO) titer and antinuclear during gestation. In this example, the newborn antibody test (ANA) has a neonatal hydrocele and family reassu- Answer: (e) Microscopic abnormal shaped rance, and observation of the neonatal hydro- RBCs or dysmorphic RBCs are more com- cele is all that is necessary. monly associated with nephrologic causes of hematuria, and normal shaped or eumorphic 4. A 14-year-old boy presents for follow-up after RBCs are more commonly associated with an extracorporeal shock wave lithotripsy 4 Clinical Pediatric Urology Study Guide

(ESWL) for an 8 mm right renal calculus. (d) 17-OH progesterone The patient is “ stone free ” on kidneys, ureter, (e) M ü llerian inhibiting substance (MIS) bladder X-ray (KUB) and sonographic imaging. Answer: (d) The most common intersex The next step is: condition is congenital adrenal hyperplasia (a) Reassurance and observation (CAH). It is important to identify CAH early (b) Metabolic evaluation to prevent any electrolyte and metabolic imbal- (c) CT scan ances such as “ salt-wasting ” . Although the other (d) Spot urine calcium/creatinine ratio tests are important in the work-up of a child (e) Initiate high-fluid therapy with intersex, serum 17-OH progesterone is critical because elevated levels identify CAH Answer: (b) The prevalence of nephrolithia- and would direct prompt intervention. sis in children is less than in the adult popula- tion and extensive laboratory assessment is 6. An 18-month-old male is referred for bilat- recommended upon initial presentation. As eral, nonpalpable cryptorchidism. Appropriate opposed to the adult patient, nephrolithiasis in laboratory testing would include consideration the pediatric population is more likely attri- of which of the following: butable to a metabolic abnormality. Standard metabolic evaluation for pediatric nephro- (a) Karyotype lithiasis includes serum tests and a 24-hour (b) Serum testosterone and gonadotropins urine study. A serum CBC, electrolytes, bicar- (c) HCG stimulation test bonate, calcium, phosphorus, blood urea (d) MIS nitrogen (BUN), creatinine, alkaline phos- (e) All of the above phatase, magnesium, and uric acid should be Answer: (e) A karyotype should be per- obtained. A 24-hour urine collection should formed to rule out an intersex condition. be obtained on a regular diet to check calcium, Serum testosterone levels may be obtained phosphorus, magnesium, oxalate, sodium, but would be low at this age and must be uric acid, citrate, cystine, creatinine, and referenced to the levels of the gonadotropins volume. to have any relevance. (Remember there is normal elevation of serum testosterone at 5. You are called to the nursery for consultation 2 – 3 months of age as well as at puberty.) An regarding a newborn with ambiguous genita- HCG stimulation test is a good way of check- lia. Inspection and examination of the genitalia ing for testosterone production from any reveals a phallic structure with a severe hypo- testicular tissue. Finally, measuring serum spadias and impalpable gonads. The most MIS levels that are produced by the Sertoli cell critical test in the work-up is: is the most sensitive indicator of testicular (a) Karyotype presence. Unfortunately, the MIS enzyme- (b) Electrolytes linked immunosorbent assay (ELISA) is not (c) Genitogram readily available in most labs. Fetal urology and prenatal 3 diagnosis

CD Anthony Herndon (Based on chapter written by Dave Thomas)

1. During the third trimester, urine constitutes (d) 125 ml/1.73 m 2 what percentage of amniotic fluid? (e) 5 ml/1.73 m 2 (a) 50 % Answer: (c) Despite a dramatic increase in (b) 100 % renal perfusion pressure, the initial GFR is (c) 70 % quite low at 30 ml/m 2 . At two years of life it (d) 90 % approaches 125 ml/m 2 . (Page 20) (e) 10 % 4. In patients with suspected bladder outlet Answer: (d) By the third trimester, urine pro- obstruction, a poor prognosis may be predicted duction equals 30– 40 ml/h and comprises up to for the following: 90% of amniotic fluid volume. (Page 19) (a) Bladder distention identified < 24 weeks 2. Amniotic fluid is vital for fetal lung develop- (b) Thickened bladder wall ment secondary to: (c) Anteroposterior (AP) diameter renal pel- vis diameter > 10 mm (a) Does not play a role (d) Male fetus (b) Mechanical properties of stenting open (e) All of the above airways (c) Provides a liquid surface that decreases Answer: (e) All of the above, plus oligohy- surface tension dramnios are believed to be a bad prognostic (d) Provides potential growth factors for indicator for early-onset renal deterioration. aveoli (Page 22) Answer: (b) and (d) Amniotic fluid provides 5. Prenatal ultrasound findings which may be growth factors for fetal lung development consistent with lower urinary tract pathology in addition to the mechanical properties of include: stenting open the airways. (Page 20) (a) Ureterocele 3. Following birth, an increase in renal perfusion (b) Visualization of a ureter occurs which results in estimated glomerular (c) Thickened bladder wall filtration rate (GFR) to be: (d) Multicystic dysplastic kidney (MCDK) (e) All of the above (a) 75 ml/1.73 m2 (b) 100 ml/1.73 m2 Answer: (e) All of the above including (c) 30 ml/1.73 m2 MCDK may be associated with pathology at 6 Clinical Pediatric Urology Study Guide

the bladder level. Vesicoureteral reflux (VUR) (e) Demonstrates a high spontaneous resolu- is associated with MCDK in up to 40% of tion rate patients. (Page 26) Answer: (a) and (e) Vesicoureteral reflux that is detected prenatally with hydroneph- 6. Vesicoureteral reflux that is detected in patients rosis is more common in boys, tends to be with prenatal hydronephrosis: high grade and bilateral, and resolves sponta- (a) Usually occurs in boys neously at a rate higher than expected. Girls (b) Usually occurs in girls tend to present with urinary tract infections (c) Tends to be low grade and unilateral postnatally and the majority of their reflux is (d) Tends to be high grade and bilateral low grade. In-office ultrasonography 4 Dennis Liu and Max Maizels

1. Which of the following ultrasound probes In an uncooperative younger child, the com- would be most appropriate to image the fort of being held by the caregiver will often kidney of a large 14-year-old boy? calm the child down and allow a better exam. (Page 36) (a) 3 mHz (b) 5 mHz (c) 7.5 mHz 3. A 7-year-old girl is referred to the office for (d) 10 mHz unilateral Grade 1 hydronephrosis. An IOUS (e) Any of the above probes would be is performed which confirms the finding of appropriate right hydronephrosis. All of the following manuevers on IOUS would help make the Answer: (a) The lower the mHz of the probe, diagnosis of vesicoureteral reflux (VUR) the better tissue penetrance is obtained. In except for: order to image the kidneys in a large 14-year- old child, the 3 Mhz probe offers the best tis- (a) Imaging of the bladder transversely while sue penetrance and would be the best choice. full (Page 35) (b) Imaging of the bladder transversely after voiding (c) Imaging of the bladder parasagittally from 2. An 18-month-old child is seen in the office for the right an in-office ultrasound (IOUS). During the (d) Imaging of the bladder parasagittally from exam, an overlying rib obscures the child' s left the left kidney and she becomes increasingly agitated (e) Imaging of the kidneys after voiding and uncooperative. Which of the following positions is optimal to obtain an image of this Answer: (c) Use of IOUS is helpful in certain kidney? situations to delineate unilateral hydroneph- rosis due to VUR from ureteropelvic junction (a) Supine with the probe placed in laterally obstruction. Resolution of hydronephrosis along the axillary line after voiding would favor a diagnosis of VUR (b) Left lateral decubitus position while persistence of unilateral hydronephrosis (c) Prone position after voiding would suggest possible uretero- (d) Have the caregiver hold the child and pelvic junction (UPJ) obstruction. (Page 38) image the kidney from the prone position Likewise, imaging for dilated distal ureters (e) Supine with the probe anteriorly in an with a full bladder and resolution after voiding intercostal space would also be suggestive of VUR. This is best Answer: (d) The prone position is the best done either transversely (ureters located pos- position to image a kidney obscured by ribs. terior to the bladder) or viewed longitudinally 8 Clinical Pediatric Urology Study Guide

by viewing the ureter from the contralateral (c) Uric acid crystals parasagittal view of the bladder. Thus, left (d) Tamm-Horsfall protein parasagittal views of the bladder would be (e) Hypercalcuria used to image the right ureter. (Page 36) Answer: (d) The echogenic characteristic of a neonate' s bladder is usually due to the 4. A two-month-old child is seen in the office for Tamm-Horsfall protein. The normal urine in evaluation of severe unilateral hydronephrosis. older children is echolucent. You perform an IOUS. Which of the follow- ing findings would be suggestive of a severely obstructed UPJ rather than a multicystic 6. A 6-year-old girl is seen in the office for dysplastic kidney? evaluation of incontinence and recurrent urinary tract infections (UTIs). An IOUS is (a) An echogenic contralateral kidney performed and her bladder capacity is calcu- (b) Contiguity of the dilated renal pelvis and lated. What is the expected bladder capacity of dilated calyces a 6-year-old child? (c) Involution of the kidney on serial follow-up

exams (a) 140 ml

(d) Elongated kidney without reniform shape (b) 180 ml

with multiple cysts (c) 240 ml

(e) All of the above (d) 280 ml (e) 340 ml Answer: (b) The differentiation between

a severely obstructed kidney and a multi- Answer: (c) The formula to estimate the cystic dysplastic kidney (MCDK) can be bladder capacity is as follows: Bladder capacity challenging. The key differential feature is (ounces) = age (years) + 2. To convert ounces the contiguity of the dilated renal pelvis with to ml, one multiples the capacity by 30. Thus, the dilated calyces that is not seen in a a 6-year-old child should have a bladder capa-

MCDK and the lack of involution of the city of 240 ml (or 8 ounces). (Page 48) severely obstructed kidney on serial exams. Furthermore, additional features suggestive 7. The following ultrasound is obtained on a of a MCDK include: an echogenic elongated 2-week-old neonate as follow-up for prenatally kidney without reniform shape; multiple detected right SERP. Which of the following noncommunicating cysts; a contralateral echo- is the likely diagnosis? genic kidney; and perhaps most importantly, involution of the MCDK over time. (Page 40)

5. An IOUS is performed on routine follow-up on a 14-day-old neonate found to have bilateral prenatal sonographically evident renal pelvis (SERP). Examination of the bladder ultra- sound demonstrates echogenic material within the bladder lumen. Which of the following is most likely responsible for this finding? (a) Bladder calculi (b) Phosphate crystals In-office ultrasonography 9

(a) Normal right kidney without hydro- nephrosis (b) Right kidney with Society of Fetal Urology (SFU) Grade 2 hydronephrosis (c) Right kidney SFU Grade 4 hydro- nephrosis (d) Right kidney with duplex system and hydronephrosis of upper pole moiety (e) Right kidney with duplex system and hydronephrosis of lower pole moiety Answer: (d) This ultrasound image obtained in the office represents an ideal way to follow-up on prenatally detected SERP. This (a) Normal kidney kidney likely has a duplex system as evidenced (b) SFU Grade 1 hydronephrosis by separation of the central echogenic com- (c) SFU Grade 2 hydronephrosis plex (CEC). The upper pole in a duplex (d) SFU Grade 3 hydronephrosis system is seen laterally on a sagittal view of the (e) SFU Grade 4 hydronephrosis kidney and the lower pole medially. In this

case, hydronephrosis is seen in the upper pole Answer: (a) This is an image of a normal moiety. (Pages 37, 40, 44) right kidney. The hypoechoic pockets seen between the renal cortex and renal sinus are prominent renal pyramids typically seen in 8. An IOUS is obtained in the office on a children. These are not to be confused with 1-month-old child as follow-up for prenatally hydronephrosis as there appears to be no detected SERP. The following ultrasound is connection with the renal pelvis. Furthermore, obtained. What best describes the ultrsound there is no evidence of dilation within the finding? CEC. (Page 37)

Pediatric renal nuclear medicine 5 Aseem R Shukla (Based on chapter written by Martin Charron)

1. The radioactive tracer providing the most Answer: (b) MAG3 does bind with plasma effective relative measure of glomerular filtra- protein, but does not filter through the glo- tion rate (GFR) in children more than 5 year merular membrane, giving it a high extraction of age is: fraction and resolution. MAG3 is the radio- (a) 99m Tc-diethylenetriaminepentaacetic acid tracer of choice for its reliable estimation of (DTPA) differential renal function and renal clearance. (b) 99m Tc-mercaptoacetyl triglycine (MAG3) The remainder of the choices are all true. (c) 131 I- or 123 I-orthoiodohippurate (OIH) 99m (d) 99m Tc-dimercaptosuccinic acid (DMSA) 3. The Tc DMSA is ideal for the evaluation of (e) 131 I radioiodine acute pyelonephritis because: Answer: (a) 99m Tc DTPA has > 95% excretion (a) It accumulates in the proximal renal by glomerular filtration and becomes a reliable tubules method of measuring relative function on (b) Impaired renal cortex and space-occupying each side and measuring GFR in children more lesions are depicted as hyperactive areas than 5 years of age. DTPA is taken up by the with DMSA kidney through glomerular filtration and is (c) DMSA accurately differentiates between not secreted or reabsorbed by the renal tubules. a renal scar or acute pyelonephritic Once it reaches the kidney, about 20% is changes accumulated and the remainder flows away. (d) The DMSA scan detects pyelonephritic changes while determining the presence 2. All of the following regarding 99m Tc MAG3 of a ureteropelvic junction obstruction are true except: (e) Acute pyelonephritis appears as a nonseg- (a) It is taken up in the proximal tubules mental reduction in cortical accumulation (b) It shows high plasma protein binding and of DMSA readily filters through the glomerular Answer: (e) DMSA accumulates in function- membrane ing renal cortical tissue for a prolonged period (c) A high extraction fraction makes high- and impaired areas appear hypoactive. The resolution imaging possible DMSA scan is a preferred adjunct in the (d) Differential renal function analysis may be diagnosis of a child with pyelonephritis, but difficult if one kidney has markedly does not provide reliable assessment of urinary decreased or delayed renal function obstruction. While hypoactive areas may (e) A poorly functioning renal unit renders a reflect acute pyelonephritis, follow-up imaging MAG3 study unreliable due to a poor is necessary to determine if a previously response to furosemide administered detected lesion is a permanent parenchymal during the exam abnormality, such as a renal scar.

Prenatal and postnatal urologic 6 emergencies

Sarah Marietti and Fernando Ferrer (Based on chapter written by Patrick McKenna and Fernando Ferrer)

1. All of the following are findings of bladder (e) CAH is the most likely diagnosis when exstrophy by screening ultrasound except: the karyotype is 46XX and uterus is not visualized on ultrasound (a) Lower abdominal bulge (b) Low set umbilicus Answer: (d) The most common cause of (c) Nonvisualization of the bladder ambi guous genitalia is CAH. The most com- (d) Hydronephrosis mon enzyme deficiency associated with CAH (e) Small penis is 21-hydoxylase deficiency. The second most common deficiency is 11-hydroxylase. Lastly, Answer: (d) Studies have identified five 3beta-hydroxysteroid dehydrogenase can also common prenatal finding on ultrasound to cause CAH but is extremely rare. CAH, if suggest bladder exstrophy: inability to visua- unrecognized and untreated, is life threatening lize bladder on multiple ultrasounds, lower in the salt-waster secondary to poor feeding, abdominal bulge, small penis with anteriorly dehydration, and hyperkalemia leading to placed scrotum, low set umbilicus, and abnor- arrhythmias and shock. When the cause is mal widening of the iliac crests. Although 21-hydroxylase, blood tests for 17-hydroxy- hydronephrosis can be found in addition progesterone and progesterone will be elevated. to bladder exstrophy, it is not indicative of These tests should be ordered after day 2 of the diagnosis. life because they may be falsely elevated early in life from the stress of delivery. 45XX 2. In regards to ambiguous genitalia which of CAH patients will still have Müllerian ducal the following is true? structures, such as a uterus, visible on pelvic ultrasound. (a) The most common cause is mixed gonadal dysgenesis 3. In regards to neonatal : (b) The most common enzyme deficiency in congenital adrenal hyperplasia (CAH) is (a) Amputation is usually associated with 11B-hydroxylase plastibell clamp use (c) CAH may be life-threatening secondary (b) The removed penile skin should be used to low serum potassium and high serum to repair a degloving injury sodium levels (c) If urethral injury occurs, a catheter should (d) 17-hydroxyprogesterone levels will be be placed to allow healing by secondary elevated when CAH is present intention 14 Clinical Pediatric Urology Study Guide

(d) Bleeding should be cauterized while the (b) Wilms ' tumor metallic clamp is in place (c) Renal cell carcinoma (d) Congenital mesoblastic nephroma Answer: (b) The mogan clamp is more com- monly associated with amputation of the glans Answer: (d) Neuroblastoma is the most com- as the clamp may be placed at an angle draw- mon solid malignancy in the neonate. Wilms' ing the frenulum and the glans up into the tumors are the most common renal tumors in clamp. The removed penile skin, if preserved, children but rarely occur in the neonate. Renal may be used to repair an injury. Injuries cell carcinoma is rare in young children. How- involving the urethra should be managed by ever, in children older than 10 years of age, immediate reconstruction within hours of the presenting with a solid renal mass, 50% will injury. Repairs performed days after the injury be of renal cell origin. Congenital mesoblastic have been reported but with decreased success nephroma is the most common solid renal rates. If bleeding is cauterized while the mass in the neonate. Many are detected prena- metallic clamp is in place, the current may be tally or in the first month after birth. The transferred to the clamp causing unintentional cellular variant is capable of metastasis and penile cauterization. once this occurs the child will require chemo- therapy. The classic variant is not capable 4. Multicystic dysplastic kidney (MCDK): of metastasis and nephrectomy alone is the (a) Is associated with contralateral renal treatment of choice. anomaly in 50% of patients 6. When treating a presumptive patient with (b) Is diagnosed by ultrasound revealing mul- CAH with prenatal corticosteroids, what frac- tiple connected cysts tion of patients are not expected to benefit? (c) Is typically followed with serial ultra- 1 sounds for spontaneous regression (a) /4 1 (d) When associated with vesicoureteral reflux (b) /2 1 is usually grade IV– V (c) /8 (d) 7/ Answer: (c) MCDK is usually unilateral and 8 (e) 3/ associated with contralateral anomalies 25% 4 of the time. The contralateral kidney anomaly Answer: (d) CAH is inherited in an auto- is most commonly UPJ obstruction or vesi- somal recessive fashion which gives a 25% coureteral reflux. When reflux is present (25% likelihood of being afflicted with the disease. of the time), the majority are low grade and Corticosteroids are given early in the first most will resolve spontaneously. Ultrasound trimester in an attempt to prevent viralization should demonstrate nonconnected cysts and of the external genitalia. In that 50% of fetuses 7 should be done serially to verify regression. In will be male, /8 of recipients will not benefit the past, nephrectomy was the recommended from treatment. treatment based on reports of malignancy within these kidneys. However, chance of 7. Anorectal malformations are associated with a % malignancy is extremely low, making conser- urologic abnormality in 20– 60 of patients. vative management standard. One of these, neuropathic bladder, should be studied early in the evaluation process with 5. The most common solid malignancy in the urodynamics. What is the expected etiology of neonate is: a neuropathic bladder in this population? (a) Neuroblastoma (a) Tethered cord Prenatal and postnatal urologic emergencies 15

(b) Spina bifida Which of the following should be performed (c) Lipomeningocele immediately: (d) Syrinx (a) Ultrasound of pelvis (e) Myelomeningocele (b) Sequential multi-channel analysis (SMA)-7 Answer: (a) When vertebral anomalies are (c) Karyotype present, a tethered spinal cord is the most (d) Genitogram common etiology of a neuropathic bladder in (e) 17-hydroxyprogesterone level this population. Although the other processes Answer: (b) Although imperative for the may occur, it is not an anticipated finding in correct diagnosis, the karyotype, steroid path- this population. way precursor studies and radiologic imaging 8. In the newborn nursery on routine examina- do not need to be performed immediately. tion 5 hours after a prolonged vaginal delivery, Salt wasting CAH can be fatal if not detected an erythematous, firm and tender right hemi- in the newborn time period. scrotum is discovered. On reviewing the deli- very examination notes, both testes were 10. During the evaluation of gross hematuria for thought to be present at birth and the scrotum a 1-week-old premature infant born to a dia- was normal. The next step should be: betic mother, a renal ultrasound reveals an edematous kidney with minimal but present (a) Allow discharge home arterial wave form. A renal scan reveals an (b) Observation in the nursery for 24 hours enlarged kidney with decreased blood flow (c) Immediate exploration for suspected and function. The most likely diagnosis is: testicular torsion (d) Attempt to detorse at the bedside and (a) Adrenal hemorrhage then obtain an ultrasound (b) Renal artery thrombosis (e) Obtain a KUB radiograph (c) Renal trauma secondary to prolonged vaginal delivery

Answer: (c) Neonatal torsion may present (d) Congenital mesoblastic nephroma in the early postdelivery time period. It is hall- (e) Renal vein thrombosis marked by a change in the scrotal exam as

compared to immediate delivery. In this set- Answer: (e) Although all of these may ting, i.e. < 6 hours, immediate exploration is result in gross hematuria in the neonate, warranted. The pathogenesis is likely to be given the above history, renal vein thrombosis extravaginal torsion. Bilateral inguinal inci- is the most likely etiology. Infants of diabetic sions are most commonly used because of the mothers may be initially dehydrated and increased incidence of a patent process. Also, result in renal vein thrombosis. A renal ultra- in rare cases, a testicular tumor is responsible sound of renal artery thrombosis typically for the torsion of the . does not demonstrate an edematous kidney and the renal scan will reveal no function 9. When evaluating a newborn with ambiguous and no blood flow. The management of renal genitalia, the phallic structure is a Prader III vein thrombosis may consist of hydration and/ with a perineal hypospadias. Labioscrotal or anticoagulants depending on the clinical folds are fused. Neither gonad is palpable. situation.

Urinary tract infections 7 in children

Hans Pohl and H Gil Rushton

1. All of the following are true of bacteriuria (b) Risk of recurrent UTIs in older girls except: approaches 60% (c) Girls with a history of bacilluria have a (a) Declines with increasing age greater incidence of bacilluria of preg- (b) More common in boys aged > 10 years nancy than healthy controls than in girls aged > 10 years (d) Children born to bacilluric women do (c) Male to female predominance (2.5:1) in not have a greater risk of bacilluria them- first year of life selves (d) Treatment with short-term oral antibiotics (e) Up to two-thirds of boys with sympto- does not usually provide lasting resolu- matic UTIs will present with temperature tion of occult bacteriuria > 38 ° C (e) Ten times more common in uncircum- cised than in circumcised boys Answer: (d) Prospective studies have shown Answer: (b) The results of several large that some children carry a lifelong propensity epidemiological studies have shown that bac- towards UTI. One study found a greater inci- teriuria occurs in 1– 1.4 % of neonates, most dence of bacteriuria during pregnancy among of those being uncircumcised boys under girls with a childhood history of bacteriuria as % % 6 months of age. The male:female ratio is compared to healthy controls (64 and 27 , reversed in older children. In a prospective respectively). This propensity persisted in the screening study in Sweden of 3581 infants children born to bacteriuric women, whereas who underwent suprapubic aspiration, the none of the children born to healthy controls incidence of occult bacteriuria was 2.5% in demonstrated bacteriuria. boys and 0.9% in girls. Symptomatic urinary tract infection (UTI) occurred with equal 3. Which of the following is true: % % frequency (1.2 of boys and 1.1 of girls) in (a) An uncircumcised adolescent boy is more both sexes. The male predominance declines likely to acquire a UTI with Proteus than after the first year of life, such that later in with E. coli childhood, bacteriuria, whether occult or symp- (b) Neonatal boys are at similar risk for UTI tomatic, is very unlikely in boys as compared caused by E. coli when compared with with girls. neonatal girls (c) Adolescent girls are at greatest risk of 2. All of the following are true of UTIs except: streptococcal UTIs (a) Risk of recurrent UTI is approximately (d) Klebsiella is a frequent cause of UTI in 25 % in infants older children 18 Clinical Pediatric Urology Study Guide

(e) Struvite stones are associated with (b) P. fimbrial receptors are “ not exposed ” Pseudomonas UTIs and are located within the connective tissue that supports the uroepithelium Answer: (a) Although the majority of UTIs (c) Some soluble urinary proteins may actually are caused by E. coli, the bacteriologic findings promote bacterial adherence by binding in the Goteborg bacteriuria study suggest that to P. fimbria the environmental factors which determine (d) A bacterium' s phenotype is invariable, what type of bacteria cause UTIs are in turn thus it reliably predicts the organism' s influenced by age and gender. ability to colonize the upper urinary tract (e) P. fimbria may be differentiated by differ- 4. Choose the best response regarding the impor- ent tip antigens which impart selectivity tance of bacterial virulence factors: for uroepithelial cells in different locations (a) K capsular antigens, hemolysin and aero- of the urinary tract bactin are critical for tissue invasion by Answer: (e) Further research has identified bacteria that not all P. fimbria are alike. Proteins located (b) Only mannose-resistant fimbria are at the tip (G-tip proteins, or tip adhesins) important in cell surface attachment determine the fimbria' s specific attachment (c) The presence of P. fimbria predisposes to properties. Three classes have been identified, nonreflux pyelonephritis of which only class II and class III P. fimbria (d) Hemolysin-producing E. coli are more have uropathogenic potential. In vitro studies virulent due to their ability to resist have found that the class III P. fimbria bind phagocytosis receptors found in higher density on bladder (e) Lewis blood group secretors are more uroepithelium. In contrast, class II tip adhesins likely to be infected by P. fimbriated strains may be more important in the evolution of of E. coli acute pyelonephritis by virtue of an increased Answer: (c) P. fimbria poses specific adhe- density of class II specific receptors located on sion molecules that are most associated the uroepithelium of the upper urinary tract. with nonreflux pyelonephritis. Epidemiolo- gical studies in children have provided con- siderable evidence that the presence of 6. All of the following are host risk factors for P. fimbriae on E. coli is a significant virulence the development of UTI except one: factor, particularly in upper UTIs. These studies (a) Perineal or perimeatal colonization have shown that 76 – 94 % of pyelonephrito- (b) Presence of low-grade vesicoureteral genic strains of E. coli are P. fimbriated com- reflux (VUR) pared with 19 – 23 % of cystitis strains, 14– 18 % (c) Presence of P. fimbriated E. coli in fecal of strains isolated from patients with asymp- isolates tomatic bacteriuria, and 7– 16% of fecal isolate (d) Absence of secreted antigens which assist strains. in clearing bacteria from the urinary tract (e) D elayed micturition and constipation 5. Regarding the role of P. fimbria in the etiology Answer: (b) VUR in and of itself does of UTIs choose the true answer: not increase a child' s risk for UTI, instead (a) P. fimbria predispose to febrile UTIs should UTI occur, the presence of VUR – only by promoting bacterial tropism particularly of higher grades – can increase the (attachment) to renal parenchyma risk for acquired renal cortical abnormalities. Urinary tract infections in children 19

7. A 7-year-old Hispanic girl recently immi- 9. All of the following regarding elimination and grated to the United States presents with the risk of UTIs are true except: frequent urination (sometimes hourly). Her (a) Greater than 50% of children being evalu- mother specifically denies posturing maneu- ated for UTI will demonstrate squatting, vers. She has no daytime incontinence and no delayed micturition and/or constipation history of fecal soiling. She remains dry (b) A majority of girls with occult bacilluria throughout the night. Her physical exam and have urodynamic evidence of dysfunc- urinalysis are both normal. Choose the best tional bladders diagnosis: (c) Dysfunctional voiding rarely contributes (a) Lazy bladder syndrome to the risk of UTI in children with ana- (b) Occult spina bifida tomically normal urinary tracts (c) Bacterial cystitis (d) Placing a child with a history of UTI on (d) Daytime frequency syndrome oral fiber supplements significantly reduces (e) Urgency-frequency syndrome the likelihood of further infections (e) Children with dysfunctional voiding often Answer: (d) The key feature of this child' s have urinary leakage despite large bladder voiding complaints is the high frequency with capacities which she is voiding throughout the day, without other features of dysfunctional elimi- Answer: (c) The presence of dysfunctional nation such as posturing, wetting or constipa- elimination should be considered, evaluated tion and fecal soiling. The daytime frequency for and treated in any child presenting with syndrome is selflimiting and not associated UTI and/or VUR, particularly since it has been with an increased risk for UTI. determined by a number of studies that both the risk of recurrent UTI and VUR resolution are beneficially influenced by treating abnormal 8. An 8-year-old girl presents for evaluation of elimination. culture documented recurrent UTIs charac- terized by dysuria and wetting in the absence 10. Choose the best response regarding reflux of fevers. Her parents note that she often nephropathy: crosses her legs and urgently seeks the toilet. She has infrequent bowel movements and has (a) Experimental models have confirmed the recently had one or two episodes of fecal soil- detrimental effect of sterile primary urinary ing. Her physical exam and urinalysis are both reflux on renal parenchyma normal. Choose the best diagnosis: (b) Focal acquired renal scarring is most likely to occur at polar regions in association (a) Dysfunctional/infrequent voider with compound calyces (b) Detrusor hyperreflexia with urge inconti- (c) Among children with dimercaptosuccinic nence acid (DMSA) scan documented acute (c) Normal for her age pyelonephritis, those with VUR are more (d) Daytime frequency syndrome likely to develop newly acquired renal (e) Ectopic ureter scarring than those without Answer: (a) This child suffers from urgency (d) In kidneys with severe VUR, clinical and urge incontinence, but the coexistence studies have demonstrated progressive of constipation suggests that she has a more acquired renal scarring despite successful pervasive elimination problem than simple control of urinary infection with anti- bladder overactivity. biotic prophylaxis 20 Clinical Pediatric Urology Study Guide

(e) Renal papilla are inherently susceptible to (b) A 2-year-old girl with high fevers and a invasion by pyelonephritogenic E. coli positive culture obtained by catheteriza- tion Answer: (e) Primary, sterile reflux of urine (c) A 5-year-old girl with a nonfebrile urine does not harm the renal parenchyma and is culture documented UTI and a history of the basis for placing children on antibiotic fecal soiling and enuresis prophylaxis while awaiting resolution of (d) A 7-year-old girl with three nonfebrile VUR. Acquired renal cortical abnormalities culture documented UTIs over the last more commonly occur on the upper and lower 2 years and a normal voiding history poles of the kidney, as a consequence of differ- (e) A 7-year-old boy with his first culture ences in papillary configuration which permits documented UTI and a normal voiding influx of urine into the parenchyma. The risk history of renal scarring (approx 40% ) is no different among children with acute pyelonephritis Answer: (d) Children with UTI are evalu- (APN) who have VUR as compared with ated in order to identify urological abnormali- those who do not have VUR. ties that place that child at risk for progressive renal dysfunction should recurrent infection occur. In the case of scenario (d), the child has 11. Choose the false statement regarding post- demonstrated no propensity for acquired renal pyelonephritic renal scarring: cortical abnormalities on the basis that all of (a) Hypertension is most common in chil- her UTIs are confined to the lower urinary dren with bilateral acquired renal scars tract. Thus, one could argue that no radio- (b) A ll renal scarring associated with severe logical evaluation is needed, but instead a reflux is acquired thorough history should be obtained and any (c) T he incidence of hypertension in children dysfunctional elimination habits addressed as with acquired renal scarring increases with part of the therapeutic course. age (d) N ot all patients with acquired renal scar- 13. Studies frequently employed in the initial eval- ring suffer hypertension uation of a child with a history of febrile UTIs (e) P renatally detected reflux may be asso- include all except: ciated with significant functional abnor- malities even in the absence of infection (a) I ntravenous pyelography (b) R enal and bladder ultrasound Answer: (b) When performed prior to the (c) Contrast voiding cystourethrogram in first febrile UTI, DMSA renal scans have boys shown renal cortical abnormalities – particu- (d) Direct radionuclide cystography in girls larly in kidneys associated with high grades of (e) DMSA renal scintigraphy VUR – which can only be ascribed to defects during induction of renal tissue. Answer: (a) Intravenous pyelography has no role in the evaluation of the child with UTI. Other imaging protocols, including the stan- 12. Which child does not require radiographic dard combination of cystography and sonog- evaluation following a UTI: raphy, or the recent “ top-down ” approach (a) A 2-year-old uncircumcised boy with a that obtains early DMSA renal scans as the positive screening urine culture confirmed first-line evaluation tool, are more appropriate by cathed specimen for evaluating. Urinary tract infections in children 21

14. All of the following statements concerning (c) A 2-year-old uncircumcised boy with a DMSA radionuclide renal scans are true positive urine culture obtained by collec- except: tion bag (a) T ransient acute pyelonephritic changes (d) A 7-year-old girl with surgically corrected may persist up to 2 months after the VUR and a postoperative isocystogram initial episode demonstrating no VUR (b) R enal scarring on DMSA scans appears (e) A 5-year-old girl with recurrent febrile as focal or generalized areas of cortical UTIs and resolved VUR contraction Answer: (e) Regardless of the reflux status, (c) U ltrasonography is as accurate as DMSA the child in (e) is at greatest risk for acquired renal scans in detecting areas of pyelone- renal cortical scarring and thus deserves phritic scarring preventive measures. (d) C hanges associated with acute pyelone- phritis on DMSA renal scans appear as areas of decreased isotope uptake and intact renal cortex without renal volume 16. All of the following regarding the treatment loss of acute urinary infection is true except: (e) D MSA scintigraphy is useful in detecting ° acute pyelonephritis in children perform- (a) A 7-year-old with fevers to 102 F should ing clean intermittent catheterization receive 1– 2 days of intramuscular Ceftri- (CIC) axone to begin therapy (b) A 2-month-old girl with a culture docu-

Answer: (c) Several comparative studies, mented febrile UTI may be managed on including one in an experimental model in the an outpatient basis with oral antibiotics piglet, have shown that sonography is half (c) O ral antibiotic therapy for acute cystitis as sensitive as DMSA renal scans for the detec- should be followed by antibiotic prophy- tion of acute pyelonephritis. laxis in children < 5 years of age until 15. Which of the following children requires anti- evaluation is completed > microbial prophylaxis: (d) A cute cystitis rarely requires 5 days of oral antibiotics (a) A 16-year-old myelodysplastic girl with- (e) P rompt treatment of acute urinary infec- out VUR on clean intermittent catheter- tion in infants is critical in preventing the ization and positive urine cultures formation of renal scarring (b) A n 8-year-old girl without VUR with three culture documented afebrile UTIs Answer: (b) An infant of this age should be within 18 months managed initially with intravenous antibiotics.

Fungal, parasitic, and other 8 inflammatory diseases of the genitourinary tract

Ilene Wong and William A Kennedy II

1. What quantity (threshold in cfu/ml) will (b) Side -effects can include fever and general diagnose true candiduria on suprapubic tap malaise and straight catheterization? (c) Creatinine should be checked only at the end of therapy Suprapubic tap Straight (d) Peak and trough values of amphotericin B catheterization should be two times the minimum inhibi- tory concentration (MIC) and one time (a) Any number of Any number of the MIC, respectively cfu/ml cfu/ml (e) The treatment course is often required for (b) Any number of > 10,000 cfu/ml several weeks cfu/ml Answer: (c) Amphotericin B treatment car- (c) > 1000 cfu/ml> 20,000 cfu/ml ries with it a significant risk of nephrotoxicity, (d) > 10,000 cfu/ml > 15,000 cfu/ml requiring creatinine monitoring every other (e) > 10,000 cfu/ml> 100,000 cfu/ml day. Goal peak and trough levels to minimize toxic side-effects are twice the MIC and one Answer: (d) The diagnosis of candiduria is time the MIC. Side-effects include fever, nau- frequently complicated by contamination of sea and vomiting as well as generalized malaise. specimens and the rapid growth of organisms Amphotericin B treatment duration is variable in urine. Kozinn et al1 have shown colony and dependent on the extent of disease. counts of > 10,000 cfu/ml to be associated with clinical infection for suprapubic tap. 3. A 7-year-old girl/status post-bone marrow Wise et al 2 showed that > 15,000 cfu/ml signi- transplant presents with hematuria and fies true candiduria in straight catheterized dysuria. A CT scan reveals a large fungus ball females, while a threshold of > 100,000 is used in her renal pelvis. Urine cultures reveal to determine candiduria in patients with an aspergillosis. What is the next recommended indwelling catheter. treatment? (a) Nephrectomy 2. You are counseling the parents of a (b) Intravenous fluconazole 4-month-old child with systemic candidiasis (c) Percutaneous nephrostomy on the potential risks of treatment. Which of (d) Percutaneous nephrostomy followed by the following statements about the treatment amphotericin B irrigation of systemic candidiasis with amphotericin B (e) Oral fluconazole are false? Answer: (d) The optimal treatment of a (a) Amphotericin B can be nephrotoxic patient with an isolated fungal ball without 24 Clinical Pediatric Urology Study Guide

systemic sepsis is amphotericin B irrigation 6. Which of the following are useful in the through a percutaneous nephrostomy tube. diagnosis of echinococcal infections of the Surgical or percutaneous removal of the urinary system: fungus ball may be required if this fails. (a) The recovery of daughter cysts in the urine If invasive aspergillosis is expected, intrave- (b) Identification of parasite in urine and nous amphotericin B is recommended. feces (c) Serum enzyme-linked immunosorbent 4. The radiographic appearance of renal coccy- assay (ELISA) diomycosis is similar to: (d) (a) and (c) (a) Schistosomiasis (e) (b) and (c) (b) Tuberculosis (f) (a), (b) and (c) (c) Renal cell carcinoma Answer: (d) Echinococcal infections in (d) Xanthogranulomatous pyelonephritis humans occur during an intermediate stage in (e) Angiomyolipoma the parasitic life cycle. Thus, diagnosis can Answer: (b) Renal coccydiomycosis is often only be made through the identification of mistaken for renal tuberculosis in its radio- scolices or daughter cysts in the urine rather graphic appearance, with infundibular stenoses, than by visualization of the parasite itself. blunted or sloughed calyces and calcified gra- Serum ELISA assays can also be used to nulomas frequently seen. Typically the organ- identify echinococcal infections; however, ism is limited to the cortex, presenting with their increased sensitivity is offset by the miliary granulomas and microabscesses. possibility of false-positive reactions with other parasites. 5. Which of the following is not one of the sequelae of schistosomiasis infection? 7. A 5-year-old human immunodeficiency virus (HIV)-positive girl presents with gross hema- (a) Hematuria turia, dysuria and urge incontinence. Urine (b) Vesicoureteral reflux cultures at the time are negative. Ultrasound (c) Bladder outlet obstruction of the bladder shows a diffusely thickened (d) Bladder calculi bladder wall. Urine immunofluorescence is (e) Transitional cell carcinoma positive for adenovirus antigen. What is the Answer: (e) Schistosomiasis can have wide- next step in treatment? spread genitourinary effects. Acute infection (a) Aggressive hydration can cause hematuria severe enough to lead (b) Cystoscopy and bladder biopsy to anemia. The inactive infection state, (c) Oral fluconazole characterized by infiltrative fibrosis, can (d) Oral corticosteroid therapy cause a wide range of lower urinary tract (e) Chemical fulguration with formalin pathology including hydronephrosis, bladder outlet obstruction and vesicoureteral reflux. Answer: (a) Adenovirus types 11 and 21 are Complications of urinary stasis, including the most common cause of viral cystitis, which urinary tract infections, bladder calculi and has a higher incidence in immunocompro- squamous metaplasia leading to squamous cell mised patients. These infections are usually carcinoma, can also occur. self-limited and can be treated with fluid Inflammatory diseases of the genitourinary tract 25

resuscitation; resolution of sonographic by Michaelis-Gutman bodies, which are bladder wall thickening typically occurs in “ owl-eye” inclusions in the cytoplasm. 2 – 3 weeks. 10. The physical exam of a child with idiopathic 8. An 8-year-old boy presents with 2 days of scrotal edema is best aided by: gradually worsening left scrotal pain. He is afebrile and a urinalysis is negative. Doppler (a) Digital rectal exam ultrasound of the scrotum reveals normal flow (b) Manipulating the testis into the superficial to bilateral testes and a left hydrocele. He has inguinal pouch for diagnostic palpation a history of dysfunctional voiding. What is the (c) Evaluation of the contralateral testicle most likely etiology for his pain? (d) Close evaluation of the penis, including retraction of the foreskin (a) Ectopic ureter draining to the vas (e) Palpation of the deferens (b) Torsion of the appendix testis Answer: (b) Characterized by edema con- (c) Hematogenous spread from a systemic fined to the superficial layers of the scrotum infection in an afebrile child with normal urinalysis, (d) Retrograde passage of urine from the idiopathic scrotal edema is typically self- ejaculatory ducts limiting. Though is often (e) Direct extension from a previous bout of performed, diagnostic palpation after manipu- orchitis lating the testis into the superficial inguinal pouch will normally reveal a nontender and Answer: (d) Epididymitis can be caused by normal testis. Rectal exam and palpation of an infectious agent or chemical irritants. How- the vas deferens are nondiagnostic, and the ever, Megalli et al3 have also theorized that penis is rarely involved. retrograde passage of sterile urine from the ejaculatory ducts may cause inflammation. 11. Which of the following are not part of the Bukowski et al4 further suggested that a dys- treatment of nonspecific vulvovaginitis? functional voiding pattern may cause retro- grade flow through due to external sphincter (a) Promoting the use of loose-fitting spasm. This is the most likely etiology in a undergarments patient who presents with no signs of systemic (b) Avoidance of bubble-baths infection and a sterile urine specimen. (c) Empiric treatment with PO tetracycline (d) Examination and identification of vaginal 9. Malacoplakia is thought to be caused by: foreign objects (a) An immunoglobin G (IgG) deficiency (e) Emphasizing good handwashing tech- (b) Abnormal macrophage function niques (c) Obstruction by Michaelis-Gutman bodies Answer: (c) The treatment of nonspecific (d) Chronic staphylococcal infections vulvovaginitis is typically geared towards (e) Lymphocyte overactivity reducing risk factors including poor hygiene, Answer: (b) Malacoplakia is a benign granu- use of chemical irritants, presence of foreign lomatous condition thought to be caused by objects (including toilet paper) and promot- defective digestion by abnormal macrophages. ing drying of the vulva by wearing loose- Its histologic appearance is characterized fitting underwear. Treatment of vulvovaginitis 26 Clinical Pediatric Urology Study Guide

with a specific cultured organism is dependent 2. Wise GJ , Goldberg P , Kozinn PJ . Genitourinary can- on the species isolated. didiasis: diagnosis and treatment. J Urol 1976; 116 : 778 – 80 . 3. Megalli M , Gursel E , Lattimer JK . Reflux of urine into References ejaculatory ducts as a cause of recurring epididymitis in children. J Urol 1972; 108 : 978 – 9 . 1. Kozinn PJ , Taschdjian CL , Goldberg PK et al. Advances 4. Bukowski TP , Lewis AG , Reeves D et al. Epididymitis in the diagnosis of renal candidiasis. J Urol 1978 ; 119 : in older boys: dysfunctional voiding as an etiology. 184 – 7 . J Urol 1995; 154 : 762 – 5 . Pain management for 9 the pediatric patient

Jason W Anast and Paul F Austin (Based on chapter written by Stephen C Brown and Patricia A McGrath)

1. Which of the following is a valid method of improve pain due to nerve compression or assessing pain intensity in a child? metastatic disease. Amitriptyline is an antide- pressant which can effectively treat neuro- (a) P atient-reported questionnaire pathic pain after 1– 2 weeks of therapy. (b) P arent-reported checklist of presumed Selective serotonin reuptake inhibitors such as pain-related behaviors fluoxetine are useful for treating depression (c) P ain index based on patient heart rate and but have not been shown to improve pain in blood pressure children. (d) (a) and (b) (e) A ll of the above 3. Which of the following regimens is not a Answer: (d) Multiple validated scales exist recommended method for controlling acute to measure pain in children. Behavioral scale postoperative pain in children? checklists that measure presumed pain beha- (a) I ntravenous fentanyl viors such as crying, limb rigidity, or facial (b) I ntravenous meperidine grimacing have been shown to accurately mea- (c) O ral ibuprofen sure acute postoperative pain. Additionally, (d) R ectal acetaminophen self-reported pain scales accurately reflect the (e) P atient-controlled administration of intra- child' s true level of pain. Although physiologic venous hydromorphone measures such as heart rate and blood pressure

provide insight into the level of distress in Answer: (a) In general, the oral route should children, presently there are no valid physio- be utilized to administer pain medications logic pain scales in children. since it avoids the additional pain of intrave- nous, intramuscular administration. The intra- 2. Which of the following medications does not muscular route should be avoided in general have a beneficial effect on pain? because it is often painful and does not deliver reliable dosages. Intravenous fentanyl should (a) G abapentin not be used for treating acute postoperative (b) D iclofenac pain due to the high risk of respiratory depres- (c) F luoxetine sion or arrest. (d) P rednisone (e) A mitriptyline 4. When performing an inguinal nerve block on an 8 kg child, what is the maximum volume of Answer: (c) Gabapentin is an anticonvulsant 0.5 % bupivicaine that should be used? which can be effective for treating neuropathic pain. Diclofenac is an anti-inflammatory drug. (a) 2 ml Prednisone is a corticosteroid which can (b) 4 ml 28 Clinical Pediatric Urology Study Guide

(c) 6 ml (e) R are complications of penile block include (d) 8 ml penile ischemia, excessive bleeding, and (e) 16 ml abscess Answer: (a) The maximum dose of bupiva- Answer: (b) The penile block is a safe and caine should not exceed 1.25 mg/kg when per- simple anesthetic method for penile surgery. forming ilioinguinal block in children < 15 kg In a prospective study, it was shown that in weight.1 For 0.5% bupivicaine, this is equiv- penile block resulted in less pain and less post- alent to 0.25 ml/kg, or 2 ml in an 8 kg child. operative pain medication than subcutaneous ring block.2 5. Which of the following statements about the penile block is not true? (a) T he targeted nerves run deep to Scarpa' s fascia References (b) T he penile block provides equivalent anes- thesia as the subcutaneous penile ring 1. Smith T , Morati P , Wulf H . Smaller children have block greater bupivacaine plasma concentration after ilioin- (c) T he needle is inserted in the subpubic guinal block. Br J Anaesth 1996; 76 : 452 – 5 . 2. Holder KJ , Peutrell JM , Weir PM . Regional anaes- region just lateral to the midline thesia for circumcision. Subcutaneous ring block of (d) T he penile ring block is an easily learned the penis and subpubic penile block compared. Eur technique J Anaesthesiol 1997; 14 : 495 – 8 . Office pediatric urology 10 Kara Saperston , Patrick Cartwright , and Brent Snow (Based on chapter written by Patrick Cartwright, Timothy Masterson, and Brent Snow)

1. By what age does 95% of physiologic phimosis (c) Urinary tract infections (UTIs) most commonly resolve? (d) Zipper trauma (a) 1 – 2 years of age Answer: (b) Non-neonatal circumcision is (b) 3– 5 years of age most commonly performed because of phimo- (c) 6 – 7 years of age sis; while balanoposthitis and zipper trauma (d) 8 – 9 years of age are causes for circumcision they are not the Answer: (b) The physiologic adherence of most common cause. UTIs in uncircumcised the preputial attachments to the glans resolves males appear to be more common only in the spontaneously around 3– 5 years of age. While first 6 months of life; after 1 year of age it is most texts agree on this age range, Oster felt the risk for UTIs diminishes significantly performed a longitudinal study that showed and does not warrant circumcision. some boys will continue to resolve their 4. A finding on voiding cystourethrogram physiologic phimosis when they are as old as (VCUG) consistent with urethral prolapse is: 10 or 11 years of age.1 (a) Narrow distal urethra 2. The recommended treatment for phimosis (b) Wide bladder neck includes all of the following except: (c) Vaginal voiding (a) Circumcision (d) Inability to void (b) Use of topical corticosteroids Answer: (a) Urethral prolapse appears as a (c) Forceful separation of the inner prepuce doughnut-shaped mass protruding at the and glans penis vulva. It is usually diagnosed based on appear- (d) Watchful waiting ance of the lesions. At times it is confirmed by

Answer: (c) Forceful separation of foreskin placing a catheter in the urethra. Imaging is adhesions is not recommended because not necessary but a narrow distal urethra has of pain and trauma to the patient; gentle been seen on VCUG. proximal traction of the foreskin may enable one to differentiate between normal and 5. Asymptomatic labial adherence is best treated abnormal phimosis. Circumcision is recom- with: mended for pathologic phimosis, and topical (a) Separation in the operating room steroids and watchful waiting may aid in (b) EMLA cream and gentle blunt separation physiologic phimosis. in the office 3. Non-neonatal circumcision is most commonly (c) Estrogen cream performed because of: (d) Observation (a) Balanoposthitis Answer: (d) Labial adherence is described (b) Phimosis as the midline fusion of the labia minora. 30 Clinical Pediatric Urology Study Guide

Its peak incidence is between birth and 2 years, (b) Nuclear medicine renal scan and again around 6– 7 years of age. Treatment (c) Renal ultrasound is not routinely required and the adhesions (d) Hemoglobin electrophoresis should lyse spontaneously, but separation in Answer: (c) All of the above except the operating room, the office or the use of for nuclear medicine renal scan aid in estrogen cream have been tried when there are assessing for hypercalciuria. A calcium:creati- associated symptoms. nine ratio with help identify idiopathic calciu- ria, a renal ultrasound will rule out a mass 6. Gartner' s duct cysts in the newborn period in the kidney or stones, and hemoglobin are: electrophoresis will rule out sickle-cell trait or (a) Remnants of nonfused portions of the disease. Müllerian ducts (b) Remnants of the urogenital sinus 9. Glomerular source of hematuria is often: (c) Posterior (d) Rudimentary portions of the Wolffian (a) Bright red and normal morphology on duct microscopic exam (b) Cola-colored urine Answer: (d) Gartner' s duct is the distal (c) Absence of red blood cells (RBCs) on rudimentary portion of the Wolffian duct in microscopic exam girls. They usually regress but if persistent (d) Associated with flank pain they are found on the anterolateral walls of the vagina. They present as a perivaginal mass in Answer: (b) Glomerular source hematuria infants, and can be large enough to protrude is often cola or tea colored; bright red and fill the vagina. and normal morphology suggests the urinary tract. The absence of RBCs on 7. Yeast infections, other than diaper rash, in micro but the positive dipstick indicates 4- and 5-year-old girls are caused by: myoglobinuria or hemoglobinuria. Flank pain often insinuates obstructive causes either (a) Depressed cellular immunity from gross hemorrhage or clot. (b) Alkaline vaginal pH (c) Prophylactic antibiotics (d) A warm, moist environment 10. Idiopathic urethrorrhagia in prepubertal boys presents with all but: Answer: (a) Yeast infections are more com- mon in adult females than in prepubertal girls. (a) 30 % have dysuria The alkaline vaginal pH of childhood is hostile (b) Blood spotting between voiding to yeast. Most yeast vaginitis is caused by (c) Normal ultrasound (US) and VCUG Candida albicans, and is secondary to host (d) Urinalysis (UA) with RBCs in 57% of factors such as depressed cellular immunity or patients broad-spectrum antibiotics. (e) Resolution with antibiotics Answer: (e) Idiopathic urethrorrhagia is 8. Helpful studies in persistent microscopic seen in prepubertal boys who present with hematuria include: blood spotting in their underwear between (a) Calcium:creatinine ratio voiding. A review at the University of Utah Office pediatric urology 31 showed 30% had dysuria, 57% had UA with Reference RBCs, a normal US and VCUG. The symp- toms would range from a few weeks to 3 1. Oster J . Further fate of the foreskin: incidence of prepu- years, and resolved spontaneously without tial adhesions, phimosis and smegma among Danish antibiotics. schoolboys. Arch Dis Child 1968; 43 : 200 – 3 .

Principles of minimally 11 invasive surgery

Walid Farhat and Pasquale Casale

1. When performing laparoscopic pediatric uro- (b) D ecreased cardiac output logical procedure, necessary intraoperative (c) U reteral compression monitoring should include all of the following (d) D ecreased renal blood flow except: (e) N one of the above (a) R outine electrocardiogram Answer: (d) The renal effects occur second- (b) N oninvasive blood pressure ary to gas insufflation manifested by decreased (c) A rterial lines glomerular filtration rate and urine output. (d) T emperature Animal studies have shown that gas insuffla- (e) I nspired oxygen concentration tion causes renal vein compression inducing decreased renal blood flow; urine output Answer: (c) Although end-tidal CO may 2 drops and diminished creatinine clearance. not accurately reflect arterial CO2 tension, its use is helpful to plan appropriate ventilation 4. Currently, the following are contraindications strategies. On the other hand, in infants and to laparoscopy except for: children with respiratory pathology, capillary (a) C ardiopulmonary morbidity or arterial blood gas analysis might be indi- (b) U ncorrected coagulopathy cated to accurately measure CO tension. 2 (c) S epsis 2. Trendelenburg position during laparoscopy (d) S mall bowel obstruction will cause all of the following except: (e) M alignant kidney tumor

(a) I ncreased heart rate Answer: (d) Contraindications to laparos- (b) D ecreased vascular resistance copy in infants, children, and adolescents are (c) D ecreased mean arterial pressure the same as for any other surgical procedure, (d) D ecreased cardiac output except for evidence of limited lung reserve function, which may be considered as relative

Answer: (b) Patient positioning during lap- contraindications. If the patients is septic, in aroscopic surgery is capable of affecting and shock, or exhibits a coagulopathy, these should further potentiating the impact of gas insuffla- be corrected before surgery is contemplated. tion. For instance, a Trendelenburg position If surgery is deemed essential under these during laparoscopy will increase the heart rate circumstances, then it probably should be and vascular resistance while decreasing mean performed open. Furthermore, although lap- arterial pressure and cardiac output. aroscopy may play a role in the staging of malignant pediatric abdominal tumors such 3. Decreased urine output during laparoscopic as Wilms or neuroblastoma, its role in the surgery is secondary to: management of these tumors has yet to be (a) D ecreased renal vascular resistance defined. 34 Clinical Pediatric Urology Study Guide

5. The boundaries of the retroperitoneal space minor vascular injury is encountered during are the following except for: dissection, such as laceration to the gonadal vessels or adrenal vessels, hemostasis should (a) P osteriorly and laterally: the transversalis be maintained with traction anteriorly on the and quadratus lumborum muscles kidney, thus stretching those vessels, and con- (b) A nteriorly: the mobile posterior parietal tinually observing and maintaining normal peritoneum and its contents patient' s vital signs. On the other hand, to (c) S uperiorly: the diaphragm decrease the risk of injury while dissecting, in (d) I nferiorly: contiguous with the extraperi- addition to paramount care while dissecting, it toneal portions of the pelvis is mandatory that there is a thorough under- (e) N one of the above standing of the anatomy, equipment, and Answer: (a) An understanding of the retro- meticulous attention to details. peritoneal surgical anatomy is mandatory before embarking on retroperitoneoscopic 7. During laparoscopic procedures: surgery. Posteriorly and laterally: the paraspi- nal, psoas, and quadratus lumborum muscles, (a) P atient positioning in the pediatric patient which are anatomically, fixed structures. rarely affects the impact of gas insuffla- Anteriorly: the mobile posterior parietal peri- tion toneum and its contents. (b) T rendelenburg position during laparos- Superiorly: the diaphragm. copy will increase heart rate and vascular Inferiorly: contiguous with the extraperito- resistance neal portions of the pelvis. (c) R everse Trendelenburg position increases mean arterial pressure and cardiac output 6. When vascular injury occurs during laparo- (d) L eft flank positioning accentuates impaired scopic procedure: venous return more than the right flank position (a) I mmediate removal of the trocars and (e) N one of the above conversion to open exploration is under-

taken Answer: (d) Trendelenburg position during (b) I t may be unnoticed, thus may not need laparoscopy will increase heart rate and vascu- any intervention lar resistance while decreasing mean arterial (c) W hen minor such as tear of gonadal pressure and cardiac output, while the oppo- vessels, traction on the kidney posteriorly site effect is seen in a reverse Trendelenburg may help stop the bleeding position. Furthermore, flank positioning espe- (d) R outine inspection of the electrocautery cially with the kidney rest up and patient flexed instruments prior to any laparoscopic will accentuate impaired venous return and procedure is mandatory increase cardiac strain. Particularly, the left (e) N one of the above lateral decibitus position produces far more significant hemodynamic and respiratory

Answer: (d) The hallmark features of vascu- changes than the right flank position. lar injuries include either bloody return from the injury site or rapid deterioration of the 8. The advantages of retroperitoneal approach hemodynamic status of the patient. Once are true all except: extensive vascular injury is suspected, the trocar should be left in place and an open (a) T he technique mimics the open urological exploration should be performed. However, if surgical procedure Minimally invasive surgery 35

(b) I t provides direct approach to the organs have a small retroperitoneal space – and close of the genitourinary tract proximity between the abdominal wall and (c) C olon or splenic injuries do not occur in the major vessels, which are primarily retro- this approach peritoneal – the closed technique is not (d) F acilitates the view of the posterior sur- recommended. In addition, since there is no face of the kidney, hence rapid access to actual pre-existing retroperitoneal space, place- the renal hilum ment of Veress needle is not accurate and may (e) P revious transperitoneal surgery does not inadvertently be positioned deep in the retro- preclude retroperitoneoscopy peritoneum causing either injury to the great vessels or pneumoperitoneum, thereby com- Answer: (c) Retroperitoneoscopy provides plicating the procedure. direct approach to the organs of the genitouri- nary tract, and requires less dissection to the colon or the spleen to expose the kidneys and 10. Relative contraindications to a retroperitoneal adrenals. Furthermore, previous transperito- approach are all the following except: neal surgery does not preclude retroperitone- (a) P rior retroperitoneal scar (kidney surgery, oscopy. Finally, it facilitates the view of the kidney biopsy or pyeloplasty) posterior surface of the kidney and hence rapid (b) P revious infectious or inflammatory retro- access to the renal hilum. On the other hand, peritoneal process the disadvantages of retroperitoneal laparos- (c) C ardiopulmonary morbidity copy are that manipulation of instruments (d) U ncorrected coagulopathy may be initially difficult due to a restrictive (e) S epsis working space. Answer: (b) Contraindications to laparos- 9. When performing a retroperitoneal laparo- copy in infants, children, and adolescents are scopic procedure, all of the following are cor- the same as for any other surgical procedure, rect except: except for evidence of limited lung reserve function, which may be considered as relative (a) A ccess may be achieved by the closed contraindications. If the patient is septic, in technique shock, or exhibits a coagulopathy, these should (b) V eress needle may rarely cause injury to be corrected before surgery is contemplated. the great vessels If surgery is deemed essential under these (c) I nsertion of the primary trocar far medi- circumstances, then it probably should be ally may result in peritoneal entry or colon performed open. injury Absolute contraindications: cardiopulmonary (d) I nitial dissection of the kidney anteriorly morbidity; uncorrected coagulopathy; sepsis. is undertaken Relative contraindications: prior retroperito- (e) N one of the above neal scar (kidney surgery, kidney biopsy or Answer: (c) Access to the retroperitoneum pyeloplasty); previous infectious or inflamma- is preferably achieved by the open (Hasson) tory retroperitoneal process (xanthogranuloma- technique, which provides visual guidance to tous pyelonephritis) except for experienced the correct retroperitoneal space. As children surgeons in this approach.

Pediatric fluid management 12 Paul F Austin (Based on chapter written by James M Robertson)

1. The daily maintenance fluid requirement for (b) 1 700 ml/day a 9 kg infant is: (c) 1 800 ml/day (d) 1 900 ml/day (a) 6 00 ml/day (e) 2 000 ml/day (b) 7 00 ml/day (c) 8 00 ml/day Answer: (c) Daily maintenance fluid require- (d) 9 00 ml/day ments can be stratified by the following patient (e) 1 000 ml/day weights: 100 ml/kg/day for 0– 10 kg; 1000 ml/ day + 50 ml/kg/day per kg for 11– 20 kg; Answer: (d) Metabolism parallels the utili- 1500 ml/day + 20 ml/kg/day per kg > 20 kg. zation of water. Holliday and Segar found In the following example, the 35 kg infant that infants and children weighing 0– 10 kg would require 1500 ml/day + (15 kg × 20 ml/ require 100 cal/kg/day. Because 1 cal of energy kg/h) = 1800 ml/day. requires a net consumption of 1 ml of water, the answer is 9 kg × 100 ml/kg/day = 900 ml/ 4. The hourly maintenance fluid requirement of day.1 a 40 kg child is:

2. The hourly maintenance fluid requirement of (a) 7 5 ml/h

this same 9 kg infant is: (b) 8 0 ml/h (c) 8 5 ml/h

(a) 2 5 ml/h (d) 9 0 ml/h

(b) 3 0 ml/h (e) 9 5 ml/h (c) 3 3 ml/h (d) 3 6 ml/h Answer: (b) Hourly maintenance fluid requi- (e) 4 2 ml/h rements can be stratified by the following patient weights: 4 ml/kg/h for 0– 10 kg;

Answer: (d) Hourly maintenance fluid requi- 40 ml/h + 2 ml/kg/h per kg for 11– 20 kg; rements can be stratified by the following 60 ml/h + 1 ml/kg/h per kg > 20 kg. In the fol- patient weights: 4 ml/kg/h for 0– 10 kg; lowing example, the 40 kg child would require 40 ml/h + 2 ml/kg/h per kg for 11– 20 kg; 60ml + (20 kg × 1 ml/kg/h) = 80 ml/h. 60 ml/h + 1 ml/kg/h per kg > 20 kg. In the following example, the 9 kg infant would 5. Which of the following statements is not require 9 kg × 4 ml/kg/h = 36 ml/h. true? (a) P otassium is the major cation in the intra- 3. The daily maintenance fluid requirement for cellular space a 35 kg child is: (b) S odium is the major cation in the extra- (a) 1 600 ml/day cellular space 38 Clinical Pediatric Urology Study Guide

(c) M ajor losses of sodium, potassium, and (c) I ntravascular volume is low and increases chloride occur via the kidney with age (d) U rine concentrating ability is impaired at (d) I ntracellular fluid is high and decreases birth with age (e) P remature infants have low renal sodium (e) I ntracellular fluid includes the interstitial loss volume and transcellular fluid Answer: (e) Premature infants, in particular, Answer: (b) Total body water (TBW) is have high intestinal and renal sodium losses. highest in the preterm infant at 80% body Sodium reabsorption is poor and intrarenal weight. By term, TBW has decreased to gradients of NaCl are not yet developed. 70 – 75% and continues to fall during the Urine-concentrating ability is impaired, and immediate postnatal period to 60– 65 % during salt and water losses are great. the latter part of the first year of life. The extracellular fluid compartment comprises the 6. The electrolyte and glucose requirements for bulk of the TBW and parallels this decrease by

a 9 kg infant are: percent body weight with maturity: 60% body + % (a) 2 7 mEq Na /day, 18 mEq K + /day, 36 g weight in the premature infant; 45 in the % glucose/day term infant; and 30 in the adolescent. (b) 1 8 mEq Na + /day, 9 mEq K +/day, 27 g Intravascular volume (ml/kg) is highest in glucose/day premature infants and diminishes with age. (c) 2 7 mEq Na + /day, 9 mEq K + /day, 27 g Intracellular fluid is the only fluid compart- glucose/day ment which increases with age based on (d) 1 8 mEq Na + /day, 12 mEq K + /day, 36 g percent body weight. The interstitial volume glucose/day and transcellular fluid are divisions of the (e) 1 8 mEq Na + /day, 12 mEq K + /day, 27 g extracellular fluid compartment. glucose/day 8. Which of the following statements regarding

Answer: (a) Holliday and Seger observed intravenous fluids is false: that human milk contained the minimal elec- trolyte requirements for infants, and electrolyte (a) H ypotonic fluids are appropriate for main- requirements for infants were 3.0 mEq/100 cal/ tenance of TBW and electrolyte balance day for sodium and 2.0 mEq/100 cal/day for when given in amounts reflective of nor- both potassium and chloride. Glucose require- mal physiologic requirements ments are 25 g/100 cal. For a 9 kg infant, their (b) I sotonic fluids that most closely match daily caloric requirement is 100 cal/kg/day or serum and plasma osmolarity in children

9 kg × 100 cal/kg/day = 900 cal/day. Subsequen- are 270– 290 mOsm/kg tly a 9 kg infant would require supplementa- (c) H alf normal saline is a hypotonic fluid tion with 27 mEq Na + /day, 18 mEq K + /day, (d) D extrose is a transient osmole and once 36 g glucose/day.1 metabolized the osmolarity of the solu- tion diminishes 7. With regard to fluid compartments in the (e) I sotonic solutions are solutions with an newborn: osmolarity that is comparable to that of intracellular fluid but greater than that of (a) T otal body water is lowest in premature the extracellular fluid compartment infants (b) T he extracellular fluid is the largest com- Answer: (e) Ideally, there is no change in the partment of the total body water intracellular and extracellular osmolar gradient. Pediatric fluid management 39

Isotonic solutions are solutions with an osmo- 10. Which of the following statements regarding larity that is comparable to that of the extra- postoperative fluid therapy is false: cellular fluid as well as the intracellular fluid (a) T he allowable blood loss may be calcu- compartment. lated by knowing the patient' s weight, estimated blood volume, initial hemoglo- 9. Which of the following statements regarding bin, and minimal allowable hemoglobin intraoperative fluid therapy is false: (b) H yperkalemia and hypercalcemia with transfusion are dependent on the rate of (a) I sotonic fluids are used intraoperatively at transfusion rates necessary to maintain intravascular (c) D ilution with saline is advised for slow volume and adequate perfusion infusions, since calcium in lactated Ring- (b) R elative to awake resting requirements, er ' s solution may promote microaggregate there is an increased need for sodium formation in the intravenous line replacement intraoperatively due to excess (d) F luid losses should be replaced with iso- free water retention from the increased tonic fluids antidiuretic hormone (ADH) that is stim- (e) I ntravenous fluid therapy in the post- ulated from the stress of surgery operative period is an extension of intraop- (c) L actated Ringer' s solution is the most erative therapy, and changes in fluid and common isotonic fluid which is used for electrolyte homeostasis are still under the ongoing replacement of intraoperative influence of surgical or traumatic stress fluid losses (d) 5 % dextrose solution is acceptable for Answer: (b) Sodium citrate is added to fluid replacement intraoperatively packed red blood cells (PRBCs) and fresh (e) B lood loss may be replaced in a one-to- frozen plasma to chelate calcium and prevent one ratio with colloid or three- to fourfold clotting. Citrate administered with these blood with crystalloid pro ducts may therefore lead to hypocalcemia. PRBCs contain a high level of potassium

Answer: (d) The administration of dextrose which may increase with storage and therefore to pediatric patients under anesthesia is can lead to hyperkalemia. debatable and although intraoperative stress increases blood sugar, the risk of preoperative hypoglycemia due to fasting is estimated at Reference 1 – 2 % . The use of 5 % dextrose solution intra- operatively has been associated with hypergly- 1. Holliday MA , Segar WE . The maintenance need for cemia and should not be used for fluid water in parenteral fluid therapy. Pediatrics 1957 ; 19 : replacement. 823 – 32 .

Embryology of adrenal, 13–14 kidney, ureteral development, renal vasculature, and gonads

Steven E Lerman , Irene M McAleer , and George W Kaplan

1. There are three sets of kidneys that develop in (d) Early obstruction of tubules from blockage mammals during embryogenesis. Which of of the developing ureteral bud these primitive kidneys do not function Answer: (c) The cysts in ARPKD are due to during development? enlargement of the cystic ducts. Also associated (a) Pronephros is periportal hepatic fibrosis. The cysts in (b) Mesonephros ADPKD are due to abnormal branching of the (c) Metanephric mass cystic duct and formation of cysts in other parts (d) All of the primitive kidneys function of the renal tubules (making (a) incorrect). at some time, however brief, during This disorder is much more common than mammalian development ARPKD. The early obstruction of the renal tubules from faulty development or blockage of

Answer: (a) The pronephros (pronephroi – the ureteral bud is thought to be responsible plural) develops during the first 21 days for the findings in multicystic dysplastic kid- of development and does not function in neys (MCDK) (making (d) incorrect). None of mammals but is analogous to the kidney in these disorders is due to abnormal formation primitive fish. The other primitive developing of the distal convoluted tubule (making (b) kidneys do function during development; the incorrect). (Ch 19, page 289) pronephric ducts are necessary for formation of the mesonephros. (Ch 19, page 285) 3. Adrenal cortical development in human embryogenesis is noted by the 8th week of 2. Autosomic recessive polycystic kidney gestation and is comprised of: disease (ARPKD) is a rare inherited disorder (a) Fetal zone, zona glomerulosa, and zona with the abnormal renal configuration fasciculata due to: (b) Primitive zona glomerulosa, fasciculata, (a) Abnormal branching of the cystic duct and reticularis and cyst formation in other parts of the (c) Fetal zone and adrenal cortical cells renal tubules (d) Primitive neural crest cells interspersed (b) Abnormal formation of the distal convo- with primitive adrenocortical cells luted tubule Answer: (c) The cortex develops around the (c) Enlargement of the cystic ducts 8th week of gestation and is comprised of the 42 Clinical Pediatric Urology Study Guide

rather large central fetal zone and the thin (c) Occur in about 35% of adult kidneys peripheral adrenal cortical cells that are the (d) Both (b) and (c) precursors to the zona glomerulosa, fasciculata Answer: (b) Accessory renal arteries are and reticularis. The zona glomerulosa and more common than the occurrence of fasiculata develop late in gestation while the accessory renal veins and are found in about zona reticularis will not be apparent until 25 % of adult kidneys (making (a), (c) and (d) after the 3rd year of life (making (a) and (b) incorrect). During renal ascent from the incorrect). The neural crest cells found in pelvis, the developing kidneys receive their the adrenal gland are in the medulla and vascular supply from those vessels that are are not interspersed during development closest to them in descent, starting with the in the cortex of the adrenal (making (d) common iliacs and ultimately arising from incorrect). (Ch 14, page 237) branches off the aorta as the kidneys stop at the adrenal gland area. If the arteries do not 4. The inferior vena cava forms at the level of the involute during the ascent of the kidney, there renal veins by: will be accessory or supernumerary renal (a) Merging the right and left subcardinal arteries (making (b) correct). Ureteral obstruc- veins tion at the renal pelvis can occur if an inferi- (b) Persistence of the right supracardinal orly placed accessory artery is present and vein crosses over the ureter in the inferior location. (c) Persistence of the right subcardinal (Ch 19, page 283) vein (d) Persistence of the left supracardinal vein 6. The posterior cardinal veins form dorsal to the developing kidneys in: Answer: (c) The inferior vena cava is comprised of the right subcardinal vein (a) The mesonephros phase making up the inferior vena cava from the (b) The pronephros phase level of the renal veins and cephalad. The (c) The metanephric phase inferior vena cava below the renal veins is com- (d) The nephrogenic cord phase prised of the right supra cardinal vein (making Answer: (a) The postcardinal veins or poste- (b) incorrect). The left supracardinal and the rior cardinal veins develop early in vasculature left subcardinal vein systems disappear almost formation to allow return of blood to the completely, except for the common stem of heart. They run dorsal to the mesonephros the left renal vein, left adrenal vein and the and are the venous supply to the mesonephros left gonadal vein, all comprised of the left (making (a) correct). Eventually the postcar- subcardinal venous system (making (d) dinal veins will disappear except for remnants incorrect). There is no merging of the right which will become the root of the azygos vein or left subcardinal systems to create the and the common iliac veins. The subcardinal inferior vena cava (making (a) incorrect). vein on the left will become the left renal vein (Ch 19, page 283) to the metanephric tissue and the subcardinal vein meeting the supracardinal vein on the 5. Accessory renal arteries are: right will be the right renal vein location (mak- (a) Less common than accessory renal veins ing (c) incorrect). The nephrogenic cord is the (b) Result from failure of involution of entire bulk of pronephros, mesonephros and precursor arteries metanephric pluripotential renal tissue, so is Embryology 43

correct but is not as specific as the mesoneph- appears that WT1 is necessary for the meta- ros (making (d) incorrect). The pronephros nephric mass to induce the ureteral bud to forms early in the 4th gestational week and form probably through regulation of GDNF involute before functioning and prior to devel- (glial-cell-line neurotrophic factor). (b) and opment of the blood supply of the developing (c) are incorrect as WT1 appears to be impor- mesonephric tissue (making (b) incorrect). tant in the development of the permanent kid- (Ch 19, page 283) ney and it is not by inducing the ureteral bud to join with the metanephric blastema. There 7. The ureteric bud, arising from a bend in the is no indication, at this time, that WT1 makes mesonephric duct, pushes into the meta- the kidney resistant to Wilms' tumor develop- nephric blastema, inducing the development ment during the prenatal time period (making of the permanent kidney in the: (a) incorrect). (Ch 19, page 288) (a) 5th gestational week (b) 8th gestational week 9. Horseshoe kidneys occur because there is (c) 10th gestational week fusion of the lower poles of the developing (d) 12th gestational week kidneys and: Answer: (a) Early development of the (a) The ascent is arrested by the superior permanent kidney occurs during the 5th mesenteric artery gestational week when the ureteric bud meets (b) Is fairly common occurring in 1 in up with the metanephric mass. By the 500 people 6th week the kidney has ascended above (c) Have no higher risk of Wilms' tumor the umbilical arteries and by the 8th week the development than the general population center of the kidney has reached its permanent (d) Both (b) and (c) are correct level just below the adrenal gland (making (b) Answer: (b) Horseshoe kidneys occur incorrect). The fetal kidney will start to secrete during the ascent of kidneys but where the urine by the 10th gestational week (making two developing metanephric masses come in (c) incorrect). From the 6th to 20th gesta- contact with the lower poles later in develop- tional weeks, the major calices subdivide ment. Horseshoe kidneys are fairly common into the 12 generations of caliceal formation occurring in 1 in 500 people and also have a (making (d) incorrect). (Ch 19, page 287) 2 – 8 times higher risk of developing Wilms' tumors than the general population (making 8. During kidney development, WT1, a Wilms' (c) and (d) incorrect). The ascent is arrested tumor suppressor gene, is suggested to: by the inferior mesenteric artery and not (a) Make the kidney more resistant to antena- the superior mesenteric artery (making (a) tally developing Wilms' tumors incorrect). (Ch 19, page 289) (b) Not have any role in the development of the permanent kidney 10. The adrenal cortex develops as two distinct (c) Cause the ureteral bud to join the areas, with the permanent cortex as an outer metanephric blastema rind comprised at birth of: (d) Allow the metanephric mass to induce (a) Zona glomerulosa and zona fasciculata ureteral bud formation (b) Zona glomerulosa, zona fasciculata, and Answer: (d) Although molecular studies zona reticularis involved deal with laboratory animals, it (c) Zona fasciculata and zona reticularis 44 Clinical Pediatric Urology Study Guide

(d) Undifferentiated cortex tissue that will born before normal adrenal development takes differentiate in the first year of life place. (Ch 14, page 238) Answer: (a) The adrenal cortex develops as a large fetal zone and a small rind perma- nent adrenal cortex which will become the Extra questions eventual adult cortex. During late gestation, 1. The bladder develops from: this thin rind will differentiate into the hormone-producing areas of the zona glom- (a) The urogenital sinus erulosa, producing aldosterone, and zona fas- (b) Mesoderm of the anterior abdominal ciculata, producing glucocorticoid hormones. wall The zona reticularis, which will produce the (c) Endoderm sex or androgen hormones, does not develop (d) Ureteral buds until about the third year of life (making (b) (e) All of the above and (d) incorrect). (d) is incorrect as there is Answer: (e) also a thin rind of the permanent adult cortex tissue around the adrenal gland at birth. (Ch 14, page 237) 2. Place the following events in correct chrono- logical order: 11. As part of the fetal hypothalamic– pituitary– (a) Pronephros involutes adrenal axis, fetal adrenal glucocorticoids (b) Urorectal septum descends stimulate: (c) Proximal urethra forms (a) The fetal hypothalamus to release (d) Müllerian ducts form corticotrophin-releasing hormone (CRF) (e) Wolffian ducts form (b) The fetal pituitary to release adrenocorti- (i) (a), (b), (c), (d), (e) cotrophin (ACTH) (ii) (a), (b), (e), (c), (d) (c) The placenta to release placental CRF (iii) (a), (b), (c), (e), (d) (d) Both (b) and (c) are correct (iv) (a), (d), (c), (b), (e) Answer: (c) The fetal hypothalamic– pituitary – (v) (a), (e), (b), (c), (d) adrenal axis is necessary for the development, Answer: (v) differentiation and maturation of vital organ systems such as the lungs, brain and liver which are critical for immediate postnatal sur- 3. The fetal gonads begin to differentiate into vival of the infant. If increased, the fetal gluco- either a testis or an ovary at approximately corticoids will cause the fetal CRF and fetal which week of gestation? pituitary ACTH to be downregulated to (a) Week 3 decrease the fetal CRF and ACTH (making (b) Week 6 (a), (b) and (d) incorrect). The fetal adrenal (c) Week 9 glucocorticoids stimulate the placenta to (d) Week 12 release CRF causing the fetal adrenal to (e) Week 15 increase production of the glucocorticoids, thought to be so critical for the transition Answer: (b) from fetal to infant life, which may explain why extremely premature infants do not 4. Testicular descent from the abdominal cavity transition well to normal infant life if they are through the and into the Embryology 45

scrotum may be influenced by all of the 5. Remnant(s) of the Müllerian duct system following except: is (are): (a) Gubernacular guidance (a) Appendix testis (b) Increased intra-abdominal pressure (b) Epoopheron (c) Local hormonal factors (c) Gartner' s duct (d) The processus vaginalis (d) (b) and (e) (e) Completed development of the vas deferens (e) All of the above Answer: (e) Answer: (a)

Radiologic assessment of 15 the adrenal

Karoush Afshar , Douglas H Jamieson , and Andrew MacNeily

1. All of the following adrenal masses demon- (b) A drenal hemorrhage strate high signal intensity on T2-weighted (c) P heochromocytoma magnetic resonance imaging (MRI) except (d) G anglioneuroma one. Choose the exception: Answer: (b) The most commonly found (a) P heochromocytoma neonatal adrenal mass is an adrenal hemor- (b) A cute adrenal hemorrhage rhage. These are commonly detected on ante- (c) N euroblastoma natal maternal fetal ultrasound and are found (d) A drenal cyst more often on the right-hand side. The latter is thought to be secondary to compression Answer: (b) Fresh blood possesses low between the liver and kidney anteriorly, and signal intensity on T2-weighted MRI. After the spine posteriorly. liquefaction of the hematoma over several weeks, signal intensity increases. All the other lesions listed possess bright characteristics on 4. An antenatal ultrasound reveals a 2 cm right T2-weighted MRI. adrenal mass. Postnatally, this is confirmed to be a 2 cm hyperechoic and septated adrenal 2. In a case of antenatally diagnosed adrenal mass. Which of the following represents the mass, which of the following features on post- best next step in management? natal ultrasounds is against the diagnosis of (a) R ight adrenalectomy neuroblastoma: (b) M RI (a) S ize > 3 cm (c) C omputerized tomography (CT) scan (b) I ncrease in size on serial ultrasounds (d) S erial ultrasounds (c) D evelopment of internal echoes Answer: (d) Small hyperechoic adrenal (d) S olid homogeneity lesions detected antenatally, and confirmed Answer: (c) Development of internal echoes, postnatally, are most commonly adrenal calcifications, or shrinkage suggest resolving hemorrhages. These can be observed with serial hematoma. Size > 3 cm, solid homogeneous ultrasounds. Enlargement, or a primarily solid mass and increase in size imply neuroblastoma. appearance, would mandate more aggressive imaging and biochemical investigations. 3. Which of the following entities represents the most common neonatal adrenal mass? 5. A 3 cm solid hyperechoic adrenal lesion is (a) N euroblastoma found incidentally on an ultrasound performed 48 Clinical Pediatric Urology Study Guide

for abdominal pain in a 4-year-old child. MRI Answer: (b) Myelolipoma is a benign adre- reveals high signal intensity on T1-weighted nal neoplasm composed of fat and myeloid images and low signal intensity on T2-weighted elements. Typically fat is hyperechoic on ultra- images. What is the most likely diagnosis? sound imaging, exhibits high signal intensity on T1-weighted images and low signal inten- (a) P heochromocytoma sity on T2-weighted imaging. The other three (b) M yelolipoma malignancies exhibit varying degrees of low (c) A drenal cortical carcinoma intenstity on T1 and high intensity on T2 (d) N euroblastoma imaging. Adrenal tumors and functional 16 consequences

Julie Franc-Guimond , Anne-Marie Houle , and Aseem R Shukla

1. Cushing' s syndrome is: modulated by the renin-angiotensin system. Aldosterone release may be affected by potas- (a) C aused by pituitary hypersecretion of sium balance, ACTH, dopamine, and atrial adrenocorticotropic hormone (ACTH) natriuretic peptide. (b) A lways diagnosed in the presence of hyperparathyroidism, pancreatic-duodenal, and pituitary tumors 3. Primary hyperaldosteronism caused by an (c) A clinical disorder seen in cases of over- aldosterone-producing adenoma (APA) is best production of cortisol regardless of the managed by: cause (a) B ilateral adrenalectomy (d) O bserved with catecholamine-secreting (b) S pironolactone administration tumors (c) P otassium supplementation (e) O bserved with several genetic syndromes (d) S alt restriction Answer: (c) Cushing' s syndrome is a clinical (e) U nilateral adrenalectomy disorder seen in cases of overproduction of Answer: (e) An adrenal mass, especially in cortisol, as opposed to Cushing' s disease which an individual with hypokalemia and hyperten- is specifically caused by the pituitary hyperse- sion, raises the suspicion of an APA. Comput- cretion of ACTH. The increased cortisol pro- erized tomography (CT) scanning or magnetic duction leads to protein catabolism, subsequent resonance imaging (MRI), and adrenal vein increased glucose production, and typical sampling of aldosterone and cortisol when symptoms of cortisol excess. required, confirms an APA that is best man- aged by a unilateral adrenalectomy. 2. Aldosterone secretion is modulated by: (a) P lasma estradiol and/or estrone 4. In cases of primary hyperaldosteronism (b) P otassium balance, dopamine, and atrial with inconclusive data or equivocal radio- natriuretic peptide graphic features, ______is recommended (c) A low plasma corticotropin-releasing hor- to differentiate between an adenoma and mone (CRH) level, associated with ele- hyperplasia. vated plasma cortisol concentration (a) A therapeutic trial with daily administra- (d) T he secretion of catecholamines and vaso- tion of amiloride active intestinal polypeptides (b) M easurement of plasma adrenal andro- (e) A ndrogenic hormones gens and testosterone Answer: (b) Aldosterone production from (c) P harmacological blockade (alpha and beta) the zona glomerulosa of the adrenal cortex is of catecholamine effects and synthesis 50 Clinical Pediatric Urology Study Guide

(d) A drenal vein sampling of aldosterone and (d) H yperparathyroidism, pancreatic-duodenal, cortisol and pituitary tumors (e) M easurements of elevated plasma estra- (e) I slet cell carcinoma of the pancreas, neu- diol and/or estrone roblastoma or hemangiopericytoma Answer: (d) When a dexamethasone sup- Answer: (c) Neonatal macrosomia, macro- pression test and CT or MRI sampling are all glossia and omphalocele comprise the Beck- equivocal, adrenal vein sampling of aldoster- with-Wiedemann syndrome, which occurs due one and cortisol is recommended to differen- to an allelic loss of 11q15. The syndrome is tiate between an adenoma and hyperplasia. associated with adrenocortical tumors along Amiloride or spironolactone are appropriate with multiple endocrine neoplasia (MEN) as medical management in cases of bilateral Type 1 (associated with hyperparathyroidism, adrenal hyperplasia. Adrenal androgens and pancreatic-duodenal and pituitary tumors). testosterone are not measured in this case since aldosterone is the main mineralocorti- 7. Familial pheochromocytomas can occur with coid secreted by the adrenal gland. or without an association to known syndromes including: 5. Which conditions lead to reduced “ effective ” (a) M EN 1 and the Beckwith-Wiedemann circulating blood volume? syndrome (a) C ongestive heart failure (CHF), hepatic (b) C arney' s complex and congenital adrenal cirrhosis, nephrotic syndrome hyperplasia (b) R enovascular disorders, coarctation of the (c) L i-Fraumeni and McCune-Albright syn- aorta, renin-secreting tumors dromes (c) G itelman' s syndrome, Bartter' s syndrome (d) M EN 2, von Hippel-Lindau and neuro- (d) D iuretic use, pseudohypoaldosteronism fibromatosis type 1 Type I (e) P olycystic ovarian syndrome (e) E motional stress, physical activity, eating, Answer: (d) Familial pheochromocytomas fever can occur with or without the associations Answer: (a) States of mineralocorticoid defi- described, and are diagnosed at a younger age cit lead to reduced circulating blood volume as and tend to have bilateral or multifocal lesions is seen in CHF, hepatic cirrhosis, and nephritic that are usually benign. syndrome. Renovascular disorders and coarc- tation of the aorta, for example, lead to the 8. Pheochromocytomas are: opposite state of hypertension. (a) F unctioning adrenocortical tumors which exhibit elevated levels of several plasma 6. Beckwith-Wiedemann syndrome may include and urinary steroids the following findings: (b) T ransitional tumors of sympathetic cell (a) O ligo-amenorrhea, hirsutism, acne, and origin which contain elements of both excessive muscle mass malignant neuroblastoma and benign (b) T emporal balding, increased libido, and ganglioneuroma clitoromegaly (c) C atecholamine-secreting tumors (c) N eonatal macrosomia, macroglossia, and (d) A ldosterone-producing adrenocortical omphalocele carcinomas Adrenal tumors and functional consequences 51

(e) C omposed of gangliocytes and mature tumors of the sympathetic nervous system. stroma The adrenal glands are the most common site for this tumor and ganglioneuroblastomas are Answer: (c) Pheochromocytomas are cate- transitional tumors of sympathetic cell origin. cholamine-secreting tumors of which 90% originate from the adrenal medulla and 10% 10. Fetal neuroblastoma: arise from extra-adrenal chromaffin tissues (paragangliomas). The tumor is of neuroecto- (a) U sually presents with cerebral and, less dermal origin, and the diagnosis is based commonly, renal metastases on measurements of catecholamine and its (b) H as been discovered as early as 9 weeks metabolic products in plasma or urine. (c) H as a very poor prognosis (d) M ay secrete vanillylmandelic acid (VMA) and homovanillic acid (HVA) 9. Ganglioneuroblastomas are: (e) I s rarely of adrenal origin (a) F unctioning adrenocortical tumors which Answer: (d) Since 90% of neuroblastomas exhibit elevated levels of several plasma secrete VMA and HVA, these urinary cate- and urinary steroids cholamines may be screened to detect the (b) T ransitional tumors of sympathetic cell presence of the neoplasm. Fetal neuroblasto- origin which contain elements of both mas may be discovered as early as 19 weeks malignant neuroblastoma and benign and have an excellent prognosis (> 90 % ). These ganglioneuroma are almost always of adrenal origin and a con- (c) C atecholamine-secreting tumors servative approach allows involution of these (d) A ldosterone-producing adrenocortical car- tumors. Newborn screening seems to detect cinomas the infant neuroblastomas which often remain (e) C omposed of gangliocytes and mature occult, regress or mature, and screening has stroma not been shown to decrease the incidence of Answer: (b) Ganglioneuromas, ganglioneu- advanced stage disease in older children nor roblastoma and neuroblastoma are neuroblastic affect survival.

Surgery of the adrenal 17 CD Anthony Herndon (Based on chapter written by Stephen Boorjian, Michael Schwartz, and Dix P Poppas)

1. Advantages to laparoscopic versus open (b) Nonadvantageous for those with a history adrenalectomy include all but: of advanced trauma (c) More direct access to the adrenal gland (a) Improved cosmesis (d) May avoid potential intestinal injuries (b) Shorter hospital stay (e) Larger working space (c) Cost (d) Decreased need for postoperative pain Answer: (e) The main disadvantage to medication retroperitoneal access is the relatively small (e) Quicker return to activity working space when compared to intraperito- neal access. Answer: (c) Laparoscopy provides an im- provement in cosmesis and postoperative pain management with shorter returns to activity. 4. In terms of left renal vein anatomic relation- However, this comes at an increased cost. ships from medial to lateral: (a) Lumbar vein, gonadal vein, adrenal vein 2. The fundamental principle for laparoscopic (b) Adrenal vein, lumbar vein, gonadal vein adrenalectomy is: (c) Gonadal vein, lumbar vein, adrenal vein (a) Always mobilize the duodenum when (d) Adrenal vein, gonadal vein, lumbar vein approaching the right adrenal gland (e) Lumbar vein, adrenal vein, gonadal vein (b) Secure adrenal gland with noncrushing Answer: (e) From medial to lateral draining grasper when dissecting gland into the renal vein, the lumbar, adrenal and (c) Retroperitoneal access should be attempted then gonadal veins are encountered. prior to transperitoneal dissection (d) An attempt should be made to dissect the patient away from the adrenal gland 5. During transperitoneal laparoscopic adrenal- ectomy which attachments should be taken

Answer: (d) Every attempt should be made down last: not to manipulate the adrenal gland during dissection. The surgical dictum “ dissect the (a) Anterior patient away from the adrenal gland” should (b) Medial be adhered to if at all possible. (c) Lateral (d) Posterior 3. Advantages in favor of retroperitoneal versus (e) Inferior intraperitoneal access include all but: Answer: (c) The lateral attachments serve as (a) Favored in those patients with prior a retractor that allows dissection of the more surgery medial vascular anatomy. 54 Clinical Pediatric Urology Study Guide

6. Intraoperative instability may be reduced (e) Maintaining single agent calcium channel during an adrenalectomy for a pheochromo- blockade cytoma by performing all but: Answer: (d) Patients with a history of pheo- (a) Aggressive intraoperative hydration chromocytoma should be maintained on either (b) Minimal adrenal gland manipulation calcium-channel blockers or alpha blockers (c) Early control of adrenal vein prior to surgery in addition to the above (d) Maintaining single agent beta-blockade recommendations. Basic science of the kidney 18 John C Thomas and John C Pope IV

1. Renal agenesis can result from improper comparison of postobstructive findings signaling between which receptor and ligand: difficult (c) A n artificially created obstruction is the (a) P ax-2 and glial cell line-derived neuro- perfect model to study obstruction which trophic factor (GDNF) occurs naturally (b) C -ret and GDNF (d) K nock-out animals have not helped (c) W nt 4 and bone morphogenic protein researchers in defining molecular changes (BMP) 4 which occur after artificially induced (d) T ransforming growth factor (TGF)-beta obstruction and Eya-1 Answer: (b) Animal models of obstruction Answer: (b) C-ret is a tyrosine kinase recep- are difficult to correlate with obstruction tor located on the tips of the developing ure- occurring in humans as nephrogenesis is com- teral buds that must bind with secreted GDNF pleted at different time points. The closest from the developing mesenchyme in order to current model of obstructive uropathy is the assure normal renal development. neonatal rat, as only 10% of its nephrons are formed at birth, while the rest develop during 2. Which one of the following is not considered the first week of life. a current potential biomarker for congenital obstructive uropathy: 4. TGF-beta contributes to fibrosis by: (a) A natomic detail from imaging studies (a) U pregulating extracellular matrix proteins (b) R adionuclide scans to determine function and recruiting fibroblasts (c) M olecular markers in the urine (b) C ombining with TGF-alpha to cause (d) M icroscopic analysis of biopsy specimens cross-linking of collagen Answer: (c) Currently, molecular markers (c) S uppressing the accumulation of fibro- are not used as a biomarker; however, they nectin, collagen, and proteoglycans hold promise in the future to help determine (d) D ownregulating matrix metalloprotein who may need surgical intervention and who inhibitors can be observed. (e) I nhibiting Smad 3 activation Answer: (a) TGF-beta upregulates extracel- 3. One difficulty in using animal models to study lular matrix (ECM) proteins and activates the congenital obstruction in humans is that: SMAD pathway, including Smad 3, to cause (a) R odents and humans have a similar period fibrosis. It increases the accumulation of of nephrogenesis fibronectin, collagen and proteoglycans, and (b) C ompletion of nephrogenesis in animals upregulated matrix metalloproteinase (MMP) and humans is different, which makes inhibitors. 56 Clinical Pediatric Urology Study Guide

5. The RAS is an important contributor to renal promotes fibrosis. The RAS promotes apop- fibrosis by which mechanism: tosis by increasing ROS. (a) I ncreased renin production (b) D ecreased vascular resistance in the affer- 6. All of the following are potential treatment ent arteriole options except: (c) I ncreasing production of TGF-beta by activating plasminogen activator inhibitor (a) T GF-beta inhibition (PAI)-1 (b) A II receptor blockers (d) S uppressing apoptosis by decreasing reac- (c) N itric oxide activation tive oxygen species (ROS) (d) C aspase inhibition (e) C auses upregulation of MMP inhibitors (e) A ll of the above directly Answer: (e) All of the listed answers have Answer: (c) Although important in normal the potential to limit fibrosis and further apop- renal development, the RAS can cause or exa- tosis following obstruction. All of these pro- cerbate fibrosis by increasing human apolipo- cesses are related to one another so inhibiting protein (apo) AII. AII increases TGF-beta by one process may have additional benefit by activating PAI-1, which inhibits the break- preventing downstream activation of another down of plasminogen to plasmin and thereby signaling pathway. Anomalies of the kidney 20 Michael Nguyen (Based on chapter written by Michael Ritchey and Susan John)

1. The kidneys ascend to their final level by the (c) Crossed renal ectopia with fusion is the end of which week of fetal life? most common type of crossed renal ectopia (a) 5th week (d) There is a slight male predominance (b) 8th week (e) Crossing from left to right is more (c) 10th week common than right to left (d) 13th week (e) 15th week Answer: (a) Crossed renal ectopia is the second most common fusion anomaly behind Answer: (b) The kidneys reach their final a horseshoe kidney. The remaining answers level by the end of the 8th week of fetal life. are all true regarding crossed renal ectopia. Axial rotation medially also occurs during the 7th and 8th week. 4. Associated anomalies in children with horseshoe kidneys include: 2. Failure of induction of the metanephric blastema by the ureteral bud leading to renal (a) Vesicoureteral reflux agenesis could result from: (b) Multicystic dysplasia (c) Turner's syndrome (a) Failure of the ureteral bud or Wolffian (d) (a) and (b) duct to develop (e) (a), (b) and (c) (b) Failure of the ureteral bud to reach the blastema Answer: (e) All have been found to be % (c) Abnormality of the metanephric blastema associated with a horseshoe kidney. A 7 (d) (a) and (c) incidence of horseshoe kidney was noted in (e) All of the above patients with Turner's syndrome.

Answer: (e) Abnormalities in any of the 5. Which of the following are true regarding interactions between the ureteral bud and anomalies of renal ascent? metanephric blastema can result in renal (a) During ascent, the kidney receives its agenesis. blood supply from the middle sacral artery, iliac artery, and the aorta 3. Which of the following is false concerning (b) The most common location of an ectopic crossed renal ectopia? kidney is in the pelvis opposite the sacrum (a) It is the most common fusion anomaly or below the aortic bifurcation (b) The ectopic kidney crosses the midline to (c) Voiding cystourethrogram (VCUG) is lie on the opposite side from its ureteral not recommended in all children with a insertion diagnosis of pelvic kidney 58 Clinical Pediatric Urology Study Guide

(d) (a) and (b) (d) All of the above (e) (a), (b) and (c) (e) None of the above Answer: (d) The most common urologic Answer: (d) A VCUG is recommended in problem in pelvic kidneys is a UPJ obstruction all children to exclude the diagnosis of vesi- due to a high ureteral insertion, or maybe coureteral reflux, which is frequently associ- secondary to anomalous vessels that obstruct ated with an ectopic kidney. the UPJ.

8. Which of the following is true regarding a 6. How often is the ipsilateral ureter absent in horseshoe kidney? unilateral renal agenesis? (a) The presence of a horseshoe kidney does (a) 8–15% adversely affect survival (b) 25–33% (b) UPJ obstruction is the second most (c) 50–87% common cause of hydronephrosis in (d) 90% horseshoe kidneys (e) None of the above (c) Vesicoureteral reflux is not found in the horseshoe kidney Answer: (c) The ipsilateral ureter is absent (d) Wilms’ tumor is the second most com- % in 50–87 of cases of unilateral renal agenesis mon tumor found in horseshoe kidneys and only partially developed in others. (e) None of the above Answer: (d) In a review of National Wilms’ 7. All of the following are urologic problems Tumor Study patients, there was a sevenfold found in pelvic kidneys except: increased risk of a Wilms’ tumor developing in patients with a horseshoe kidney. UPJ obstruc- (a) Renal stones tion is the most common cause of hydroneph- (b) Ureteropelvic junction (UPJ) obstruction rosis in the horseshoe kidney, occurring in (c) Hydronephrosis 30% of patients. Fetal and neonatal renal function 21 Paul F Austin (Based on chapter written by Billy S Arant Jr)

1. In the development of the fetal kidney: (e) Placental circulation ceases after the second trimester (a) New glomeruli are formed until 28 weeks gestation Answer: (d) The other answers are incorrect (b) The human metanephros is formed about because systemic vascular resistance is kept 2 weeks after conception low in the fetus except in the pulmonary and (c) Each kidney will contain one billion renal circulations. These regional differences nephrons of vascular resistances are in part regulated by (d) All glomeruli are contained within the vasoactive substances (e.g. angiotensin, pros- cortex taglandins and nitric oxide) which are expressed (e) The essential role of the fetal kidney is to in the developing kidney. Placental circulation contribute the majority of the amniotic continues throughout pregnancy and is inter- fluid beginning in the first trimester rupted at birth. Answer: (d) The other answers are incorrect 3. Renal blood flow in the newborn infant: because: (a) Occurs primarily in the glomeruli residing • New glomeruli are formed until in the inner cortex 34–36 weeks gestation (b) Is measured by the clearance of inulin • The human metanephros is formed at (c) Increases postnatally as renal vascular about the 5th week after conception resistance decreases • Each kidney contains approximately one (d) Is 750 ml/min/1.73 m2 million nephrons (e) Is unaffected by vasoactive factors • The essential role of the fetal kidney is to contribute the majority of the amniotic Answer: (c) Renal blood flow initially is fluid after the first trimester distributed to the glomeruli residing in the outer cortex and becomes evenly distributed to all cortical glomeruli with maturity. This 2. Which of the following best characterizes the redistribution occurs secondary to vasoactive hemodynamics of the fetus? factors (e.g. angiotensin II) which are also (a) Systemic vascular resistance is elevated involved in the diminished renal vascular resis- (b) Cardiac output in the fetus and preterm tance that follows the increase in renal blood infant is lower than adults flow with birth. Para-aminohippurate (PAH) (c) Vasoactive substances are expressed in clearance is the technique for measuring renal low levels in the developing kidney blood flow in infants. Renal blood flow in the (d) Pulmonary and renal circulation vascular newborn infant is 290 ml/min/1.73 m2 and resistance is elevated increases with maturity. 60 Clinical Pediatric Urology Study Guide

4. Which of the following statements is not area of 1.73 m2 in a child and compared to true? adult GFR. GFR can be measured directly by a clearance technique that requires a timed (a) Glomerular filtration rate (GFR) is low urine collection and one or more blood sam- during fetal development until 34 weeks ples using either endogenous creatinine or an gestation at which time it increases dra- exogenous marker such as inulin or iothali- matically mate infused intravenously. There are no nor- (b) GFR will increase within 1–2 weeks in a mal data published for infants regarding 28 week gestational age infant after birth radioisotope techniques to estimate GFR and (c) The average GFR in the normal adult nuclear medicine studies provide no actual is 125 ml/min, which is achieved when estimate of GFR in infants. The change in the average adult body surface area is GFR which occurs at 34–36 weeks gestation 1.73 m2 is mediated through changes in renal vascular (d) Developmental changes in GFR occur in resistance. every mammalian species (e) GFR increases as renal vascular resistance decreases 6. With regard to serum creatinine in the infant: Answer: (b) GFR is low during fetal deve- lopment until 34 weeks gestation at which (a) Serum creatinine concentration will be time it increases dramatically. This timeline is reduced by 50% during the first week unaltered in a premature infant. In this example of life in an infant born after 34 weeks of a 28 week premature infant, GFR will not gestation dramatically increase for 6 more weeks until it (b) Premature infants’ serum creatinine con- reaches 34 weeks corrected gestational age. centration will be higher than maternal values at birth 5. With respect to GFR: (c) Creatinine clearance is inversely related to (a) After 6 months of age, the absolute value GFR for GFR (ml/min) cannot be corrected (d) Normal serum creatinine concentration for a body surface area of 1.73 m2 in a is approximately 0.7 mg/dl in full-term child and compared to adult GFR infants by 1 month of age (b) PAH clearance is the technique for (e) Premature infants will attain a normal measuring GFR in infants serum creatinine concentration compar- (c) Nuclear medicine studies are good meth- able to full-term infants by 1 month ods of providing an actual estimate of of age GFR in infants Answer: (a) All newborns, regardless of pre- (d) Changes in newborn GFR occur inde- maturity, will have maternal values of serum pendent of changes in renal vascular resis- creatinine concentration during the first days tance of life. Creatinine clearance is directly related (e) A serial change in serum creatinine con- to GFR. The normal serum creatinine concen- centration is a clinically useful estimate of tration should be ≤0.4 mg/dl by the end of the GFR in neonates. first month of life in full-term infants but pre- Answer: (e) The absolute value for GFR term infants may not reach this level until (ml/min) can be corrected for a body surface 3 months of age. Fetal and neonatal renal function 61

7. Which of the following statements regarding Answer: (e) Each statement is a marker for urine volume is false: the end of postnatal diuresis.

(a) Premature infants usually void more fre- 9. Which of the following statements regarding quently and in larger volumes relative to tubular handling of sodium is false: body weight and GFR when compared with full-term infants (a) Classic teaching previously touted that (b) Premature infants have less extracellular premature infants could not conserve fluid to excrete following birth than do sodium and full-term infants could not full-term infants excrete sodium (c) A full-term newborn may not void in the (b) Renal handling of sodium is reflective of first 12 hours of life changes in effective arterial blood volume (d) Initially, the urine is less concentrated in a (c) The status of effective arterial blood vol- newborn ume overrides any signaling from body (e) Both premature and full-term infants fluid osmolarity are capable of concentrating their urine (d) The fractional excretion of sodium (FENa) to a specific gravity of >1.020 once extra- is <1% in the setting of diuretics, obstruc- cellular fluid volume has been reduced tion or fluid overload sufficiently to stimulate the baroreceptor- (e) An increase in the effective arterial blood mediated release of arginine vasopressin volume results in the renal excretion of from the hypothalamus sodium and water in the mature kidney % Answer: (b) Total body water (TBW) is Answer: (d) FENa will increase >1 with highest in the preterm infant at 80% body an increase in effective arterial blood volume, weight. By term, TBW has decreased to by tubular injury as with diuretics, obstruction 70–75% and continues to fall during the or renal disease, or by any maneuver which immediate postnatal period to 60–65% increases the fractional excretion of water such during the latter part of the first year of as fluid overload. life. The extracellular fluid compartment com- prises the bulk of the TBW and parallels this 10. Which of the following statements regarding decrease by percent body weight with matu- acid–base balance is false: rity: 60% body weight in the premature infant; 45% in the term infant; and 30% in the ado- (a) Unless demand becomes overwhelming, lescent. the neonatal kidney is capable of reab- sorbing bicarbonate filtered by the glom- 8. Markers for the end of the postnatal diuresis erulus and secreting hydrogen ions in a normal infant, regardless of maturity, are: (b) An increased effective arterial blood volume results in reabsorption of all sub- (a) A reduction in the frequency and volume stances presented to the distal tubule of urine excreted (c) Most of the acidemia measured in prema- (b) An increase in urinary osmolarity ture infants can be accounted for by respi- (c) A stabilization of the infant's body ratory acidosis weight (d) Most so-called renal tubular acidosis in (d) (a) and (c) infants is a complication of diarrheal disease (e) All of the above with the intestinal loss of bicarbonate 62 Clinical Pediatric Urology Study Guide

(e) Distal tubular injury from obstruction or Answer: (b) An increased effective arterial infection impairs the ability of that part of blood volume results in reabsorption of all the nephron to secrete hydrogen ions and substances presented to the proximal tubule – potassium, resulting in a metabolic acido- not just sodium and chloride, but also bicar- sis and hyperkalemia. bonate. Cystic kidney disease 22 Larry A Greenbaum

1. Which of the following cell structures appears (c) Autosomal recessive cystic kidney disease to be involved in cyst formation in a variety of (d) Autosomal dominant cystic kidney disease inherited cystic kidney diseases? (e) Medullary sponge kidney

(a) Mitochondria Answer: (c) The majority of patients with (b) Primary cilium autosomal recessive polycystic kidney disease (c) Lysosomes present during the first year of life, with a (d) Ribosomes significant percentage being diagnosed during (e) Nuclear pores the neonatal period. The other entities are Answer: (b) In a variety of cystic kidney typically diagnosed later in childhood or in diseases – including autosomal dominant adults. polycystic kidney disease, autosomal recessive polycystic kidney disease, and nephronoph- 4. A 14-year-old child is seen in your office thisis – there are mutations in genes encod- because of gross hematuria, which occurred ing proteins which are components of or during football practice. Among cystic kidney associated with the primary cilium of renal diseases, which of the following is the most tubular cells. likely diagnosis?

2. You palpate an enlarged liver in a 6-year-old (a) Nephronophthisis child with autosomal recessive polycystic (b) Medullary cystic kidney disease kidney disease. What is the likely etiology? (c) Autosomal recessive cystic kidney disease (a) Intrahepatic aneurysm (d) Autosomal dominant cystic kidney (b) Autoimmune hepatitis disease (c) Liver cysts (e) Multicystic dysplastic kidney (d) Liver displacement due to enlarged Answer: (d) Hematuria occurs in patients kidneys with autosomal dominant polycystic kidney (e) Hepatic fibrosis disease and the hematuria may be associated Answer: (e) Hepatic fibrosis occurs in with physical activity. Gross hematuria is patients with autosomal recessive kidney rarely reported in the other entities. disease. Clinical consequences of hepatic fibro- sis include cholangitis and manifestations of 5. You are seeing a patient with autosomal portal hypertension such as esophageal varices dominant polycystic kidney disease. Which and splenomegaly. of the following is not a possible extrarenal 3. Which of the following diseases is likely to manifestation? present clinically during the first year of life? (a) Ruptured intracranial aneurysm (a) Nephronophthisis (b) Hepatic cysts (b) Medullary cystic kidney disease (c) Tapetoretinal degeneration 64 Clinical Pediatric Urology Study Guide

(d) Inguinal hernia of the nose), limb deformities, and pulmonary (e) Mitral valve prolapse hypoplasia that is incompatible with life. Answer: (c) Mitral valve prolapse and ingui- 8. You are seeing a neonate with a unilateral nal hernias are more common in children multicystic dysplastic kidney (MCDK) that with autosomal dominant polycystic kidney was detected on prenatal ultrasound. The disease (ADPKD). Ruptured intracranial diagnosis was confirmed by a postnatal renal aneurysms and hepatic cysts are rare in ultrasound. You should tell the parents: children with ADPKD, but occur in adults. Tapetoretinal degeneration does not occur in (a) The child requires immediate nephrec- ADPKD. tomy of the MCDK to prevent malignant transformation 6. You are seeing a 9-year-old boy due to pri- (b) The child should have a nephrectomy of mary nocturnal enuresis. By history, he has the MCDK at about 1 year of age polydipsia and polyuria. He also craves salty (c) The child requires regular renal ultra- foods. What cystic kidney disease do you need sound follow-up to consider in your differential diagnosis? (d) No further follow-up is indicated since the contralateral kidney is normal (a) Nephronophthisis (e) The child may develop cysts in the con- (b) Medullary sponge kidney tralateral kidney (c) Autosomal recessive cystic kidney disease (d) Autosomal dominant cystic kidney disease Answer: (c) MCDK are typically asymp- (e) Multicystic dysplastic kidney tomatic and the majority completely involute and disappear during the first 6 years of life. Answer: (a) High urine output and salt Hence, routine nephrectomy is no longer rec- wasting are common in nephronophthisis. ommended. Ultrasound follow-up is currently This leads to polydipsia, and may cause recommended due to an extremely low risk of enuresis and salt craving. The high urine malignant transformation. output persists even when patients develop renal failure. 9. You are seeing a 1-year-old child with a uni- lateral MCDK. The child is otherwise healthy. 7. What is the typical outcome in a child The parents are now considering having a sec- with bilateral multicystic dysplastic kidney ond child and they want to know the risk of a disease? MCDK in subsequent children. What is the inheritance of MCDK? (a) Senior Loken syndrome (b) Potter syndrome (a) Autosomal recessive (c) Pomeranz syndrome (b) Autosomal dominant (d) Fanconi syndrome (c) X-linked recessive (e) Ask-Upmark syndrome (d) Genetic imprinting (e) None of the above Answer: (b) Children with bilateral multi- cystic dysplastic kidneys die at birth due to Answer: (e) Most cases of MCDK are spo- Potter syndrome, which is caused by severe radic, without a genetic etiology. There are a oligohydramnios. Manifestations of Potter few families described with putative autosomal syndrome include facial features (hypertelor- dominant inheritance and MCDK is increased ism, low-set ears, receding chin, and flattening in certain genetic syndromes. Cystic kidney disease 65

10. Which of the following diseases can be associ- renal manifestations. Angiomyolipomas are ated with cystic kidney disease? the most frequent complication, and they may cause bleeding and renal insufficiency in late (a) Tuberous sclerosis childhood and adulthood. Cysts are less com- (b) Neurofibromatosis mon and are often due to a chromosomal (c) Cystic fibrosis deletion which affects two adjacent genes: one (d) Gordon syndrome of the tuberous sclerosis genes and one of the (e) Sickle-cell disease autosomal dominant polycystic kidney disease Answer: (a) Tuberous sclerosis, one of the genes. neurocutaneous syndromes, has significant

Acute renal failure 23 Lawrence Copelovitch , Bernard S Kaplan , and Kevin EC Meyers

1. Acute renal failure (ARF) may be associated nephritis, and dehydration are characterized with which of the following? by oliguria.

(a) Anuria 3. Which of the following is associated with a (b) Oliguria high likelihood of recovery in ARF? (c) Polyuria (d) Normal urine output (a) Oliguria (e) All of the above (b) Multiple organ dysfunction (c) Requirement for dialysis Answer: (e) Acute renal failure, the sudden (d) Polyuria decrease in normal kidney function compro- (e) Prolonged renal insult mises normal renal regulation of fluid, electro- lyte, and acid–base homeostasis. ARF is a Answer: (d) Recovery from ARF is predi- reduction in glomerular filtration rate (GFR), cated on the amount of urine output and the which results in an increase in the creatinine underlying cause. Patients with nonoliguric or and blood urea nitrogen, and may be charac- high-output ARF have lower complication terized with anuria, oliguria, polyuria or rates and higher survival rates than those with normal urine output. anuria or oliguric ARF.

4. Which of the following is not an endogenous 2. Which of the following is usually associated toxin which can cause ATN? with nonoliguric ARF? (a) Myoglobin (a) The maintenance phase of ischemic- (b) Hemoglobin induced acute tubular necrosis (ATN) (c) Methemoglobin (b) Acute interstitial nephritis (d) Uric acid (c) Dehydration (d) Acute glomurelonephritis Answer: (c) Myoglobin or hemoglobin- (e) Cardiac tamponade induced ARF is more likely to develop in the presence of dehydration, metabolic acidosis Answer: (b) Children with acute interstitial or multiple organ failure as the pigment may nephritis or nephrotoxicity from medications precipitate in the tubules and cause tubular are more likely to have normal or increased injury. Uric acid may similarly precipitate in urine output ARF. The maintenance phase of tubules while methemoglobinemia does not ischemia-induced ATN, acute glomerulo- cause ATN. 68 Clinical Pediatric Urology Study Guide

5. Which of the following is not a common met- much on the fluid status of the patient abolic complication of ARF? and whether the ARF is oliguric, anuric or polyuric. (a) Hypernatremia (b) Hypocalcemia 8. Which of the following is incorrect with (c) Hyperkalemia regards to urine studies? (d) Hyperphosphatemia (e) Metabolic acidosis (a) A high urine specific gravity suggests pre- renal ARF Answer: (a) Serum electrolytes are carefully (b) Red blood cell casts strongly suggest a monitored in a patient with ARF as they are glomerular lesion important guides to predicting morbidity and (c) Significant proteinuria (>3+) is highly mortality. Hyponatremia may occur due to suggestive of ATN fluid overload, but hypernatremia is not com- (d) A low fractional excretion of sodium monly seen with ARF. The remainder of the (<1%) suggests prerenal ARF choices above are also common metabolic (e) A positive dipstick for blood with no red complications. blood cells on microscopic examination is suggestive of myoglobinuria 6. Which of the following is an indication for initiating dialysis? Answer: (c) A dipstick test revealing signifi- cant proteinuria indicates intrinsic ARF with (a) Volume overload unresponsive to conser- glomerular damage and not ATN. Prerenal vative management ARF, ATN, and postrenal ARF are indicated (b) Hyperkalemia unresponsive to medical by less proteinuria. therapy (c) Uremia 9. Which of the following agents has been (d) Prolonged malnutrition associated with toxin-mediated ATN? (e) All of the above (a) Amphotericin Answer: (e) All of the above, as well as the (b) Aminoglycosides patient's overall condition, must be evaluated (c) Acyclovir before dialysis is considered. (d) Ethylene glycol (e) All of the above 7. Which is not a principle in the management Answer: (e) Renal injury from toxins usually of ARF? presents with a decrease in urine concentrat- (a) Maintain renal perfusion ing capacity, and is usually transient, mild and (b) Administer large volumes of isotonic nonoliguric. Risk factors include extremes of crystalloid regardless of volume status age, hypovolemia, and preexisting renal insuf- (c) Avoid nephrotoxic agents ficiency. (d) Maintain electrolyte balance (e) Provide nutritional support 10. Pyelonephritis falls into which category of intrinsic renal disease? Answer: (b) Renal hypoperfusion is a predis- posing factor to developing ARF, so establi- (a) Interstitial shing a fluid balance is critical in management. (b) Glomerular The volume of resuscitation depends very (c) Vascular Acute renal failure 69

(d) Tubular result in interstitial injury. Fever, rash, arthral- (e) None of the above gias and flank pain are typical symptoms. Urine output is usually increased or normal Answer: (a) Pyelonephritis, systemic diseases, and interstitial injury is the most frequent tumor infiltrates or genetic conditions most cause of nonoliguric ARF.

Renal transplantation 24 Nafisa Dharamsi , Curtis Sheldon , and Jens Gobel

1. If a short renal vein is encountered during a maximizing ureteral vascularity and to prevent renal transplant then: clot colic. (a) It can be extended by using adjacent cavil flaps 3. Advantages of the extravesical approach to (b) A small patch of cava or aorta can be used ureteral reimplant include all the following to provide the extension except: (c) The recipient venous tree is mobilized (a) Shortening operating room time extensively (b) Low incidence of anastomotic obstruction (d) Vascular reconstruction is completed with (c) Reduction in postoperative hematuria techniques of dividing and spatulating the (d) Reduction in urinary extravasation renal vein (e) Decreased risk of vesicoureteral reflux Answer: (c) In most cases, if a short renal Answer: (d) Options for the ureterovesical vein is encountered, additional length can be reimplantation include transvesically or gained by mobilizing the recipient venous extravesically. Advantages of the extravesical tree. If this is not sufficient to create a tension- approach include all of the above except free anastomosis, the renal vein can be extended for reducing urinary extravasation post- by tubularizing adjacent caval flaps. operatively.

2. Ureteral stents are utilized in transplants for 4. The best solution for preservation of all all the following reasons except: abdominal organs is: (a) To prevent ureteral obstruction from (a) EuroCollins anastomotic edema (b) Ringer's lactated solution with heparin (b) To decrease potential risk of extravasation (c) Sach's solution (c) To maximize ureteral vascularity (d) Collins 2 (d) To prevent clot colic postoperatively (e) University of Wisconsin (UW) solution (e) To increase intrarenal reflux postopera- tively Answer: (e) University of Wisconsin solution favors graft function and survival over Answer: (e) All are true and stents do EuroCollins. increase reflux into the kidney, however, they are not utilized for this purpose during renal 5. The standard method of reconstructing the transplantation. The stents are utilized to pre- ureter during the renal transplant is: vent ureteral obstruction from anastomotic edema, to decrease extravasation from the (a) Ureteropyelostomy anastomosis, to stabilize the ureter thereby (b) Ureteroureterostomy 72 Clinical Pediatric Urology Study Guide

(c) Cutaneous ureterostomy 8. Common causes of end-stage renal disease (d) Ureteroneocystotomy (ESRD) in children are all of the below (e) Transureteroureterostomy except: Answer: (d) The ureter is most commonly (a) Focal segmental glomerulosclerosis reimplanted into the bladder – ureteroneocyst- (b) Obstructive uropathy, e.g. from posterior otomy – using a transvesical or extravesical urethral valves approach. A less frequently used option is (b) (c) Renal dysplasia and, rarely, (a), (c) and (e). (d) Diabetic nephropathy (e) Reflux nephropathy 6. Etiology of urolithiasis posttransplantation Answer: (d) The common causes of ESRD includes all the following except: in children are different from adults where (a) Retained suture material diabetes and hypertension play a large role. (b) High citrate and low phosphate excretion The common causes in children are the con- in the urine genital obstructive and dysplastic conditions. (c) Decreased urine output (d) Alkaline urine 9. Kidney transplantation is contraindicated in (e) Increased frequency of urinary tract the presence of: infections and urine stasis (a) Malignancy Answer: (b) Urolithiasis is seen after 5% (b) Chronic infection, e.g. tuberculosis of pediatric renal transplants. The etiology of (c) Significant noncompliance the stones is multifactorial; however, retained (d) Ongoing substance abuse suture material, decreased urine output, (e) All of the above alkaline urine, and increased frequency of Answer: (e) Contraindications are poten- urinary tract infections would all facilitate tially modifiable with “safe” periods after cura- stone formation. High citrate is protective tive treatment for malignancy and chronic against urolithiasis. infections are available. Some centers offer kidney transplantation to certain patients 7. All of the following are indications for infected with human immunodeficiency virus: pretransplant nephrectomy except: noncompliance and substance abuse can be overcome in some instances. However, these (a) Renal tumor all need to be addressed carefully. (b) Hypertension controlled with one agent (c) Severe hydronephrosis 10. Commonly used immunosuppressants in (d) Recurrent renal infection pediatric kidney transplantation include: (e) Stones not managed with minimally inva- sive procedures (a) A calcineurin inhibitor (b) A corticosteroid Answer: (b) All of the above are indications (c) Rapamycin for pretransplant nephrectomy except hyper- (d) Mycophenolate mofetil tension, which can be controlled with one (e) All of the above agent. If, however, the hypertension cannot be controlled easily with one agent, a neph- Answer: (e) Given the relatively small rectomy should be considered. number of pediatric kidney transplants and Renal transplantation 73

the increasing number of drugs available, most Answer: (e) Least powerful antirejection evidence for immunosuppressive regimens is prophylaxis is required by first-time Caucasian derived from adult studies. A typical protocol recipients of a live donor kidney who have to be used initially in pediatric kidney trans- no evidence of presensitization. Enhanced plantation currently consists of triple therapy immunosuppresion is required when: the with a calcineurin inhibitor (CI) and steroids, recipient is of a repeat transplant, especially possibly paired with a course of induction one from a deceased donor; the recipient has with a nondepleting anti-T cell antibody. evidence of presensitization; the recipient is However, these are constantly changing and African-American. adjusted to individualized treatment based on the recipient's estimated risk for acute rejection 13. In the immediate posttransplant period, the balanced against side-effects. New treatment intravascular volume of pediatric recipients of protocols are attempting to avoid steroids functioning kidneys from adult donors needs completely and to reduce the use of CIs. to be: (a) Aggressively restricted to prevent pulmo- 11. Complications seen with increased frequency nary edema, hypertension and congestive when transplanting laparoscopically obtained heart failure adult kidneys specifically into small children (b) Restricted to balance the avid tendency of are: freshly transplanted kidneys to concen- (a) Delayed graft function and rejection trate the urine (b) Infection and malignancy (c) Generous to provide good perfusion of (c) Wound dehiscence and respiratory the new allograft compromise (d) Generous to compensate for the typically (d) Cardiovascular compromise and decreased limited ability of freshly transplanted end-organ perfusion kidneys to concentrate the urine (e) Ureteral anastomotic insufficiency (e) (c) and (d) Answer: (a) Small children, less than the Answer: (e) The relatively large adult donor age of 5, have repeatedly been shown to have kidney in a pediatric recipient requires excel- inferior outcomes in terms of delayed lent perfusion – recommendations include a graft function and early rejection following a central venous pressure (CVP) between 12 1 laparoscopic nephrectomy. Graft survival and 16 cmH2 O to provide adequate renal tur- and rejection rates in older children appear gor. The generous requirements to maintain a not to differ on the method of harvest. central venous pressure sufficient for good blood flow to the transplant persist during 12. Enhanced immunosuppression should be and after the surgery may continue long term, considered in the setting of: with even night-time or bolus supplementa- (a) Poor human leukocyte antigen (HLA) tions being required. matching 14. Not included in the differential diagnosis of (b) Recipient presensitization early allograft dysfunction is: (c) Delayed graft function/acute tubular necrosis (a) Acute rejection (d) African-American recipient race (b) Acute CI toxicity (e) All of the above (c) Vascular thrombosis 74 Clinical Pediatric Urology Study Guide

(d) Chronic allograft nephropathy (CAN) their grafts and have demonstrably poorer (e) Urinary tract obstruction renal function. Noncomplaint teenagers are more often girls than boys and frequently Answer: (d) Early allograft dysfunction is the cause for the noncompliance is due to triggered by all of the above except CAN, the side-effect profile of the immunosup- which is a long-term complication affecting pressant. kidney transplant recipients. Other long-term complications include the high incidence of 16. Challenges ahead in the management of cardiovascular (CV) disease and malignancies pediatric kidney transplant recipients are: compared to the general population. (a) Chronic allograft nephropathy 15. The age group of pediatric kidney transplant (b) Accelerated CV disease recipients at highest risk for adverse outcomes, (c) Skeletal problems, e.g. suboptimal growth in part because of noncompliance, is: (d) Malignancies such as posttransplant lymphoproliferative disease and infections (a) Infants as indicators of overimmunosuppression (b) Preschoolers (e) All of the above (c) Preteens (d) Teenagers Answer: (e) (e) Young adults Answer: (d) Transplantation is a complex therapy that requires adherence and close Reference follow-up. It has been documented that only 50% of patients will adhere to a long- 1. Troppman C , McBride MA , Baker TJ , Perez RV . Laparoscopic live donor nephrectomy: a risk factor term medical regimen and studies have shown for delayed function and rejection in pediatric kidney that most noncompliant patients are teenag- recipients? A UNOS analysis. Am J Transplant 2005; ers. These children are more likely to lose 5 : 175 – 82 . Renal calculus disease 25 Parmod Reddy and Eugene Minevich

1. The prevalence of hypercalciuria is greater in Cystinuria is an autosomal recessive pediatric patients with vesicoureteral reflux defect in reabsorptive transport of cystine and (VUR) than in the general population. True the dibasic amino acids ornithine, arginine, or False? and lysine from the luminal fluid of the renal proximal tubule and small intestine. The Answer: True The high prevalence of clinical manifestation of cystinuria is cystine hypercalciuria found in the parents, and the urolithiasis, which often recurs throughout a high prevalence of urolithiasis in other rela- patient's lifetime. Surgical intervention is often tives of children with VUR, points to a genetic necessary to treat the calculi. origin of the hypercalciuria. Data suggest that Primary hyperoxaluria is a very rare but the inheritance of hypercalciuria in patients serious disorder caused by a congenital defect, with VUR is autosomal dominant as it has resulting in very high levels (>200 mg/d) of been described for idiopathic hypercalciuria, endogenous oxalate production. Renal failure although with a higher probability to be inhe- develops in 80% of these patients by the third rited from the mother. This may be due to a decade. Normal dialysis for uremia cannot mechanism known as genetic imprinting, remove enough serum oxalate to protect the which has been observed in many autosomal kidneys and other organs from widespread dominant transmitted diseases in which an calcium oxalate deposition (i.e. oxalosis) and autosomal gene has a different behavior calcium oxalate stone production. Type I depending on the parent from whom it has hyperoxaluria is the more common variety. It been inherited.1, 2 occurs in 1 per 120,000 live births and is transmitted as an autosomal recessive trait, 2. Which of the following conditions are caused by a deficiency of the peroxisomal inherited in an autosomal recessive manner? liver-specific alanine:glyoxylate aminotrans- ferase gene (AGT). The median age for pre- (a) Cystinuria sentation of initial symptoms related to (b) Renal tubular acidosis hyperoxaluria is 5 years. Oxalate deposition (c) Primary hyperoxaluria occurs in multiple organs (e.g. bones, joints, (d) (a) and (b) eyes, heart). In particular, bone tends to be (e) (a) and (c) the major repository of excess oxalate in Answer: (e) Cystinuria and primary hyper- patients with primary hyperoxaluria. It is oxaluria are inherited as autosomal recessive recommended that all pediatric patients who disorders, whereas renal tubular acidosis and have stones should be screened for hyperox- the syndrome of idiopathic calcium oxalate aluria. The discovery of the condition in a urolithiasis are inherited as autosomal domi- child permits the siblings to be screened for nant conditions. the disease and preemptive treatment. Liver 76 Clinical Pediatric Urology Study Guide

transplantation prior to the development of (d) Glyceraldehyde-3-phosphate dehydroge- overt renal failure may preserve the individu- nase (GAPDH) al's native renal function, thus avoiding renal (e) Hepatic peroxisomal alanine-glyoxylate transplantation. Renal transplantation alone is Answer: (e) The enzyme deficiency in pri- insufficient because the liver defect causing the mary hyperoxaluria is hepatic peroxisomal hyperoxaluria is not corrected. Type II hyper- alanine-glyoxylate, therefore the only treat- oxaluria is due to a deficiency of D-glyceric ment for these patients is a liver transplant.4 dehydrogenase and this condition is less common than type I. End-stage renal disease 5. The enzyme deficiency that results in Lesch- is slightly less common in patients with type II Nyhan syndrome is: primary hyperoxaluria.3 , 4 (a) GAPDH 3. The most common cause of hypocitriuria in (b) 1,8 Dihydroxyadenyl transferase children is: (c) HPRT (d) Xanthine dehydrogenase (a) Renal tubular acidosis (e) Hepatic peroxisomal alanine-glyoxylate (b) Hypokalemia (c) Urinary tract infection (UTI) Answer: (c) Lesch-Nyhan syndrome is a (d) (a) and (b) condition that results in hyperuricosuria and is (e) (b) and (c) due to a deficiency of the enzyme HPRT. Lesch-Nyhan syndrome is inherited as an Answer: (a) Hypocitriuria is defined as uri- X-linked disease, therefore the disease is seen nary citrate excretion of <250 mg in 24 hours. mainly in males. It is characterized by increased Urinary citrate forms a soluble complex with blood and urine uric acid levels. The features calcium which inhibits the formation and of the Lesch-Nyhan syndrome are mental propagation of crystals. It is a common cor- retardation, spastic cerebral palsy, choreo- rectable cause of recurrent calcium stones. athetosis, uric acid urinary stones, and self- Females excrete more citrate and have lower destructive biting of fingers and lips. The incidence of stone formation than age-matched excess uric acid levels cause children to develop males. Urinary citrate is mainly derived endog- gout-like swelling in some of their joints. Uric enously through the tricarboxylic acid cycle acid urolithiasis and renal dysfunction develop and is excreted by renal tubular cells. Intracel- because of the excess uric acid levels.5 lular acidosis, acidic diets (diets rich in animal proteins), and hypokalemia decrease urinary 6. Which of the following is/are naturally citrate excretion. Fruits such as oranges and occurring inorganic inhibitors in urine grapefruits are the main exogenous sources of that affect the Ksp (solubility product) of urinary citrate. Hypocitraturia in type I renal lithogenic ions? tubular acidosis is thought to be due to a defect in distal tubule function. (a) Magnesium (b) Sodium 4. The enzyme deficiency that results in primary (c) Calcium hyperoxaluria is: (d) Nephrocalcin (e) (a) and (d) (a) Hypoxanthine-guanine phosphoribosyl transferase (HPRT) Answer: (a) The supersaturation theory (b) 1,8 Dihydroxyadenyl transferase is based on the binding of salts, which (c) Xanthine dehydrogenase occurs after a certain concentration is obtained. Renal calculus disease 77

A compound's thermodynamic solubility • History of concurrent diseases such as product (Ksp) defines the saturation of a com- cystic fibrosis, neoplasms, etc. pound in a solution. The Ksp of a compound • Nutritional habits – ketogenic diet is equal to the product of a pure chemical in Failure to thrive might indicate underlying equilibrium between a solid and solvent in gastrointestinal (GI) malabsortive conditions, solution. If a compound's concentration renal tubular acidosis. exceeds the Ksp, it precipitates. Temperature, pH, and the presence of inhibitors or promot- 9. The single most important predisposing ers in the solution also affect the compound's metabolic factor in pediatric urolithiasis is: solubility. Magnesium is an inorganic inhibi- tor that is found in urine and serves as an (a) Hyperuricosuria inhibitor of lithogenesis. While nephrocalcin (b) Hyperoxaluria is also an inhibitor found in urine it is an (c) Hypovolemia organic compound. Sodium and calcium (d) Hypercalciuria promote lithogenesis.6 (e) Hypocitriuria Answer: (d) Hypercalciuria is the single 7. Where does lithogenesis (urinary crystal most important risk factor in the development formation) begin? of pediatric urolithiasis. Hypercalciuira occurs (a) Glomerulus in 53–81% of all children with calcium (b) Efferent duct stones.7 (c) Ascending loop of Henle (d) Papillary duct 10. What is the underlying defect in the type of (e) Renal pelvis renal tubular acidosis which is associated with stone formation? Answer: (d) The papillary duct is the site of initial crystal formation and retention, which (a) Failure of bicarbonate reabsorption in the subsequently results in stone formation. proximal tubule (b) Chronic renal parenchymal damage 8. Important factors that are relevant in the (c) Hyperchloremic metabolic acidosis medical history of a child with urolithiasis (d) Inability of the distal nephron to maintain are: a hydrogen ion gradient between the tubular fluid and the blood (a) Recurrent skeletal fractures (e) Bicarbonate wasting (b) Prematurity (c) Nutritional habits Answer: (d) Renal tubular acidosis is a term (d) Failure to thrive applied to several conditions in which (e) All of the above metabolic acidosis is caused by specific defects in renal tubular hydrogen ion secre- Answer: (e) While obtaining a history in a tion. Three types of renal tubular acidosis are child with urolithiasis one should be sure to generally recognized based on the nature of ask about the following: the tubular defect. Nephrolithiasis occurs only • History of prematurity, especially the use in type I renal tubular acidosis, a condition of furosemide or calcium supplements marked by an abnormality in the generation during the new-born period and maintenance of a hydrogen ion gradient • History of recurrent skeletal fractures – by the distal tubule. Type I renal tubular ?hyperparathyroidism acidosis is a heterogeneous disorder which 78 Clinical Pediatric Urology Study Guide

may be hereditary, idiopathic or secondary to long-term impact and implications. J Urol 1998; 160 : a variety of conditions.8 341 – 4 ; discussion 344–5. 4. Cochat P , Basmaison O . Current approaches to the management of primary hyperoxaluria. Arch Dis Child 2000; 82 : 470 – 3 . References 5. Nyhan WL , Wong DF . New approaches to under- standing Lesch-Nyhan disease. N Engl J Med 1996; 1. García-Nieto V , Siverio B , Monge M , Toledo C , 334 : 1602– 4 . Molini N . Urinary calcium excretion in children with 6. Schrier RW . Diseases of the Kidney and Urinary Tract. vesicoureteral reflux. Nephrol Dial Transplant 2003; Philadelphia: Lippincott Williams & Wilkins, 2006. 18 : 507 – 11 . 7. Rudolph AM , Kamei RK , Overby KJ . Rudolph's Funda- 2. Guizar J , Kornhauser C , Malacara J , Sanchez G , mentals of Pediatrics. New York: McGraw-Hill , 2002 . Zamora J . Renal tubular acidosis in children with 8. Rodríguez-Soriano J . New insights into the patho- vesicoureteral reflux. J Urol 1996; 156 : 193 – 5 . genesis of renal tubular acidosis – from functional 3. Chow GK , Streem SB . Contemporary urological to molecular studies. Pediatr Nephrol 2000 ; 14 : intervention for cystinuric patients: immediate and 1121– 36 . Endourology for stone disease 26 Steven Lukasewycz and Aseem R Shukla (Based on chapter by Aseem R Shukla and Michael Erhard)

1. What percentage of pediatric patients diag- noncontrast helical CT is the “gold standard” nosed with urolithiasis complain of flank pain for detecting urolithiasis. at initial evaluation? 3. One hour after undergoing extracorporeal (a) 10% shock-wave lithotripsy (SWL) for a 1 cm (b) 20% right renal pelvic stone a 13-year-old female (c) 40% patient complains of excruciating right-sided (d) 50% flank pain. On examination, vital signs are (e) 80% stable and examination reveals no ecchymosis. Answer: (d) Only 50% of pediatric patients KUB shows stone fragments in the renal diagnosed with urolithaisis complain of flank pelvis but no evidence of ureteral calculi. The pain. The other common presentations include next step is: gross or microscopic hematuria, incidental (a) Administration of morphine to decrease findings, and urinary tract infections. expected postoperative pain (b) Administration of ketorolac to decrease 2. In the evaluation for pediatric urolithiasis expected postoperative pain ultrasound has: (c) Ureteroscopy to evaluate and remove stone fragments (a) Equivalent sensitivity when compared to (d) Renal ultrasound to evaluate for peri- computerized tomography (CT) nephric hematoma (b) No role in the contemporary evaluation (e) Transfusion of renal stones (c) Increased sensitivity when compared Answer: (d) Though uncommon, clinically to CT significant subcapsular hematoma can occur (d) Decreased sensitivity when compared after SWL. Often, these patients present with to CT extreme flank pain though they are clinically (e) None of the above otherwise stable. Evaluation should include ultrasound or CT scan to evaluate for this Answer: (d) Although ultrasound is often condition, but only conservative management the first choice of imaging modality in the and observation are typically required. pediatric population – and does demonstrate large renal calculi or presence of hydronephro- 4. Hydrophilic-coated guidewires are best used sis – it has very low sensitivity for detecting for/as: urolithiasis, especially for stones located in the ureter. While radiation exposure is cer- (a) Initial cannulization of the ureteral orifice tainly a concern in the pediatric population, a to minimize trauma 80 Clinical Pediatric Urology Study Guide

(b) A safety wire because the hydrophilic power with instrument deflection and retro- coating allows other instruments to pass grade stone migration are shortcomings of with minimal resistance ballistic lithotripsy. (c) Getting access past an impacted ureteral stone or narrow stricture 6. All of the following methods are commonly (d) Percutaneous tract dilation used to access a narrow ureteral orifice for (e) All of the above ureteroscopy except : Answer: (c) Hydrophilic-coated guidewires (a) Soft graduated ureteral dilator are primarily used for negotiating tortuous (b) Pre-stenting the ureter for passive dilation and narrowed ureters, and for gaining wire (c) Balloon dilation of ureteral orifice access past an impacted stone. These wires are (d) Incision of the ureteral orifice with a often extremely slippery and therefore are not Ho:YAG laser used for routine access of the ureteral orifice or as a safety wire as they can become easily Answer: (d) Though ureteroscopy can be dislodged. Extra-stiff guidewires are chosen effectively performed in the pediatric popula- for percutaneous tract dilation. tion, ureteral dilation is necessary in approxi- mately 30% of children undergoing this 5. Which of the following statements are true procedure. Options for dilation include use of regarding intracorporeal lithotripsy? a graduated single-shaft dilator, balloon dila- tion or temporary placement of a ureteral (a) Ultrasonic lithotripsy can be effectively stent. An incision with laser may be consid- performed through a flexible nephro- ered for a ureteral stricture, but this is unusual scope for standard ureteroscopy. (b) Shortcomings of ballistic lithotripsy include loss of power with instrument 7. Ureteroscopic stone extraction should be con- deflection and retrograde stone migration sidered first-line treatment for ureteral stones (c) Electrohydraulic lithotripsy (EHL) facili- in the pediatric population. True or False? tates stone fragmentation by creation of a cavitation bubble with maximal energy Answer: True According to the American 5 mm from the probe tip Urological Association (AUA) treatment (d) Due to the differential absorption of the guidelines ureteroscopic stone extraction is Ho:YAG laser, damage to urothelium or safe and effective in the pediatric population, the endoscope is not a significant concern and should be considered first-line treatment. Answer: (b) Ultrasonic lithotripsy probes 8. Factors that are likely to decrease stone-free lose power with deflection and therefore are rates for ESWL monotherapy include which not effectively used through flexible scopes. of the following? EHL does produce stone fragmentation through creation of a cavitation bubble and (a) Presence of an ectopic kidney maximal power is approximately 1 mm from (b) Presence of cysteine stones the probe tip. Though very effective for (c) Stone within a narrow-neck calyceal diver- stone fragmentation, Ho:YAG can cause sig- ticulum nificant damage to both urothelium and (d) Stones within a narrowed lower pole equipment. Only (b) is true in that ballistic infundibulum (pneumatic) lithotripsy probes may lose (e) All of the above Endourology for stone disease 81

Answer: (e) ESWL can be quite efficacious (c) Use of a single as opposed to multiple in the pediatric population with stone-free access tracts rates quoted at >80%. However, several (d) None of the above factors decrease the effectiveness of ESWL (e) (a) and (c) therapy including presence of an ectopic or horseshoe kidney, presence of “hard stones” Answer: (a) Percutaneous nephrolithotripsy such as cysteine or brushite stones, stones (PCNL) can be effectively used to extract within calyceal diverticula, and stones within large stones in the pediatric population. elongated narrowed lower pole infundibula. Though renal parenchymal bleeding can occur, only the use of an access tract <22F is associ- 9. Which factors have been shown to decrease ated with a decreased risk of transfusion. transfusion rates in the pediatric population? Multiple access tracts may be necessary and (a) Use of an access tract <22F have not been shown to increase the risk of (b) Use of an access tract <12F transfusion.

Renal parenchymal imaging 27 in children

Jason W Anast and Paul F Austin (Based on chapter by J Michael Zerin)

1. Which of the following is the least sensitive (e) In the acute phase of renal vein throm- test for diagnosing pyelonephritis? bosis, MAG3 renal scintography reveals a markedly enlarged kidney with increased (a) DMSA uptake and no excretion (b) Ultrasound (c) Computerized tomography (CT) Answer: (c) CT and MRI are rarely per- (d) Magnetic resonance imaging (MRI) formed in children since renal vein thrombosis can usually be easily diagnosed with ultra- Answer: (b) DMSA, CT, and MRI all have sound or MAG3 renal scintigraphy. The sensitivity >90% for diagnosing pyelonephri- classic presentation of renal vein thrombosis tis. While renal ultrasound is often ordered in is gross hematuria, hypertension, and an patients with suspected pyelonephritis, subtle enlarged, palpable kidney. In the acute phase, changes in parenchymal echogenicity are the kidney is enlarged, the parenchyma is often difficult to differentiate. Additionally, diffusely echogenic on ultrasound, and MAG3 purulent urine can be difficult to differentiate scintigraphy revealed reduced or absent uptake sonographically from desquamated cellular with no excretion of the tracer. In the chronic material in a chronically obstructed, unin- phase, renal function diminishes and the kid- fected system. ney becomes small and remains echogenic, and calcifications may appear within throm- 2. Which of the following is true regarding renal bosed intrarenal veins. vein thrombosis in children? (a) CT and MRI are useful imaging tests for 3. Which of the following is not true regarding diagnosing renal vein thrombosis imaging in Wilms’ tumor? (b) The classic presentation of renal vein thrombosis is gross hematuria, hypoten- (a) The ultrasound appearance is usually a sion, and an enlarged, palpable kidney solid, hyperechoic renal mass (c) Risk factors for renal vein thrombosis (b) The CT appearance is usually a low include prematurity, mother with dia- attenuation before contrast and variable betes, severe dehydration, sepsis, or coag- enhancement on postcontrast scans ulopathy (c) Postnephrectomy surveillance is usually (d) In the chronic phase of renal vein throm- performed with ultrasound of the bosis, the kidney becomes enlarged on surgical bed and of the contralateral ultrasound with decreased echogenicity kidney 84 Clinical Pediatric Urology Study Guide

(d) CT and MRI generally provide the (e) In leukemia and lymphoma, renal involve- same information about tumor extent ment is usually unilateral, with tumors and are superior to ultrasound for tumor appearing as distinct masses on ultra- staging sound (e) Ultrasound and MRI are generally more Answer: (d) Malignant rhabdoid tumor sensitive than CT at identifying renal vein is an aggressive neoplasm which usually and inferior vena cava (IVC) thrombus arises from the renal hilum and may extend Answer: (e) The ultrasound appearance of into the parenchyma. Clear cell sarcoma is Wilms’ tumor typically demonstrates a solid, usually indistinguishable from Wilms’ tumor hyperechoic renal mass, while CT imaging on abdominal imaging. Renal medullary usually reveals a low-attenuation solid mass carcinoma is a very aggressive tumor, with prior to contrast and variable enhancement heterogeneous enhancement on CT and postcontrast. MRI typically shows a low sig- MRI. The appearance of renal cell carcinoma nal renal mass on T1-weighted sequences and in children is similar to that in adults, with a bright solid mass on T2 images. In general, the tumor appearing hyperechoic on ultra- CT and MRI provide more accurate tumor sound and hyperdense on CT. In leukemia staging than ultrasound. CT and MRI are also and lymphoma, renal involvement is usually more accurate than ultrasound for detecting bilateral, with leukemic infiltrates presenting renal vein and IVC tumor thrombus. Postne- as diffuse renal enlargement with disruption phrectomy, Doppler ultrasound is usually uti- of the normal corticomeduallary junction, lized for routine surveillance and can aid in and lymphocytic infiltrates presenting as identifying small masses. nonspecific renal enlargement with diffuse increase in parenchymal echogenicity on ultrasound. 4. Which of the following is true about imaging of malignant renal tumors in children? 5. A 3-week-old boy is noted to have an enlarged (a) Malignant rhabdoid tumor is an aggres- right kidney with multiple cysts of varying sive, infiltrating renal tumor which can be sizes on abdominal ultrasound. A MAG3 renal detected with ultrasound, MRI, or CT, scan reveals 3% function of the right kidney. and usually arises in the parenchyma with What is the most likely diagnosis? invasion into the renal hilum (a) Multicystic dysplastic kidney (b) Clear cell sarcoma is usually a solid, com- (b) Autosomal recessive polycystic kidney plex mass on ultrasound, CT, or MRI, disease with an appearance usually distinguish- (c) Autosomal dominant polycystic kidney able from Wilms’ tumor disease (c) Renal medullary carcinoma is a very (d) Multilocular cystic nephroma aggressive tumor, with minimal enhance- (e) Juvenile nephronophthisis ment on CT or MRI (d) The imaging characteristics of renal Answer: (a) The description of a kidney cell carcinoma in children are similar to with multiple cysts of varying sizes with mini- those in adults, with the tumor appearing mal or no function is characteristic of multi- hyperechoic on ultrasound and hyper- cystic dysplastic kidney. Autosomal recessive dense on CT polycystic kidney disease presents as enlarged Renal parenchymal imaging 85 and echogenic kidneys. Autosomal dominant multilocular mass within the kidney. Juvenile polycystic kidney disease usually demonstrates nephronophthisis is an autosomal recessive a functioning kidney with few small cysts early disorder characterized by kidneys with in life that enlarge throughout childhood and increased echogenicity, with microscopic adulthood. Multilocular cystic nephroma is a medullary cysts that gradually develop into benign condition that presents as a complex, visible larger cysts.

Assessment of renal obstructive 28 disorders: ultrasound, nuclear medicine, and magnetic resonance imaging

James Elmore and Andrew J Kirsch

1. A 3-year-old girl is found to have bilateral Answer: (a) Rationale: Ultrasonography is a SFU (Society of Fetal Urology) grade 3 useful screening tool because it is inexpensive, hydronephrosis during a work-up for a febrile readily available, and does not use radiation. urinary tract infection (UTI). A voiding Ultrasonography does not provide functional cystourethrogram (VCUG) is performed and information. is negative. The best study to determine dif- ferential renal function, the degree of obstruc- 3. The radiopharmaceutical that is 90% bound tion, and to assess for a crossing vessel is: to plasma proteins and is principally cleared (a) Renal ultrasound by tubular secretion is: (b) Computerized tomography (CT) abdomen and pelvis with contrast (a) Tc-Mercaptoacetyltriglycine (MAG3) (c) MAG3 renal scan (b) Tc-Diethylenetriaminepentaacetic (DTPA) (d) Gadolinium enhanced magnetic resonance (c) Tc-Dimercaptosuccinic acid (DMSA) imaging (MRI) (d) Tc-Glycerinaldehydic acid (GAA) Answer: (d) Rationale: Although all of these Answer: (a) Rationale: Tc-MAG3 is primar- studies would provide important information ily bound to plasma proteins and is cleared by in this case, none except for MRI provides tubular secretion. Tc-DTPA is cleared by differential function, assesses the degree of glomerular filtration. Tc-DMSA is unique in obstruction, and provides the anatomical that it tightly binds the renal tubular cells and detail needed to see a crossing vessel. very little is excreted in the urine. These properties make each useful in different 2. All of the following are reasons that ultra- scenarios. sonography is a useful screening tool for the evaluation of a child with a history of a febrile UTI except : 4. The renal handling of gadolinium-DTPA (Gd-DTPA) during MR urography is deter- (a) It provides functional information mined by the: (b) There is no exposure to radiation (c) It is inexpensive (a) Gadolinium (d) It is readily available (b) DTPA 88 Clinical Pediatric Urology Study Guide

(c) The ratio of gadolinium to DTPA 7. A patient with myelodysplasia is referred for (d) Creatinine clearance MR urography. The presence of potentially harmful ferromagnetic implants: Answer: (b) Rationale: DTPA is handled identically by the kidney whether it is attached (a) Will be evaluated by the radiologist to radionuclide or to a contrast agent. (b) Can be neglected due to compensatory MR settings 5. A major disadvantage to performing MR (c) Needs to be evaluated by the referring urography in children is: physician (a) The use of ionizing radiation (d) New implants are all MR compatible (b) Adverse biological long-term effects Answer: (c) Rationale: The physician order- (c) Restricted use to normal renal function ing any radiographic study should be aware of (d) The need for sedation or anesthesia potential contraindications. Metallic implants Answer: (d) Rationale: The major disadvan- are a contraindication to MR urography and tages of MR urography are the need for these patients should undergo alternative sedation in children younger than about studies. 5 years of age and the cost.

6. A boy with a functionally solitary kidney is dia- 8. A patient with left flank pain is evaluated gnosed with SFU grade 2–3 hydronephrosis. by Doppler ultrasonography. During the The main advantage of obtaining an MR study a resistive index (RI) is calculated. urogram in comparison to more conven- Regarding the RI, urinary obstruction tional imaging modalities (i.e. diuretic would be suggested by all of the following renography) is: except: (a) The low cost of the imaging study (a) An interrenal RI difference >0.10 (b) Accurate functional data even in poorly (b) An abnormal RI response to a diuretic functioning renal units challenge (c) Gadolinium never causes allergic reactions (c) An RI >0.7 (d) Acquisition of anatomic and functional (d) A relatively high diastolic flow compared data in a single study to systolic flow Answer: (d) Rationale: MR urography Answer: (d) Rationale: A high RI index offers both functional and anatomical infor- (>0.7) is suggestive of obstruction. Since the mation which equals or surpasses that obtained RI is based on the ratio of diastolic flow to with any other imaging modality. Answer (b) systolic flow, increasing diastolic flow reduces is also true but not the principal advantage in the RI and is therefore not suggestive of this scenario. obstruction. Assessment of renal obstructive 29 disorders: urodynamics of the upper tract

Thomas E Novak and Yegappan Lakshmanan

1. In the evaluation of hydronephrosis, upper the upper threshold for normal renal pelvic tract infusion pressure/flow studies differ from pressure. (Page 463) nuclear renal scans and sonograms in their measurement of: 3. When using upper tract pressure/flow studies in children, consideration for which of the (a) Outflow resistance following parameters allows for a more (b) Renal blood flow meaningful interpretation: (c) Glomerular filtration rate (d) Degree of dilation (a) Maximum physiologic urine output (b) Presence of microscopic hematuria Answer: (a) In any biological fluid conduit (c) Calculation of renal blood flow system, the resistance of the conduit is directly (d) Detrusor leak point pressure proportional to pressure/flow. Both pressure and flow must be accounted for in the mea- Answer: (a) A limitation of the initial surement of resistance. (Page 461) infusion pressure/flow study as described by Whitaker is that infusion rates did not account 2. Animal models and human studies suggest for the age and size of a child. One can that undesirable physiologic renal changes estimate a patient's maximum physiologic begin to take place above what pelvic urine output based on the 90th percentile pressure? glomerular filtration rate (GFR) for their body surface area (available in nomograms). (a) 10 cmH O 2 The equation given on page 464 assumes that (b) 14 cmH O 2 under nonpathologic conditions, maximum (c) 18 cmH O 2 diuresis is 20% of the GFR. This calculation (d) 22 cmH O 2 allows the urologist to individually tailor Answer: (b) Although the exact precise infusion rates based on the age and size of the threshold pressure above which renal injury child. (Page 463) occurs is not known, studies in rats and pigs suggest that this number lies between 10 and 4. An 8-year-old girl has ongoing intermittent

20 cmH2 O. In humans, the intrarenal arterial right-sided abdominal pain, nausea and emesis pressure rises acutely above renal pelvic 1 year following pyeloplasty for uretero-pelvic

pressures of 14 cmH2 O. Based on these junction obstruction (UPJO). A sonogram

observations, 14 cmH2 O has been accepted as demonstrates residual hydronephrosis of the 90 Clinical Pediatric Urology Study Guide

right kidney with mild cortical thinning. The (d) The renal pelvic pressure subtracted from left kidney is normal. On diuretic renography, the abdominal pressure the right kidney clears after administration of Answer: (b) Periodic fluoroscopic monitor- furosemide at T1/2 = 18 min (indeterminate ing is an important part of infusion pressure- range). An infusion pressure flow study of the flow studies of the kidney. The presence of a right kidney is performed. The maximum ureteral opening pressure of >14 cmH O is renal pelvic pressure following administration 2 strongly predictive of a positive individualized of furosemide is 10 cmH O. The next step is: 2 pressure-flow study. (Page 470) (a) Termination of study (b) Drainage of the bladder 6. Patients with a positive infusion pressure-flow (c) Individualized infusion pressure-flow study of the upper tract should: study (a) Have the ureter stented while under (d) Antegrade nephrostogram anesthesia Answer: (c) Infusion pressure/flow studies (b) Maintain the urethral catheter for 1 week are useful in the evaluation of postsurgical prior to removal upper tract dilation. Renal pelvic pressures (c) Undergo diuretic Doppler sonography

>14 cmH2 O during the diuresis pressure- with calculation of the resistive index flow portion of the study are indicative of (d) Be placed on oral prophylactic antibiotics obstruction. In that event, an antegrade until definitive correction nephrostogram is performed and study is Answer: (d) One of the disadvantages of concluded. If the pressure does not exceed infusion pressure-flow studies of the upper 14 cmH O during diuresis, an individualized 2 tract is that they are invasive. Infection is a infusion pressure-flow study should be possible complication and this can be espe- performed. (Page 473) cially troublesome in an obstructed kidney. Routine preprocedural urine culture and pro- 5. The ureteral opening pressure is defined as: phylactic antibiotics prior to introduction of the needle are recommended. Patients with a (a) The difference calculated between the positive examination should remain on a bladder pressure and renal pelvic pressure prophylactic dose of oral antibiotics until during maximum infusion definitive surgical correction. (Page 473) (b) The renal pelvic pressure at which contrast is first visualized in the ureter under fluoroscopy Reference (c) The renal pelvic pressure measured immediately after introduction of the 1. Whitaker RH . Diagnosis of obstruction in dilated transducer into the renal pelvis ureters. Ann R Coll Surg Engl 1973; 53 : 153 – 66 . Ureteropelvic junction obstruction 30 and multicystic dysplastic kidney: surgical management

Michael C Carr

1. In a 6-week-old infant who has <10% ureter along with a very generous renal pelvis function to a kidney with a radiographically can be optimally managed with which repair: diagnosed ureteropelvic junction obstruction, (a) Foley Y-V plasty appropriate options for their management (b) Davis intubated ureterotomy would be all of the following except: (c) Scardino-Prince vertical flap pyelouretero- (a) Temporary nephrostomy tube drainage plasty (b) Placement of double stent for 4–6 weeks (d) Anderson-Hynes dismembered pyeloplasty with repeat renal scan to assess for (e) Retrograde endopyelotomy improved function Answer: (c) The Scardino-Prince vertical flap (c) Dismembered pyeloplasty and double can be fashioned to provide a long flap which stent placement at time of surgery can be fashioned even onto the posterior (d) Laparoscopic retroperitoneal dismembered aspect of the renal pelvis. In doing so, even a pyeloplasty ureteral narrowing of several centimeters can (e) Nephrectomy if cortical cysts are encoun- be overcome. A Foley Y-V plasty would work tered at the time of exploration for only limited length; an intubated uretero- Answer: (d) Temporary drainage may dif- tomy would be similar in use to an endopyelo- ferentiate those kidneys that have recoverable tomy and is generally not a good choice for function from those that have irreversible longer narrowed ureters. A dismembered injury or renal dysplasia and/or hypoplasia. pyeloplasty may be limiting if the ureter and/ Early surgery may allow for an easier repair or kidney cannot be mobilized to provide for than one in which a double J stent or a a tension-free anastomosis. nephrostomy tube causes trauma to the renal pelvis. The presence of cortical cysts generally 3. Repairs of an infant's ureteropelvic junction correlates with diffuse glomerulosclerosis. obstruction should involve all of the following Laparoscopic surgery is a viable option but except: the technical nuances in a 6-week-old infant preclude its use. (a) Judicious handling of tissues and use of traction sutures 2. A ureteropelvic junction obstruction in which (b) Either internal stenting with a double J there is a 4 cm narrowing of the proximal stent or nephrostomy tube, or placement 92 Clinical Pediatric Urology Study Guide

of a Penrose drain without internal The advent of laparoscopy with 5 mm instru- stenting ments allows for easy, safe removal of a kidney (c) Aggressive reduction of the renal pelvis with minimal morbidity giving the best cos- prior to completing a dismembered metic outcome and thus would be the favored pyeloplasty approach. (d) A running or interrupted closure with eversion of the mucosa to provide for a 5. In the case of a redo pyeloplasty in a 3-year-old watertight anastomosis child in which there was known to be urinary Answer: (c) The technical aspects in per- extravasation postoperatively along with forming an infant pyeloplasty amount to aggressive tailoring of the renal pelvis, the minimizing tissue trauma to decrease postop- preferred approach to the repair would be: erative edema and potential devascularization. (a) A redo open dismembered pyeloplasty Aggressive reduction of the renal pelvis risks (b) A laparoscopic dismembered pyeloplasty inadvertent infundibular injury. Creation (c) A retrograde endopyelotomy with laser of a funnel-shaped dependent tension-free (d) A ureterocalicostomy with amputation of anastomosis is the most important objective. the lower pole parenchyma (e) Antegrade Acucize endopyelotomy 4. A multicystic kidney which fails to involute over a period of several years may warrant Answer: (d) The anatomy of the persistent surgical removal due to the risk of malignancy ureteropelvic junction obstruction will dictate or future hypertension, albeit very rare. Opti- the repair, but significant urinary extravasa- mal surgical management in an 8-year-old tion and aggressive tailoring of the renal pelvis child with a noninvoluting 7 cm multicystic will make a dismembered pyeloplasty difficult, dysplastic kidney would be: either open or laparoscopically. A ureterocali- costomy, in which there is already thinning (a) Anterior subcostal open nephrectomy of the lower pole parenchyma and a dilated (b) Laparoscopic nephrectomy calyx, should provide the most durable long- (c) Flank incision and open nephrectomy term outcome. A focal stenosis in which the (d) Posterior lumbotomy and nephrectomy anastomosis was in a dependent position Answer: (b) Surgical removal of a multicys- may lend itself to an endopyelotomy, either tic dysplastic kidney has been accomplished antegrade or retrograde, depending on the readily through any number of incisions. patient's age. Laparoscopic nephrectomy and 31 pyeloplasty

CD Anthony Herndon (Based on chapter by Alaa El-Ghoneimi)

1. For the laparoscopic lateral retroperitoneal should be visualized first. The renal vein is approach for nephrectomy, which attachment above this and the renal pelvis sits most should be taken down last: anterior. If a renal artery is seen anterior to the ureter during this approach, then it (a) Posterior represents a crossing vessel and the repair (b) Inferior should be performed while reflecting this (c) Anterior lower pole vessel posteriorly. (d) Superior (e) Lateral 3. In which approach is it necessary to Answer: (c) With the retroperitoneal mobilize the colon prior to laparoscopic renal approach, the anterior surface of the kidney surgery? should be dissected last. It serves as an anchor (a) Right lateral retroperitoneal to keep the kidney reflected anteriorly, and (b) Left transperitoneal allows for easy identification of the hilar ves- (c) Left lateral retroperitoneal sels and ureter posteriorly as they lie on the (d) Right transperitoneal psoas muscle which is the posterior limit of (e) Prone retroperitoneal dissection. Answer: (d) Mobilization of the colon is 2. For the lateral retroperitoneal pyeloplasty, usually required for transperitoneal right renal which is identified upon first entry into the surgery. The transmesenteric approach is not retroperitoneum: ideal and typically is impeded by the duode- num or vena cava. By mobilizing the colon, (a) Aorta the Kocher maneuver can be avoided and (b) Renal artery excellent exposure for the renal hilum and (c) Renal vein renal pelvis can be achieved. (d) Renal pelvis (e) Adrenal gland Answer: (b) The renal artery lies most posterior, during retroperitoneal access it

Wilms' tumor 32 Sarah Marietti and Fernando Ferrer (Based on chapter by Michael Ritchey and Fernando Ferrer)

1. All of the following syndromes are associated (b) Mutations are associated with Wilms' with Wilms' tumors except: tumor 90% of the time (c) Is located on chromosome 11p13 (a) Wilms', aniridia, genital anomaly, mental (d) Mutations are associated with blastemal retardation (WAGR) syndrome pathology (b) Denys-Drash syndrome (c) Coloboma of the eye, heart defects, Answer: (c) The WT1 gene is located on atresia of the choanae, retardation of 11p13 of chromosome 11 and is responsible growth and/or development, genital for normal genitourinary development. and/or urinary abnormalities, and ear Mutations have been attributed to extensive abnormalities and deafness (CHARGE) genitourinary defects including pseudoher- syndrome maphroditism. Wilms' tumors are associated (d) Soto's syndrome with mutations of the WT1 gene <10% of the (e) Beckwith-Wiedemann syndrome (BWS) time. Germline mutations have been found in familial Wilms' tumors, and when present Answer: (c) 1% of Wilms' tumor patients are associated with a predominant stromal are afflicted with the WAGR syndrome. pathology. A WT1 mutation is rarely found in Denys-Drash syndrome consists of mesangial the Wilms' tumor patient without a specific sclerosis, pseudohermaphroditism and Wilms's syndrome or genitourinary anomaly. tumors. There have been reports of Wilms' tumors associated with Soto's syndrome. The 3. Unfavorable histology: syndrome consists of excessive growth in the first 2 years of life, mental retardation, hypo- (a) Is present in 50% of Wilms' tumors tonia and speech impairment. Patients with (b) Is defined by large polyploid nuclei BWS have an incidence of Wilms' tumor of (c) Is most commonly present in the first 10–20%. CHARGE syndrome consists of fail- 2 years of life ure of ability to close the eye, heart defects, (d) Is associated with resistance to chemo- blocked nasal passages, mental retardation, therapy underdeveloped sexual organs and ear abnor- Answer: (d) Unfavorable histology is associ- malities. This syndrome is not associated with ated with increased rates of relapse and death. Wilms' tumors. This histology is present in approximately 10% of Wilms' tumors, but responsible for 2. Which of the following is true regarding the over 50% of deaths from Wilms' tumors. It is WT1 gene? defined by large polyploid nuclei and is rare in (a) Is necessary for normal genitourinary the first 2 years of life, however, in children development over 5 years of age the incidence increases 96 Clinical Pediatric Urology Study Guide

to 13%. It is associated with resistance to for patients with stage III and IV favorable chemotherapy and can be divided into focal histology. These markers can be used as or diffuse patterns. When all anaplastic independent prognostic factors. components are removed outcomes are still very good. Thus, unfavorable histology is 5. Complications of chemotherapy include all of not a marker for aggressiveness of the the following except: tumor, but predominantly a marker for chemoresistance. (a) Doxorubicin use increases the chance of developing secondary malignancy 4. According to data obtained from National (b) Alkylating agents can chronically damage Wilms' Tumor (NWTS) and Children's germ cell production in both the pre- and Oncology Group (COG) renal tumor studies, postpubertal testis which statement is true? (c) Congestive heart failure is a known complication of vincristine treatment (a) Stage I and II favorable histology tumors (d) Growth disturbance is a direct effect of should be treated with radiation spinal irradiation and chemotherapeutic (b) Stage IV favorable histology disease effects on chondrocytes treated with vincristine, dactinomycin, doxorubicin and cyclophosphamide have Answer: (c) Children treated with Wilms' significant improvement in survival tumors are at increased risk for developing (c) Risk of death for stage III and IV second malignant neoplasms. The risk of favorable histology increased with loss leukemia or lymphoma is greatest in the of heterozygosity for chromosome 16q 8 years following treatment, but the risk of and 1p solid tumors continues to rise as the child (d) The presence of lymph node metastasis gets older. The use of doxorubicin has been does not negatively affect prognosis associated with increased risk of second malig- nancy. Doxorubicin is also cardiotoxic and is Answer: (c) The NWTS-3 demonstrated associated with increased risk of congestive that children with favorable histology stage I heart failure, the risk is proportional to the and II tumors treated with vincristine, dose of doxorubicin received. Alkylating dactinomycin and radiation did no better agents have been found to damage germ cells than those whose radiation was omitted at all ages, although prior reports believed that % with overall survival rates reaching 91 . the prepubertal testis was resistant to chronic They also demonstrated that patients with toxicity. Growth disturbances occur as a direct stage IV favorable histology treated with result of chondrocyte damage. The reduction dactinomycin, vincristine, doxorubicin and in height is a direct result of amount of radia- cyclophosphamide did no better than those tion, however, with the current radiation treated with only the three drugs dactinomy- doses given, the effect should not be clinically cin, vincristine and doxorubicin. Thus, the use apparent. of cyclophosphamide was omitted for this group. NWTS-5 was a single-arm therapeutic 6. The best test to detect inferior vena cava trial designed to collect information about the thrombosis and extent is: biologic features of the tumors. They found that loss of heterozygosity for both 16q and (a) Computerized tomography (CT) scan 1p increased the risk of relapse and death with contrast Wilms' tumor 97

(b) Ultrasound however, the determination of extent of (c) Magnetic resonance imaging (MRI) thrombus is best visualized with MRI. Ultra- (d) CT scan without contrast sound may also be used to evaluate for pres- ence or absence of thrombus, but precision is Answer: (c) Caval thrombus may be visible limited in demonstrating exact anatomical on CT scan with intravenous contrast; location and extent.

Surgical approaches for 33 renal tumors

João L Pippi Salle , Armando J Lorenzo , and Elizabeth R Williams (Based on original chapter by JL Pippi Salle and Roman Jednak)

1. Which one of the following is not a beneficial due to the risk of venous congestion of effect of a mannitol when used in the surgical the left kidney management of renal tumors? (c) Computerized tomography (CT) scan of the abdomen and pelvis as an imaging (a) Increases renal plasma flow study is enough in order to fully evaluate (b) Minimizes cellular edema the presence and extension of an inferior (c) Increases intravascular resistance vena cava tumor thrombus (d) Promotes osmotic diuresis (d) Left-sided renal tumors pose a difficult (e) None of the above challenge when associated with an inferior Answer: (c) Mannitol is a drug commonly vena cava thrombus as supra renal resec- used in the management of renal tumors, tion of the inferior renal cava without particularly in partial nephrectomies. There reestablishment of caval venous flow may are several potential benefits from using it, result in overwhelming venous conges- including stimulation of diuresis, decrease in tion of residual right kidney cellular edema, and increase in renal plasma (e) All of the above are important surgical flow. Experimental evidence indicates that issues to consider in this step of manage- mannitol induces a decrease in renal intravas- ment of tumors cular resistance. Answer: (d) Involvement of the inferior vena cava poses a difficult challenge. In patients 2. For renal tumors in which thrombus invasion with suprahepatic extension preoperative che- involves the inferior vena cava, which of the motherapy is advised. In order to define the following statements best reflects good level of thrombus involvement preoperatively, surgical management? Doppler ultrasound (including transesopha- (a) Resection of the involved segment of the geal echo) and/or magnetic resonance imag- vena cava should be avoided at all cost, ing (MRI) are obtained in those patients in even if it includes leaving small amounts whom there is clinical or radiological suspi- of residual tissue in the vessel wall cion of involvement. A CT scan is somewhat (b) Right-sided renal tumors with involve- limited in the evaluation of this problem. In ment of vena cava cannot undergo supra patients who undergo surgical exploration, renal resection of the inferior vena cava evidence of involvement of the vena caval wall 100 Clinical Pediatric Urology Study Guide

indicates surgical resection whenever possible. in children with bilateral tumors or tumors If a large segment of vena cava is resected, the involving a solitary kidney after chemother- blood supply to the contralateral kidney may apy. Also, it is advocated for children with be compromised. Due to the large network of syndromes that put them at high risk for collaterals which the left kidney commonly metachronous ipsilateral and contralateral has (i.e. gonadal, adrenal and lumbar veins as disease. Renal cell carcinoma is rare in chil- tributaries to the renal vein), obstruction of dren and appears to have a more favorable caval blood flow after resection of a right renal course than in adults. The data for partial tumor is tolerated. On the contrary, the right nephrectomy in pediatric renal cell carcinoma kidney has poor collateral venous drainage, are limited. There is no evidence that preopera- and may not fare well after resection of a left- tive immunotherapy improves survival or sided renal mass with caval thrombus. Thus, facilitates surgical resection. Furthermore, in such cases, reconstruction of the vena cava experience in adult patients would suggest to avoid overwhelming venous congestion of that cytoreductive surgery (i.e. tumor removal the right kidney is paramount. prior to systemic therapy) may be of benefit in some patients. 3. In the management of renal masses in children, which of the following statements 4. Regarding the management of congenital regarding partial nephrectomy is true? mesoblastic nephroma, which of the following statement(s) is/are false? (a) Partial nephrectomy is the surgical approach of choice for all unilateral renal (a) Congenital mesoblastic nephroma is one masses in children, as the goal is to of the most common solid renal tumors in preserve renal mass even if the risk of local infancy recurrence is slightly higher (b) Partial nephrectomy is the treatment of (b) Patients with a bilateral Wilms' tumor choice for congenital mesoblastic neph- benefit from partial nephrectomy prior to roma chemotherapy (c) There is a small but real risk of local reoc- (c) Partial nephrectomy plays a role in currence in patients with a cellular variant the management of a patient with of congenital mesoblastic nephroma Wilms' tumor associated with Beckwith- (d) Dissection of the renal hilum is particu- Wiedemann syndrome or Wilms', aniridia, larly important as this site is the most genital anomaly, mental retardation common location for invasion and local (WAGR) syndrome reoccurrence (d) Patients with biopsy-proven renal cell car- (e) All of the above statements are true cinoma should undergo immunotherapy Answer: (b) Congenital mesoblastic neph- or chemotherapy protocols prior to roma represents the most common solid renal attempting partial nephrectomy mass in infants, particularly in the first (e) The prognosis of renal cell carcinoma 6 months of life. The treatment of choice is after partial nephrectomy in children is nephrectomy as the risk of residual disease is equal to that of adults with similar stage an important problem, particularly in those of disease patients with the cellular variant of the tumor. Answer: (c) Partial nephrectomy is offered The renal hilum dissection is also of crucial in selected cases in children with Wilms' importance, where attention must be paid to tumors. Currently, protocols support its use complete removal of the tumor. Surgical approaches for renal tumors 101

5. Which of the following are true danger points and transected until the tumor is considered to be particularly considered in the resection resectable. If early transection of the ureter is of large renal masses in children? carried out and the tumor cannot be removed, the risk of postoperative obstruction or urine (a) A transmesenteric approach is preferred leak is increased. as it provides better exposure of the renal vessels as they enter the aorta and the vena cava 6. Regarding tumor rupture and/or spillage, (b) Great care should be paid to identify and which of the following statements is false? avoid ligation of the superior mesenteric (a) Preoperative chemotherapy increases the artery when exposure of left renal masses risk of tumor rupture with manipulation is attempted (b) Rupture of the renal capsule or presence (c) Dissection of the renal hilum should be of tumor spillage will upgrade the tumor performed after complete mobilization of stage of the kidney the kidney in order to allow proper (c) Gerota's fascia should be removed with exposure of the vessels in most cases the kidney in order to try to minimize the (d) The renal vein is longer on the right side, chances of leaving residual microscopic therefore more easily controlled disease behind, and decrease the chances (e) Early ligation of the ureter is recom- of rupture mended in order to allow exposure and (d) Peritoneal soiling is currently considered mobilization along the medial aspect of a stage 3, which adds radiation and the kidney towards the renal hilum chemotherapy to the management of Answer: (b) Although an uncommon occur- these children rence, dissection of the superior mesenteric (e) Bloody peritoneal fluid is a sign of major artery can be carried out by mistakenly identi- spillage fying it as the left renal artery. This can be Answer: (a) Tumor rupture or spillage is of seen in cases with large masses and displace- great concern. Due to the increased risk of ment of the aorta. Keeping this danger point local recurrence, it calls for more aggressive in mind is of crucial importance, as ligation of management including radiation. Conse- the superior mesenteric artery can lead to fatal quently, it increases the stage. Evidence of complications (mesenteric ischemia). The peritoneal soiling – either by detection of peri- approach to most renal masses is not trans- toneal implants or the presence of bloody mesenteric; the colon should be reflected peritoneal fluid – is considered diffuse medially after incision of the Line of Toldt. It soiling (not limited to the retroperitoneum). is advocated that the renal hilum should be Preoperative chemotherapy in general makes controlled prior to complete mobilization in the tumors less friable and potentially decreases order to decrease the chance of tumor cells the chances of tumor rupture. accessing the vascular system during aggres- sive manipulation. Furthermore, by securing 7. Regarding the arterial vascularization of the the renal artery early, the tumor may be easier kidney, which of the following statements is to dissect out and remove. Admittedly, this true? can not be done in all cases and some tumors need to by completely dissected in order to (a) There are no defined vascularized seg- gain access to the renal artery from a posterior ments and the vascular supply is extremely approach. The ureter should not be ligated variable between patients 102 Clinical Pediatric Urology Study Guide

(b) Multiple renal arteries occur in up to 23% extensive tumor thrombus (i.e. above the of patients hepatic veins), the decision to call a tumor (c) Ligation of a branch of the renal arteries inoperable should be finalized in the operating is of no consequence due to the rich room (after exploration). In cases where the collateral blood supply of the kidney involvement of surrounding structures pre- (d) Warm ischemia by temporary occlusion cludes safe resection, the tumor should be of the renal artery can be safely tolerated biopsied in order to have histological informa- for >1 h without any important conse- tion prior to chemotherapy (following the quences for renal function Children's Oncology Group protocols). (e) All of the above statements are true Lymph node sampling is wise to perform but a full lymph node dissection is not required. Answer: (b) Multiple renal arteries are not In general, pathology specimens should be uncommonly seen. Up to 23% of patients sent fresh and not in formalin. have multiple arteries in one kidney. The arterial blood supply to the kidney has 9. Which one of the following is the best surgical well-defined segments and is devoid of collat- approach for a unilateral renal mass in a erals. Therefore, ligation of a branch leads to child who has not received previous ischemia of the segment it supplies. Warm chemotherapy? ischemia is not well tolerated. After 30 min of warm vascular occlusion the progression of (a) Retroperitoneal through a flank incision ischemic cellular damage increases. Therefore, (b) Retroperitoneal through a dorsal lumbo- cooling the kidney is an important maneuver tomy incision to exercise in prolonged partial resections. (c) Transperitoneal through a subcostal or transverse abdominal incision 8. In regards to the surgical management of (d) Transperitoneal using a laparoscopic Wilms' tumors, the decision to call the tumor approach inoperable ______: (e) All of the above are acceptable options (a) Should be reached prior to surgery with- Answer: (c) The surgical approach of choice out exploration in children with possible Wilms' tumor is (b) Intra-operative biopsy should be avoided trasperitoneal, usually through a subcostal at all costs in patients who are deemed or transverse abdominal incision. The laparo- inoperable and those children should scopic approach has been used in a few institu- undergo chemotherapy based on clinical tions after administration of chemotherapy characteristics (following the Society of Pediatric Oncology (c) A full lymph node dissection is required (SIPO) protocols favored in Europe and in patients that undergo a radical nephrec- parts of South America). The retroperitoneal tomy for Wilms' tumor approach through a flank or dorsal lumbo- (d) A pathology specimen should be sent in tomy incision leads to limited exposure in formalin children and is generally avoided. (e) Surgical exploration is avoided for patients who have preoperative imaging consistent 10. After hilar dissection during a partial nephrec- with bilateral disease and/or suprahepatic tomy, you are about to clamp the renal artery. extension of tumor thrombus Preparation to minimize the effects of ischemia includes all of the following except: Answer: (e) Aside from children with bilat- eral tumors, tumors in a solitary kidney and (a) Cooling the kidney with ice Surgical approaches for renal tumors 103

(b) Generous hydration prior to clamping brain. All patients with this diagnosis should (c) Simultaneously clamping both the artery undergo either a CT or MRI of the brain. and vein Additional testing may be necessary, but none (d) Administering mannitol prior to clamping of the other tests listed are indicated in the (e) Minimizing traction on the kidney during absence of associated symptoms. resection 13. A 4-year-old girl presents with a right renal Answer: (c) Simultaneous occlusion of both mass with no evidence of extension beyond vessels produces additional venous congestion the right kidney on preoperative imaging. and renal damage. All of the other maneuvers During dissection of the mass off the adrenal, described above help to minimize the effects a 0.5 cm incidental incision is made into the of ischemia on the kidney and are considered renal capsule without evidence of gross tumor routine during nephron-sparing surgery. spillage and with careful avoidance of perito- neal contamination. On final pathology, the 11. While examining a patient with a known tumor is identified as a Wilms' tumor. The left renal mass, which of the following findings kidney is uninvolved with tumor on inspec- is not consistent with intravascular tumor tion and there is no evidence of spread beyond extension? the kidney. What is the final staging? (a) Bilateral lower extremity edema (a) Stage I (b) Varicocele (b) Stage II (c) Dilated superficial abdominal veins (c) Stage III (d) Proteinuria (d) Stage IV (e) Peripheral neuropathy (e) Stage V Answer: (e) Venous congestion due to Answer: (b) The tumor is completely excised tumor infiltration of the vascular system can except for local tumor spillage into the flank manifest in a variety of ways. All of the choices when the capsule was violated. Tumor spillage above result from inferior vena cava (IVC) into the peritoneum would have upgraded the extension of tumor thrombus except for case to a stage III. peripheral neuropathy. If any of these findings is discovered on exam, MRI or color Doppler 14. Which of the following would not be an ultrasonography is used to confirm the pres- indication for partial nephrectomy for a Wilms' ence of intravascular extension. tumor? (a) Solitary kidney 12. A 13-month-old male undergoes a left (b) Bilateral tumors nephrectomy and the pathology reveals rhab- (c) Renal failure doid tumor of the kidney. In addition to a (d) Beckwith-Wiedemann syndrome hematology-oncology referral, which of the (e) 2 cm tumor based on CT findings following should be ordered? Answer: (e) Partial nephrectomy is not the (a) Bone scan standard of care for management of Wilms' (b) MRI of the head tumors. However, there are situations in (c) Duplex ultrasound of the lower extremities which partial nephrectomy is indicated. Partial (d) Transthoracic echocardiogram nephrectomy is utilized in cases mandating Answer: (b) The most common metastatic nephron-sparing surgery (solitary kidney, site for rhabdoid tumor of the kidney is the bilateral masses, renal insufficiency) in order 104 Clinical Pediatric Urology Study Guide

to minimize the risk of subsequent dialysis. (b) Open Gerota's fascia and examine the Additionally, certain syndromes such as Beck- contralateral kidney with-Wiedemann and WAGR predispose (c) Perform a lymph node sampling patients to developing Wilms' tumors. Partial (d) Perform a partial nephrectomy nephrectomy is indicated since these patients (e) Perform a right radical nephrectomy with have an increased likelihood of recurrent careful avoidance of tumor spillage disease. Tumor size is not a criterion used Answer: (d) When a Wilms' tumor is during evaluation for partial nephrectomy. encountered intraoperatively, certain surgical modifications need to be implemented. The 15. After making a subcostal incision in a incision often needs to be extended in order to 6-year-old girl with a right renal mass adequately assess for tumor in the contralat- suspicious for renal cell carcinoma, you note eral kidney. A focused lymph node sampling peritoneal metastases. Frozen section of one should be performed, including any suspicious of these metastases reveals Wilms' tumor. lymph nodes and nodes along the iliac, para- Which of the following is not appropriate aortic, and celiac regions. A partial nephrec- surgical technique? tomy is not appropriate in the setting of (a) Extend the surgical excision across the normal renal function and no risk factors for midline recurrent Wilms' tumor. Pediatric upper tract trauma 34 Douglas A Husmann

1. A patient that has gross hematuria trauma is most likely to miss the majority of disproportionate to the degree of trauma what type of injuries: sustained suggests the finding of a: (a) UPJ disruption (a) Uretero-pelvic junction (UPJ) disruption (b) Grade 2 renal injury (b) Intraperitoneal bladder rupture (c) Traumatic intimal renal artery disruption (c) Traumatic arterial-venous (AV) fistula (d) Perinephric urinoma developing from a formation major renal laceration (d) Thoracic or lumbar spinal fracture (e) Subcapsular hematomas which will (e) Preexisting congenital renal anomaly cause a Page kidney affect (posttraumatic hypertension) Answer: (e) The presence of gross hematuria disproportionate to the severity of the history Answer: (a) A single-phase CT study is of the traumatic injury classically suggests that beneficial in determining renal perfusion, and a preexisting renal anomaly is present. Specifi- will accurately diagnose renal artery intimal cally, genitourinary (GU) anomalies found tears, grade 2 renal injuries and the presence with this history in order of decreasing fre- of subcapsular hematomas. It may on occa- quency are UPJ obstruction, hydrouretero- sion miss the presence of urinary extravasation nephrosis (refluxing, obstructive or nonob- developing as a consequence of a major renal structive–nonrefluxing variants), and renal laceration, it is however notoriously famous fusion anomalies (horseshoe kidneys). Approx- for missing the vast majority of isolated % imately 30 of patients with UPJ disruption ureteral injuries and UPJ disruptions. A will have no degree of hematuria, with the triphasic CT study (precontrast study, vast majority of patients with a UPJ disrup- followed by a study immediately following tion having severe life-threatening associated injection and then a 15–20 min delayed study) injuries which resulted in inadequate radio- is the most sensitive method for diagnosis graphic staging studies being performed. The and classification of renal trauma, and is presence of multiple life-threatening injuries highly accurate in diagnosing isolated ureteral and the need for emergent life-saving surgery injuries. results in inadequate radiographic staging, leading to a missed or delayed diagnosis of the UPJ disruption in >50% of patients. 3. Following a traumatic injury a CT of the abdomen reveals a renal laceration that extends 2. A single-phase computerized tomography into 2 cm past the renal cortex, no urinary (CT) scan obtained in the assessment of GU extravasation is seen. The injury is associated 106 Clinical Pediatric Urology Study Guide

with a devitalized renal fragment, the grade of urinary drainage bag filled with grossly bloody renal injury is: urine associated with clots. The patient's vital signs are stable, no orthostatic blood pressure (a) 1 (BP) changes are noted, and a stat hematocrit (b) 2 (Hct) reveals a 2 point decrease from prior (c) 3 values. Your next step is: (d) 4 (e) 5 (a) Reassurance, continue activity and force Answer: (c) fluids (b) Recommend bed rest, increase intravenous Grading of renal injuries (IV) fluids and serial Hct assessment (c) Recommend bed rest, increase IV fluids Grade of Description and stat CT scan Renal Injury (d) Recommend bed rest, increase IV fluids, emergent angiography 1 Renal contusion or subcapsular (e) Emergent surgical exploration hematoma 2 Nonexpanding perirenal hematoma, Answer: (d) Approximately 25% of patients <1 cm parenchymal laceration, no with a grade 3–4 renal trauma, managed in a urinary extravasation, all renal nonoperative fashion, will have persistent or fragments viable delayed hemorrhage. Classically, delayed 3 Nonexpanding perirenal hematoma, hemorrhage will present 10–14 days postin- >1 cm parenchymal laceration, no jury but may occur up to 1 month after the urinary extravasation, renal fragments may be viable or devitalized insult. Delayed hemorrhage usually arises from the development of bleeding arteriovenous 4 Laceration extending into the collecting system with urinary extravasation, renal fistulas or ruptures of traumatic-induced fragments may be viable or devitalized pseudoaneurysms. The traumatic induced or arterial-venous malformations (AVMs) will not spontaneously resolve and may be associ- Injury to the main renal vasculature with contained hemorrhage ated with life-threatening hemorrhage. Man- agement should be by restriction of 5 Completely shattered kidney, by definition multiple major lacerations of activity, increase IV fluids, and angiographic >1 cm associated with multiple embolization of the bleeding site. Remember, devitalized fragments in a child, shock and orthostatic BP changes or may be one of the latter signs of severe bleed- Injury to the main renal vasculature with ing. Delay in management of this serious uncontrolled hemorrhage, renal hilar complication places the child's life in jeopardy. avulsion Recurrence of gross hematuria with clots after their urine has grossly cleared is one of the key 4. A 13-year-old boy sustained a grade 4 renal findings suggestive for the development of injury, secondary to a skiing accident 1 week bleeding from an AV malformation or rupture ago. He has been managed nonoperatively. of a vascular pseudoaneurysm. Patients with His urine has been grossly clear for the past a history of recurrent gross hematuria with 24 h and he is allowed to ambulate. You are clots after their urine has grossly cleared should contacted by the junior urology resident on undergo urgent/emergent angiographic evalu- the service that shortly after ambulation the ation and embolization of the bleeding site. Upper tract trauma 107

5. A 9-year-old boy sustained a left major renal 6. A 6-year-old boy on IV prophylaxis with laceration associated with a perinephric uri- cephazolin undergoes angiographic emboliza- noma. Three days postinjury a repeat CT scan tion of a traumatic AV fistula associated confirms a major renal laceration with peri- with a grade 4 left traumatic renal injury. On nephric urinoma of approximately 5–6 cm in the night of the embolization he spikes a diameter, the left distal ureter is visualized, temperature to 40°C. BP is stable. Physical and the right kidney is normal. Your next examination reveals a tender left flank and step is: the absence of bowel sounds, no rebound or indirect tenderness is found. His urine is (a) Continued observation grossly clear. You recommend acetaminophen, (b) Cystoscopy and left ureteral stent blood and urine cultures, and: placement (c) Left percutaneous nephrostomy tube (a) Continued observation placement (b) Change in antibiotic coverage to pipera- (d) Percutaneous drainage of left perinephric cillin and metronidazole urinoma (c) Retroperitoneal/renal ultrasound (e) Cystoscopy, left ureteral stent placement, (d) CT of abdomen and aspiration of and percutaneous drainage of perinephric perinephric hematoma/urinoma urinoma (e) Emergent surgical exploration and removal of necrotic tissue Answer: (a) Approximately 75% of traumatic- Answer: (a) Postembolization syndrome induced urinomas will spontaneously resolve. is manifested by pyrexia up to 40°C, flank Intervention should be considered only if pain, and adynamic ileus. Symptoms there is persistent flank pain, adynamic ileus, should resolve within 4 days after the and/or recurrent fevers suggestive of infection. embolization. When pyrexia develops, blood In these patients endoscopic intervention and urine cultures to rule out bacterial with cystoscopy, retrograde pyleography, seeding of the necrotic tissue are necessary. and ureteral stent placement is the preferred Consideration for a repeat CT scan with treatment. Although percutaneous nephros- possible aspiration, culture, and drainage tomy tube placement and ureteral stent place- of a perinephric hematoma/urinoma should ment are equally efficacious, the major be given if febrile response persists for >96 h advantage of stent placement is quality of life or if the patient's clinical course should rapidly during convalescence. A disadvantage of a worsen. No indications for altering antibiot- ureteral stent is that both stent placement and ics, the performance of a renal ultrasound or removal require anesthesia in a small child. In emergent surgical exploration are present in addition, the small ureteral stent 4–5F used in this patient. small children may be obstructed by blood clots and fail to adequately drain the kidney, resulting in persistent urinoma. Percutaneous 7. A 2-year-old boy is a victim of child abuse, he nephrostomy tube placement is preferred in a was thrown across the room striking his left patient failing endoscopic management. In a flank on the arm of a chair. A single-phase CT patient whose medical condition precludes the scan reveals a massive spleenic rupture, use of general anesthesia, or in patients where coexisting with a large liver laceration and a fair amount of blood clots exist in the renal hemoperitoneum, both kidneys are visualized. pelvis, occlusion of the small ureteral stent is His vital signs are rapidly deteriorating likely to occur. and the child undergoes an emergent partial 108 Clinical Pediatric Urology Study Guide

heptectomy and spleenectomy. Perihepatic 8. Recommendations for radiologic follow-up of drains are noted to have >250 ml output per a grade 2 renal injury are repeat CT scan: shift beginning the first postoperative day. (a) Only if a patient develops systemic or Drain creatinine of 15 mg/dl is noted. CT scan localized signs or symptoms reveals good bilateral renal perfusion, large (b) At 2–3 days posttraumatic injury left perinephric urinoma, no distal left ureter (c) At 3–4 weeks posttraumatic injury is seen. Contrast is seen in the right distal ure- (d) At 3 months posttraumatic injury ter.Vital signs and hematocrit are normal and (e) At 2–3 days postinjury and again at stable. Your next step is: 3 months posttraumatic injury (a) Continued observation Answer: (a) Follow-up renal imaging is not (b) Left percutaneous nephrostomy tube recommended for grade 1–2 renal injuries and (c) Cystoscopy, retrograde pyelogram, and for grade 3 lacerations where all fragments are stent placement viable. In patients with grade 3 renal lacera- (d) Cystoscopy, retrograde pyelogram, and tions associated with devitalized fragments, surgical exploration with primary ureteral grade 4 and salvaged grade 5 renal injuries, a repair repeat CT scan with delayed images should be (e) Surgical exploration with primary repair obtained 2–3 days following the traumatic insult. This study serves the purpose of assess- Answer: (d) The majority of patients ing the extent of the hematoma/urinoma and sustaining a traumatic UPJ disruption will will serve as a baseline evaluation in case present with vascular instability resulting secondary hemorrhage or infection should in minimal to no emergent preoperative occur. An additional 3-month follow-up imaging. This results in the average time from triphasic CT scan is obtained to verify resolu- injury to diagnosis of a UPJ disruption of tion of any perinephric urinoma and to define 36 h. Patients with this injury usually present the anatomic configuration of the residual with increased drainage from intraoperative functioning renal parenchyma. Irrespective of drains, postoperative fever, ileus or flank pain. the grade of the injury, repeat imaging with a Classic findings on cystogram are a perinephric triphasic CT scan is recommended for patients urinoma, the absence of renal parenchymal with a history of renal trauma that have a injuries, and nonvisualization of the ipsilateral persistent and/or increased fever, worsening ureter. A retrograde pyelogram is extremely flank pain, or persistent gross hematuria >72 h helpful in delineating the injury. If the diag- following the traumatic insult. nosis is made within 5 days of the injury, and the patient is clinically stable, open surgi- 9. Retroperitoneal (renal) exploration is recom- cal repair is preferred. If a delay in diagnosis mended when: of ≥6 days occurs, or if the patient is clinically unstable, a percutaneous nephrostomy tube (a) A stab wound to the flank associated with may be placed with plans for delayed repair a grade 2 renal injury at 6–8 weeks postinjury. Predelayed repair, (b) A 22 caliber gunshot wound (GSW) assessment of ipsilateral renal function, and resulting in a grade 2 renal injury antegrade and retrograde pyelograms can (c) A motor-vehicle accident (MVA) results be performed to delineate the extent of in an isolated grade 3 renal injury with the injury. Surgical alternatives for repair devitalized renal fragments would be primary uretero-ureterostomy, (d) A nonhemorrhagic, nonexpanding retro- ileal ureter, autotransplanatation or primary peritoneal mass found on surgical explo- nephrectomy. ration following a GSW Upper tract trauma 109

(e) An MVA resulting in the need for emer- 10. A 7-year-old boy is involved in a pedes- gent laparotomy due to vascular instabil- trian MVA. A CT scan taken 1 h following ity, retroperitoneal hemorrhage found on trauma reveals no perfusion, and subsequently exploration no function, of the left kidney; the right kid- ney Answer: (e) is within normal limits. No associated intraabdominal injuries are noted. Vital signs Consensus Recommendations for Management of are stable and hemoglobin is normal. Your Renal Trauma next step is: Clinical findings and/or Recommended grade of renal injury treatment (a) Observation Grade 1 or 2 renal injury Nonoperative (b) Angiography with stenting of left renal irrespective of traumatic artery etiology (c) Angiographic infusion of streptokinase Isolated grade 3, 4, and Nonoperative hemodynamically stable into left renal artery grade 5 renal injuries (d) Systemic heparinization Uncontrollable renal Absolute requirement (e) Surgical exploration hemorrhage/vascular for surgical Answer: (a) In a patient sustaining renal instability (usually grade 4 intervention vascular or grade 5 injuries) arterial trauma, the clinical triad of hemody- Persistent or delayed Absolute requirement namic instability, inadequate collateral blood hemorrhage not responding for surgical flow, and warm ischemic time almost invari- to angiographic embolization intervention ably results in the inability to salvage renal Expanding pulsatile Absolute requirement function. Due to this finding, no attempt to retroperitoneal mass found for surgical repair injuries to segmental renal vessels should on surgical exploration for intervention (verify be considered, and repair of the traumatically coexisting intraabdominal contralateral renal injured main renal artery is seldom if ever injuries function prior to exploration) indicated if a normal contralateral kidney is present. Penetrating trauma, Retroperitoneal inadequate preoperative (renal) exploration Reconstruction of the main renal artery fol- radiographic staging due to recommended (verify lowing trauma is only a consideration in vascular instability of patient, contralateral renal patients that are hemodynamically stable with retroperitoneal hemorrhage function) an injury to a solitary kidney or in patients found on exploration where bilateral renal arterial injuries are found. Blunt trauma, inadequate Retroperitoneal The infrequent exception to this rule is the preoperative radiographic (renal) exploration presence of an incomplete arterial injury where staging due to vascular recommended (verify instability of patient, contralateral renal perfusion to the kidney has been maintained retroperitoneal hemorrhage function) by flow of blood through either the partially found on exploration occluded main renal artery or via collateral Blunt/penetrating trauma, Retroperitoneal vessels. radiographic screening (renal) exploration studies reveal grade 3 with with renorrhaphy and devitalized renal fragments, repair recommended grade 4 or 5 renal injury, coexisting intraabdominal injuries, especially, duode- num, pancreas and colon

The ureter 35 Martin Kaefer (Based on chapter by Cem Akbal and Martin Kaefer)

1. All of the following statements regarding (c) Spontaneous electrical activity initiated by ureteral embryology are true except: pacemaker cells in the renal pelvis and upper ureter (a) The development of the ureter begins as (d) Innervation provided by peptidergic an out pouching of the mesonephric duct nerves (e.g. tachykinins, substance P) termed the ureteric bud during the fourth (e) Nitric oxide week of gestation (b) Transient luminal obstruction of the ure- Answer: (c) Although the ureter receives ter occurs with subsequent recanalization neurologic input from numerous sources, during the 7th week of gestation spontaneous electrical activity initiated by the (c) As the ureter enters into the bladder its pacemaker cells appears to be the dominant sole contribution to trigonal formation is control necessary for providing electrical at the level of the superficial trigone activity and inducing coordinated peristaltic (d) The renin-angiotensin system appears to muscular activity. Evidence to support this play a major role in ureteral development contention includes the persistence of peristal- sis after renal transplantation or denervation. Answer: (c) As the ureter penetrates the Propagation of electrical activity proceeds bladder, its muscular layers disperse and through tight areas of cell-to-cell contact contribute to formation of both the deep termed intermediate junctions. and superficial trigone. The outer layer melds into the detrusor in the upper part of the 3. All of the following statements regarding the hiatus to form Waldeyer's sheath, which prevention of retrograde urine flow are true attaches the ureter to the bladder and is in with the exception of: continuity with the deep trigone. As the ureter makes its transition to an intramural location (a) The ratio of intramural ureteral length/ the muscle fibers take on a primarily longitu- ureteral diameter plays an important dinal orientation and these fibers spread out to role in the prevention of vesicoureteral form the borders of the superficial trigone. reflux (b) In order for the antirefluxing mechanism to operate effectively there must exist 2. Which appears to play the most important adequate muscle backing against which role in ureteral peristalsis? the ureter can be compressed (a) The parasympathetic nervous system (c) The distal end of the ureter must be (b) The sympathetic nervous system adequately anchored to the base of the 112 Clinical Pediatric Urology Study Guide

bladder so that with filling it does (b) Superimposed infection in the presence of not migrate laterally and experience a obstruction can further reduce efficient shortening of tunnel length peristalsis of urine (d) Maintenance of unidirectional urine flow (c) Over time, the increase in urine volume at the ureterovesical junction (UVJ) is a held by the ureter results in decreased purely passive process amplitude of peristaltic contraction waves Answer: (d) Although the traditional concept (d) After prolonged obstruction and the resul- of the ureterovesical antireflux mechanism tant deleterious effects on ureteral func- consisted solely of a passive flap valve mecha- tion, the rate of urine transport becomes nism, a number of investigators have noted dependent on hydrostatic forces gener- sufficient anatomic and physiological evidence ated by the kidney to point to an active sphincteric mechanism. Although an anatomically defined circum- Answer: (a) As the ureter fills with urine, ferential sphincter has not been identified the the peristaltic contraction waves become longitudinal muscles of the UVJ are postu- smaller and are unable to coapt the ureteral lated to provide this “active” component by wall. Urine transport then becomes dependent closing the meatus. on hydrostatic forces generated by the kidney. As predicted by the Law of Laplace, an 4. All of the following statements regarding increase in luminal size results in a decrease ureteral physiology are true with the in intraluminal pressure, thereby decreasing exception of: the force propelling urine in a forward direction. (a) Under normal conditions, peristaltic contractions occur 2–6 times/min (b) Peristaltic waves elevate ureteral pressure 6. Urinary concentrating ability is the first aspect

to 20–80 cmH2 O of renal function to be affected by obstruction. (c) Peristaltic contractions advance down the With increasing urine flow rates (due to the ureters at a rate of 2–6 ml/s production of a larger volume of dilute urine), (d) The ureter maintains a higher resting pres- the initial response of the ureter is to: sure than the bladder, thereby facilitating (a) Increase bolus volume flow of urine in the caudal direction (b) Increase peristaltic frequency Answer: (d) The resting pressure within the (c) Decrease the pressure gradient at the

ureter, 0–5 cmH2 O, is similar to the intra- level of the UVJ in order to facilitate vesical pressure of the nondistended bladder. the more rapid transport of urine into the Peristaltic waves elevate this pressure to bladder

20–80 cmH2 O. If the intraureteral pressure (d) Experience luminal dilation beginning at exceeds the intravesical pressure urine will its most proximal segment proceed into the bladder. Answer: (b) The first change in ureteral physiology resulting from an increase in urine 5. Which one of the following is false regarding load is to increase peristaltic frequency. After alterations in upper urinary tract physiology the maximal frequency is achieved, further secondary to obstruction? increases in urine transport occur by means of (a) Increased luminal diameter secondary to increases in bolus volume. Eventually, the obstruction results in an increase in boluses coalesce and the ureter becomes filled intraluminal pressure with a continuous column of fluid. The ureter 113

7. Which of the following statements regarding megaureter in which there exists an adynamic refluxing megaureters is false? segment of distal ureter with adequate ureteral (a) Relative to obstructed megaureters there diameter. With all forms of ureteral obstruction appears to be a higher collagen to smooth the hierarchal order of the ureteral pacemakers muscle ratio within the ureteral wall may become altered. When alternate pace- (b) Multiple infections of the refluxing ureter makers fire, electrical activity and subsequent can result in progressive fibrosis peristalsis can propagate both caudally and (c) The collagen subtype III appears to be cranially from the site at which it is initiated. abnormally elevated This lack of coordination affects the efficiency (d) Surgical results are generally reported to of urine transport and contributes to upper be better in the repair of refluxing mega- tract dilatation. The effect of ureteral diameter ureters relative to obstructed megaureters on the ability to generate effective peristaltic contractions provides the rationale for ureteral Answer: (d) Refluxing megaureters have tapering in the patient with an increased ure- been shown to have a twofold increase in tis- teral size. The Law of Laplace would result in sue matrix ratio of collagen to smooth muscle higher intraluminal pressures and a resultant when compared to patients with primary improvement in urine transport. obstructed megaureter and controls. Investi- gators subsequently demonstrated that there 9. Which statement is true regarding gender was a greater contribution of type III collagen differences in regard to vesicoureteral reflux in the refluxing megaureter and felt that this identified in the newborn? may play a role in the pathophysiology of refluxing megaureters. Because type III colla- (a) Males are more likely to have abnormali- gen is a less distensible fiber, it may cause an ties at the level of the UVJ that predispose intrinsically stiffer ureter and play a role in the to vesicoureteral reflux lower surgical success in the reimplantation of (b) Intravesical pressures are generally refluxing megaureters. higher in newborn males than females, potentially explaining why newborn 8. Which of the following statements regarding males tend to present with higher the obstructed megaureter is false? grades of reflux than their female (a) Long-standing obstruction can result in cohorts altered hierarchy of the ureteral pacemak- (c) Females tend to have higher grades of ers, resulting in cranially directed peristal- reflux than males sis thereby contributing to upper tract (d) Females have a higher genetic predisposi- dilatation tion to develop reflux (b) In the primary obstructed megaureter, Answer: (b) Transient urodynamic abnor- obstruction is due to compromise of the malities have been identified in neurologically intravesical ureteral diameter normal children who present with urinary (c) The Law of Laplace predicts that surgi- tract infections (UTIs) and reflux in the first cally reducing the ureteral diameter will year of life. One study identified abnormali- increase intraluminal pressures ties, including detrusor hyperreflexia and (d) Concentrating ability is the first aspect elevated filling pressures, in 97% of boys and of renal function to be affected by 77% of girls who presented in infancy with obstruction reflux. Intravesical pressures in boys were Answer: (b) The classic example of a func- found to be higher than in girls, a finding tional obstruction is the primary obstructed confirmed by other investigators. This finding 114 Clinical Pediatric Urology Study Guide

may account for gender differences in degree Answer: (d) The most common anatomic of reflux. cause of secondary vesicoureteral reflux is posterior urethral valves, in which approxi- 10. The most common cause(s) of secondary mately 50% of boys are affected. However, reflux is/are: functional causes of increased intravesical (a) Posterior urethral valves pressure, including detrusor sphincter dyssyn- (b) Bladder diverticuli ergia in patients with neuropathic bladder (c) Ureteroceles dysfunction secondary to myelomeningocele, (d) Bladder dysfunction are far more common. The imaging of reflux and 37 ureteral disease

Jeanne S Chow and David A Diamond

1. A voiding cystourethrography (VCUG) is the best first imaging test would be ultra- definitely preferred to a radionuclide cystogram sound. Ultrasound is readily available, nonin- (RNC) when studying: vasive, and does not emit ionizing radiation. (a) Abnormalities of the male urethra 3. Some of the main clinical indications for (b) Patients with suspected duplex renal ultrasound include: anomalies (c) Patients with ureteroceles (a) Evaluation of reflux and prenatal hydro- (d) Patients with sibling reflux nephrosis (e) Patients with cystitis (b) Evaluation of stone disease and ureteral trauma (i) (a) (c) Evaluation of prenatal hydronephrosis (ii) (a) and (b) and stone disease (iii) (a), (b) and (c) (d) Evaluation for a mid-ureteral stricture and (iv) (a), (b), (c) and (d) reflux (v) All of the above (e) Evaluation for stone disease and reflux Answer: (iii) The VCUG enables one to Answer: (c) Ultrasound misses 75% of precisely define the anatomy of the urethra, reflux. Therefore, (a), (d) and (e) would be bladder, and ureters when reflux is present, incorrect answers. It is also a poor modality, whereas the RNC provides less anatomic in general, for evaluating ureteral trauma rela- detail and essentially determines the presence tive to a contrast study such as CT scan which or absence of reflux. Therefore, to obtain can better delineate urinary extravasation. anatomic definition of the male urethra, a ureterocele within the bladder or to determine 4. MAG3 studies are useful in the evaluation of: that a ureter is part of the duplex system, the (a) Reflux VCUG is required. (b) Hydronephrosis 2. What is the initial imaging study recom- (c) Pyelonephritis mended for the evaluation of the ureter? (d) Complex ureteral anatomy (e) Posterior urethral valves (a) Computerized tomography (CT) Answer: (b) MAG3 is a substance that is (b) Ultrasound filtered and excreted by the kidney, and can be (c) Voiding cystourethrogram used to assess renal function and drainage. It (d) Retrograde urethrogram is, therefore, an ideal substance with which to (e) Magnetic resonance imaging (MRI) assess hydronephrosis. Because it does not Answer: (b) Although each of these imaging afford precise anatomic detail it is not useful studies may be useful to evaluate the ureter, in assessing complex ureteral anatomy. 116 Clinical Pediatric Urology Study Guide

5. The main disadvantage of MR urography (d) Inadequate visualization of the ureters compared to other imaging studies is: (e) Poor contrast resolution (a) Long imaging time often requiring Answer: (a) MR urography affords precise sedation definition of ureteral anatomy and is not (b) Difficulty imaging patients with poor dependent upon renal function. However, it renal function does require a long imaging time (30–45 min). (c) Difficulty in imaging infants with “renal For young or uncooperative children this is immaturity” likely to require sedation. Ureteral anomalies and their 38 surgical management

Christopher Cooper

1. Which of the following mid-ureteral surgical Answer: (e) Most patients with a retrocaval techniques is most likely to result in ureteral ureter are asymptomatic, but some present stricture? with obstructive symptoms. The retrocaval ureter is almost always on the right and is (a) Renal mobilization displaced medially most often at the L3 level. (b) Ureteral stents The persistence of the sub cardinal vein results (c) Removal of periureteral adventitia in the development of the retrocaval ureter. (d) Peri-anastomotic drains (e) Spatulation of both proximal and distal 3. A 14-year-old child presents with intermittent ureteral segments left flank pain and gross hematuria. Intra- Answer: (c) The blood supply to the mid- venous pyelogram (IVP) reveals a long, ureter is more tenuous than that of the proxi- smooth-filling defect in the distal ureter. mal and distal ureter. The adventitia supports Cystoscopy reveals a polypoid lesion project- multiple blood vessels, and to preserve the ing from the ureteral orifice and biopsy reveals adventitia and blood supply the ureter epithelial-lined fibrovascular fronds. Appro- should be mobilized with as much surround- priate treatment may require: ing soft tissue as possible. Removal of the (a) Nephrectomy adventitial layer may result in ischemic injury (b) Nephroureterectomy and subsequent ureteral stricture. Renal (c) Transurethral resection and Bacille mobilization, drainage, and spatulation are all Calmette-Guérin (BCG) techniques which may be employed to result (d) Upper tract instillation of BCG in a tension-free wide-open anastomosis. (e) None of the above 2. A retrocaval ureter is most often associated Answer: (e) This patient has a benign with which of the following: fibroepithelial polyp. Appropriate treatment is by complete resection. This may be (a) Persistence of the supracardinal vein approached by ureteroscopy from the (b) The left ureter bladder or percutaneously from the kidney. (c) Hematuria At times, ureterotomy may be required to (d) Lateral ureteral displacement completely remove the polyp and the base of (e) Absence of symptoms its stalk.

Megaureter 39 Douglass Clayton

1. The ureter receives its blood supply predomi- 3. Patients with refluxing megaureter are found nately from: to have an increase in which of the following types of collagen fibers: (a) Renal artery (b) Aorta (a) Type I (c) Internal iliac artery (b) Type II (d) Gonadal artery (c) Type III (e) All of the above (d) Type IV Answer: (e) The blood supply of the ureter Answer: (c) Previous authors have shown a is not based solely on one vessel, instead it is relative increase in the presence of Type I and supplied by multiple branches from various Type III collagen both in the obstructed mega- arteries. The blood flow to the proximal ureter ureter as well as the refluxing megaureter. originates in the ipsilateral renal artery while However, in the refluxing megaureter, the the mid-ureter receives its blood supply both proportion of Type III collagen is increased from direct aortic branches and from branches predominately and may result in a ureter which of the gonadal artery. Finally, the distal is less distensible than other megaureter types. ureteral blood supply arises from the internal iliac artery as well as the superior and inferior 4. A diagnosis of megaureter is identified in vesical arteries. All of these various sources approximately how many asymptomatic of blood supply then form an anastomotic neonates: network of vessels that travels within the (a) 5 % adventitia of the ureter. (b) 10 % (c) 25 % 2. Megaureter is defined as having a ureteral (d) 50 % diameter greater than: (e) 75 % (a) 4 mm Answer: (c) Approximately 25% of neonates (b) 5 mm are found to have megaureter on routine fetal (c) 6 mm sonography. The only entity more commonly (d) 7 mm noted is that of hydronephrosis which is (e) 8 mm identified in up to 41% of fetal ultrasounds. Answer: (e) Studies of ureteral diameter in 5. When evaluating the function of the mega- children involving both postmortem analysis ureter, the diagnostic study which provides and excretory urography analysis have shown the greatest amount of data is: the normal diameter of the pediatric ureter is < 6 mm in size. Thus, the accepted definition (a) Ultrasonography for megaureter is a ureter > 8 mm in diameter. (b) Renal scintigraphy 120 Clinical Pediatric Urology Study Guide

(c) Magnetic resonance urography Answer: (b) A subset of patients with (d) Whitaker test obstructed megaureter and renal impairment (e) Voiding cystourethrography (VCUG) may benefit from temporary urinary diver- sion. Diversion may allow for decompression Answer: (b) Of the studies mentioned of the ureter, resolution of infection, and above, renal scintigraphy is the only study improvement in renal function. Of the which provides adequate functional data. Both available diversion procedures, cutaneous ultrasonography and VCUG are necessary ureterostomy is the procedure of choice due components of the evaluation of megaureter. to its low morbidity and effectiveness. Ultrasound provides good anatomic data of both ureter and the kidney. Likewise, the VCUG characterizes the anatomy of the lower 8. Definitive reconstruction of the megaureter is urinary tract and is essential in evaluating for best undertaken when the child is: vesicoureteral reflux. Magnetic resonance (a) Newborn urography is a burgeoning test and may (b) 6 months of age provide improved anatomic detail. The (c) 8 months of age Whitaker test is invasive and is not routinely (d) 12 months of age performed today. (e) 24 months of age

6. The radiotracer of choice when performing Answer: (d) The technical difficulties of neonatal renal scintigraphy for the evaluation surgical correction of the megaureter are less of the megaureter is: when performed after 12 months of age. Previous studies have demonstrated that as (a) DMSA many as 10% of infants undergoing recon- (b) DTPA struction at < 8 months of age will require (c) MAG3 repeat procedures. Answer: (c) Because of the immature state of the neonatal renal unit when renal scintig- 9. The principles of surgery for the megaureter raphy is performed with radiotracers which include which of the following: approximate glomerular filtration rate such as DTPA, limited data are provided regarding the (a) Adequate straightening of the ureter degree of ureteral obstruction. Of the avail- (b) Tapering without devascularization able agents for neonatal scintigraphy, mercap- (c) 5:1 ureteral tunnel to diameter length toacetyltriglycine (MAG3) is the agent of (d) Antirefluxing repair choice because it provides functional data based (e) All of the above on the effective renal plasma flow of the kidney Answer: (e) All of the principles listed not the neonatal glomerular filtration rate. above should be considered when repair of the megaureter is undertaken. Extensive 7. The procedure of choice for temporary uri- ureteral dissection may result in devasculari- nary diversion in the patient with obstructed zation of the ureteral segment. Tapering megaureter is: should be limited to the intramural portion (a) Percutaneous nephrostomy of the ureter only. A gradual transition (b) Cutaneous ureterostomy should occur from the tapered to nontapered (c) Vesicostomy segment to decrease the risk of a pseudo- (d) Pyelostomy obstruction. Megaureter 121

10. Persistent vesicoureteral reflux after tapered performed at 6 months. Once the absence of reimplant for megaureter occurs in what reflux is confirmed, any antibiotic prophylaxis percentage of patients: may be discontinued. Up to 5% of patients will have persistent reflux after a tapered reim- (a) 5 % plant. This should be monitored initially as (b) 15 % many will resolve within a few months of sur- (c) 20 % gery. Those patients with continued reflux (d) 30 % may require a repeat procedure. (e) 50 % Answer: (a) After a tapered ureteral reim- plant, a voiding cystourethrogram should be

Ureteral duplication anomalies: 40 ectopic ureters and ureteroceles

Jorge R Caso and Michael A Keating

1. Which of the following statements about distance, and become more laterally exagger- ureteral development is false? ated. Vesicoureteral reflux can result. (a) W olffian duct remnants in the female include Gartner' s duct, the epoophoron 2. Which of the following statements about and oophoron; an ectopic ureter venting ureteroceles is false? into these bypasses the external sphincter, (a) T heories about the origins of ureteroceles causing incontinence include: persistence of Chwalla' s mem- (b) A n ectopic ureter in a male will drain brane; abnormal muscular development into structures proximal to the urinary of the terminal ureter, possibly in con- sphincter, such as the seminal vesicle, vas junction with meatal or perisphincteric deferens, or prostatic urethra obstruction; and abnormal widening of (c) I f the ureteral bud originates from a more the mesonephric duct between its insertion cephalad than usual position on the meso- in the urogenital sinus and the ureteral nephric duct, primary reflux from the bud resultant short submucosal ureter can (b) U reteroceles commonly cause unilateral occur hydronephrosis from ureteral blockage (d) A fter the common excretory duct is (c) S alvageable renal function is usually seen absorbed into the bladder and the ureteral with extravesical ureteroceles, while intra- orifice begins its migration, the excretory vesical ureteroceles are associated with duct continues moving toward the mid- poor function line, fusing to its contralateral partner to (d) U reteroceles occur more frequently than form the primitive trigone ectopic ureters (1:500 versus 1:2000), are (e) I n a duplicated kidney, the superior/upper more common in females, and are associ- pole ureteric bud may assume an abnor- ated with duplex systems in females 95% mally high position along the mesonephric of the time whereas in 66% of males there duct, while the lower pole bud may be is a single system positioned more caudally than usual; the resultant relationship of ureteral orifice to Answer: (c) Intravesical ureteroceles are more draining renal segment is defined by the commonly associated with salvageable renal Weigert-Meyer law function and extravesical ureteroceles with Answer: (c) After the ureteral orifice is absor- poor function. This may be explained by an bed into the bladder it migrates in a cranial abnormal takeoff of the ureteral bud inducing and lateral direction. A more caudally posi- less favorable polar tissue of the metanephric tioned ureteral bud will be absorbed into blastema to develop. Cystic dysplasia could the bladder more quickly, migrate a greater result. 124 Clinical Pediatric Urology Study Guide

3. Which of the following statements about the (d) I n males, ectopic ureters may cause prosta- classification of ureteroceles is false? titis, seminal vesiculitis, and epididymitis (e) A n extravesical ureterocele is the most (a) A ccording to Stephens' classification, intra- common cause of bladder outlet obstruc- vesical ureteroceles are either stenotic or tion in newborn girls and boys nonobstructed, while extravesical uretero- celes are classified as sphincteric, sphinc- Answer: (e) Posterior urethral valves are the terostenotic, cecoureteroceles, or blind most common cause of bladder outlet obstruc- ectopic tion in newborn boys; an extravesical uretero- (b) I ntravesical stenotic and extravesical cele is the second most common cause. sphincteric ureteroceles are the most com- mon variants 5. All of the following statements about the (c) S phincteric ureteroceles always open radiographic appearance of ureteroceles and proximal to the sphincter ectopic ureters are true except: (d) C ecoureteroceles open into the bladder, (a) E ctopic ureters that displace the bladder but have a blind pouch extending into the can often be differentiated from uretero- submucosa of the urethra which may celes by noting the thickness of the wall cause bladder outlet obstruction when that separates the structure from the blad- distended with urine der lumen (e) B lind ectopic ureteroceles are similar to (b) R adionuclide scans in the newborn that the sphincteric variants, but without a are completed soon after delivery offer ureteral orifice the most accurate assessment of renal Answer: (c) Generally, sphincteric uretero- function with both anomalies celes open proximal to the external sphincter. (c) V esicoureteral reflux occurs in the ipsila- However, in females the meatus is sometimes teral lower pole moiety with both uretero- distally positioned. At rest, the bladder neck celes and ectopic ureters about 50% of the and sphincter contract, which can obstruct the time ureter and orifice. (d) E xcretory urography is especially useful in the diagnosis of ectopic ureters and may be suggestive of nonfunctioning, occult 4. All of the following statements about the duplications presentation of ureteroceles and ectopic ureters are true except: Answer: (b) The transitional physiology of the newborn kidney affects its handling of (a) P rolapsed ureteroceles have a pink, smooth radionuclides and contrast. Function can be wall, unlike the grape-like appearance of underestimated and later recovery overesti- sarcoma botryoides or the yellow-white mated if scans are completed during the first color of a periurethral duct cyst few days of life. Radionuclide studies should (b) U reteroceles typically present with a be deferred for at least 4– 6 weeks. urinary tract infection (UTI), failure to thrive, and colic; ectopic ureters com- 6. Which of the following statements regarding monly present with flank pain, fever, and the management of ectopic ureters is false? an abdominal mass (c) T he diagnosis of ureteral ectopia should (a) A fter performing nephrectomy for an ecto- be entertained in any girl who reportedly pic ureter associated with a minimally func- has never been fully toilet trained tioning kidney, a nonrefluxing, obstructed Ureteral duplication anomalies 125

ureteral stump should be left open to (d) I n a duplex system having a functioning drain upper pole, the majority of patients will (b) F or a duplex system with marginally require surgery of the upper and lower functioning upper pole, when considering urinary tracts when vesicoureteral reflux is salvage and evaluating relative function, present in two or more associated renal one should note that the upper pole moieties moiety provides about one-third of the Answer: (d) The majority of these patients, affected kidney' s function in a normal especially older children, can be managed duplex kidney solely at the bladder level, excising the uretero- (c) I n a single system anomaly having sal- cele and correcting the reflux, thus eliminating vageable function, reimplantation should the need for secondary surgery. be carried out disregarding the distal-most ureteral segment entering the urethra or 8. Which of the following is not recommended bladder neck when performing cystoscopic incision of a (d) I n a duplex system having salvageable ureterocele? function, ureteropyelostomy (the lower pole renal pelvis) or ureteroureterostomy (a) A ggressive hydration and minimal use are recommended treatments. of intravesical irrigation are helpful for keeping the ureterocele distended

Answer: (a) When a ureteral stump is left, it (b) T he puncture can be performed with a should be taken as distally as possible, ligated fine Bugbee electrode using cutting cur- with absorbable sutures, and urine aspirated rent; a laser can also be used from its distal end. Reflux can be associated (c) S elective punctures of intravesical uretero- with obstructed ectopic ureters but can be celes are created along their lateral edge unappreciated unless cyclic voiding cystoure- (d) F ollow-up includes an ultrasound 10– thrography is included in the work-up. In such 14 days later to document resolution of cases a urinary fistula can develop if the stump hydronephrosis and decompression of is left open. the ureterocele (e) A voiding cystourethrogram should be 7. Which of the following treatments regarding performed 3– 6 months later in evaluation the management of ureteroceles is false? of reflux Answer: (c) Selective incisions are placed at (a) W ith single system ureteroceles, nephrec- a distal position along the medial base of the tomy is the treatment of choice for those ureterocele. The intent is to preserve enough associated with minimal or nonfunction- submucosal length to obviate the development ing kidneys of reflux. With cecoureteroceles and sphinc- (b) U pper pole heminephrectomy is recom- teric ureteroceles, the meatus should be incised mended for duplex systems having mini- vertically and the cut extended above the blad- mally or nonfunctioning upper poles; der neck. however, lower urinary reconstruction is still required of a significant (20– 30 % ) 9. Which of the following statements regarding number of patients surgical technique is false? (c) I n a single system associated with a sal- vageable kidney, selective incision and (a) T raction should be minimized in all puncture is the procedure of choice patients when mobilizing the kidney, but 126 Clinical Pediatric Urology Study Guide

especially in babies, who are prone to vas- a newborn, it is reasonable to address the cular spasm and intimal tears; papaverine problem at about 1 year of age can be applied topically if vascular spasm (b) A ddressing ureteroceles at an age prior to persists toilet training is optimal because children (b) H eminephrectomy defects should be who are corrected at an early age rarely closed with chromic mattress sutures develop bladder dysfunction incorporating the parenchyma and fascia; (c) I f vesicoureteral reflux is noted on the pararenal fat may also be mobilized and initial follow-up cystogram after uretero- tacked into position to close the defect cele repair, a trial of medical management (c) A fter performing a heminephrectomy, is reasonable; however, the rate of spon- tacking the capsule of the lower pole of taneous resolution is less than that encoun- the kidney to muscle is recommended to tered with primary reflux avoid postoperative torsion (d) I n general, prophylactic antibiotics should (d) W hen performing a lower ureterectomy, be continued after surgery until postop- the ureter can be identified by ligating erative studies confirm the adequacy of the obliterated hypogastric artery and the repair following it laterally until the ureter is (e) A fter the anomaly has been corrected, a encountered posteriorly reasonable follow-up would include ultra- sound studies at 2 and 5 years, as well as Answer: (d) Once the hypogastric artery is exams by the primary physicians to include identified, it should be followed posteriorly checking urine, blood pressure, and interval until the ureter is encountered medially. somatic growth Answer: (b) Bladder dysfunction is quite 10. Which of the following statements regarding common after surgical repair even when per- the postoperative management of ureterocele formed prior to toilet training (up to two- surgery is false? thirds of children). Infrequent voiding, large (a) W hen bladder reconstruction becomes capacities, and significant post-void residuals necessary after incision of a ureterocele in are typical findings. Laparoscopic management of 41 duplication anomalies

Stephen Lukasewycz and Aseem R Shukla (Based on chapter written by Alberto Lais and Craig Peters)

1. What are the technical advantages of a retro- 2. All of the following are true when comparing peritoneal versus a transperitoneal approach a laparoscopic partial nephrectomy and an to laparoscopic management of duplication open approach in the pediatric population anomalies? except: (a) A retroperitoneal approach offers wider (a) Shorter operative time field of view (b) Shorter hospital stay (b) A retroperitoneal approach allows for easy (c) Decreased need for postoperative analgesia access to other genitourinary structures (d) High rate of conversion of laparoscopic allowing for concomitant surgery (i.e. procedure to open procedure ) (e) None of the above (c) A retroperitoneal approach allows for a Answer: (a) At some centers with a large larger working space and therefore larger laparoscopic experience, laparoscopic surgery insufflation volumes often allows for a shorter hospital stay and a (d) A retroperitoneal approach allows essen- decreased need for postoperative analgesia; tially direct access to the renal unit and however, operative times are generally equiva- minimizes the risk of postoperative adhe- lent or longer compared to open surgery. The sions secondary to bowel mobilization rate of conversion to an open procedure is (e) A retroperioneal approach results in a very low in most series. significantly shorter operative time Answer: (d) Owing to the size of the perito- 3. If during a laparoscopic upper pole partial neum compared to the retroperitoneal space nephrectomy, the lower pole remnant begins created by insufflation, a transperitoneal to appear blue and ischemic, one should: approach allows for a wider field of view and (a) Perform lower pole nephrectomy a larger working space. Additionally, a trans- (b) Release all tension off the lower pole peritoneal approach allows for easier access to vessels other genitourinary structures compared to a (c) Apply neosynephrine directly to the lower retroperitoneal approach. Recent studies pole vessels have failed to show a difference in operative (d) Decrease pneumoperitoneum pressure time in a transperitoneal versus a retroperito- (e) (b) and (c) neal approach. A retroperitoneal approach does offer essentially direct access to the renal Answer: (b) During laparoscopic partial unit and, as the peritoneum is not entered, nephrectomy, manipulation of the lower pole limits the risk of future adhesions. moiety may lead to arterial vasospasm and 128 Clinical Pediatric Urology Study Guide

decreased blood flow. Releasing tension on (d) Lack of tactile feedback requiring reliance the lower pole vessels and applying papaverine on visual cues or lidocaine directly to the affected vessels may (e) None of the above reverse this condition. Answer: (c) A limitation of robotic laparos- copy is that once the robot is engaged, no 4. All of the following are true for robotic lap- further adjustment to patient position is aroscopy as compared to traditional laparos- possible without disengaging the robot and copy in pediatric urologic surgery except: resetting the arms into newly placed ports. (a) Improved visualization with high defini- Robotic laparoscopy also lacks tactile feed- tion and three-dimensional capabilities back, and the surgeon must rely on the appear- (b) Increased speed and easier fine suturing ance of tissue during grasping and an capabilities understanding of the inherent strength of the (c) Increased field of surgery due to versatil- robotic arm when manipulating intraabdomi- ity and mobility of robot nal structures. Vesicoureteral reflux: anatomic 42 and functional basis of etiology

John M Park

1. Which of the following statements is not true 3. Which of the following demographic state- of the normal ureterovesical junction (UVJ) ments is true of vesicoureteral reflux (VUR)? anatomy and function? (a) In infancy, VUR is more common and of (a) The inner longitudinal muscle fibers of the severe grade in girls ureter extend into the superficial trigone (b) VUR is most common in patients with (b) Waldeyer' s sheath anchors the intramural Asian ancestry ureter and prevents displacement during (c) The incidence of VUR in normal infants bladder filling and children is estimated to be 0.4– 1.8 % , (c) Contraction of the trigonal musculature whereas in children with UTI it can be up causes lateral displacement of the ureteric to 30– 50 % orifice (d) In familial analysis, VUR does not appear (d) Embryologically, both ureter and trigone to be a heritable disorder develop from mesonephric duct origin Answer: (c) In infants, VUR is more com- Answer: (c) Contraction of the trigonal mus- mon and of severe grade in boys. VUR is culature anchors the ureter and prevents the most commonly found in patients with Cauca- lateral displacement of the ureteric orifice. All sian racial background and is rare in those of other statements are true. Asian and African ancestry. There is an increased incidence of VUR among siblings 2. The “ flap-valve” mechanism of the UVJ is as well as children of patients with VUR, reinforced by the following except : indicating its heritable nature. (a) An adequate length of the intramural 4. The most commonly found voiding dysfunc- ureter and submucosal tunnel tion in older girls with VUR is: (b) Sturdy back wall support of the detrusor muscle (a) Detrusor instability with detrusor sphinc- (c) Well-integrated ureter-trigone muscle ter dyssynergia complex (b) Detrusor areflexia (d) Paraureteral diverticulum (c) High-pressure voiding (d) High postvoid residual Answer: (d) Paraureteral diverticulum lacks an adequate detrusor muscle support and Answer: (a) Detrusor areflexia is possible but causes the intramural ureter to displace extra- uncommon. High-pressure voiding is most fre- vesically during bladder filling. Consequently, quently seen in infant boys with VUR. High there is a shortening of the intramural ureter postvoid residual can be seen but is not the and a loss of flap-valve mechanism. most common voiding dysfunction. 130 Clinical Pediatric Urology Study Guide

5. The appropriate clinical management of VUR mechanism, bladder dysfunction, predisposi- in children must include consideration of: tion to UTI, and renal anomalies. (a) Anatomic UVJ competency 6. The secondary VUR may be caused by the (b) Potential bladder dysfunction following entities except : (c) Predisposition to urinary tract infection (UTI) (a) Posterior urethral valves (d) Underlying renal dysplasia (b) Neurogenic bladder dysfunction (e) All of the above (c) Non-neurogenic bladder dysfunction (d) Primary nocturnal enuresis Answer: (e) The distinction between primary and secondary VUR is important in the con- Answer: (d) Primary VUR is thought to ceptualization of contributing pathophysiol- represent an abnormal anatomy and function ogy and relevant anatomic/functional etiologies of the UVJ, whereas secondary VUR implies which need to be addressed clinically. In all an acquired condition as a result of increased patients presenting with VUR, the proper intravesical pressure in the setting of neuro- management must include a global view of genic bladder dysfunction, non-neurogenic the entire urinary tract, including UVJ bladder dysfunction, or outlet obstruction. Nonsurgical management of 43 vesicoureteral reflux

Jack S Elder

1. A 4-year-old girl with left grade III vesi- item is scored from 0 to 3; 0 is almost never, coureteral reflux (VUR) develops a urinary 1 is less than half the time, 2 is about half the tract infection (UTI) with left flank pain and time, and 3 is almost every time. A higher a temperature of 103° F. A DMSA scan shows score is indicative of more severe voiding acute pyelonephritis: dysfunction.2 (a) I n < 5 % of cases (b) I n 15– 20 % of cases 3. In the DVSS which of the following compo- (c) I n 40– 50 % of cases nents yields a low score: (d) M ost often in the presence of voiding (a) F low rate > 18 ml/s dysfunction (b) P ostvoid residual urine volume < 10% (e) O nly if treatment is delayed for 4– 7 days predicted maximum bladder volume Answer: (b) Children with reflux who are (c) A bsence of uninhibited bladder contrac- receiving antibiotic prophylaxis are at risk for tions on urodynamic studies acute pyelonephritis. According to Szlyk et al, (d) B owel movement once per day the risk is 17% , irrespective of age, gender, (e) R eflux grade II or less 1 and presence of preexisting renal scarring. Answer: (d) The DVSS includes ten items pertaining to voiding and bowel habits; seven 2. In the dysfunctional voiding scoring system items pertain to bladder function, two pertain (DVSS), which of the following is not to bowel function, and one pertains to a stress- included: ful event related to their voiding pattern. Each (a) W hen I pee it hurts item is scored from 0 to 3; 0 is almost never, (b) I wet the bed at night 1 is less than half the time, 2 is about half the (c) I have to push to pee time, and 3 is almost every time. A higher (d) W hen I wee myself, my underwear is score is indicative of more severe voiding soaked dysfunction. It does not include urodynamic 2 (e) I only go to the bathroom one or two data. times per day 4. When administered before a voiding cystoure- Answer: (b) The DVSS includes ten items thrography (VCUG), oral midazolam often pertaining to voiding and bowel habits; seven causes: items pertain to bladder function, two pertain to bowel function, and one pertains to a stress- (a) A nterograde amnesia ful event related to their voiding pattern. Each (b) S edation 132 Clinical Pediatric Urology Study Guide

(c) O xygen desaturation (d) 8 0 % (d) H ypotension (e) N early 100% (e) I mpaired voiding Answer: (c) In the International Reflux Study, Answer: (a) Oral midazolam causes anxioly- at 5 years, only 10 % of children with grade sis and anterograde amnesia, but not significant III – IV reflux randomized to medical therapy sedation. Although children are monitored, showed spontaneous resolution. However, at changes in oxygen saturation or vital signs are 10 years, 39% had shown spontaneous resolu- uncommon. It does not inhibit voiding to any tion. In contrast, with unilateral grade III– IV significant degree. Unlike propofol, it does VUR, 73% had shown spontaneous resolu- not cause significant sedation.3 tion, and in children with grade III– IV on one side and grade I– II on the other, 49% had 5 5. A 5-year-old girl develops a febrile UTI. The resolved. renal sonogram is normal. A VCUG shows bilateral grade II VUR. If medical manage- 7. In children, compared with sulfamethoxazole- ment is recommended, 2 years after diagnosis, trimethoprim, which of the following side- the risk of persistent reflux is approximately: effects of nitrofurantoin is more common: (a) S tevens-Johnson syndrome (a) 20% (b) H ives (b) 40 % (c) P hotosensitivity (c) 60 % (d) H epatotoxicity (d) 8 0 % (e) H emolytic anemia (e) H igher than if there was only unilateral reflux Answer: (e) Photosensitivity and delayed allergic reaction are the most common side- Answer: (c) According to the data generated effects associated with treatment with sulfame- by the AUA Pediatric Reflux Guidelines panel, thoxazole-trimethoprim. Other less common in grade I and II VUR, the spontaneous reso- side-effects include leucopenia, hepatotoxicity, lution rate is similar for unilateral and bilateral and, in rare cases, Stevens-Johnson syndrome. reflux, and does not vary with age. With Side-effects with nitrofurantoin also are com- grade II reflux, 80% show reflux resolution mon and include hepatotoxicity, cutaneous at 5 years. The likelihood of spontaneous reso- reaction, pulmonary fibrosis, and hemolytic lution is fairly constant each year, although in anemia in patients who have glucose-6- the first few years the resolution rate is slightly phosphate dehydrogenase deficiency. Nausea higher.4 is far more common with the nitrofurantoin suspension, but less common with macro- 6. A 2-year-old girl develops a febrile urinary crystals.6 tract infection. A DMSA scan shows renal scarring in the left kidney. A VCUG demon- 8. In children with an overactive bladder, treat- strates grade III VUR on the right and grade ment with anticholinergic medication is asso- IV VUR on the right. If long-term medical ciated with all of the following except: management is chosen, the likelihood of per- (a) X erostomia sistent reflux at 10 years is approximately: (b) B lurred vision (a) 2 0 % (c) H igher resolution rate of VUR (b) 4 0 % (d) D iarrhea (c) 6 0 % (e) D rowsiness Nonsurgical management of VUR 133

Answer: (d) Typical side-effects of anticho- associated with the highest likelihood of linergics include xerostomia (dry mouth), spontaneous reflux resolution? facial flushing, reduced sweating, constipa- tion, and less commonly, blurred vision and (a) U nilateral grade III VUR, diagnosed at drowsiness. Side-effects are dose related and 4 years of age may be eliminated with dose reduction. (b) U nilateral grade IV VUR, diagnosed at Alteration in mental function has not been 2 years of age observed. (c) B ilateral grade II VUR, diagnosed at 6 years of age (d) B ilateral grade III VUR, diagnosed at 9. In children with dysfunctional voiding and 1 year of age VUR, biofeedback therapy: (e) B ilateral grade IV VUR, diagnosed at (a) S hould be supplemented with anticholi- 1 year of age nergic therapy Answer: (c) According to the AUA Practice (b) I s associated with increased likelihood of Guidelines, reflux resolution for grades I and reflux resolution II VUR is independent of the age of the (c) S hould be supplemented with laxative patient and laterality (unilateral versus bilat- therapy eral). In contrast, with grade III VUR, the (d) S hould not be used if the postvoid residual likelihood of reflux resolution is lower with urine volume is > 15 % of the child' s higher age at diagnosis and for bilateral versus expected bladder capacity unilateral VUR. For grade IV VUR there are (e) I s usually unsuccessful in children with insufficient patient numbers to draw any day and night incontinence conclusions regarding age at diagnosis and Answer: (b) In children with dysfunctional laterality. At 5 years, the likelihood of reflux voiding, typically there is inconsistent relax- resolution for unilateral grade III VUR diag- ation of the external sphincter during voiding. nosed at 4 years of age is 50 % , for unilateral This pattern may occur because of an overac- grade IV VUR diagnosed at 2 years of age is tive bladder which causes the child to tighten 50 % , for bilateral grade II VUR diagnosed at his or her sphincter during a detrusor contrac- any age is 80% , for bilateral grade III VUR tion. A typical treatment session might include diagnosed at 1 year of age is 60% , and for a uroflow study, sonographic evaluation of bilateral grade IV VUR diagnosed at any age postvoid residual urine volume, and a 1 h is 10% .4 session with perineal patch electrodes for elec- tromyography. Often, this treatment is used instead of anticholinergic therapy or in chil- dren who fail that treatment. However, some children need supplemental anticholinergic References therapy. In addition, in children with signifi- 1. Szlyk GR , Williams SB , Majd M , Belman AB , Rushton cant constipation, oral laxative therapy is HG . Incidence of new renal parenchymal inflammatory important. It is associated with an increased changes following breakthough urinary tract infection likelihood of reflux resolution.7 in patients with vesicoureteral reflux treated with anti- biotic prophylaxis: evaluation by 99MTechnetium dimercapto-succinic acid renal scan. J Urol 2003; 170 : 10. According to the AUA Practice Guidelines 1566 – 8 . report, in a child with reflux diagnosed 2. Farhat W , Bagli DJ , Capolicchio G et al. The dysfunctio- following a UTI, which of the following is nal voiding scoring system: quantitative standardization 134 Clinical Pediatric Urology Study Guide

of dysfunctional voiding symptoms in children. J Urol 5. Smellie JM , Jodal U , Lax H et al. Outcome at 10 years 2000; 164 : 1011 – 15 . of severe vesicoureteric reflux managed medically: 3. Elder JS , Longenecker R . Premedication with oral report of the International Reflux Study in Children. midazolam for voiding cystourethrography in children: J Pediatr 2001; 139 : 656 – 63 . safety and efficacy. Am J Roentgenol 1995; 164 : 6. Karpman E , Kurzrock EA . Adverse reactions of nitro- 1229 – 332 . furantoin, trimethoprim, and sulfamethoxazole in 4. Elder JS , Peters CA , Arant BS Jr et al. Pediatric Vesi- children. J Urol 2004; 172 : 448 – 53 . coureteral Reflux Panel Guidelines summary report on 7. Palmer LS , Franco I , Rotario P et al. Biofeedback the management of vesicoureteral reflux in children. therapy expedites the resolution of reflux in older J Urol 1997; 157 : 1846 – 51 . children. J Urol 2002; 168 : 1699 – 702 . Surgery for vesicoureteral reflux 44 John W Brock III and Romano T DeMarco

1. Which of the following statements regarding (d) Can be performed extravesically the surgical correction of vesicoureteral reflux (e) Is only performed in children with nondi- (VUR) is correct? lating reflux (a) All patients require an indwelling urethral Answer: (c) The original Politano-Leadbetter catheter following surgery technique described in 1958 involved the (b) Endoscopic subureteric injection is the intravesical blind creation of a new suprahiatal current surgical “ gold standard” for muscular location for the ureter. Modifica- treatment tions of Politano and Leadbetter' s original (c) Persistent ureteral obstruction is common description were made to lengthen the sub- following surgical correction mucosal ureteral tunnel and prevent inadver- (d) New contralateral VUR following unilat- tent injury to adjacent structures during the eral neocystostomy typically requires sur- transfer of the ureter to its new suprahiatal gical correction location. This versatile technique is success- (e) Open surgery has a 95% or greater fully used to correct all grades of reflux. success rate 3. The major drawback related to the cross- Answer: (e) Open ureteral reimplantation trigonal repair described by Cohen is? surgery is highly successful, with reported success rates of ≥ 95 % independent of tech- (a) The high rate of postoperative VUR nique. While newer, less invasive, and novel (b) The extensive ureteral mobilization alternatives to antireflux surgery exist, open required in this technique leading to surgical repair of VUR is the only technique devascularization of the ureter with a well-documented durable response. (c) The difficult intubation of the ureteral Routine urethral catheterization after surgery orifices encountered following surgery is not required, and may prolong convales- (d) The substantial risk of postoperative cence and delay anticipated discharge in some urinary retention patients. Ureteral obstruction has been (e) The inability to create an adequate reported to occur in about 1% of patients after intravesical tunnel surgery. New-onset contralateral reflux is typi- Answer: (c) The cross-trigonal repair des- cally low grade and usually resolves sponta- cribed by Cohen is arguably today' s most neously with good voiding habits. popular open technique for ureteral reimplan- tation. The success rate using this approach is 2. The modified Politano-Leadbetter repair: close to 99% in those without dilating (a) Involves laparoscopic mobilization of the reflux. This method requires less ureteral ureter mobilization than the Politano-Leadbetter (b) Was originally described 10 years ago repair, as the submucosal tunnel is created (c) Was described to decrease complications along the posterior bladder wall without related to the original procedure transferring the ureter to a new muscular 136 Clinical Pediatric Urology Study Guide

hiatus, virtually eliminating kinking of the dis- (b) The Gil-Vernet advancement repair tal ureter. The major drawback to this opera- is a good option for any patient with tion is the subsequent difficult intubation of VUR the ureteral orifices related to their transtrigo- (c) Routine voiding cystourethrography nal location. Transient postoperative urinary (VCUG) is routinely performed in all retention is occasionally encountered in chil- patients following reimplant surgery dren following bilateral extravesical ureteral (d) Open surgery is not as durable a repair as reimplantation. endoscopic subureteric injection for the treatment of VUR 4. The Glenn-Anderson repair: (e) Patients following bilateral extravesical ureteral reimplantation are at risk for devel- (a) Is well suited for those patients with oping postoperative urinary retention laterally displaced ureters (b) Is similar to the transtrigonal technique Answer: (e) An extravesical approach to the (c) Can be performed in an intra- or extra- correction of VUR was almost simultaneously vesical fashion described in the USA and in Europe in the (d) Has a high failure rate early 1960s. Following bilateral extravesical (e) Was the first method described to correct reimplantation, transient postoperative blad- VUR der retention has been reported and a voiding trial prior to discharge is recommended. A Answer: (a) The Glenn-Anderson intravesical simple advancement technique for those ureteral advancement technique is highly suc- patients with lateral ectopia and a widened cessful in the select patient with reflux. Those “ megatrigone ” was described by Gil-Vernet in patients with laterally displaced ureters and a 1984 and is used in very select patients with sufficient posterior bladder wall are ideal can- reflux. Because of the high rate of success fol- didates for this advancement procedure and lowing open ureteral reimplantation surgery, success rates in these properly selected patients routine postoperative voiding studies are not rival other open techniques. necessary when the surgery is performed by an experienced surgeon. In contrast to uretero- 5. Which of the following statements is correct? neocystostomy, questions with regard to the (a) The Lich-Gregoir technique was initially durability of endoscopic subureteric injection described by two Russian surgeons persist. Injection therapy for 46 vesicoureteral reflux

Michael A Poch and Anthony A Caldamone

1. Proper technique of injection therapy includes collagen as 3% of patients will have a hyper- all of the following except: sensitivity reaction. However, a negative result does not completely eliminate the risk of an (a) Bladder distended to ¾ capacity allergic reaction. Leonard et al describe a small (b) Needle bevel up at the 6 o' clock position subset of patients who developed antibovine (c) Monitor for bulging after initially inject- collagen antibodies who had negative preinjec- ing 1– 2 ml tion skin testing. While those patients became (d) The use of multiple injection sites seropositive the clinical relevance is unclear. In Answer: (d) Proper technique for the stan- only one of those patients a local reaction occur- dard subureteric transurethral injection red which may have been immunogenic.2 (STING) therapy involves the following steps: (1) emptying the bladder to ¾ capacity so as 3. The mechanism of long-term sustained not to displace the ureter laterally; (2) place- efficacy of dextranomer/hyaluronic acid is: ment of the needle bevel up into the posterior (a) Giant cell reaction and granuloma portion of the ureter; (3) visualization of the formation creation of the mound with initial injection of (b) Fibrous encapsulation 0.1 – 0.2 ml of material. Multiple injection sites (c) Fibroblast migration and collagen implies there was difficulty in obtaining a ingrowth proper mound in the correct plane, as well as (d) Rapid absorption and replacement with increasing the possibility of leakage of inject- calcium carbonate able material. Kirsch et al reported improved results by modifying the standard technique Answer: (c) Dextranomer/hyaluronic acid by performing hydrodistention of the ureteral is a dual component material. The dextrano- orifice and injection into the submucosa of the mer is cross-linked dextran microspheres ureteral tunnel.1 80 – 250 µ m in diameter in a gel of hyaluronic acid. The dextranomer, the main bulking 2. Which of the following injectable materials agent, is slowly degraded by hydrolysis. The requires preinjection skin testing: hyaluronic acid represents the transport medium and slowly dissipates over 12 weeks. (a) Polytetrafluoroethylene (PTFE) Volume stability over time results from (b) Dextranomer/hyaluronic acid fibroblast migration and collagen ingrowth (c) Cross-linked bovine collagen into the hyaluronic acid matrix between the (d) Polymethysiloxane dextranomer microspheres. Stenberg et al Answer: (c) Preinjection skin testing is found that histologic samples taken at the time necessary for the use of cross-linked bovine of open reimplantation after failed injection 138 Clinical Pediatric Urology Study Guide

therapy revealed granulomatous reaction of (c) Requires the same amount of injected the giant cell type, inflammatory cell infiltra- material as nonreimplanted ureters tion and implant pseudoencapsulation.3 (d) Should be performed with a full bladder

4. A 5-year-old female with spina bifida has a Answer: (c) Endoscopic correction of reflux poorly compliant bladder with stable hydro- after failed reimplantation has been shown to nephrosis and bilateral grade III vesicoure- be effective, although less so than for the teral reflux (VUR). Her bladder management nonreimplanted ureters. Perez-Brayfield et al includes intermittent catheterization and anti- describe a cohort of complex patients with cholinergic therapy. Which of the following is reflux after failed open reimplantation, in neu- true regarding the endoscopic correction of rogenic bladders and in duplicated systems. reflux in this patient: They describe endoscopic correction success rates of 88% , 78 % and 74% , respectively. (a) It has no role in the management of reflux Despite the potential for challenging anatomy, in patients with neuropathic bladders the total volume of dextranomer/hyaluronic (b) Its efficacy is equal to that of patients with acid used and technique performed was simi- nonneuropathic bladders lar to patients with uncomplicated reflux. (c) Postinjection obstruction is more com- Ectopic ureters with orifices at the bladder mon than with nonneuropathic bladders neck posed a very challenging problem with (d) Bladder management should continue success rates of only 14% . 4 after injection therapy Answer: (d) Injection therapy in those patients with neuropathic bladders has been 6. A 6-year-old boy undergoes unilateral shown to be effective. Perez-Brayfield et al endoscopic injection therapy for grade III describe a success rate of 78% . Neuropathic reflux. Excluding treatment failure, the bladder and injection therapy present a certain most common complication of endoscopic set of challenges. Neuropathic bladders often injection is: have severe trabeculation and fibrosis, and (a) Development of contralateral reflux placement of the injectable material in the cor- (b) Postprocedure cystitis rect position and with adequate mound con- (c) Ureteral obstruction figuration is often more technically challenging. (d) Recurrent hematuria In addition, due to high voiding pressures, neuropathic bladders are at increased risk for Answer: (a) The complications of injection mound shifting. These patients should be therapy include failure, obstruction, and devel- maintained on a bladder-management regi- opment of contralateral reflux. Elmore et al men including anticholinergic medication and and Menezes et al both reported a 10% inci- clean intermittent catheterization in order to dence of new contralateral reflux, with females keep bladder pressures low.4 under 5 years of age being at most significant risk.5 , 6 The rate of obstruction for all injectable 5. A 4-year-old female is found to have recurrent materials is < 1 % . Puri et al described the inci- reflux after open ureteral reimplantation. Her dence of obstruction requiring open reimplan- parents are interested in the least invasive tation of PTFE to be 0.33% . 7 Vandersteen treatment options. Endoscopic injection in et al reported similar results for dextranomer/ this scenario: hyaluronic acid with an obstruction rate (a) Is contraindicated of 0.6% per ureter.8 The development of (b) Has the same efficacy compared to the recurrent hematuria has not been described in nonreimplanted ureters any study. Injection therapy for VUR 139

7. Reasons for failure of injection therapy include (c) Cross-trigonal all of the following except: (d) Proximally into the ureter

(a) Technical difficulty with the injection Answer: (b) The most common direction (b) Material properties of mound shift is medially and caudally (c) Presence of urinary tract infection (UTI) towards the bladder neck. Studies have (d) Bladder dynamics shown that autologous chondrocytes, PTFE, Answer: (c) Many factors can affect the effi- and dextranomer/hyaluronic acid all shift cacy of injection therapy. Technical difficulties toward the bladder neck if shifting occurs. due to challenging ureteric anatomy by the Diamond et al describe distally shifted presence of duplication, bladder trabeculation mound in 75% of their technical failures.10 or diverticula are the most common. Material The mechanism of shifting is thought to be due properties and volume injected also play a role to the bladder contraction creating pressure in regard to migration and absorption. Addi- gradient toward the bladder neck. Capozza tionally, patients with abnormal bladder et al found an association between mound dynamics and increased voiding pressures are shift and those patients with voiding dysfunc- at increased risk for mound shifting. While the tion symptoms. Twenty-five of 27 patients rates of cystitis and febrile UTIs following with treatment failure had voiding dysfunc- endoscopic injection are 6% and 0.75% , tion symptoms including frequency, urgency, respectively, there is currently no evidence and incontinence.11 at this time relating the presence of UTI to treatment failure.9 10. Concern for PTFE particle migration after 8. A 6-year-old female undergoes cystoscopy and injection therapy is related to particles being open reimplantation after failed injection less than: therapy for grade IV reflux. The most likely µ finding of the ureteral orifice at the time of (a) 50 m µ cystoscopy is: (b) 500 m (c) 80 µ m (a) Shifting of mound (d) 8 µ m (b) Volume loss (c) Shifting and volume loss Answer: (c) PTFE is a chemically inert (d) Normal appearance at the ureteral orifice substance with particle size ranging from Answer: (a) In a study by Diamond et al, of 4 to 100 µ m; 90 % of particles are < 40 µ m. a total of 57 ureters, the most common find- Concern for migration of PTFE is based ing in patients with failed injection of autolo- upon the fact that macrophages are able to gous chondrocytes was mound shifting (35% ), engulf particles < 60 µ m. Once engulfed, followed by shifting and volume loss (25% ), PTFE is unable to be degraded, resulting in isolated volume loss (13% ), and normal cell death. As the macrophage dies the appearing mound (11% ). Similar technical PTFE particles are freed and potentially failures have been described in collagen, PTFE, initiate the formation of a granuloma. Addi- and dextranomer/hyaluronic acid.10 tionally, those particles can theoretically migrate to other organs, including lymph 9. The most common direction for mound nodes, lung and brain tissue. Therefore, the shifting is: optimal size of particles is larger than that (a) Laterally and superiorly which can be engulfed by macrophages, so the (b) Toward the bladder neck ideal size is > 80 µ m. 140 Clinical Pediatric Urology Study Guide

References 6. Menezes M , Mohanan N , Haroun J et al. New contrateral vesicoureteral reflux after endoscopic correction of unilateral reflux – is routine contralateral 1. Kirsch AJ , Perez-Brayfield M , Smith EA , Scherz HC . injection indicated at initial treatment? J Urol 2007; The modified STING procedure to correct vesi- 178 : 1711 – 13 . coureteral reflux: improved results with submucosal 7. Puri P , Granata C . Multicenter survey of endoscopic implantation with the intramural ureter. J Urol 2004; treatment of vesicoureteral reflux using polytetrafluoro- 171 : 2413 – 16 . ethylene. J Urol 1998; 160 : 1007 – 11 ; discussion 1038. 2. Leonard MP , Decter A , Hills K , Mix LW . Endoscopic 8. Vandersteen DR , Routh JC , Kirsch AJ et al. Post- subureteral collagen injection: are immunological con- operative ureteral obstruction after subureteral injection cerns justified? J Urol 1998; 160 : 1012 – 16 . of dextranomer/hyaluronic acid copolymer. J Urol 3. Stenberg A , Larsson E , Lackgren G . Endoscopic 2006; 176 : 1593 – 5 . treatment with dextranomer/hyaluronic acid for vesi- 9. Elder JS , Diaz M , Caldamone AA et al. Endoscopic coureteral reflux: histological findings. J Urol 2003 ; therapy for vesicoureteral reflux: a meta-analysis. I. 169 : 1109 – 13 . Reflux resolution and urinary tract infection. J Urol 4. Perez-Brayfield M , Kirsch AJ , Hensle TW et al. 2006; 175 : 716 – 22 . Endoscopic treatment with dextranomer/hyaluronic 10. Diamond DA , Caldamone AA , Bauer SB , Retik AB . acid for complex cases of vesicoureteral reflux. J Urol Mechanisms of failure of endoscopic treatment of 2004; 172 : 1614 – 16 . vesicoureteral reflux based on endoscopic anatomy. 5. Elmore JM , Kirsch AJ , Lyles RH et al. New contra- J Urol 2003; 17 : 1556 – 8 ; discussion 1559. lateral vesicoureteral reflux following dextranomer/ 11. Capozza N , Lais A , Matarazzo E et al. Influence of hyaluronic acid implantation: incidence and identifica- voiding dysfunction on the outcome of endoscopic tion of a high risk group. J Urol 2006 ; 175 : 197 – 100 ; treatment for vesicoureteral reflux. J Urol 2002 ; 168 : discussion 1100– 1 . 1695 – 8 . Basic science of the 47 urinary bladder

Scott R Manson (Based on chapter written by Armando J Lorenzo and Darius J Bä gli)

1. Normal function of the urinary bladder can developing cells with the extracellular matrix, best be described as: other cells, and the presence of appropriate mechanical stimulation which combine to (a) An active neuromuscular process promote the differentiation of developing (b) A passive function to accommodate filling smooth muscle cells. of the bladder (c) Dynamically responsive to changing 3. Contraction of the bladder smooth muscle physical and mechanical conditions layer during filling and emptying of the blad- (d) All of the above der is primarily mediated by ______upon Answer: (d) The urinary bladder is a unique stimulation by ______whose concentration organ that demonstrates a remarkable integra- gradients are dependent upon ______levels: tion of neuromuscular, mechanical, and physi- (a) Actin– myosin; sodium; adenosine triphos- cal properties to perform its function. Bladder phate (ATP) contraction is largely an active neuromuscular (b) Actin– myosin; calcium; ATP process. Bladder filling is thought to represent (c) Microtubules; calcium; ATP a largely passive phenomenon which can also (d) Microtubules; calcium; guanine triphos- be modulated by changes in the neuromuscu- phate (GTP) lar properties of the bladder in order to main- tain low intravesical pressure. Disruption of Answer: (b) In regulating the filling and the molecular mechanisms underlying any of emptying of the bladder, contraction of the these normal bladder functions places both bladder smooth muscle cell is mainly the result continence and renal function at risk. of contractile forces generated by actin and myosin following phosphorylation of myosin 2. Development of the bladder smooth muscle light chain by calcium-dependent kinases. Cal- layer from the mesenchyme of the urogenital cium is maintained in intracellular stores by sinus is dependent upon: ATP-dependent calcium pumps; upon neuro- stimulation, calcium is released from these (a) Cell – cell interactions stores to stimulate the contractile apparatus. (b) Cell – matrix interactions (c) Mechanical stimulation 4. Mechanotransduction can best be described as: (d) All of the above (a) The process by which neurostimulation Answer: (d) Normal prenatal bladder devel- is transduced into mechanical forces in opment results from a complex interaction of muscle contraction 142 Clinical Pediatric Urology Study Guide

(b) The process by which physical forces are 6. The primary neurotransmitter receptors converted into biochemical signals and described in activation of contractions of the then integrated into a cellular response detrusor muscle are: (c) A process involved in stimulating hyper- (a) The muscarinic receptors trophy and hyperplasia in the bladder in (b) The purinergic receptors response to an increased mechanical load (c) The adrenergic receptors on the bladder wall (d) None of the above (d) (b) and (c) (e) All of the above Answer: (a) Innervation of the detrusor Answer: (d) The process of converting muscle is primarily controlled by the muscar- physical forces into biochemical signals and inic receptors. However, both purinergic and integrating these signals into a cellular response adrenergic receptors have been described is known as mechanotransduction. This is an to play a role in innervation of the human important concept in bladder development bladder, particularly in developmental and and in the response of the bladder to numer- pathologic states. ous physiologic and pathologic stimuli. 7. As knowledge of the basic science of the 5. Which of the following processes may be bladder increases, we can expect: involved in the remodeling of the bladder (a) The identification of potential pharmaco- in response to physiologic and pathologic logic targets for the treatment of bladder stimuli? diseases (a) Changes in the degradation of extracel- (b) An improvement in the early diagnosis of ullar matrix proteins bladder diseases (b) Changes in the synthesis of extracellular (c) An improvement in the development matrix proteins of tissue engineering approaches to the (c) Changes in the behavior of bladder smooth treatment of bladder diseases muscle cells as a result of the altered com- (d) An increasing awareness for the complex- position in extracellular matrix proteins ity of the biology of the bladder (d) (a) and (b) (e) All of the above (e) All of the above Answer: (e) Recent times have witnessed Answer: (e) As the bladder develops, or a changing view of the bladder from a adapts to different stimuli, changes in both simple organ to a complex biological system matrix production and degradation result in that is responsive to physiologic and remodeling of the extracellular matrix in the pathologic stimuli. Basic science research of bladder. Since the behavior of bladder smooth the underlying molecular signaling cascades muscle cells is closely tied to their physico- which regulate bladder physiology has already mechanical environment, these changes may led to a better understanding of the bladder result in further changes in the behavior of and new potential modes of treatment, and bladder smooth muscle cells (i.e. proliferation, this should continue with further study of the differentiation). bladder. Basic science of prostatic 48 development

Ellen S Shapiro

1. The endodermal urogenital sinus is derived 8th week of gestation.3 Unlike development from the: of the Wolffian duct (WD) derivatives, which are dependent solely on testosterone, the (a) Allantois differentiation of the UGS is dependent (b) Vitelline duct on the 5α reduced form of testosterone, (c) Terminal hindgut dihydrotestosterone (DHT). DHT is essential (d) Cloacal membrane for the growth and development of the (e) Ureteral bud .3– 6 By 10 weeks, the prostatic ductal Answer: (c) The prostate develops from the network develops from solid epithelial out- endodermal urogenital sinus (UGS) which is growths, or prostatic buds, which evaginate derived from the terminal end of the hindgut from the endodermal UGS immediately or “cloaca”, which is Latin for sewer. Septa- below the bladder and penetrate into the tion of the cloaca by the urorectal septum surrounding urogenital mesenchyme (UGM).5 begins at about 28 days of gestation.1 The The prostatic ducts rapidly lengthen, arborize rectum and primitive UGS form by the and canalize. By 13 weeks, 70 primary 44th day. The primitive UGS proximal to ducts are present and exhibit secretory the mesonephric duct develops into the vesi- cytodifferentiation.7 courethral canal, whereas the region distal of the mesonephric duct becomes the definitive 3. Which of the following is not characteristic of UGS. The UGS adjacent to the bladder 5α-reductase deficiency syndrome: (pelvic urethra) differentiates into the lower (a) Normal vas deferens portion of the prostatic and membranous (b) Persistent Müllerian ducts urethra.2 (c) Severe penoscrotal hypospadias (d) A vaginal pouch 2. Testosterone production by the fetal testis (e) Normal testes peaks at: Answer: (b) The 5α-reductase deficiency (a) 8 weeks syndrome is a form of autosomal recessive (b) 10 weeks male pseudohermaphroditism characterized (c) 13 weeks by severe penoscrotal hypospadias, a blind (d) 16 weeks vaginal pouch, and normal testes with normal (e) 18 weeks epididymes, vasa deferentia, and seminal Answer: (c) Prostate growth and develop- vesicles.8 The ejaculatory ducts terminate in ment are dependent on androgen production the blind-ending vagina, and the prostate is by the fetal testes, which begins at about the small or undetectable. 144 Clinical Pediatric Urology Study Guide

4. In the fetal prostate, 5α-reductase expression organization of prostatic SM.11 Experiments is never observed in the: combined adult prostatic epithelium (PRE) with UGM or seminal vesicle mesenchyme (a) Urogenital epithelium (SVM) or BLE with UGM or SVM. Prostatic (b) Prostatic buds ducts developed in all tissue recombinants (c) Posterior lobe ductal epithelium when UGM was used with either epithelium. (d) Ejaculatory duct stroma SM cells also organized into sheets resembling (e) Ventral stroma prostate. When SVM was combined with Answer: (c) Shapiro et al have shown andro- either epithelium, the prostatic ducts were sur- gen receptor expression in the stroma and rounded by thick SM cells resembling seminal luminal epithelium of the UGS of the devel- vesicle. The SM was unorganized in grafts of oping human prostate as early as 7 weeks, SVM or UGM. These experiments suggest while 5α-reductase expression is observed that male UGM dictates spatial organization, only in the prostatic stroma at 7 weeks and in but SM differentiation is induced by epithe- the basal layer of the UGS epithelium at lium. Urothelium may also direct the organi- 9 weeks.9 Regional 5α-reductase expression zation of SM tissue, since urothelium is thought varies from 7 to 16 weeks, with increasing to be a potent inducer of SM differentiation. expression observed in a caudal to cranial Cunha12 has shown that the proximal segments direction at every gestational age. Diminished of prostatic ducts near the urethra express expression occurs after 18–20 weeks. The urothelial membrane antigen and have associ- most posterolateral stroma and the ductal ated thick layers of SM cells surrounding them. epithelium in the posterior prostatic lobe All of these tissue recombinant experiments never express 5α-reductase. The lateral lobes show that interactions between the mesen- show distal ductal epithelial expression, while chyme and epithelium are reciprocal. the ducts emanating from above the ejacula- tory ducts (EDs) express 5α-reductase in their proximal and distal ductal epithelium. These References localization studies support the notion that DHT serves largely as a signal amplification 1. Stephens FD . Congenital Malformation of the Urinary Tract. New York: Praeger Publishers, 1993. for prostatic development and is directly 2. Hamilton WJ , Mossman HW . The urogenital system. associated with the distance the target tissue is In: Human Embryology: Prenatal Development of from the testicular androgen source.9 , 10 Form and Function, 4th ed. New York: Macmillan , 1976; 201 . 5. When tissue recombination experiments 3. Siiteri PK , Wilson JD . Testosterone formation and combine adult bladder epithelium (BLE) with metabolism during male sexual differentiation in male urogenital mesenchyme, the following the human embryo. J Clin Endocrinol Metab 1974 ; 38 : 113 . structure forms: 4. Cunha GR . Epithelio-mesenchymal interactions in pri- (a) Vagina mordial gland structures which become responsive to (b) Prostatic ducts androgenic stimulation. Anat Rec 1972 ; 172: 179. 5. Imperato-McGinley J , Binienda Z , Arthur A et al. (c) Seminal vesicle The development of a male pseudohermaphroditic (d) Smooth muscle rat using an inhibitor of the enzyme 5α-reductase. (e) Ejaculatory ducts Endocrinology 1985; 116 : 807 . 6. Raghow S , Shapiro E , Steiner MS . Immunohis- Answer: (b) Tissue recombinant experiments tochemical localization of TGF-alpha and TGF-beta have been used to examine epithelial–mesen- during early human fetal prostate development. chymal interactions on the differentiation and J Urol 1999; 162 : 509 – 513 . Prostatic development 145

7. Lowsley OS . The development of the human 10. Mahendroo MS , Cala KM , Hess DL , Russell DW . prostate gland with reference to the development Unexpected virilization in male mice lacking steroid of other structures at the neck of the urinary bladder. 5α-reductase enzymes. Endocrinology 2001 ; 142 : Am J Anat 1912; 13 : 299 – 346 . 4652– 62 . 8. Imperato-McGinley J , Guerrero L , Gautieer T 11. Cunha GR , Battle E , Young P et al. Role of epithelial- et al. Steroid 5α-reductase deficiency in man. An mesenchymal interaction in the differentiation and inherited form of pseudohermaphroditism. Science spatial organization of visceral smooth muscle. Epith 1974; 186 : 1213. Cell Biol 1992; 1 : 76 – 83 . 9. Shapiro E , Tang R , Wang B et al. The expression of 12. Cunha GR , Donjacour AA , Cooke PS et al. The endo- 5α-reductase and the androgen receptor in the human crinology and developmental biology of the prostate. fetal prostate. J Urol 1996; 155 : 534 (Abstract). Endocrine Reviews 1987; 8 : 388 .

Embryology of the anterior 49 abdominal wall, bladder, and proximal urethra

Steven E Lerman , Irene M McAleer , and George W Kaplan

1. The abdominal wall is formed from: by rupture of or failure of closure of the lower abdominal wall. (Ch 49, page 734) (a) Concentric closure of the somatopleure (b) Entoderm replacing mesoderm 3. The cloaca arises from ___ during the ____ (c) Mesoderm from the notochord week(s) of gestation: (d) Ectoderm (e) Three sheets of myeloderm (a) Endoderm; 5th– 7th (b) Endoderm; first 3 Answer: (a) During the first 3 weeks of (c) Mesoderm; first 3 embryogenesis the anterior abdominal wall is (d) Mesoderm; 5th– 7th represented by the somatopleure, which forms (e) None of the above the ventral surface of the embryonic disk. The somatopleure closes concentrically around the Answer: (b) The cloaca arises as a caudal umbilical ring and is invaded by mesoderm, expansion of the endodermal hindgut during which splits into three muscle layers. (Ch 49, the first 3 weeks. (Ch 49, page 735) page 733) 4. Separation of the cloaca into separate urinary 2. Problems in embryogenesis of the abdominal and fecal tracts results in: wall result in which of the following (a) The trigone problems: (b) Ejaculatory ducts (a) Eagle-Barrett syndrome (c) Cowper ' s glands (b) Exstrophy (d) Rathke ' s folds (c) Cloacal exstrophy (e) The perineum (d) (a), (b) and (c) Answer: (e) Rathke ' s folds are involved in (e) None of the above the separation of the urogenital and fecal Answer: (d) All three problems are consid- tracts. The trigone arises from the urogenital ered to arise because of problems with the sinus as do the ejaculatory ducts. Cowper' s abdominal wall. The Eagle-Barrett syndrome glands arise in the bulbous urethra. The includes disorganized muscle, presumably perineum forms after the cloacal membrane because of faulty mesenchyme. Exstrophy and ruptures and the urogenital and fecal tracts are cloacal exstrophy are thought to be produced separated. (Ch 49, page 736) 148 Clinical Pediatric Urology Study Guide

5. The bladder develops from: incorporated into the sinus in the trigone. The mesoderm of the anterior abdominal wall is (a) Ectoderm thought to form part of the anterior aspect of (b) Mesoderm the bladder. (Ch 49, page 735) (c) Endoderm (d) (a) and (e) (e) (b) and (c) 7. Place the following events in correct chrono- logical order: Answer: (e) The trigone is initially meso- derm but subsequently fills in with epithelium (a) Pronephros involutes from the urogenital sinus and is eventually (b) Urorectal septum descends endoderm. The bladder itself forms from (c) Proximal urethra forms endoderm from the urogenital sinus and the (d) Müllerian ducts form detrusor is formed from mesoderm. (Ch 49, (e) Wolffian ducts form page 735) (i) (a), (b), (c), (d), (e) (ii) (a), (b), (e), (c), (d) 6. The bladder develops from: (iii) (a), (b), (c), (e), (d) (a) The urogenital sinus (iv) (a), (d), (c), (b), (e) (b) Mesoderm of the anterior abdominal wall (v) (a), (e), (b), (c), (d) (c) Endoderm Answer: (v) The pronephros involutes as the (d) Ureteral buds mesonephric (Wolffian ducts) form. Then (e) All of the above the urorectal septum divides the cloaca into Answer: (e) The endoderm forms the primi- the urogenital sinus and rectum. The proximal tive cloaca which splits to form the urogenital urethra forms from the urogenital sinus. The sinus and the rectum. The ureteral buds Müllerian ducts form from the Wolffian ducts arise from the mesonephric ducts near their if there is no testis to produce Müllerian entrance into the urogenital sinus and are then inhibiting factor. (Ch 13, page 231) Radiologic assessment of 50 bladder disorders

Douglas E Coplen

1. The best anatomic and functional assessment (c) In late filling of the bladder is obtained with a: (d) During voiding (a) Voiding cystourethrography (VCUG) Answer: (a) A ureterocele appears as a filling (b) Nuclear cystogram defect at the bladder base. This is best seen (c) Urodynamics during early bladder filling since it can col- (d) Ultrasound lapse and actually efface at capacity giving the (e) Computerized tomography (CT) appearance of a diverticulum. A filling defect in the urethra is occasionally, but not repro- Answer: (a) Fluoroscopic imaging gives the ducibly, seen during voiding. best anatomic detail of the bladder, urethra and ureters (in cases of reflux). While not as 4. Superior bladder and pelvic spatial imaging accurate as urodynamics with regards to blad- resolution is obtained with: der function, the VCUG does give functional information with regards to bladder emptying (a) US and the appearance of the bladder outlet (b) CT during voiding. (c) MRI (d) VCUG 2. In pediatric patients the best initial bladder Answer: (c) MRI has the advantage of supe- imaging modality is: rior soft tissue resolution when compared to (a) VCUG CT and US. Different spin-echo sequences (b) CT allow delineation of fluid-filled structures from (c) Magnetic resonance imaging (MRI) the bladder wall. (d) Ultrasound (US) (e) Bladder scan 5. On bladder US, a dilated distal ureter may be confused with all of the following except: Answer: (d) US is ideally suited for the evaluation of bladder symptoms in children (a) Ovarian cyst because it is painless and noninvasive. The US (b) Urachal diverticulum can direct other imaging. (c) Bladder diverticulum (d) Uterine duplication with hydrocolpos 3. During a VCUG the best visualization of a (e) Ureterocele ureterocele occurs: Answer: (e) Cystic structures are frequently (a) In early filling identified posterior and lateral to the bladder. (b) In mid filling Real-time US imaging can usually make a 150 Clinical Pediatric Urology Study Guide

definitive diagnosis. A urachal diverticulum is (c) 0.7 × length × width × depth present at the dome of the bladder and is not (d) 0.5 × length × width × depth in the differential diagnosis. Answer: (c) The bladder is not a perfect circle or a square. The best formula is 6. Bladder volume as determined by US is best 0.7 × length × width × depth. estimated by the formula: (a) Length × width × depth (b) 4/3 × π × width3 Urodynamics of the upper and 51 lower urinary tract

Jennifer J Mickelson and William E Kaplan

1. A healthy 8-year-old girl was referred to a (c) Barium enema pediatric urologist with concerns of recurrent (d) Spinal MRI urinary tract infections. None of the infections Answer: (d) Children with both fecal and are culture proven. The remaining past medi- urinary incontinence should have lumbosacral cal history is negative and she is not on any images. In patients with clearly abnormal medications. Her mother states that she has studies, spinal MRI is performed. (Page 750) bowel movements every 2– 3 days. Your initial investigation besides a physical exam would 3. A 14-year-old female presents with multiple include: culture proven urinary tract infections. Urin- (a) Renal ultrasound alysis in the office is negative. VCUG shows (b) Magnetic resonance imaging (MRI) of no reflux but a trabeculated bladder. Renal the spine and bladder ultrasound shows no hydro but (c) Voiding history and voiding diary a thickened bladder wall. A uroflow is (d) Voiding cystourethrography (VCUG) performed as the patient complains that it is difficult for her to void. She voids 200 ml, Answer: (c) An accurate voiding history is which shows a prolonged, intermittent flow most important; the use of a voiding diary pattern. The best screening parameter from may also be helpful. (Page 748) uroflow for bladder function is: (a) The shape of the uroflow curve 2. A 12-year-old male developed new onset (b) The maximum flow daytime wetting. He has never been dry at (c) Average flow night. Urinalysis and urine glucose are (d) Total voided volume negative. Renal ultrasound is normal and VCUG shows no reflux. Plain film kidney Answer: (a) In a study of 180 school children ureter bladder (KUB) showed an abdomen age 7 – 16 years, 98% had a bell-shaped flow packed with stool and the sacrum could not be pattern. Although the maximum flow, the well visualized. He has occasional episodes of average flow, and time to maximum flow can encopresis, which his parents attributed to be of diagnostic interest, the shape of the flow constipation. The next investigation which appears to be the best screening parameter. should be ordered is: (Page 752) (a) DMSA renogram 4. A 4 kg male newborn is having urodynamic (b) Computerized tomography (CT) abdomen investigation following a diagnosis of an 152 Clinical Pediatric Urology Study Guide

absent sacrum. He has been voiding well on study shows his first leak at 68 ml at a

his own, VCUG is negative for reflux and detrussor pressure of 41 cmH2 O; the leakage ultrasound is normal. An 8F urodynamics was minimal. At capacity his pressure was

catheter is used for the study. The urodynam- 60 cmH2 O with no reflux. His total capacity ics show a capacity of 29 ml, no unstable con- was 220 ml. The next step in his management tractions but a significantly elevated leak-point should include: pressure. This is most likely due to: (a) Bladder augment (a) Urethral stricture (b) Bilateral ureteral reimplants (b) A noncompliant bladder (c) Addition of an alpha-blocker (c) A spinal syrinx (d) Bladder neck reconstruction (d) Catheter used for the study is too large Answer: (a) It has been shown in several Answer: (d) Male newborns usually require centers that in the myelodysplastic popula- a 5F feeding tube for urodynamic evaluation. tion, children with leak-point pressures of > We try to take care to avoid overly large 40 cmH2 O are at greater risk for upper catheters in newborns, which can obstruct the tract damage (hydronephrosis and reflux) urethra and produce an abnormally high leak- than patients who have leak-point pressures < point pressures, as fluid tries to leak around 40 cmH2 O. (Page 759) the catheter. (Page 753) 7. Rate of filling in urodynamic testing is deter- 5. An 8-year-old girl was referred for problems mined by the bladder capacity. A medium fill of infrequent voiding and constipation. Her rate is based on which percentage of bladder history is consistent with dysfunctional elimi- capacity: nation syndrome. Her imaging studies are % normal. A urodynamic investigation was (a) 2 % performed which showed bladder capacity of (b) 10 % 450 ml and a bilateral grade 1 reflux. Her (c) 15 (d) 20 % capacity pressure was 18 cmH2 O. Her initial management would include: Answer: (d) Joseph1 noted that a change (a) Bilateral subureteric Deflux injections in detrussor pressure, as well as maximum (b) Bladder augment detrussor pressure, was adversely affected (c) Bowel management and biofeedback by increasing the rate of filling from slow % (d) Bilateral ureteral reimplants (approximately 2 of estimated bladder capacity, 0– 10 ml/min) to medium fill (approx-

Answer: (c) (Figure 51.12, page 757) imately 20% of estimated bladder capacity, 10– 100 ml/min). In general, it is recommended 6. A 14-year-old male has a history of spina to fill at < 10 ml/min in children. (Page 754) bifida. He managed his bladder with clean intermittent catheterizations since he was 8. Examination of pressures in the upper 3 years old. He takes 20 mg of oxybutinin XL urinary tract can be performed using the in the morning and catheterizes every 2– 3 h. Whittaker test. A normal unobstructed At night he uses an overnight drainage bag. ureteral pressure is: Previous urodynamics have shown a low < detrussor leak-point pressure and good com- (a) 5 cmH2 O < pliance. Recent spinal MRI is normal. A repeat (b) 10 cmH2 O Upper and lower urinary tract 153

< (c) 15 cmH2 O rectal catheter was calibrated appropriately < (d) 20 cmH2 O with electromyography (EMG) leads in place. A 6F catheter was used along with a fill rate of Answer: (c) (Table 51.3 Summary of the 15 ml/min of normal saline. The abnormalities Whittaker Test, Page 762) in the exam may be due to: 9. A 10-year-old male is referred regarding a (a) Filling the bladder too slowly painful perineum. If a postvoid bladder scan (b) The use of normal saline as opposed to was performed prior to the evaluation, you carbon dioxide gas would expected the patient' s postvoid residual (c) Vesicoureteral reflux volume to be: (d) Filling the bladder too quickly (a) Negligible Answer: (d) It is not recommended to use (b) 20 % of bladder capacity carbon dioxide gas for filling. Joseph1 noted (c) 25 % of bladder capacity that a change in detrussor pressure, as well as (d) 35% of bladder capacity maximum detrussor pressure, was adversely affected by increasing the rate of filling from Answer: (d) Estimation of residual urine can slow (approximately 2% of estimated bladder be an important clue to overall bladder func- capacity, 0– 10 ml/min) to medium fill (approx- tion. Except in infants, the child' s bladder imately 20% of estimated bladder capacity, should empty to completion with each void. 10– 100 ml/min). In general, it is recommended (Page 752) to fill at < 10 ml/min in children. (Page 754) 10. A 5-year-old male is having urodynamic assessment for symptoms associated with Reference a suspected tethered spinal cord. The

cystometrogram (CMG) showed elevated 1. Joseph DB . The effect of medium-fill and slow-fill saline detrussor pressures and multiple unstable cystometry on detrusor pressure in infants and children bladder contractions at low volumes. The with myelodysplasia. J Urol 1992 ; 147: 444 – 6.

Neurologic control of storage 52 and voiding

Julian Wan and John M Park

1. Norepinephrine is used as a neurotransmitter receptors are more concentrated in the in which of the following? bladder neck and base; whereas beta-adrenergic receptors are more concentrated in the (a) Sympathetic postganglionic synapses fundus. Nitrous oxide is a major inhibitory (b) Sympathetic preganglionic synapses neurotransmitter. Neuropeptides such as (c) Parasympathetic preganglionic synapses substance P modulate the sensation brought (d) Parasympathetic postganglionic synapses by afferent nerves. (e) Purinergic synapses Answer: (a) Norepinephrine is used in 3. Which of the following is correct? sympathetic postganglionic synapses. Puriner- (a) Pelvic nerves are sympathetic and origi- gic synapses use peptides such as adenosine nate from the caudal end of the lumbar triphosphate (ATP) and gamma-aminobutyric spinal cord acid (GABA). All parasympathetic and sym- (b) The parasympathetic postganglionic neu- pathetic preganglionic synapses use acetyl- rons of the bladder are located next to the choline. lumbar cord (c) Hypogastric nerves have somatic, para- 2. Which of the following is not correct? sympathetic, and sensory fibers (a) Alpha-adrenergic receptors in the bladder (d) The pudendal nerve nucleus is in the lateral are concentrated more in the trigone and ventral horn of the sacral spinal cord bladder neck (e) Afferent pathways are unaffected by (b) Beta-adrenergic receptors in the bladder inflammatory conditions and neuropathy are concentrated more in the fundus Answer: (d) The pudendal nerve nucleus is (c) Cholinergic receptors of the bladder Onuf's nucleus and it resides in the lateral smooth muscle are primarily nicotinic ventral horn of the sacral spinal cord. Pelvic (d) Nitrous oxide is an inhibitory neurotrans- nerves are parasympathetic and originate mitter of urethral smooth muscle from the sacral cord. Parasympathetic post- (e) Neuropeptides modulate signals from ganglionic neurons of the bladder are located afferent sensory nerves within the detrusor wall and in the pelvic Answer: (c) Cholinergic receptors of the plexus. Hypogastric nerves carry sympathetic bladder smooth muscle are primarily muscar- and sensory fibers. Afferent pathways can inic. There are now five distinct subtypes change after injury or disease. Unmyelinated recognized and this fact is exploited to make c-fibers become more active with inflamma- more precise acting drugs. Alpha-adrenergic tion and neuropathy. 156 Clinical Pediatric Urology Study Guide

4. During urine storage, which of the following of micturition. Sympathetic and somatic path- statements is most accurate? ways are inhibited during micturition, and allow rhabdosphincter relaxation and bladder (a) Distention of the bladder decreases contractions. Parasympathetic pathways relax afferent nerve activity the bladder neck by inhibiting contraction and (b) During bladder filling the activity of the cause micturition by stimulating bladder rhabdosphincter increases smooth muscle contraction. Transection of (c) During bladder filling, the electromyog- the brain above the brainstem will not abolish raphy (EMG) of the urinary sphincter inhibition of the rhabdosphincter during mic- shows less activity as it is relaxing to help turition. Transection of the spinal cord below hold more urine the brainstem disrupts the coordinating influ- (d) Squirming and crossing of legs helps a ences of the pontine micturition center and child hold urine by mechanically obstruct- will lead to dyssynergia. ing the urethra and bladder neck (e) Storage occurs without any input from the brainstem or cerebrum 6. Voluntary control of voiding depends on all of the following except ? Answer: (b) The rhabdosphincter increases activity during normal bladder filling as part (a) Frontal cortex of the guarding reflex. Distention of the (b) Preoptic region of the hypothalamus bladder increases afferent nerve activity. It also (c) Brainstem increases EMG activity of the urinary sphinc- (d) Cerebellum ter. Avoidance maneuvers such as squirming (e) Paracentral lobule and crossing of the legs are believed to trigger Answer: (d) All of the other parts of the somatic afferent pathways to the pudendal brain have been found to have a role in the nerve which help inhibit bladder contractions. voluntary control of urine. During storage there are active links between the brainstem and cerebrum and efferent and 7. Which of the following is true about matura- afferent nerves from the sacral spinal cord. tion of voiding control? 5. During voiding which of the following is not (a) At birth the neonatal bladder is a com- true about micturition? plete and effective urine storage vessel (a) During micturition sympathetic and (b) The primitive voiding reflex of neonates is somatic pathways are inhibited replaced by the mature micturition reflex (b) Parasympathetic pathways inhibit the by 4 years of age in most children bladder neck and stimulate the bladder (c) Animals such as cats and rats are born smooth muscle innately knowing how to void (c) Transection of the brain above the (d) Once maturation changes have occurred, brainstem will abolish reflex inhibition of they are permanent and cannot regress the rhabdosphincter with micturition (e) Downregulation of interneuron to excit- (d) Transection of the spinal cord beneath the atory preganglionic neuron synapses brainstem will lead to dyssynergia Answer: (e) Downregulation of interneuron (e) The guarding reflex is inhibited by the pon- to excitatory preganglionic neuron synapses, tine micturition center prior to voiding possibly by competition for synaptic space, is Answer: (e) The pontine micturition center thought to be crucial to the maturation process. signals inhibit the guarding reflex at the start All of the other choices are incorrect. Neurologic control of storage and voiding 157

8. The chilled water test will demonstrate: pathways is rapid and abrupt in animals (a) No effect in normal mature children such as rats and kittens. In humans this (b) No effect in normal adults process may take several years and there will (c) Bladder contractions in infants under be a period of time when neither group of 3 years of age pathways predominate. Voiding dysfunction (d) Bladder contractions in patients with neu- and neuropathology may be due to the ropathic bladders reemergence of primitive pathways or the (e) (c) and (d) incomplete transition over to mature micturi- tion processes. Answer: (e) Cold receptors are mediated by c-fibers. In normal mature children and adults there are inhibitory controls which mask the 10. Which of the following are correct? primitive neonatal reflexes. Infants under 4 years of age will normally have a positive (a) Nerve endings innervate each individual response with involuntary bladder contrac- smooth muscle cell of the human tions. Likewise, patients with neuropathology bladder will also respond, demonstrating that the loss (b) Removal or alteration of the innervation of an inhibitory control may be an effect of results in no lasting changes to bladder neuropathology. histology (c) Smooth muscle cells under repetitive 9. Which of the following are not correct? stress elaborate greater amounts of (a) As babies develop and mature into toddlers myosin and myotropin primitive urinary reflexes are supplanted (d) Normal smooth muscle cells propagate by more mature micturition processes excitation by direct neuronal cell-to-cell (b) The transition in infants is as rapid and excitation abrupt as it is in mammalian animals (e) Pathological states may manifest their (c) Plasticity of the micturition pathways effects by altering the properties of the suggests that reemergence of primitive extracellular matrix pathways may explain some forms of Answer: (e) Nerve endings do not innervate neuropathology each individual smooth muscle cell. Smooth (d) Plasticity of the micturition pathways muscles are separated from each other and no suggests that voiding dysfunction may direct cell-to-cell excitation has yet been be due to incomplete transition to mature shown. Electromechanical coupling, rather pathways or the reemergence of primitive than direct neuronal excitation, propagates pathways detrusor contractions. Repetitive stress to (e) There is a period where neither primitive smooth muscle cells results in greater amounts nor mature micturition pathways are of collagen and elastin being laid down, but clearly preeminent not myosin or myotropin. Pathological states Answer: (b) The transition from more often have changes in the relative amounts of primitive micturition reflexes to more mature the different types of collagen.

Neurogenic voiding dysfunction 53 and nonsurgical management

Richard N Yu and Stuart Bauer

1. Which vitamin plays a major role in the will typically have intact sacral reflex arc prevention of neural tube defects? function involving the sacral spinal roots. But, they commonly have detrusor– sphincter (a) C arotene dyssynergy. (b) N iacin (c) F olic acid (d) R iboflavin 3. Urinary tract deterioration within the first (e) B 12 3 years of life is most likely associated with which of the following factors? Answer: (c) The addition of folic acid sup- plements to the diet of women of childbearing (a) D etrusor areflexia age has reduced the incidence of spina bifida (b) D etrusor– sphincter synergy by > 80 % . In some cases, despite increased (c) D etrusor– sphinter dyssynergy folic acid intake, women may still have a fetus (d) D etrusor overactivity with spina bifida owing to antibodies that (e) I ncomplete bladder emptying develop to folate. Answer: (c) Seventy-one percent of new- borns with dyssynergic voiding had urinary 2. Which statement is true about children with a tract deterioration within the first 3 years of thoracic level myelomeningocele? life. Deterioration was noted in only 17% % (a) L eg function is usually normal of children with synergic voiding and 23 of (b) I nfants with this level lesion often have children with sphincter denervation. Bladder detrusor– sphincter dyssynergy outlet obstruction is a major contributor to (c) I nfants with this level lesion have an the development of urinary tract deterioration underactive detrusor in these children. (Figure 53.2) (d) O lder children tend to have a normal upper urinary tract by ultrasound 4. Cred é voiding is best avoided in which of the (e) O lder children tend to easily achieve following conditions? urinary continence with just clean inter- (a) T he presence of vesicoureteral reflux with mittent catheterization (CIC) alone a reactive external urethral sphincter Answer: (b) The neurologic lesion produced (b) T he presence of vesicoureteral reflux with by a myelomeningocele can be quite variable. a severely denervated external urethral The bony vertebral level may not correlate sphincter to the exact neurologic level or deficits (c) T he presence of vesicoureteral reflux with produced. Children with thoracic level lesions a low leak-point pressure 160 Clinical Pediatric Urology Study Guide

(d) A n overactive detrusor with a low leak- (b) A subcutaneous mass overlying the tho- point pressure racic spine (e) A compliant bladder with a low leak-point (c) A n asymmetric gluteal cleft presssure (d) A draining pilonidal dimple (e) O ne leg slightly longer than the other Answer: (a) Cred é voiding should be avoided in children with reflux and a reactive external Answer: (b) An occult spinal dysraphism urethral sphincter. The increased sphincter includes lipomeningocele, intradural lipoma, tone and associated bladder outlet resistance diastematomyelia, tight filum terminale, der- requires higher bladder pressures to expel moid cyst/sinus, aberrant nerve roots, anterior urine and may aggravate the degree of reflux. sacral meningocele, and cauda equina tumor. Cred é voiding can be used in patients Physical examination may demonstrate a cuta- with low bladder outlet resistance, which is neous abnormality overlying the lower spine, associated with severe external urethral such as a small dimple or skin tag, tuft of hair, sphincter denervation, and with a low leak- asymmetric gluteal cleft, vascular malforma- point pressure on urodynamic studies. tion, or subcutaneous lipoma. The child may also have leg abnormalities from nerve root injury. 5. When considering an antireflux operation in a child with a myelomeningocele, treating which urodynamic parameter beforehand is thought 7. The conus medullaris normally resides oppo- to be most important for achieving a success- site which vertebral body at birth? ful result? (a) T horacic 11 level (a) P oor compliance (b) T horacic 12 level (b) P resence of detrusor overactivity (c) L umbar 1 level (c) A high leak-point pressure (d) L umbar 2 level (d) (a) and (b) (e) L umbar 3 level (e) (a) and (c) Answer: (d) Under normal circumstances, the conus medullaris ends just below the L-2 Answer: (d) Untreated poor bladder com- vertebra at birth and recedes upwards to T-12 pliance and high leak-point pressure may by adulthood. result in persistent reflux after antireflux sur- gery. Therefore, children who cannot empty their bladder spontaneously should be placed 8. The conus medullaris normally resides oppo- on intermittent catheterization to ensure site which vertebral body in a normal adult? complete emptying. Anticholinergic medica- (a) T horacic 11 level tions may be useful in patients with poor (b) T horacic 12 level detrusor compliance in order to lower storage (c) L umbar 1 level pressure and improve bladder compliance. (d) L umbar 2 level (e) L umbar 3 level 6. All of the following may be an outward

physical sign of an occult spinal dysraphism Answer: (b) Under normal circumstances, except: the conus medullaris ends just below the L-2 vertebra at birth and recedes upwards to T-12 (a) S pinal scoliosis by adulthood. Neurogenic voiding dysfunction and nonsurgical management 161

9. Which of the following maternal factors may (e) T he incidence of neurovesical dysfunction be responsible for sacral agenesis in a newborn is unrelated to the presence of a sacral child? bony defect (a) E xposure to progestational agents early in Answer: (d) The Pe ñ a procedure utilizes a the pregnancy posterior, midline approach which was deve- (b) E xposure to progestational agents later in loped for correction of a high imperforate the pregnancy anus while minimizing the risk of injury to the (c) E arly gestation, insulin-dependent diabetes pelvic floor nerves. However, 37% of patients (d) I nsulin dependency later in the pregnancy do have long-term problems with fecal incon- (e) M aternal exposure to tetrahydrocanna- tinence. Neurogenic bladder is more common binol in patients with supralevator level lesions and is related to the presence of a sacral bony Answer: (c) Insulin-dependent mothers defect. Thirty to 70 percent of children with have a 1 % chance of giving birth to a child imperforate anus have a spinal abnormality. with sacral agenesis. This condition has been reproduced in chicks that were exposed to 11. Most children with cerebral palsy have what insulin during embryogenesis. Maternal drug type of lower urinary tract function on urody- exposure has also been reported to cause namic testing? sacral agenesis. However, progestational agents and tetrahydrocannabinol have not (a) A normally reflexic bladder with bladder been associated with sacral malformation or sphincter synergy during voiding agenesis. (b) A hyperactive bladder with denervation in the external urethral sphincter 10. Which statement is true regarding neurogenic (c) A hyperactive bladder with bladder bladder dysfunction in children with imper- sphincter dyssynergy during voiding forate anus? (d) A n underactive bladder with denervation in the external urethral sphincter (a) T he Pe ñ a operation leads to pelvic (e) A n underactive bladder with a normally nerve injury resulting in bladder dys- innervated external urethral sphincter function (b) N eurogenic bladder dysfunction is almost Answer: (a) Most children with cerebral never seen in patients with a supralevator palsy have a normally reflexic bladder with level lesion bladder –sphincter synergy during voiding (c) N eurogenic bladder dysfunction is fairly and they can develop total urinary control. common in patients with an infralevator Urodynamic findings may demonstrate a level lesion partial, upper motor neuron lesion type of (d) A spinal cord abnormality in association dysfunction with exaggerated sacral ref lexes, with imperforate anal lesions can be seen detrusor overactivity, and/or detrusor– in 30% of these children sphincter dyssynergia.

Diurnal and nocturnal enuresis 54 Rosalia Misseri (Based on chapter written by Mark Horowitz and Rosalia Misseri)

1. When desmopressin acetate is administered, agement of regular daytime voiding and void- fluid intake should be adjusted downward in ing before bed may ameliorate the situation. order to: 3. Pathophysiologic mechanisms causing noctur- (a) D ecrease the potential occurrence of water nal enuresis: intoxication and hyponatremia (b) D ecrease overnight urine production (a) A re related to abnormalities in the (c) D ecrease urine osmolality circadian rhythm of antidiuretic hormone (d) A void hypertension related to its weak secretion pressor activity (b) A re related to nocturnal polyuria (e) A void headache and nausea (c) M ay be related to poor arousability (d) (a) and (c) Answer: (a) Desmopressin is an analogue of (e) A ll of the above vasopressin with increased antidiuretic activity without vasopressor activity. The risk of water Answer: (e) Nocturnal enuresis may be due intoxication and hyponatremia should be dis- to one or all of the above. cussed with parents prior to starting therapy. 4. A healthy child with a normal physical exami- 2. A 4-year-old girl who was potty trained at nation and severe nocturnal enuresis refrac- 2 years 7 months is referred for evaluation of tory to therapy with desmopressin acetate and primary nocturnal enuresis. Her siblings, 7 alarm therapy should be: and 9 years of age, wet the bed until age 6. (a) R eferred for evaluation by a behavioral She has no daytime urinary urgency, frequency therapist or incontinence. Initial therapy includes: (b) E valuated for functional and anatomic (a) O ral desmopressin bladder abnormalities (b) P arental reassurance (c) R eassured that approximately 15% will (c) E ncourage regular daytime voiding sche- become dry yearly dules and biofeedback (d) E valuated with a lumbosacral magnetic (d) E ncourage regular daytime voiding sche- resonance imaging (MRI) scan dules and encourage emptying the bladder (e) R eferred for evaluation of obstructive before bed sleep apnea (e) E valuation for obstructive sleep apnea Answer: (b) Children without a history of Answer: (d) Approximately 25% of 4-year- behavioral disturbances, cutaneous abnor- old children frequently wet the bed. Though malities overlying the lumbosacral spine, and parents should be reassured that the majority snoring would not benefit from choices (a), (d) of children will outgrow this, simple encour- and (e). Although it is true that approximately 164 Clinical Pediatric Urology Study Guide

15 % of children with nocturnal enuresis will (c) B ehavioral modification become dry yearly, consideration of under- (d) P rophylactic antibiotics lying bladder dysfunction, such as small func- (e) A cupuncture tional bladder capacity and detrusor instability, Answer: (d) Desmopressin acetate, alarm should be explored as the cause of severe therapy, and behavioral modification are the refractory enuresis. most commonly used therapies for nocturnal enuresis. Alternative therapies such as tradi- 5. Nocturnal enuresis may be treated with all of tional Chinese acupuncture and laser acupunc- the following except: ture have been used. There is no role for (a) D esmopressin acetate antibiotic prophylaxis in children with primary (b) A larm therapy nocturnal enuresis. Operations for the weak 55 bladder outlet

CD Anthony Herndon (Based on chapter written by Anthony J Casale)

1. Possible side-effects from medications used to 3. Potential complications following placement of treat an incompetent bladder neck include the the artificial urinary sphincter (AUS) include: following except: (a) V aginal obstruction (a) R hinorrhea (b) U rethral erosion (b) P alpitations (c) D ifficulty with catheterization (c) H ypertension (d) A US infection (d) H eadaches (e) D etrusor decompensation with the need (e) A nxiety for augmentation

Answer: (a) Alpha-agonist medication is the Answer: (a– e) All of the above listed com- primary medical treatment of the incompetent plications may result after placement of the bladder neck; complications include palpita- AUS. Approximately one-third of patients tions, headache, anxiety and hypertension. will require bladder augmentation after place- ment of the AUS.

2. Which population is least likely to experience 4. Of the following bladder neck procedures, success with a urethral/bladder neck sling which one is most likely to allow volitional procedure: voiding: (a) N onambulatory male (a) Y oung-Dees-Leadbetter (b) C ombined with a bladder augmentation (b) K ropp (c) A mbulatory male (c) P ippi Salle (d) A mbulatory female (d) A US (e) C ombined with another bladder neck (e) U rethral/bladder neck sling procedure such as a Young-Dees and

augmentation Answer: (d) Out of the above listed options, volitional voiding is most expected in patients Answer: (c) Of the above options, the ambu- after placement of the AUS. In fact, the latory male patient historically is the most ideal patient appears to be one in whom difficult patient to achieve continence after a the bladder dynamics are normal (i.e. female urethral sling procedure. epispadias).

Bladder augmentation: 56 current and future techniques

Jennifer J Mickelson , Bradley P Kropp , and Earl Y Cheng

1. Gastrocystoplasty is an appropriate form of (c) N a + /K + ATPase enzyme activity + augmentation for patients in renal failure (d) Ammonium (NH4 ) reabsorption because: Answer: (d) Metabolic acidosis may develop (a) H yperchloremic hypokalemic metabolic whenever urine is in contact with ileal or acidosis occurs colonic mucosa. The mechanism by which (b) I t reduces metabolic acidosis by inducing this hyperchloremic metabolic acidosis occurs + a metabolic alkalosis is related to ammonium (NH4 ) reabsorption. (c) I t absorbs more chloride (Page 876) (d) I t improves renal function Answer: (b) Gastrocystoplasty is a better 4. Regardless of the type of bowel segment, option for renal failure patients because it certain reconstruction techniques should induces a hypochloremic hypokalemic meta- always be employed when performing entero- bolic alkalosis. This results in less electrolyte cystoplasty. These include: disturbance for this vulnerable patient group. (a) U reteral reimplantation (Table 56.1, page 872) (b) B ladder neck reconstruction (c) D etubularization and bowel reconfi- 2. Antireflux surgery should be considered in guration patients undergoing augmentation if: (d) A ppendicovesicostomy (a) R eflux occurs at low intravesical pressure Answer: (c) The goal of bladder augmenta- (b) T here is a history of infection tion is to provide a large-capacity, low-pressure, (c) T he bladder has poor compliance urinary reservoir. Detubularization of bowel (d) T he patient is prone to constipation segments prevents synchronous contractions Answer: (a) Although reflux has been of circular muscles of the gut. The radius of reported to resolve spontaneously after entero- the reservoir is directly related to the volume. cystoplasty, antireflux surgery should be con- The greater the radius in the augmented blad- sidered at the time of reconstruction, especially der the larger pressure low-pressure reservoir if reflux occurs at low intravesical pressures. this translates to. (Page 879) (Page 873) 5. A 12-year-old female patient with spina bifida 3. The mechanism of the metabolic acidosis seen is about to undergo bladder augmentation in bladder augmentation with ileum is due to: for a noncompliant, small capacity bladder. (a) I ncreased chloride secretion In addition, she is incontinent of stool. (b) N et bicarbonate reabsorption She has between five and six loose bowel 168 Clinical Pediatric Urology Study Guide

movements per day. Which anatomic bowel (b) M alone Antegrade Continence Enema segment is most important to preserve for this (MACE) procedure patient? (c) C ystoscopy (d) A ppendicovesicostomy (a) T ransverse colon (b) I leocecal valve Answer: (c) Although the incidence of (c) I leum tumor formation following enterocystoplasty (d) S igmoid colon is unknown, lifelong follow-up with yearly cystoscopy and urine cytologies should be Answer: (b) The incidence of bowel dys- considered beginning 6– 10 years after aug- function following enterocystoplasty in all mentation. (Page 886) patients is 10– 54 % . The incidence in the spina bifida population is 20% . Since the overall incidence of bowel problems in children 8. An 8-year-old girl with a history of bladder with spina bifida is further increased when the augmentation 2 years ago presents with ileocecal valve is removed, this segment should abdominal pain, distended abdomen, and

be preserved. (Page 884) fever. She last catheterized 2 h ago and obtained, 50 ml of urine. A catheter is placed

6. A 16-year-old girl has had an augmentation and 10 ml of cloudy urine is obtained. She has cystoplasty and appendicovesicostomy per- a history of chronic constipation. Routine lab formed for management of her neurogenic work has been drawn. The next best step in bladder. She catheterizes regularly without her management is: difficulty but has persistent problems with (a) U ltrasound recurrent stones. The most common stone (b) E nemas composition following enterocystoplasty is: (c) U rinalysis and culture (a) C alcium oxide monohydrate (d) C omputerized tomography (CT) cysto- (b) U ric acid gram (c) M agnesium ammonium phosphate Answer: (d) It may be difficult to distin- (struvite) guish pyelonephritis from spontaneous blad- (d) C alcium phosphate der perforation. Thus, a standard CT cystogram Answer: (c) The risk of stone formation is recommended in any patient with an aug- after enterocystoplasty ranges from 7 to 52% . mented bladder who presents with the above The usual stone composition reported is symptoms. Although the diagnostic role of a magnesium ammonium phosphate (struvite), cystogram has been questioned in the past, it although several others have been reported. is the most specific diagnostic radiographic (Page 885) test. (Page 887)

7. A 21-year-old male had an enterocystoplasty 9. A 16-year-old boy has a gastrocystoplasty. created 14 years prior. On follow-up ultra- Routine follow-up urodynamics show low < sound there is no hydronephrosis. Video amplitude phasic contractions 30 cmH2 O. urodynamics show a large-capacity, low- Occasionally he will have episodes of pressure reservoir. He infrequently leaks urine. coffee-colored urine that doesn' t resolve with He catheterizes per urethra. The next most increased fluid intake. The next best step in important step in his management is: management is: (a) B ladder neck procedure (a) B icarbonate supplementation Bladder augmentation: current and future techniques 169

(b) A n angiotensin-converting enzyme (ACE) (b) R educed compliance and increased inhibitor capacity (c) A nticholinergic medication (c) N ormal compliance and decreased (d) H 2 receptor blockers capacity (d) N one – preoperative urodynamic studies Answer: (d) Hematuria-dysuria syndrome do not reliably predict success (HDS) is unique to gastrocystoplasty and occurs in one-third of patients. Despite the Answer: (d) Although it appears that relatively high incidence of HDS following patients with increased compliance and large gastrocystoplasty, most patients respond well capacity bladder do better with autoaugmen- either to H2 receptor blockers or proton pump tation, mixed results have been obtained blockers such as omeprazole. (Page 891) clinically with regards to postoperative symp- tomatic and urodynamic improvement in the 10. Preoperative studies which reliably predict autoaugmented bladder. Evaluation of the success for autoaugmentation include: available data indicates that there is no direct (a) I ncreased compliance with near-normal correlation between preoperative urodynamic bladder capacity findings and success. (Page 892)

Urinary diversion 57 Richard Rink and Mark P Cain

1. The most common complication of a tempo- 3. The benefits of a loop ureterostomy compared rary vesicostomy is: to a Sober ureterostomy include: (a) U rinary tract infection (a) E nables continuous cycling of bladder (b) P arastomal hernia (b) S impler to close (c) S tomal prolapse (c) M ore complex to create but lower stenosis (d) D ermatitis rate (d) L ess prone to upper tract infection Answer: (c) The most common complica- tions of temporary cutaneous vesicostomy in Answer: (b) The Sober Y-ureterostomy is order of occurrence are: stomal prolapse the only high diversion which guarantees (9– 17 % ); stomal stenosis (3– 12 % ); peri- some degree of bladder cycling, but it is much stomal dermatitis (3– 14 % ); stone formation more difficult technically to construct. (3 – 7 % ); and peristomal hernia (3– 4 % ). 4. The indications for a surgically placed open 2. A 7-year-old male has a transverse colon con- cystostomy tube include all but: duit created for urinary diversion following (a) Extensive lower urinary tract recon- radiation and excision of a pelvic rhabdomyo- s truction sarcoma. He does well until postoperative day (b) Urethral or bladder trauma 7, when the left ureteral stent is inadvertently (c) Complex hypospadias repairs removed and he has marked increased (d) Prior extensive pelvic surgery with bowel drainage from the penrose drain. The and adhesions to dome of bladder creatinine on the fluid is > 20 mg/dl. Renal ultrasound shows no hydronephrosis. The Answer: (c) All are true except (c). While best management would be: complex hypospadias repairs may be an indi- cation for cystostomy tube, particularly in (a) O bservation the older child, it would nearly always be (b) P lacement of a closed-system percuta- placed percutaneously rather than as an open neous drain procedure. (c) I mmediate reoperation and left to right transureteroureterostomy 5. Contraindications for percutaneous nephros- (d) P ercutaneous nephrostomy and antegrade tomy tube placement are: stent placement (a) T hrombocytopenia Answer: (d) The most likely explanation is (b) P yonephrosis an anastomotic leak, which should be amena- (c) F ungal bezoar ble to percutaneous drainage and antegrade (d) I mpacted ureteropelvic junction stone stent placement. Placement of a foley catheter in the conduit may also be beneficial to mini- Answer: (a) Relative contraindications for mize extravasation. percutaneous nephrostomy tube placement 172 Clinical Pediatric Urology Study Guide

include thrombocytopenia, solitary kidney, (b) I leum and abnormal renal position which would (c) C ecum make placement of the tube dangerous. All (d) T ransverse colon the other listed problems are indications for (e) S igmoid colon percutaneous tube placement. Answer: (b) Because of the thicker muscu- 6. Intestinal conduit diversion is still occasionally laris of the stomach wall and the colon, the performed in children. All of the following submucosal ureteral tunneling techniques are bowel segments except one would be accept- more successful than with ileum. With ileal able for conduit creation: segments, the split-cuff nipple technique or the LeDuc technique have been successful in (a) J ejunum preventing reflux. (b) I leum (c) C ecum 9. As a general rule, the lowest sustained intra- (d) T ransverse colon vesical pressure, either in native bladder or (e) S igmoid colon continent diversion, which will lead to a high Answer: (a) Incontinent urinary diversion percentage of patients having upper urinary is rarely used in children and this type of tract deterioration is: diversion is generally only considered in (a) 10 those patients that are physically or mentally (b) 20 unable to manage continent diversion. Ileum (c) 30 and colon are the most commonly used bowel (d) 40 segments. Jejunum is unacceptable due to (e) 50 the severe electrolyte disorders which may occur. Answer: (d) Since McGuire' s early work it is now well accepted that intravesical pressures < 7. With respect to incontinent ileal or colon con- must be maintained 40 cmH2 O. Newer stud- < duit diversion, which of the following compli- ies have suggested that pressures 40 cmH2 O cations occur most commonly? may be deleterious to the ureter and kidneys, but the current standard is 40 cmH O. (a) U rinary leak 2 (b) R enal deterioration 10. Reconfiguring the intestinal segment from a (c) U reteroenteric stricture cylinder to a more spherical shape will accom- (d) R enal calculi plish all the following except: (e) S tomal stenosis/stomal prolapse (a) I mprove capacity Answer: (e) The complications of ileal or (b) I mprove electrolyte abnormalities colon conduit diversion include urine leak (c) I ncrease wall tension (3– 5 % ), renal deterioration (15– 18 % ), ure- (d) D ecrease contractility teroenteric stricture (8– 9 % ), renal calculi (e) D oes not eliminate all bowel contractions (5 – 7 % ), and stomal problems in > 20 % . Answer: (b) Reconfiguring and detubular- 8. Achieving a nonrefluxing submucosal tun- izing the bowel will improve capacity and neled ureterointestinal anastomosis is most dampen contractility, but will not completely difficult with which intestinal segment? eliminate all contractions. Wall tension will be (a) S tomach increased by increasing the radius (LaPlace' s Urinary diversion 173

Law T = Pr). It will not improve the risk of (b) C ontinent vesicostomy electrolyte abnormalities. (c) R econfigured ileum (d) S tomach tube 11. Vitamin B12 deficiency can be caused by and

result in which of the following? Answer: (c) While all the listed tubes have been satisfactorily used, reconfigured (a) R emoval of the sigmoid colon ileum (Monti-Yang tube) is the most reliable, (b) R emoval of large segments of right colon with a success rate similar to the “ gold (c) R esults in microcytic anemia standard ” appendicovesicostomy. The ureteral (d) N europathy tube has a high stomal complication rate (e) C an occur within days of removing the and continent vesicostomy is the worst, with

distal 20 cm of ileum a stomal stenosis rate of 40% . The reim- Answer: (d) Vitamin B12 deficiency usually planted stomach tube has a high risk of skin occurs from removing a large segment of the excoriation. distal ileum. Deficiency leads to megaloblastic anemia and neuropathy. Vitamin B12 stores 14. Which form of continent urinary reservoir is can last for several years and deficiency would most likely to result in severe hypokalemia? take many years to become manifest. (a) K och pouch 12. Resection of a large segment of ileum can (b) I ndiana pouch result in all but one of the following: (c) I leal augmentation and catheterizable (a) D iarrhea channel (b) V itamin B12 deficiency (d) U reterosigmoidostomy (c) S teatorrhea (e) P enn pouch (d) H ypochloremic metabolic alkalosis

(e) I ncreased risk of cholelithiasis Answer: (d) Ureterosigmoidostomy may lead to profound hypokalemia. Ileal segments Answer: (d) When ileum is incorporated will reabsorb some of the potassium when into the urinary tract it will frequently exposed to high levels in the urine, but colon result in hyperchloremic metabolic acidosis, will not. which is worse in the face of renal insuffi- ciency. When > 100 cm of ileum is used risk of steatorrhea, diarrhea, and cholelithiasis 15. Which bowel segment when incorporated into increases. the urinary tract is most likely to result in hyperkalemia? 13. A 10-year-old child is undergoing lower urinary tract reconstruction requiring a cath- (a) S tomach eterizable abdominal wall urinary channel (b) J ejunum and the appendix has been used in a Malone (c) I leum Antegrade Continence Enema (MACE) pro- (d) C olon cedure. What tissue should be used to create Answer: (b) Stomach, ileum, and colon are the channel? all associated with hypokalemia, but jejunum (a) D istal ureter with transureteroureteros- will result in hyponatremic, hypochloremic, tomy (TUU) hyperkalemic metabolic acidosis.

The Malone Antegrade 58 Continence Enema procedure

Martin A Koyle (Based on chapter written by Martin A Koyle and Padraig SJ Malone)

1. The highest success rate utilizing the Malone (c) A lax pelvic floor Antegrade Continence Enema (MACE) tech- (d) B ile salt diarrhea nique is in: (e) M egacolon (a) S pina bifida Answer: (a) Mouriquand3 and others have (b) A norectal abnormalities strongly advocated the virtues of distal place- (c) D owns syndrome ment of the MACE procedure in those with (d) H irshspung' s disease severe constipation, with a spastic pelvic floor. (e) F unctional constipation Liloku4 and Churchill5 reported shorter Answer: (d) In a survey performed by washout times when a left-sided MACE was Malone and Curry1 of British Association of performed. Paediatric Surgeons (BAPS) members, over 300 cases were collated and success was depen- 4. When performing MACE irrigation, the irrig- dent on the original diagnosis. ant containing which of the below is associated with potential life-threatening complications: 2. Quality of life in patients in patients who have (a) F ree tap water undergone the MACE procedure is lowest in: (b) H ypotonic saline (a) C erebral palsy with poor upper extremity (c) G o Lytely coordination (d) P hosphate (b) I mperforate anus after Soave procedure (e) M agnesium (c) T ransverse myelitis with residual bowel Answer: (d) Although in North America tap dysfunction only water is primary irrigant utilized and has a safe (d) H irshprung' s disease with residual mega- profile, phosphate enemas are utilized more colon commonly in Europe and have been associ- (e) W heelchair-bound spina bifida ated with potentially fatal toxicity. Thus, if Answer: (e) Malone,2 using a quality of life such a regimen is used, failure of one enema score where 5 was perfect, found that wheel- to produce a return, should not be followed chair-dependent children with spina bifida had by a second phosphate enema.6 lower scores than ambulatory patients. 5. The most common complication related to 3. A left-sided MACE should be strongly consid- the MACE procedure is: ered in patients with: (a) S toma incontinence (a) S evere constipation (b) F alse passage of the channel (b) S hort gut syndrome (c) S toma stenosis 176 Clinical Pediatric Urology Study Guide

(d) P ain on infusion (b) L ess likely if concurrent urinary tract (e) S toma prolapse reconstruction has been undertaken (c) P redictable and occur within 30 minutes Answer: (c) Both Denver7 and Southampton 8 of irrigant instillation series yielded similar results showing that stoma (d) M ay take as long as 6 months to occur continence was present in about 98% (e) B est when the appendix is used of patients but stenosis occurred in about 20 – 30 % of patients. Answer: (c) It may take up to 6 months for a steady state to occur and hence success to be 6. The ideal patient for consideration on the determined in patients who have under gone MACE procedure should be: the MACE procedure, and depends on under- lying condition, bowel motility, patient and > (a) 5 years of age parental motivation, and ability to deal with (b) F emale “ trial and error ” . (c) R ight-hand dominant (d) W heelchair bound 9. Most rapid mean washout times when a (e) D evoid of prior abdominal surgery MACE procedure has been performed are Answer: (a) The ideal patient is > 5 years of when it is: age, motivated, tried and failed all conserva- (a) P laced on the left side tive nonsurgical therapy, and have a diagnosis (b) P laced in the transverse colon of neuropathic bowel, anorectal malformation (c) I n situ appendix 4 or Hirschprung' s disease. (d) P erformed using a button or Chait tube in the cecum 7. In the absence of an intact appendix, the (e) P erformed using minimally invasive lap- MACE procedure: aroscopic techniques (a) S hould only be approached laparoscopi- Answer: (a) The mean washout time for cally (LACE) right-sided MACEs in situ, or right sided, is (b) I s contraindicated > 30 min, whereas this can be reduced signifi- (c) I s likely to fail cantly, especially in those with constipation, (d) S hould be performed on the left side when a left-sided MACE is performed.12 (e) C an be performed utilizing the Monti technique 10. A 10-year-old boy with history of cured spinal Answer: (e) There are many options includ- cord neuroblastoma is incontinent of stool ing Chait tubes/Mic-Key buttons, colonic and urine. Continent reconstruction of both extension or flaps which can successfully be systems is strongly considered if: utilized when the appendix is deficient or (a) T he appendix is intact and can be split for absent. Sugarman suggests the Monti tech- dual stomas nique as the procedure of choice in such sce- (b) A colostomy is considered as a primary narios.10 option (c) H e is not wheelchair bound 8. Success with the establishment of a MACE (d) H e is highly motivated channel should be: (e) H ad stage IV S neuroblastoma (a) R eadily assessable before discharge from Answer: (d) Although being wheelchair hospital bound may be associated with a less successful MACE 177

outcome, being > 5 years of age and highly 5. Churchill BM , De Ugarte DA , Atkinson JB . Left-colon motivated is the key to successful outcome. antegrade continence enema (LACE) procedure for The appendix may be split in selected circum- fecal incontinence. J Pediatr Surg 2003; 38 : 1778 – 80 . 6. Aksnes G , Dieseth TH , Heleseth A et al. Appendicos- stances but is not absolutely necessary for tomy for antegrade enema: effects on somatic and either the MACE or Mitrofanoff technique. psychosocial functioning in children with myelo- Initial tumor stage is irrelevant. In a child meningocele. Pediatrics 2002; 109 : 484 – 9 . being considered for urinary continence recon- 7. McAndres HF , Malone PS . Continent catheterizable struction, it is paramount that the patient is conduits: which stoma, which conduit and which reservoir? BJU Int 2002; 89 : 86 – 9 . assessed for fecal continence as well, as simul- 8. Barqawi A , DeValdenebro M , Furness PD 3rd , Koyle taneous reconstruction does not jeopardize MA . Lessons learned from stomal complications in the success of either individual component. children cutaneous catheterizable continent stomas. BJU Int 2004; 94 : 1344 – 7 . 9. Curry JL , Osborne A , Malone PS . How to achieve a successful Malone antegrade continence enema. J Pediatr References Surg 1998; 33 : 138 – 41 . 10. Sugarman ID , Malone PS , Terry TR , Koyle MA . 1. Curry JL , Osborne A , Malone PS . The MACE proce- Transversely tubularized ileal segments for the Mitro- dure: experience in the United Kingdom. J Pediatr fanoff or Malone antegrade continence enema proce- Surg 1997; 34 : 338 – 40 . dures: the Monti principle. BJU 1998; 81 : 253 – 6 . 2. Malone PS , Wheeler RA , Williams JE . Continence in 11. Wedderburn A , Lee RS , Denny A et al. Synchronous patients with spina bifida: long term results. Arch Dis bladder reconstruction and antegrade continence Child 1994; 70 : 107 – 10 . enema. J Urol 2001; 165 : 2392 – 3 . 3. Mouriquand P , Mure PY , Feyaerts A et al. The left 12. Kajbafzadeh AM , Chubak N . Simultaneous Malone Monti Malone. BJU Int 2000; 85 : 65 . antegrade continent enema and Mitrofanoff principle 4. Liloku RB , Mure PY , Braga L et al. The left Monti- using the divided appendix: report of a new technique Malone procedure: preliminary results in seven cases. for prevention of stoma complications. J Urol 2001 ; J Pediatr Surg 2002; 37 : 228 – 31 . 165 : 2404 – 9 .

Minimally invasive approaches to 59 lower urinary tract reconstruction

Christina Kim and Steven G Docimo

1. Which of the following is an anticipated 3. The most compelling argument against per- benefit of laparoscopic-assisted bladder forming laparoscopic bladder autoaugmenta- reconstruction over open reconstruction: tion in a child with poor bladder volume and compliance is: (a) Improved bladder capacity (b) Lower risk of stone formation (a) Low long-term success of autoaugmenta- (c) Lower risk of intra-abdominal adhesions tion in this scenario (d) Lower risk of stomal stenosis (b) Converting an extraperitoneal operation (e) Improved continence ] to a transperitoneal operation (c) High risk for bladder perforation Answer: (c) Laparoscopic surgery is associ- (d) Expected difficulty dissecting detrusor off ated with a significantly lower risk of of the bladder mucosa developing intra-abdominal adhesions in both (e) High risk of bladder calculi children and adults. The other outcomes are Answer: (a) Bladder autoaugmentation is not dependent on the approach. (Page 958) an ideal operation for adaptation to laparos- copy. Unfortunately, the long-term results, 2. A contraindication to laparoscopic-assisted especially in children with low bladder bladder augmentation is: volume, have been poor. The operation can be (a) Prior abdominal surgery done transperitoneally or preperitoneally, (b) Poor adherence to a catheterization the risk for bladder perforation and stones regimen is probably lower than for gastrointestinal (c) Ventriculoperitoneal shunt augmentation, and the dissection of bladder (d) Nonneurogenic bladder dysfunction muscle off of detrusor, while potentially (e) Need for a Mitrofanoff stoma difficult, is reasonably accomplished laparo- scopically. (Page 959) Answer: (b) The majority of laparoscopic- assisted reconstructions have been performed 4. All of the following are important technical in patients with prior abdominal surgery. points in laparoscopic ileocystoplasty except: Ventriculoperitoneal shunt is not a contra- indication to laparoscopy. The indications (a) Ileal detubularization for bladder reconstruction do not limit (b) Wide bladder opening the approach, whether open or laparoscopic. (c) Tension-free anastomosis No bladder reconstruction should be (d) Adequate postoperative drainage attempted without assurance that the child (e) Intracorporeal bowel anastomosis and/or family will adhere to an emptying Answer: (e) Laparoscopic ileal augmenta- schedule. (Page 958) tion has been done with both extra- and 180 Clinical Pediatric Urology Study Guide

intracorporeal bowel anastomosis, with little (d) Is best obtained directly through effect on outcome. The others are important posterior umbilicus with Veress needle to any intestinal bladder augmentation, regard- (e) Is enlarged in order to perform the open less of approach. (Page 962) portion of the procedure Answer: (b) In the typically described 5. Umbilical access as most commonly described laparoscopic-assisted reconstruction, a poste- for a laparoscopic-assisted bladder reconstruc- rior umbilical flap is created in order to tion with appendicovesicostomy: make a concealed umbilical stoma at the (a) Incorporates a 3 mm trocar end of the case. Open access is then (b) Is obtained through an open approach after obtained using a radially dilating 10 mm creating a posterior umbilical skin flap trocar. The open portion of the procedure is (c) Should not be used in order to preserve performed through a low incision, usually a umbilicus for stoma Pfannensteil. Genitourinary 60 rhabdomyosarcoma and other bladder tumors

Paul A Merguerian and Antoine E Khoury (Based on chapter written by Paul A Merguerian, Lisa Cartwright, and Antoine E Khoury)

1. Rhabdomyosarcoma (RMS) is associated with 2. Translocation involving chromosome 2 and the following syndromes except: 13 is associated with:

(a) Neurofibromatosis (a) Wilms' tumor (b) Denys-Drash syndrome (b) Embryonal RMS (ERMS) (c) Beckwith-Wiedemann syndrome (c) Alveolar RMS (ARMS) (d) Li-Fraumeni syndrome (d) Neuroblastoma Answer: (c) ARMS reveal a translocation Answer: (b) RMS may be associated with involving chromosomes 2 and 13, t(2,13) congenital anomalies including the nervous (q35;q14). There have also been several reports system and the genitourinary (GU) system. of translocation involving chromosomes 1 and Neurofibromatosis is the most common 13, t(1,13)(p36;q14). In a series of mapping syndrome in which RMS has been described. experiments, the chromosome 2 locus disrupted The incidence is 0.5% (Intergroup Rhab- by the t(2;13) was found to be PAX3, and domyosarcoma Study-IV study). The disease translocation breakpoints were localized to the in these patients is very progressive and it is final intron of the PAX3 gene. The chromo- recommended that patients with RMS and some 13 gene was cloned and found to be a neurofibromatosis be treated with intensive novel widely expressed member of the fork head contemporary therapy protocols. Costello transcription factor family. Based on homology syndrome is associated with increased risk to this family, the chromosome 13 gene was of solid tumors including RMS. Other con- named FKHR for “fork head in rhabdomyosar- genital syndromes associated with RMS are coma”. The PAX3–FKHR fusion in ARMS is Gorlin's basal cell nevus syndrome, Rubinstein- characterized by rearrangement of chromo- Taybi syndrome, trisomy 21, Beckwith- somes 2 and 13, t(2,13)(q35;q14), in which Wiedemann syndrome, fetal alcohol syndrome, the PAX3 gene within band 2q35 is fused with and Li-Fraumeni syndrome. Whereas Wilms' the FKHR gene within band 13q14. tumor is the most common malignancy associated with Beckwith-Wiedemann syn- 3. PAX3–FKHR fusion is associated with: drome, in the first 7 years of life, RMS, adrenocortical carcinoma, and hepatoblastoma (a) Wilms' tumor may be seen. (b) ERMS 182 Clinical Pediatric Urology Study Guide

(c) ARMS although it can rarely occur at other body (d) Neuroblastoma sites. ARMS carries a poorer prognosis with only a 53% 5-year survival rate. Answer: (c) See explanation in answer 2.

4. The cambium layer is found in which 6. In a child with pelvic RMS, the presence of condition: posttreatment residual rhabdomyoblasts in the tissue specimen: (a) Neuroblastoma (b) Wilms' tumor (a) Occurs more often in ARMS (c) Botryoid embryonal RMS (BRMS) (b) Is indicative of a worse prognosis (d) Nephrogenic adenoma of the bladder (c) Their numbers increase with time (d) Their presence is not an indication for Answer: (c) The International Classification further therapy of Rhabdomyosarcoma (ICR) criterion for (e) None of the above diagnosis of BRMS requires demonstration of a cambium (condensed layer of rhabdomyo- Answer: (d) The most difficult assessment blasts) tumor layer underlying an intact epi- by pathologists is the prognostic significance thelium in at least one microscopic field. This of residual rhabdomyoblasts in posttherapeu- microscopic criterion supersedes any gross tic RMS tissue specimens. Studies support the demonstration of a “grape-like” tumor mass. concept that posttherapeutic cytodifferentia- An extensive degree of rhabdomyoblast differ- tion occurs more frequently in BRMS or entiation can be evident both in the cambium ERMS (Heyn 19971 , Coffin 19972 , Smith layer and elsewhere in the tumor. The impor- et al 20023 ). In BRMS, cytodifferentiation tance of diagnosing this subtype is evident and decreased proliferation activity were asso- given its superior prognosis with a 95% 5-year ciated with favorable outcome. Unchanged or survival. increased posttherapeutic proliferation activity suggested aggressive biologic potential in 5. Which variants of RMS have superior ERMS or ARMS. prognosis? Smith et al3 reported on the posttreatment cytodifferentiation and clinical outcome from (a) ERMS 19 of 31 IRS-IV cases which were adequate (b) Spindle-cell ERMS for evaluation. None of the 10 cases with (c) BRMS extensive cytodifferentiation (2 BRMS, 8 (d) (a) and (c) ERMS) failed, while among the 5 cases with (e) (b) and (c) moderate cytodifferentiation, only the 2 ARMS Answer: (e) Classic ERMS with a polypoid patients failed, and all 4 patients (1 ERMS, growth pattern was associated with a more 3 ARMS) with mild or no cytodifferentiation favorable prognosis than the diffuse intra- failed. The authors concluded that posttreat- mural growth pattern. The 10-year survival ment cytodifferentiation is more common in for the polypoid tumor was 92% versus only both ERMS and BRMS than in ARMS, and 68% for the diffuse intramural growth postulated that this difference is due to differ- pattern. BRMS has a superior prognosis with ent mechanisms to cellular response to ther- a 95% 65-year survival. The spindle-cell vari- apy. They also noted that sparse persistent ant also enjoys a superior prognosis with an tumor cells in ARMS/BRMS did not 88% 5-year survival rate. This variant occurs appear to affect outcome (Smith et al 2002 3 ). exclusively in the paratesticular region, Myoglobin is specific for terminally Rhabdomyosarcoma 183

differentiated myocytes, and as such is a useful 8. The incidence of immunopositivity for anti- marker for differentiated tissue in organs such sera in RMS diagnostic studies is highest for: as bladder and prostate, which normally do (a) Polyclonal desmin not contain skeletal muscle. (b) Monoclonal desmin RMS with persistent well-differentiated (c) Myoglobin rhabdomyoblasts without mitotic activity dur- (d) MIC-2 ing and after the completion of therapy dem- onstrated rhabdomyoblast persistence, but Answer: (a) Polyclonal antidesmin (P-DES) decrease in number with time. All six patients antibody is positive in 99% of RMS speci- were alive with no evidence of disease mens. Monoclonal antidesmin (M-DES) anti- 37–233 months after the completion of body was negative in many of the same tumors therapy. The authors conclude that, while the (38%). Antimuscle specific actin (MSA) biologic nature of these cells is not known, antibody was positive in 94% of the RMS. their presence is not an indication for further Antimyoglobin (MYO) antibody was positive therapy (Ortega et al., 20004 ). in 78% and was negative mostly in the less differentiated tumors. Antialpha smooth mus- cle actin (SMA) antibody reacted positively 7. The following are all important prognostic only in a minority (4%) of embryonal RMS factors predicting survival of patients with (Qualman 19986 RMS except: ). The current approach at the IRSG (a) Age at presentation Pathology Center in cases where the diagnosis (b) Primary site of RMS is in question is to screen the case (c) Clinical group with immunostaining for three antibodies: (d) Tumor size P-DES, MSA, and MIC-2; the latter helps to (e) ICR based on tumor histology rule out the diagnosis of an extraosseous Ewing's sarcoma/primitive neuroectodermal Answer: (a) The significant difference in tumor. survival between ERMS and ARMS makes it imperative for the pathologist to accurately diagnose variants of RMS so that the biologi- 9. A 7-year-old male was found to have a large cal course of the disease can be predicted and pelvic tumor extending from the bladder caus- appropriate therapy initiated. ing bilateral hydronephrosis. Biopsy of the In 1995, a consensus classification of RMS tumor confirmed ERMS. There is no evidence was published (Newton 19955 ) that was based of distant metastasis. What is the clinical group on a review of a large number of tumors from of this tumor? IRS-II by 16 international pathologists from eight pathology groups. This produced a (a) Clinical group IIIB classification which was both reproducible (b) Clinical group IIC and could predict outcome by univariate (c) Clinical group IIIA analysis. A multivariate analysis of this new (d) Clinical group IIB ICR indicated that a survival model which Answer: (c) included the ICR along with known prognos- Clinical group I Localized tumor completely tic factors of primary site, clinical group, and resected: tumor size was significantly better at predict- ing survival than a model with only the known (a) Confined to site of origin prognostic factors. (b) Infiltration beyond site of origin 184 Clinical Pediatric Urology Study Guide

Clinical group II Microresidual or regional In >90% the lesions are solitary. Pathologi- spread: cally, 80% are grade I or papilloma, 20% are grade I–II, and only 3% invasive through the (a) Localized with micropositive margins, lamina propria. The recurrence rate is low grossly resected (25%). Cystoscopy allows definitive diagno- (b) Regional disease with complete sis, staging, and treatment of these tumors. Its resection, most distal nodes negative role in surveillance remains ill defined. (c) Regional disease resected with microresidual or most distal resected nodes involved Clinical group III Gross residual: References (a) Biopsy only 1. Heyn R , Newton WA , Raney RB et al. Preservation (b) Gross/major primary resection of the bladder in patients with rhabdomyosarcoma. (>50%) J Clin Oncol 1997; 15 : 69 – 75 . 2. Coffin CM , Rulon J , Smith L et al. Pathologic features Clinical group IV Distant metastasis at of rhabdomyosarcoma before and after treatment: A diagnosis (lung, liver, bone marrow (BM), clinicopathologic and immunohistochemical analysis. Mod Pathol 1997; 10 : 1175– 87 . bone, brain, distant muscle and/or cytology 3. Smith LM , Anderson JR , Coffin CM . Cytodifferentia- cerebrospinal fluid (CSF), pleural fluid, tion and clinical outcome after chemotherapy and peritoneal fluid). radiation therapy for rhabdomyosarcoma (RMS). Med Pediatr Oncol 2002; 38 : 398 – 404 . 10. Transitional cell carcinoma of the bladder in 4. Ortega JA , Rowland J , Monforte H et al. Presence of children is usually: well-differentiated rhabdomyoblasts at the end of therpy for pelvic rhabdomyosarcoma: implications for (a) Muscle invasive the outcome. J Pediatr Hematol Oncol 2000; 22 : (b) Low grade tumor 106 – 111 . (c) Has high recurrence rate 5. Newton WA Jr , Gehan EA , Webber BL et al. Classification of rhabomyosarcomas and related (d) Multifocal sarcomas. Pathologic aspects and proposal for a new Answer: (b) Transitional cell carcinoma of classification – an Intergroup Rhabdomyosarcoma Study. Cancer 1995; 76 : 1073– 85 . the bladder in children is low grade which 6. Qualman SJ , Coffin CM , Newton WA et al. seldom recurs. Some of these tumors occur Intergroup Rhabdomyosarcoma Study: up date for secondary to cyclophosphamide therapy. pathologists. Pediatr Dev Pathol 1998; 1 : 550 – 61 . Exstrophy and epispadias 61 Linda A Baker and Richard W Grady

1. Single-stage reconstruction using the com- urethra deep into the pelvis, and reestablish- plete primary exstrophy repair technique offers ment of normal anatomy. This relationship several advantages over staged reconstruction will allow bladder cycling and continence, except: which some believe may preclude the need for a formal bladder-neck repair. (Page 1015) (a) T he possibility to correct the penile, blad- der, and bladder-neck abnormalities of bladder exstrophy with one operation 3. The following postoperative factors have been (b) T he ability to achieve urinary continence shown to increase the success of reconstruc- without bladder-neck reconstruction tion for bladder exstrophy except: (c) C orrection of vesicoureteral reflux at the time of surgery (a) I mmobilization with external fixators, (d) L ower complication rates than previous Buck ' s traction, a Spica cast, etc . attempts at single-stage reconstruction (b) A ntibiotic therapy (c) P rolonged nil per os (NPO) status or high Answer: (c) Complete primary closure resin diet to avoid excessive bowel move- affords the above mentioned advantages. ments which may soil the dressing and However, it is typically performed in the cast neonatal period which precludes the surgical (d) U rinary diversion through ureteral stent- correction of reflux. (Page 1014) ing and suprapubic urinary drainage (e) A dequate nutritional support 2. Single-stage reconstruction using the com- plete primary repair of exstrophy (CPRE) Answer: (c) Factors recognized to increase technique relies on the following to achieve success of the initial reconstruction are: use of urinary continence except: osteotomies; immobilization; use of postop- erative antibiotics; urinary diversion; adequate (a) R eestablishment of normal anatomic rela- pain control; avoiding abdominal distention; tionships adequate nutrition; and fixation of urinary (b) B ladder-neck reconstruction at the time drainage catheters. (Page 1013) of primary surgery (c) O steotomy at the time of single-stage reconstruction 4. Single-stage reconstruction using the complete (d) S imultaneous epispadias repair primary exstrophy repair (CPRE) technique (e) N one of the above can be safely performed because: Answer: (b) CPRE involves division of (a) T he neurovascular bundles of the corporal the intersymphyseal band which allows the bodies lie laterally rather than dorsally on placement of the bladder and reconstructed the corporal bodies 186 Clinical Pediatric Urology Study Guide

(b) T he cavernosal bodies and urethral wedge extent of the pubic diastasis, and the extent of are not actually separated from each other other comorbidities such as spina bifida should with this technique factor into a decision to proceed with CPRE (c) T he blood supply to the corporal bodies of cloacal exstrophy. (Page 1036) and urethral wedge are independent of each other 7. Complications of the complete primary exstro- (d) T he blood supply is quickly reestablished phy repair (CPRE) technique include: once the components are “ reassembled ” (e) (a) and (c) (a) M yogenic bladder failure (b) T esticular atrophy Answer: (e) The corporal bodies and ure- (c) U rethrocutaneous fistula thral wedge are separated into three distinct (d) H ip dislocation parts with their respective independent blood

supply. Dissection should be carried out Answer: (c) CPRE should provide bladder above Buck' s fascia to avoid injury to the later- cycling which prevents myogenic bladder ally placed neurovascular bundle (NVB) on failure. Notable complications in the largest the corpora. The wedge should be wide prox- series include hydronephrosis, hypospadias, imally to assure adequate vascuralization dur- bladder-neck urethrocutaneous fistula, partial ing tubularization of the urethra. (Page 1015) or complete penile loss. (Pages 1009, 1022)

5. The proximal limits of dissection using the 8. Other commonly affected organ systems asso- complete primary exstrophy repair (CPRE) ciated with cloacal exstrophy include: technique are: (a) L iver (a) T he intersymphyseal band (b) S pine (b) T he muscles of the pelvic floor complex (c) E ars (c) T he rectum (d) U pper gastrointestinal tract (d) T he corpora spongiosa (e) (b) and (c) Answer: (b) The proximal dissection should Answer: (e) Spina bifida and short gut include division of the intersymphyseal bands, syndrome are two relatively common organ which is achieved with the penile disassembly. systems affected by cloacal exstrophy. Other The dissection terminates at the level of systems may be affected as well. (Page 1003) the pelvic floor musculature. Failure to reach this point will result in tension on the repair. 9. The cloacal membrane separates the coelemic (Page 1015) cavity from the amniotic space during the _____ week of embryogenesis 6. Factors that mitigate against using a single (a) 1st stage reconstruction technique for cloacal (b) 12th exstrophy include the presence of: (c) 3rd (a) A large omphalocele (d) 24th (b) A wide pubic diastasis (e) 8th (c) A concomitant myelomeningocele Answer: (c) The cloacal membrane sepa- (d) A ll of the above rates the coelemic cavity from the amniotic Answer: (d) The size of the omphalocele, space and can be first identified at 2– 3 weeks the size of the hindgut and bladder plates, the gestation. (Page 1006) Exstrophy and epispadias 187

10. Left untreated, most patients with exstrophy Answer: (e) Closure of bladder exstrophy is will experience: not mandatory but chronic exposure of the (a) I ncontinence bladder urothelium will lead to all of the above (b) U rinary infections mentioned complications. (Page 1009) (c) D ifficulty with sexual intercourse (d) I nflammation of the bladder plate (e) A ll of the above

Unusual conditions of the 62 bladder, including bladder trauma, urachal anomalies, and bladder diverticula

Marc Cendron

1. During embryological development of the into the urethra, vestibule or Gratner's duct in bladder: females or prostatic urethra in male. This, in turn will cause distal urethral obstruction early (a) The urogenital sinus forms at 12 weeks in development of the upper urinary tract. gestation Dilatation of the ureters and dysplasia of the (b) The allantois will form the dome of the renal parenchyma are the end result. bladder (c) Histological organization of the bladder will be complete by 12 weeks gestation 3. A 3-year-old girl sustained blunt trauma to the (d) Structural and functional organization is lower abdomen. In the emergency depart- complete by 16 weeks gestation ment, she presents with gross hematuria after voiding spontaneously once. On physical Answer: (d) With urine production starting examination her abdomen is benign and no at 9–10 weeks gestation, urine flow into the orthopedic issues are present. The next step in bladder starts. Filling and emptying (cycling) management is: into the bladder has been documented by prenatal ultrasonography. Histologic studies (a) Place a Foley catheter for drainage have shown that the organization of the (b) Carry out a cystogram to evaluate for a bladder is complete by 16 weeks gestation. possible bladder leak (c) Obtain an ultrasound or computerized 2. Bladder agenesis: tomography (CT) scan of the abdomen (a) Can be associated with survival mostly in (d) Admit for observation males Answer: (c) The patient may have either (b) Is associated with a high rate of ectopic bladder contusion or bladder rupture. A CT ureter and dysplastic kidney scan or an ultrasound will reveal thickened, (c) Is invariably associated with stillbirth irregular bladder wall with no extravasation (d) Is an isolated congenital anomaly of fluid. If bladder rupture has occurred, a Answer: (b) Failure of bladder development collection of extravesical fluid will be causes the ureters to become incorporated observed. 190 Clinical Pediatric Urology Study Guide

4. A 5-year-old boy is involved in a motor vehicle (b) Suprapubic urinary diversion accident and at the time was wearing a seatbelt. (c) Exploration and closure of the bladder He is suspected of having a rupture of his (d) Exploration to look for bony spicules if a bladder on CT scan. A cystogram is equivocal. pelvic fracture has been diagnosed He is admitted for observation and over the next 6 h is increasingly uncomfortable with Answer: (a) Extraperitoneal bladder rupture increased abdominal girth and displays can be managed nonoperatively with urethral peritoneal signs. The next step in management catheter drainage for 7–10 days with follow-up is to: cystogram to rule out persistent extravasation. Surgical exploration is reserved for cases (a) Place a Foley catheter where more severe injuries have been noted (b) Obtain STAT serum electrolytes such as pelvic fracture with presence of bony (c) Consult general surgery spicules in the bladder wall. (d) Repeat the CT scan

Answer: (b) Urine electrolytes will show 7. Congenital bladder diverticula are associated decreased levels of bicarbonate indicative of with the following congenital syndromes acidosis. Additionally, with an intraperitoneal except: bladder rupture increased serum creatinine (a) Trisomy 23 and potassium will be noted. Hyponatremia (b) Ehlers-Danlos syndrome may also be reported. (c) Williams syndrome (d) Menkes syndrome 5. During imaging of the bladder for a suspected bladder injury, the most important step is: Answer: (a) Trisomy 23 is not associated with Down's syndrome. All the other (a) Using intravenous (IV) contrast for the syndromes are known to have presence of CT scan bladder diverticula. (b) Inserting a Foley catheter and injecting contrast material using a syringe (c) Obtaining postdrainage images after blad- 8. A patient with Menkes, kinky hair syndrome, der emptying is noted to have multiple bladder diverticulae (d) Obtaining delayed films of the abdomen and suffers from recurrent urinary tract infec- after CT scan tion. The most appropriate management would be: Answer: (c) Obtaining bladder drainage films to evaluate for possible extravesical (a) Placement of an indwelling Foley catheter extravasation of contrast material. This can (b) Surgical excision of the bladder diver- also be obtained after CT scan and retrograde ticula bladder filling. (c) Clean intermittent catheterization (d) Chronic antibiotic prophylaxis 6. Extraperitoneal bladder rupture can be Answer: (c) Clean intermittent catheteriza- managed by: tion, as this will allow for frequent emptying (a) Simple urethral catheter drainage for of the bladder and prevention of urinary 7–10 days stasis. Unusual conditions of the bladder 191

9. Differential diagnosis of bladder outpouching a urachal cyst. The next step in the manage- in the pediatric age group includes all of the ment would be: below except: (a) Admit for antibiotic therapy and observa- (a) Bladder ears tion (b) Bladder diverticulum (b) Discharge on oral antibiotics (c) Everting ureterocele (c) Incision and drainage in the operating (d) Ectopic ureteral insertion room under antibiotic coverage with (e) Urachal cyst surgical excision at a later stage Answer: (e) Urachal cyst is distinct from the (d) Surgical excision while on antibiotic bladder and is usually identified by either CT coverage scan or ultrasound between the bladder dome and the umbilicus. Answer: (c) Incision drainage with antibiotic coverage followed by surgical excision once the 10. A 7-year-old child presents to the emergency inflammatory process has subsided. Excision department with a tender subumbilical mass of the cyst and closure of the skin may allow with erythema of the skin. A CT scan revealed for persistence of the infection.

Posterior urethral valves 63 Stephen A Zderic and Douglas A Canning

1. The end-stage bladder following partial outlet (c) Steady collagen mRNA, no change in type I obstruction is best associated with: and III collagen, no change in TIMP (d) A decline in collagen mRNA, a decline in (a) A drop in the extracellular matrix fraction type I and III collagen, increased matrix within the bladder wall metalloproteinase expression (b) A decline in average smooth muscle (e) A rise in collagen mRNA, a rise in type I bundle size and III collagen, a rise in TIMP activity, (c) Bladder wall hypoxia during bladder and a decline in matrix metalloproteinase filling activity (d) A minimal postvoid residual (e) A minimal postobstructive diuresis Answer: (e) The deposition of large amounts of extracellular matrix elements affect the pas- Answer: (c) Much evidence supports the sive properties of the bladder wall and thus notion of a bladder wall microcirculation alter its storage characteristics. This increase in which serves to maintain adequate oxygen collagen protein expression is predominately tension within the bladder wall during normal of the types I and III, and is mediated by a rise 1 filling cycles. Lapides first suggested nearly in mRNA expression brought about by the 40 years ago that outlet obstruction resulted impact of stretch on the bladder wall. This is in bladder wall hypoxia, and modern molecu- accompanied by a loss of matrix metallopro- lar and physiology studies demonstrate he was teinase activity which is mediated by TIMP. correct. The cyclic drop in perfusion followed by reperfusion leads to free-radical-mediated 3. Which of the following statements best damage. Partial outlet obstruction leading to describes the use of fetal electrolytes to assess the end-stage bladder is clearly associated with the renal prognosis: significant increases in smooth muscle bundle (a) Fetal electrolytes are highly predictive of size, extracellular matrix deposition, postvoid future renal failure residual urine, and a postobstructive diuresis. (b) The higher the osmolarity of the fetal urine, the better the renal prognosis 2. Bladder compliance following partial bladder (c) The best renal prognosis is seen if Na outlet obstruction is diminished as a result of <100 meq/l, Cl <90 meq/l, and osmolarity the following steps: is <210 mOsm/l (a) A rise in collagen mRNA, a rise in type I (d) A single bladder aspiration is as reliable as and III collagen, no change in tissue multiple taps β inhibitor of the matrix metalloproteinases (e) The addition of -2 microglobulin has (TIMP) supplanted fetal electrolytes as the marker (b) A rise in collagen mRNA, a rise in type I of choice and III collagen, an increase in matrix Answer: (c) Severe fetal obstructive uropathy metalloproteinase expression is associated with an isotonic urine. In contrast, 194 Clinical Pediatric Urology Study Guide

a healthy fetal kidney will produce a very (d) Urinary ascites dilute urine. Despite nearly 25 years of clinical (e) Perinephric urinoma experience, fetal electrolytes still fail to fully Answer: (a) All of the above findings except predict renal failure. Up to one-third of for bilateral vesicoureteral reflux are associated patients with “favorable” fetal renal electro- with a better long-term renal prognosis. These lytes still go on to develop renal insufficiency. anatomic variants serve as pressure pop-off Multiple bladder taps can increase the accuracy. mechanisms and are associated with improved The search for better markers with enhanced renal as well as bladder function. predictive power continues. 6. A neonate presents following an antenatal 4. Fetal intervention for obstructive uropathy diagnosis of severe bilateral hydrouretero- secondary to posterior urethral valves has been nephrosis. Postnatal imaging confirms a diag- shown to: nosis of posterior urethral valves with a (a) Diminish the incidence of end-stage renal severely trabeculated bladder wall and bilateral disease vesicoureteral reflux. Despite an indwelling (b) Be associated with a high rate of fetal catheter, the neonate's creatinine rises to 3.1. demise There is a vigorous postobstructive diuresis (c) Lead to improved pulmonary function in noted and after 1 week the neonate is stable the neonate from a fluid and electrolytes standpoint, and (d) Be most effective when accomplished by the creatinine has settled at 2.6. Two days open fetal surgery following a valve ablation, the creatinine rises (e) Be associated with neonatal respiratory again to 3.0 and a repeat ultrasound shows failure severe bilateral hydroureteronephrosis. Your best option for management is: Answer: (c) Fetal intervention for posterior urethral valves has not been shown to lower (a) Repeat a voiding cystourethrography the incidence of end-stage renal disease but, in (VCUG) to search for a recurrent properly selected cases, it can offer the benefit valve of improved neonatal pulmonary function. (b) Perform a vesicostomy Neonates with severe obstructive uropathy die (c) Institute clean intermittent catheteriza- of respiratory failure due to a noncompliant tion (CIC) and hypoplastic lung. While the fetal mortality (d) Perform bilateral ureterostomies has been reported to be high in older series (e) Observation with serial sonograms with open procedures, fetal survival has Answer: (b) This patient's clinical course is improved with the less invasive method of best explained by a smaller and noncompliant fetal bladder shunting. bladder coupled with a high urine output due to poor concentrating ability. This means 5. Which of the following clinical radiographic that the entire system will reach a high and findings is not associated with an improved unfavorable storage pressure with a small long-term renal prognosis? amount of urinary volume, and because of (a) Bilateral vesicoureteral reflux the postobstructive diuresis, this volume will (b) Unilateral renal reflux into a dysplastic be reached in a short period of time. While kidney some centers might advocate CIC in this (c) Bladder diverticulum setting, this means that the CIC would have to Posterior urethral valves 195

be performed at such frequent intervals as to to understand why the bactrim prophylaxis become impractical. Furthermore, CIC in a failed, i.e. was it the taste of the antibiotic or patient with reflux carries the risk of develop- an inability to pay for it. Even postcircumci- ing a febrile urinary tract infection (UTI). sion, this child should be maintained on pro- Bilateral ureterostomies would lead to a phylaxis. A DMSA scan will be less accurate in defunctionalized bladder, which in this setting the setting of severe cortical atrophy. A left can aggravate the contracture process. Among nephroureterectomy in this setting would these choices, we feel a vesicostomy is optimal mean that all of the urine output from the because it will still allow for bladder cycling right kidney would be delivered to a bladder and yet prevent the high pressures from com- with less than optimal storage properties. promising both the bladder wall as well as the Preserving the refluxing and nonfunctional upper tracts. left renal unit preserves compliance of the overall lower urinary tract and serves to 7. A 3-year-old boy with a history of posterior protect the right kidney. urethral valves is transferred to your care with a fever of 104°F and a positive urine 8. An 8-year-old boy presents with a history of culture. He was maintained on bactrim posterior valves ablated in the neonatal period. prophylaxis and is noted to be uncircumcised. He is proud to say that he is finally dry all day Sonography reveals a right kidney with long and in underwear. But he is bothered by moderate hydroureteronephrosis, and a his nighttime wetting. His creatinine which left kidney with severe cortical atrophy and was stable at 1.0 has now climbed to 1.5 and severe hydroureteronephrosis. A VCUG an ultrasound shows no significant change in demonstrates unilateral high-grade reflux the hydronephrosis seen over the years. Your and a trabeculated bladder wall with no next step should be: residual valve leaflets. The urine culture (a) Praise him for being dry by day reveals E. coli sensitive to bactrim. Your (b) Tell his family that the creatinine goes up management recommendation in this setting as boys get bigger should be: (c) Get a MAG3 renal scan to look for a (a) Change the suppression antibiotic over to kinking obstruction Keflex (d) Get a uroflow with postvoid residual (b) Start CIC (e) Have him keep a 72 voiding log with (c) Perform a circumcision measured volumes and perform videouro- (d) Obtain a DMSA scan dynamics (e) Perform a left nephroureterectomy Answer: (e) This boy will benefit from a Answer: (c) Wiswell2 was correct; prepuce videourodynamic study to assess his safe zone. presence portends the presence of perilous The real question for this boy is at what vol- periurethral pathogens. While it is true that in ume does bladder pressure rise to a point normal males one must circumcise 99 boys to which compromises renal function. Clinical save 1 afebrile UTI, in the case of this toddler and experimental studies have clearly shown with severe posterior urethral valves, the stakes that elevated bladder pressures are transmitted are much higher. Thus, in this instance, where back to the renal pelves. Thus, as this young it is likely that there was noncompliance with man seeks so desperately to be dry like his the prescribed medical course, a circumcision peers, he stresses his bladder by keeping it is reasonable. It is important to take a history filled to the right side of the cystometry curve, 196 Clinical Pediatric Urology Study Guide

which is in the zone of poor compliance. The cannot even sense fullness until they are in key question is at what bladder volume is he the noncompliant danger zone of their out of his safe zone and this is best answered cystometric curve. This young man voided by a carefully done videourodynamic study. with a slow stream but he carried a significant This study also will determine his postvoid residual urine. For him timed voiding, and residual urine. These data must then be cor- perhaps even double voiding by day, will work related with his voiding log. In this manner to keep him in the bladder's compliant safe his voiding habits can be restructured so as to zone. However, he has a significant postob- keep his bladder working within its safe structive diuresis as manifested by the fact that zone. while 700 ml of urine was instilled during the videourodynamic study, 1000 ml was voided 9. A 13-year-old boy presents for a videouro- or aspirated during the determination of the dynamic study performed because of his residual urine. What this means is that during persisting day and night wetting despite a a 35 min study, he made an additional 300 ml successful neonatal valve ablation. A double of urine, i.e. he had a postobstructive diuresis. lumen suprapubic catheter is inserted and In such an instance, overnight catheter drain- 600 ml is drained. Fluoroscopic confirmation age will preserve renal function and ultimately, shows the bladder to be empty and bladder in many instances, restore renal concentrating filling commences at 20 ml/min. At 35 min ability. (700 ml) he reports the urge to void. He voids 500 ml with a slow stream and bilateral 10. Which of the following statements is reflux is noted. The residual urine from the not true about the nature of postobstructive catheter is aspirated for and additional 500 ml diuresis in the adolescent male with posterior with fluoroscopic confirmation to determine urethral valves? that both the bladder and upper tracts are (a) They have decreased aquaporin expres- contrast free. The optimal management for sion in the ascending tubule this boy is: (b) They have noncompliant bladders (a) Timed voiding (c) They have diminished Cl- pump activity (b) CIC in the ascending loop of Henle (c) Overnight catheter drainage (d) The peak interstitial osmotic gradient (d) Timed voiding by day and overnight within the medulla will be 800 mOsm/l catheter drainage (e) There will be evidence of hypoxia within (e) Bilateral reimplantation the medullary zone of the kidney Answer: (d) During their adolescent years it Answer: (d) A characteristic feature of boys is not uncommon to see valve patients present with posterior valves is their diminished abil- with the syndrome of polyuria, urinary incon- ity to concentrate urine. As infants they are tinence, and renal insufficiency. This subset of thirstier than their peer group. As they grow patients can be saved from end-stage renal dis- older, parents often comment on the striking ease if their bladder function is rigorously clarity of their urine. These boys will have a monitored and optimized. A valuable clue in diminished interstitial medullary osmolarity; this young man's case is that it took 700 ml for they will not have anything near the normal him to feel a need to void. For many of these 800 mOsm/l gradient that serves to draw free boys, changes in the sensory innervation of water back in from the ascending tubule in the bladder have taken place such that they the loop of Henle or the distal collecting ducts. Posterior urethral valves 197

As the intrapelvic pressure remains elevated, References substantial changes take place with diminished medullary blood flow which leads to hypoxia, 1. Lapides J , Diokno AC , Silber SJ , Lowe BS . Clean, which in turn contributes to a loss of the intermittent self-catheterization in the treatment of medullary concentration gradient. Decreased urinary tract disease. Trans Am Assoc Genitourin Surg aquaporin expression as shown in experimen- 1971; 63 : 92 – 6 . 2. Wiswell TE , Miller GM , Gelston HM Jr , Jones SK , tal models of obstructive uropathy would also Clemmings AF . Effect of circumcision status on serve to increase the volume of dilute urine periurethral bacterial flora during the first year of life. that is excreted. J Pediatr 1988; 113 : 442 – 6 .

Prune belly syndrome 64 R Guy Hudson and Steven J Skoog

1. Prune belly syndrome is classically defined by is due to a deficiency of the underlying mus- all of the following abnormalities except: culature. The order of severity of involvement from most to least involved is: transversus (a) Deficiency of the abdominal wall muscu- abdominis; rectus abdominus below the umbi- lature licus; internal oblique; and rectus abdominis (b) Hypospadias above the umbilicus. (c) Bilateral cryptorchidism (d) Dilated, dysmorphic urinary tract 4. The most important/immediate factor which Answer: (b) Hypospadias is not a classic affects prognosis in the newborn with prune finding associated with prune belly syndrome. belly syndrome is: (a) Renal involvement 2. Based on Woodard's classification, the major- (b) Pulmonary involvement ity of patients with prune belly syndrome (c) Cardiac involvement reside in which category? (d) Orthopedic abnormalities (a) I (e) Gastrointestinal abnormalities (b) II Answer: (a) Prognosis of a child with prune (c) IIa belly syndrome is directly related to renal (d) III function and the degree of renal dysplasia. It (e) IV has been proposed that the renal dysplasia Answer: (d) Woodard described three major seen in prune belly syndrome is due to combi- categories for children with prune belly syn- nation of a ureteric bud and metanephric drome, with Category III, the last category, to defect. be the mildest. Category III patients are affected by the external abdominal features and 5. All of the following are mandatory in the undescended testes, but neither pulmonary nor evaluation of a newborn with prune belly renal function is significantly impaired. syndrome except: (a) Chest X-ray 3. The specific muscle layer most severely (b) Serum electrolytes involved in prune belly syndrome is: (c) Urinalysis (a) Transversus abdominis (d) Renal ultrasound (b) Rectus abdominis below the umbilicus (e) All of the above are mandatory in the ini- (c) Rectus abdominis above the umbilicus tial evaluation of a newborn with prune (d) Internal oblique belly syndrome (e) External oblique Answer: (e) The neonatal evaluation in a Answer: (a) The wrinkled, floppy abdominal child diagnosed with prune belly syndrome wall which characterizes prune belly syndrome requires the physician to rule out pulmonary 200 Clinical Pediatric Urology Study Guide

involvement, renal dysplasia and the presence affected – an important consideration at the of a urinary tract infection (UTI) among other time of reconstructive surgery? factors. (a) Proximal (b) Middle 6. Which of the following statements best (c) Distal describes the contour and capacity of the blad- (d) Intramural der in a patient with prune belly syndrome? (e) None of the above: all portions of the (a) Thin walled, trabeculated, small capacity ureter are similarly affected in patients (b) Thick walled, trabeculated, small capacity with prune belly syndrome (c) Thick walled, nontrabeculated, large Answer: (a) The ureters are elongated, tor- capacity tuous, and dilated in prune belly syndrome, (d) Thin walled, trabeculated, large capacity and the lower third of the ureter is more (e) Thick walled, non-trabeculated, small affected than the proximal portion. True capacity obstructive lesions of the ureter are rare. Answer: (c) The bladder in a child with prune belly syndrome is thick walled, enlarged, 9. All of the following can contribute to the and nontrabeculated. The normal bladder etiology of infertility in patients with prune contour is deviated by a pseudodiverticulum belly syndrome except: caused by the urachal remnant. The (a) Prostatic hypoplasia bladder neck is characteristically wide and (b) Lack of continuity between the ductuli vesicoureteral reflux is present. efferentes and rete testis (c) Abnormally thickened vas deferens and 7. The theory of mesodermal arrest as the ectopic drainage etiology of prune belly syndrome fails to (d) Atretic, absent or dilated explain: (e) All of the above can contribute to the (a) Absence of anatomic obstruction in most etiology of infertility in patients with patients prune belly syndrome (b) Male predominance of the syndrome Answer: (e) Infertility is strongly associated (c) High association with megalourethra with prune belly syndrome and this is due to (d) Abnormalities of the prostate and ure- all of the anatomical factors described in thra choices (a)–(d). An open bladder neck and (e) Abnormalities of testicular descent hypoplastic prostate result in retrograde ejacu- Answer: (b) The mesodermal arrest theory – lation; abnormalities of the prostatic epithe- that the abdominal wall and urinary tract lium and seminal vesicles affect the seminal would be vulnerable to a mesodermal defect fluid; and vas deferens obstruction affects occurring between 6 and 10 weeks of gesta- sperm delivery. tion – does not explain male predominance and the fact that abdominal wall abnormalities 10. Undescended testes in patients with prune may be preset even with a normal urinary belly syndrome are usually located: tract. (a) In the superficial inguinal pouch (b) In the inguinal canal 8. Which section of the ureter in patients (c) Intraabdominal, at the internal inguinal with prune belly syndrome tends to be less ring Prune belly syndrome 201

(d) Intraabdominal, overlying the iliac ves- range from urethral atresia to fusiform sels megalourethra. Both scaphoid and fusiform (e) Intraabdominal, overlying the ecstatic megalourethra may be associated with prune ureters at the pelvic inlet belly syndrome. Answer: (e) Bilateral cryptorchidism is a hallmark of prune belly syndrome and the tes- 12. The most frequent musculoskeletal/orthopedic tes are usually in an intraabdominal portion abnormality in patients with prune belly overlying the ecstatic ureters at the pelvic inlet. syndrome is: The intraabdominal location of the testes is associated with the known risks of infertility (a) Talipes equinovarus and malignant degeneration. (b) Polydactyly (c) Congential dislocation of the hips 11. The most common anterior urethral abnor- (d) Syndactyly mality associated with patients with prune (e) Scoliosis belly syndrome is: Answer: (c) Intrauterine compression and (a) Megalourethra oligohydramnios may cause lateral dimples of (b) Hypospadias the elbows and knees at the milder end of the (c) Microurethra spectrum, and, most commonly, congenital (d) Urethral duplication dislocation of the hips at the severe end. (e) Nonobstructing bulbar urethral stricture Scoliosis, talipes equinovarus, polydactyly and Answer: (a) The anterior urethral abnor- syndactyly are rare anomalies associated with malities associated with prune belly syndrome prune belly syndrome.

Basic science of genitalia 65 CD Anthony Herndon (Based on chapter written by Laurence S Baskin)

1. The critical gene responsible for male sexual DAX-1 gene does not appear to be active differentiation is: when present with the SRY gene and the Y chromosome. (a) L IM-1 (b) S OX-9 (c) S RY 4. In the presence of the SRY gene, which com- (d) D AX-1 pound facilitates the conversions of choles- (e) W T-1 terol to pregnenolone? Answer: (c) Products of the SRY gene located on the short arm of chromosome Y ini- (a) 11-B hydroxylase tiate the cascade of events which leads to male (b) 5-alpha reductace differentiation via development of the testis. (c) S teroidogenesis acute regulatory (STAR) protein (d) 21-alpha hydroxylase 2. Germ cells are derived from which layer: (e) 19-aromatase (a) E ctoderm Answer: (c) In the normal male, SRY over- (b) M esoderm whelms the DAX-1 gene resulting in the STAR (c) E ndoderm protein being the first step in steroidogenesis (d) N one of the above facilitating the conversion of cholesterol to Answer: (c) Germs cells are derived from pregnenolone. the endoderm cell layer.

5. Which protein induces the regression of the 3. The critical gene responsible for female sexual M ü llerian ducts? differentiation is: (a) L IM-1 (a) F ree testosterone (b) S OX-9 (b) D HT (c) S RY (c) M IS (d) D AX-1 (d) S RY (e) W T-1 (e) S TAR protein Answer: (d) Once thought to be the default Answer: (c) MIS produced from the Sertoli pathway, the DAX-1 gene located on the cells directly inhibits formation of the Mü lle- X chromosome is felt to be critical in the path- rian duct system. This acts locally and latera- way of female sexual development. The lizes to the affected side. 204 Clinical Pediatric Urology Study Guide

6. Which structures are not derived from the Answer: (e) The clitoris is derived from the M ü llerian duct system? genital tubercle. All other structures are derived from the Mü llerian duct system. The (a) U pper one-fifth of vagina urethra and lower vagina are derived from the (b) F allopian tube urinogenital sinus and urethral folds. (c) U terus (d) C ervix (e) C litoris Basic science of the testis 66 Stephen Lukasewycz and Aseem R Shukla (Based on chapter written by Julia Spencer Barthold)

1. Regarding gonadal determination, which of (b) Pre-Leydig cells begin expressing SRY at the following statements is false? 41 days gestation (c) SRY is the sex-determining region on the (a) In the human fetus, formation of the Y choromosome indifferent gonadal ridge occurs at (d) WT1 and SF1 are the two genes known 33 days to be required for gonadal determination (b) Expression of WT1, though possibly and not development associated with murine gonadal determi- (e) All of the above are true nation, has not yet been determined to play a role in human gonadal Answer: (b) Pre-Sertoli cells begin express- development ing SRY at 41 days gestation; Leydig cells do (c) Mutations in SF1, a transcription factor, not express SRY. The remainder of the choices are associated with XY gonadal dysgenesis above are true. in humans (d) SF1 interacts with many other transcrip- 3. What percentage of individuals with XY tion factors including Sry, Sox9, and gonadal dysgenesis have mutations of SRY? AMH in its role in gonadal determi- (a) 5–10% nation (b) 15–20% (e) All of the above are true (c) 25–30% Answer: (b) The indifferent gondal ridge (d) 35–40% does begin to form at 33 days gestation. (e) 50–65% Mutations in SFI have been associated with Answer: (b) Although SRY is critical in tes- XY gonadal dysgenesis but not with failure of tis development, only 15–20% of individuals ovarian or adrenal development. SF1 does with XY dysgenesis have mutations of SRY. interact with numerous downstream factors. Other genetic and environmental factors are Answer (b) is false because WT1 is a transcrip- likely the cause in other individuals with XY tion factor required for normal development dysgenesis. of the urogenital ridge and for germ cell development in both mice and men. 4. Regarding Amh and AMH (anti-Müllerian hormone) which of the following are false? 2. All of the following are true except: (a) Amh is a member of the transforming (a) The SRY gene is involved with gonadal growth factor-β family development and not gonadal deter- (b) Secretion of AMH is inhibited by Wnt4 mination and Dax1 in the ovary 206 Clinical Pediatric Urology Study Guide

(c) Males with a deletion of Amh have gubernacular growth when bound by persistence of Müllerian structures but G-protein coupled receptor affecting have normal fertility testicular descent (GREAT) (d) AMH is a sensitive indicator of the func- (b) The majority of infants with isolated tion of Sertoli cells cryptorchidism will demonstrate a muta- (e) All of the above are true tion in INSL3 (c) INSL3 is a hormone in the relaxin family Answer: (c) AMH is a member of the trans- that is a nonandrogenic testicular factor forming growth factor-β family and is a sensi- (d) Homozygous deletion of Insl3 and Rxfp2 tive measure of Sertoli cell function. AMH is in mice is associated with perirenal testes inhibited by Wnt4 and Dax1 in the ovary. and failure of gubernacular development Clinically, males with a deletion of Amh have (e) All of the above are true persistence of Müllerian structures, and also have testicular dysgenesis and infertility. Answer: (b) Insl 3 and the GREAT were expected to be candidate genes for human nonsyndromic cryptorchidism, but a mutation 5. All of the below are true regarding the role of in one of these genes is present in <5% of INSL3 in testicular descent except? cases. A multifactorial etiology involving (a) Insl3 is a G-protein coupled receptor on several genes and external exposures is likely the gubernaculum thought to stimulate responsible. Intersex 68 Hsi-Yang Wu and Howard M Snyder III

1. A patient who is a true hermaphrodite is found (d) Testosterone on gonadal biopsy to have a right ovotestis (e) Dihydrotestosterone (DHT) and a left ovary. Which arrangement of the internal genital anatomy would cause you Answer: (a) By physical exam, this patient is to question the adequacy of the gonadal most likely a female due to a lack of palpable biopsy? gonads and the likely diagnosis is congenital adrenal hyperplasia (CAH). If a male pseudo- (a) Bilateral fallopian tubes hermaphrodite had this physical exam, then (b) Right fallopian tube, left fallopian tube the testosterone and DHT levels could be and vas elevated or low. Finding an elevated 17- (c) Right fallopian tube and vas, left fallopian hydroxyprogesterone level would rule out any tube of the male pseudohermaphrodite conditions. (d) Right fallopian tube, left fallopian tube with atretic hemiuterus (e) Right fallopian tube, left fallopian tube 3. You are seeing as 8-year-old patient, who is a with normal hemiuterus recent immigrant, for ambiguous genitalia, Answer: (b) The internal structures ipsilat- and who has other relatives with ambiguous eral to an ovotestis can be either a fallopian genitalia. The left gonad is descended and the tube or a fallopian tube and vas. However, a right one is at the pubic tubercle. The urethra vas cannot be found ipsilateral to an ovary. If has a perineal opening and the phallus is small. a patient has an ovotestis in which the testicu- Which lab test will allow you to determine the lar portion is deep to the ovarian portion, then diagnosis without performing a gonadal it is possible to miss the deeper portion with a biopsy? superficial gonadal biopsy. (a) 17-hydroxyprogesterone 2. A newborn is noted to have no palpable (b) Cortisol gonads, moderate phallic enlargement and (c) Testosterone/DHT ratio fusion of the labial/scrotal folds with a (d) Dihydroepiandrosterone urethral meatus at the base of the phallus. The (e) Luteinizing hormone (LH), follicle- lab test most likely to indicate the correct stimulating hormone (FSH) diagnosis is: Answer: (c) The patient is a male pseudo- (a) 17-hydroxyprogesterone hermaphrodite, most likely with 5α-reductase (b) Corticosterone deficiency. A testosterone/DHT ratio of 50:1 (c) Cortisol will confirm the diagnosis. 208 Clinical Pediatric Urology Study Guide

4. Which intersex patient is potentially (d) Müllerian-inhibiting substance fertile? (e) 5α-reductase (a) Partial androgen insensitivity Answer: (d) This is the classic presentation (b) True hermaphrodite, female sex of rear- of persistent Müllerian duct syndrome, a male ing who is discovered on inguinal exploration to (c) Gonadal dysgenesis, 46,XY karyotype have female internal gonadal structures. (d) Mixed gonadal dysgenesis Patients with StAR protein deficiency are (e) 20,22 desmolase (StAR protein) defi- phenotypically female and the remainder ciency have ambiguous genitalia rather than appear- ing male. Answer: (b) From this group, only true her- maphrodites with a female sex of rearing have a chance of being fertile (Table 68.1). True 7. A phenotypic female presents at 1 year of age hermaphrodites with a male sex of rearing with a left inguinal hernia. The sac contains a have abnormal testicular function which makes gonad without an associated fallopian tube. fertility rare. The only intersex patients who The diagnosis can be established by all have normal fertility potential are those with except: CAH. (a) Rectal exam to palpate cervix (b) Pelvic ultrasound 5. In which intersex state is there no increased (c) Vaginoscopy risk of gonadal malignancy? (d) 17-hydroxyprogesterone (a) Persistent Müllerian duct syndrome (e) Magnetic resonance imaging (MRI) of (b) Gonadal dysgenesis, 46,XX karyotype pelvis (c) Mixed gonadal dysgenesis Answer: (d) The concern is whether this (d) True hermaphrodite, male sex of rearing child has unrecognized complete androgen (e) Complete androgen insensitivity insensitivity syndrome. The presence of a Answer: (b) The presence of a Y chromosome uterus can be determined by (a), (b), (c) or and an undescended testis increases the risk of (e). Without clitoral hypertrophy, it is unlikely gonadal malignancy; therefore, the patient that 17-hydroxyprogesterone will be elevated. with a 46,XX karyotype is not at increased risk. 8. You are discussing prenatal treatment with a family whose previous girls have been affected 6. A phenotypic male presents with a nonpalpa- with CAH. As you discuss the risks and ble right testis and a large left inguinal benefits of treatment, what is the likelihood hernia. The left hernia sac is found to contain that prenatal steroid treatment will prevent a uterus, fallopian tubes, and a gonad that masculinization of the fetus? looks like a testis. This condition is due to a (a) 100%, it is an autosomal dominant defect in: condition (a) 20,22 desmolase (StAR protein) (b) 50%, it is autosomal dominant but only (b) 3β-hydroxysteroid dehydrogenase females would benefit (HSD3B2) (c) 25%, it is an autosomal recessive (c) 21-hydroxylase (CYP21) condition Intersex 209

(d) 12.5%, it is an autosomal recessive 10. During performance of a right gonadectomy condition but only females would benefit and hypospadias repair on a boy with mixed (e) Unknown, it is a random mutation gonadal dysgenesis and 45,XO/46,XY karyo- Answer: (d) Only females would benefit type, a small hemiuterus is encountered and from treatment of this autosomal recessive removed. The left testis is descended in condition. (Page 1153). the scrotum. Why did this child have a hemiuterus? 9. Patients with CAH may be hypertensive because: (a) He has undiagnosed persistent Müllerian (a) 17-hydroxyprogesterone, a strong gluco- duct syndrome and has a hemiuterus on corticoid is elevated in 21-hydroxylase the left as well deficiency (b) The streak gonad did not express Müllerian (b) Dihydroepiandrosterone, a sex steroid is inhibiting substance (MIS), therefore the elevated in 3β-hydroxysteroid dehydroge- uterus did not involute nase deficiency (c) He has low 5α-reductase levels (c) Progesterone affects renal vascular (d) He is an undiagnosed true hermaphrodite resistance (e) He has undiagnosed pure gonadal (d) Cholesterol distention of the adrenal dysgenesis causes excess renin release (e) Deoxycorticosterone, a strong mineralo- Answer: (b) Although the presence of the α corticoid is elevated in 11 -hydroxlase hemiuterus is due to the lack of MIS, it deficiency affects only the side ipsilateral to the streak Answer: (e) Excess mineralocorticoid causes gonad. He has normal Wolffian structures hypertension. Most patients with CAH require on the left side. DHT affects only external replacement of mineralocorticoid and may genital development. The karyotype rules present with shock. (c) and (d) are not out true hermaphroditism or pure gonadal physiological mechanisms. dysgenesis.

Imperforate anus and cloaca 69 Curtis Sheldon and William R DeFoor Jr

1. All of the following clinical characteristics the age of toilet training with bowel manage- suggest a “high” level imperforate anus ment. which will likely require a neonatal diverting colostomy except: 3. Initial neonatal urologic evaluation includes all of the following except: (a) Meconium passage from the urethra or bladder (a) Renal and bladder ultrasound (b) “Rocker-bottom” perineum (b) Filling cystometrogram (c) Perineal fistula (c) Voiding cystourethrogram (d) Rectovaginal or rectovestibular fistula (d) Postvoid residual (e) Radiographically determined distance (e) Spinal ultrasound between the rectal pouch and perineum Answer: (b) A thorough anatomical evalua- exceeding 1 cm tion of the urinary tract and spine is essential in Answer: (c) The presence of a perineal the newborn period to assess for associated fistula generally denotes a “low” level lesion anomalies. A urodynamics study can be helpful that may be amenable to a neonatal posterior in diagnosing and following a neuropathic blad- sagittal anorectoplasty and thus avoid a der but is difficult to interpret in the newborn diverting colostomy. and is not critical to the initial evaluation.

2. Neonatal management of patients with 4. Severe bladder dysfunction diagnosed in the anorectal malformations includes all of the neonatal period may require which of the following except: following: (a) Normalization of anorectal anatomy (a) Clean intermittent catheterization (b) Protect the upper urinary tracts from (b) Cutaneous vesicostomy obstruction and stasis (c) Anticholinergic medication (c) Ensure low-pressure bladder drainage (d) Antibiotic prophylaxis (d) Minimize any neurologic deficit that (e) All of the above might arise from treatable spinal Answer: (e) High bladder pressures with pathology incomplete emptying and/or detrusor–external (e) Achieve fecal continence sphincter dyssynergia may lead to urinary sta- Answer: (e) Neonatal management of ano- sis, vesicoureteral reflux, and upper tract rectal malformations includes minimizing injury. Urologic intervention is indicated complications to the genitourinary system and when a neuropathic bladder is suspected and assessing for associated neurological condi- is more common with “high” anorectal mal- tions. Fecal continence is addressed around formation lesions. 212 Clinical Pediatric Urology Study Guide

5. The following type of fecal diversion is pre- abnormality. Hyperchloremic metabolic aci- ferred in patients with a high anorectal mal- dosis may be avoided by the creation of a low, formation: fully diverting colostomy and by ensuring effective bladder drainage. (a) Transverse (b) Descending loop 7. The level of the rectal insertion in a female (c) Descending with completely separated with a persistent cloaca should be clinically or stomas radiographically determined in the initial (d) Ascending neonatal assessment. True or false? (e) Ileostomy Answer: False From a surgical perspective, Answer: (c) A colostomy performed at the the level of insertion of the rectum is generally junction of the descending and sigmoid colon immaterial, as all patients are managed with with separated stomas is preferable to prevent an initial diverting colostomy. The presence of fecal contamination of the urinary tract and a cloacal malformation is an urgent indication completely decompress the distal colon. for the creation of a colostomy in order to pre- vent fecal contamination of the urinary tract. 6. Metabolic complications of urine passing from The colostomy should be positioned so as to the bladder, through a rectovesical or recto- be well away from the lower midline abdo- urethral fistula and into the defunctionalized men, where a subsequent vesicostomy may be distal limb of colon following initial diverting necessary. colostomy includes the following: (a) Hypernatremia 8. Initial management of a patient with a (b) Metabolic alkalosis persistent cloaca includes all of the following (c) Hyponatremia except: (d) Hyperchloremic metabolic acidosis (e) Hyperkalemia (a) Diverting colostomy (b) Drainage of hydrocolpos Answer: (d) This condition usually occurs as (c) Genitourinary evaluation the result of urine passing from the bladder, (d) Spinal evaluation through a rectovesical or rectourethral fistula (e) Posterior sagittal anorectal urethrovagi- and into the defunctionalized distal limb of noplasty (PSARUVP) colon following initial diverting colostomy. The resultant absorption of urinary constitu- Answer: (e) During the first phase of man- ents by the bowel results in the metabolic agement, attention is directed towards protec- abnormality. The occurrence of this complica- tion of the upper urinary tracts, the maintenance tion requires the presence of a urorectal fistula of low-pressure urinary drainage, normaliza- and is facilitated by a long segment of defunc- tion of perineal anatomy, and minimalization tionalized colon (large absorptive surface) of any neurologic deficit related to spinal cord communicating with the fistula. Additionally, anomaly. The presence of a cloacal malforma- the presence of distal urinary obstruction, tion is an urgent indication for the creation which may be structural (stricture) or func- of a colostomy in order to prevent fecal con- tional (neurovesical dysfunction), facilitates tamination of the urinary tract. The defini- this risk. Urinary tract infection and renal insuf- tive repair of the cloacal malformation is ficiency may also exacerbate this metabolic deferred until 3–6 months of age after a careful Imperforate anus and cloaca 213

endoscopic assessment of the common channel these patients at early puberty by imaging and and a thorough bowel preparation. perhaps vaginoscopy.

9. Long-term follow-up of cloacal patients 10. Patients with a cloaca should never have their includes periodic gynecological assessments as anorectal and urogenital sinus malformations well as colorectal and urological evaluations. corrected separately. True or false? True or false? Answer: True The best results are attained Answer: True Long-term gynecologic prob- by a single-stage PSARUVP. Those patients lems have been reported after cloacal recon- who present for correction of a urogenital struction. A significant number of patients sinus malformation who have previously have reached adolescence and suffered from undergone an anorectal pull-through proce- hematometra or hematocolpos. Recommen- dure are best corrected by reoperative PSAR- dations have been made to detect atresias of UVP complete with repeat mobilization of the Müllerian system early in life and reassess the rectum.

Surgery for intersex 70 disorders/urogenital sinus

Mark C Adams and Romano T DeMarco

1. In regards to genital development, which of 2. A patient with a urogenital sinus abnormality the following statements is correct? ____: (a) T he vagina has two embryologic compo- (a) E xhibits a common channel into which nents, the paramesonephric ducts, and the urethra, vagina, and rectum empty sinovaginal bulb (b) C ommonly has associated abnormalities (b) T he secretion of Mü llerian-inhibiting sub- of the clitoris, labia, and external genitalia stance (MIS) from the Leydig cells causes (c) W ill most likely have pure gonadal dys- the mesonephric ducts to involute genesis as their medical diagnosis (c) T he paramesonephric ducts give rise to (d) S hould undergo a thorough medical the epididymis, vas deferens, and seminal evaluation electively as an outpatient at vesicles 1 – 2 months of age (d) U nder the influence of the H-Y gene the (e) R equires only renal ultrasound during bipotential gonad should differentiate radiographic evaluation into an ovary around 7 weeks gestation (e) E mbryologic errors in the urogenital tract Answer: (b) A urogenital sinus abnormality have a relatively narrow spectrum of is present when the urethra and vagina are severity joined and exit the perineum as a common Answer: (a) Under the influence of SRY (the channel. A persistent cloaca or cloacal anom- sex-determining region of the Y chromosome) aly exists when a single perineal opening the bipotential gonad differentiates into a testis drains the bladder, vagina, and rectum. near the 7th week of gestation. Normal pheno- Patients with urogenital sinus anomalies typic expression of male differentiation is actively commonly have abnormalities of the clitoris, controlled by secretion of MIS by Sertoli cells labia, and external genitalia. A thorough and testosterone by Leydig cells. MIS causes medical evaluation of the newborn with a uro- regression of the paramesonephric (Mü llerian) genital sinus should be completed as many of ducts between 8 and 10 weeks of gestation. these children will have congenital adrenal In females, the absence of the Y chromosome hyperplasia (CAH). Delays in diagnosis can and SRY protein secretion leads to the forma- lead to life-threatening fluid and electrolyte tion of ovaries. Without MIS and androgen abnormalities. In addition to abdominal and secretion, the paramesonephric ducts give rise pelvic sonography, all patients should undergo to the fallopian tubes, uterus, and the upper contrast genitography to help define the two-thirds of the vagina. The distal third of the length of the common sinus and the level of vagina is formed from the sinovaginal bulb. confluence. 216 Clinical Pediatric Urology Study Guide

3. Which of the following statements is correct (d) T he use of the flap vaginoplasty easily regarding current surgical techniques for corrects the urogenital sinus abnormality clitoroplasty? during flap mobilization and tailoring (e) T he cutback vaginoplasty technique (a) T hey are typically performed in the ado- involves complicated flap harvesting and lescent period subsequent tissue modeling (b) A dorsal tunical incision is made in the hope of preserving clitoral nerve supply Answer: (c) A cutback vaginoplasty is per- (c) C litoral amputation is a reasonable surgi- formed in the patient with labial fusion and is cal alternative rarely indicated. This simple procedure (d) S ubtunical resection of the corpora may involves making an incision in the fused skin preserve glanular blood supply posteriorly to the perineum and then oversew- (e) E rectile tissue should only be excised in ing the incised lateral edges. Flap vaginoplasty patients with massive clitoral enlarge- is best suited for those patients with a low ment confluence of the urogenital sinus. Maintain- ing a U rather than narrow V shape at the tip

Answer: (d) Current clitoroplasty tech- helps maintain vascularity. Modification of the niques are based on Schmid' s description of distal flap into a rhomboid or omega shape corporal body excision and neurovascular allows for subsequent use of the labia to pro- bundle conservation. Subtunical resection of vide posterior closure of the introitus. Flap the corporal erectile tissue with preservation vaginoplasty does not correct the urogenital of the dorsal tunica may protect the glanular sinus. Without any separation of the vagina 1 nerve and blood supply. Baskin and colleagues from urethra, the patient is still left with a have suggested that a more ventral tunical urogenital sinus. incision may be prudent after carefully dem- onstrating lateral fanning of the clitoral nerve supply.2 Erectile tissue is routinely excised from 5. Surgical correction of the patient with a high the proximal bifurcation out to the level of vaginal confluence using the pull-through the corona along with the ventral tunics. Cli- vaginoplasty technique ____: toral reduction is typically performed within (a) I nvolves relatively easy tissue dissection the first year of life. Early intervention may and separation carry the benefit of thicker, more vascular skin (b) H ad early results which produced an iso- and easier dissection of the clitoris and para- lated and anteriorly displaced urethra vaginal tissue related to maternal estrogen (c) H as had several recent modifications to stimulation. improve surgical exposure (d) H ad rare problems related to vaginal 4. In patients with a low vaginal confluence stenosis following aggressive mobilization which of the following statements is correct? of the vagina (e) P laces the patient at particular risk for the (a) C utback vaginoplasty is a common and development of a urethrovaginal fistula at highly successful method of treatment the most distal aspect of the repair (b) T he flap should be V-shaped to avoid dis- tortion of the perineum Answer: (c) The pull-through vaginoplasty (c) D uring flap vaginoplasty a broad-based technique is a challenging procedure as estab- perineal flap is advanced into the incised lishing the proper plane between the proximal vagina posteriorly urethra and anterior urethra is very difficult. Intersex disorders 217

Early results produced an unappealing, iso- 7. Which of the following statements is correct lated and separate vaginal opening. Vaginal regarding vaginal replacement? stenosis has been reported and is likely due to (a) B owel vaginoplasty has as its advantages, tension created by attempts to mobilize the natural lubrication and less frequent issues relatively thin vagina all the way to the with contracture of the neovagina perineum. Most urethrovaginal fistulae occur (b) C hronic vaginal dilatation of the peri- at the level of the initial confluence where the neum typically provides an accommoda- exposure and repair are most tenuous. Modifi- ting vagina with few problems related to cations of this technique to improve exposure dyspareunia include the transtrigonal, posterior sagittal, (c) S plit-thickness skin grafts have minor and anterior sagittal approach.3–5 problems with graft contracture and rarely require chronic vaginal dilatation 6. The technique of total urogenital mobilization (d) T he preferred bowel segment used during _____: the creation of a bowel neovagina is the transverse colon (a) I s used extensively in patients with a low (e) F ull-thickness skin grafts provide substan- vaginal confluence tially more lubrication than split-thickness (b) H as been shown by several well-designed skin grafts and are preferred for vaginal studies to cause transient urinary and fecal replacement incontinence, which improves with pelvic floor exercises Answer: (a) Advantages of bowel vagino- (c) H as revolutionized the treatment of plasty include natural lubrication from the patients with high vaginal confluences intestinal segment, fewer problems with dys- (d) I s best suited for those patients with a pareunia, and infrequent contracture or steno- mid-level confluence, allowing for sub- sis, usually without any need for routine sequent flap vaginoplasty following mobi- vaginal dilatation. Sigmoid colon or ileum are lization the preferred segments for the creation of a (e) I s performed solely in the low lithotomy neovagina because of their proximity and ease position of mobility of these bowel segments into the pelvis. Full-thickness skin grafts have less reli- Answer: (d) Urogenital mobilization is gen- able ingrowth of blood supply but contract less erally not necessary for patients with a low once in place in comparison to split-thickness confluence. The best indication for its use may skin grafts. Chronic vaginal dilatation of the be patients with a mid-level lesion in whom perineum does not create an adequate vagina the level of confluence may be converted to a and leads to dyspareunia. low one suitable for flap vaginoplasty. The role of total urogenital mobilization for 8. Long-term results following genital recon- patients with a high level confluence remains struction demonstrate _____: to be determined. A full, lower body surgical prep from the nipple to toes, front to back, (a) N ormal orgasm and sexual satisfaction in allows exposure to the entire lower body in almost all patients both the prone and supine position, which (b) R are problems associated with vaginal may be required for exposure. Quality reviews stenosis using older, time-tested methods documenting long-term results following (c) T hat all patients undergoing vaginoplasty feminizing genitoplasty or vaginoplasty in the would benefit from the total urogenital setting of a urogenital sinus are lacking. mobilization technique 218 Clinical Pediatric Urology Study Guide

(d) T hat novel laparoscopic approaches are complications. Laparoscopy is not a technique helpful with dissection and separation of currently employed to help in the dissection the proximal vagina and urethra or exposure of the urogenital sinus during (e) T he need for an improved analysis of all reconstruction. patients in regards to urinary and sexual function, and quality of life issues References Answer: (e) Quality reviews documenting

long-term results following feminizing genito- 1. Kogan SJ , Smey P , Levitt SB . Subtunical total plasty or vaginoplasty in the setting of a reduction clitoroplasty: a safe modification of existing urogenital sinus are lacking, particularly any techniques. J Urol 1983; 130 : 746 – 8 . study with careful consideration of the initial 2. Baskin LS , Erol A , Li YW et al. Anatomical studies of anatomy or the surgical technique employed. the human clitoris. J Urol 1999; 162 : 1015 – 20 . 3. Passerini-Glazel G . A new 1 stage procedure for clito- A number of studies have evaluated the vagina rovaginoplasty in severely masculinized female pseudo- after reconstruction in terms of vaginal steno- hermaphrodites. J Urol 1989; 142 : 565 – 8 . sis and these reviews reported high rates 4. Pena A , Filmer B , Bonilla E , Mendez M , Stolar C . ( > 30 % ) of secondary procedures performed Transanorectal approach for the treatment of the uro- to correct stenosis. Total urogenital mobiliza- genital sinus: preliminary report. J Pediatr Surg 1992; tion is most helpful in patients with mid-level 27 : 681 – 5 . 5. Rink RC , Pope JC , Kropp BP et al. Reconstruction of lesions providing improved exposure and the high urogenital sinus: early perineal prone approach potentially easier dissection leading to without division of the rectum. J Urol 1997; 158 : decreased operative time, and possibly fewer 1293 – 7 . Hypospadias 71 Warren T Snodgrass , Aseem R Shukla , and Douglas A Canning

1. A boy presents 5 years after distal hypospadias 2. The main contraindication to primary tubu- repair complaining of a slow stream and larized incised plate (TIP) urethroplasty is: straining to void. On physical examination a (a) Proximal hypospadias faint white line is seen along the margins of a (b) A narrow urethral plate small-appearing meatus. He voided 170 ml (c) Ventral curvature >45° after the penis is with a peak flow of 4 ml/s and a plateau-shaped degloved curve. Intervention most likely to correct this (d) Repair in adults problem is: (e) A flat urethral plate with minimal glans (a) Meatal dilation groove (b) Meatotomy Answer: (c) There is no apparent glans– (c) Reoperative urethroplasty using a flip-flap urethral plate configuration (i.e. a narrow from the ventral skin versus wide plate or a flat versus deeply (d) Single-stage reoperation using a buccal grooved plate) which prevents TIP repair. inlay graft Similarly, it potentially can be used for any (e) Two-stage buccal graft urethroplasty extent of hypospadias. The main contraindica- Answer: (e) Late presentation of obstructive tion is significant ventral curvature persisting symptoms and white discoloration at the after the penis is degloved, which leads to meatus suggest balantis xerotica obliterans transection of the plate for straightening. (BXO). While any of the mentioned therapies might have short-term success, control of 3. All of the following are thought to reduce BXO requires all involved tissues be excised. likelihood of fistulas in distal hypospadias The resultant defect is best corrected using a repair except: staged buccal graft, since graft-take is more (a) Taking care not to tubularize the neoure- reliable in reoperations with a staged thra too far distally approach and BXO recurs in genital and (b) Two-layer closure of the neourethra nongenital skin, but has not yet been (c) Subepithelial stitching of the neourethra reported in buccal mucosa. Intraoperative (d) Covering the neourethra with a dartos biopsy for frozen section has in our experience flap not been feasible to obtain sufficient tissue (e) Using a urethral stent postoperatively for diagnosis while preserving enough tissue for single-stage repair if results are Answer: (e) Urinary diversion with a urethral negative. catheter has been a source of controversy in 220 Clinical Pediatric Urology Study Guide

primary hypospadias repair, especially for 6. Chordee, or ventral penile curvature, often distal hypospadias. There is no clear evidence associated with hypospadias, is now believed that catheters reduce the fistula rate in distal to be the result of: operations. Also, reoperations to close small (a) Fibrous bands attaching along the ventral fistulas can be done successfully without half of the penis along the corpora caver- catheters. nosa and reaching into the intercorporeal septum 4. While a definite etiology for hypospadias (b) A dysgenetic urethral plate which tethers remains elusive, all of the following are the glans of the penis causing ventral considered contributing factors except: tilting (a) Abnormal androgen production by the (c) Poorly vascularized subepithelial tissue fetal testis dorsal to the urethral plate (b) Mutation at the Great/Lgr8 gene (d) Ventral corporeal disproportion causing lack of binding at the insulin-3 (e) Persistence of embryologic ventral bow- receptor strings which bridge between glans and (c) Limited androgen sensitivity in target the penoscrotal junction tissues of the developing genitalia Answer: (d) While early surgeons believed (d) Premature loss of androgenic stimulation that excision of “fibrous bands” at the ven- due to early atrophy of Leydig cells of the trum of the penis was required to straighten testis the penis, chordee is now believed to be caused Answer: (b) The Great/Lgr8 gene regulates by ventral corporeal disproportion. Either the the receptor for insulin-3, a hormone Nesbit dorsal plication or incision and corpo- which is a primary stimulus to gubernacular real grafting are now utilized to correct corpo- growth. All of the other listed factors are ral disproportion. Subepithelial biopsies of the felt to contribute to the incomplete urethral plate revealed only well-vascularized embryologic development which results in connective tissues without fibrosis. hypospadias. 7. The TIP urethroplasty and onlay island flap hypospadias repairs share all of the following 5. The urethral plate, corpus spongiosum, and except: corpora cavernosa are derived from the: (a) Both repairs rely upon local urethral tissue (a) Endoderm for the reconstruction of the neourethra (b) Mesoderm (b) Both repairs may be used for both (c) Ectoderm proximal and distal repairs as long as (d) Genital swelling the chordee can be corrected without (e) Coronal sulcus transecting the urethral plate Answer: (b) Androgen stimulation causes (c) Properly completed, the TIP and island the genital tubercle to elongate and the onlay result share a low risk of fistula and urogenital folds to migrate toward the midline meatal stenosis and fuse moving proximally to distally. As the (d) Incising the distal urethral plate during an plate tubularizes, mesoderm within the island onlay repair, before the glansplasty, urethral folds differentiates into the urethral results in a slit-like urethral meatus as is plate and corporal bodies. obtained after a TIP repair Hypospadias 221

(e) Subcuticular suture placement seems to caliber of the tube as the neourethra meets improve cosmesis and decrease the fistula the native urethra rate for both types of repairs (c) Measure the combined width of the pre- served urethral plate and flap to be about Answer: (a) The onlay island flap hypospa- 20 mm, and no wider at the anastomosis dias repair is based on the inner prepuce, than at the urethral urethral meatus transposed ventrally on a pudendal arterial (d) Incorporate the neourethra with every supplied dartos mesentery skin flap that is stitch of a glansplasty overlaid on the urethral plate. The TIP repair (e) Remove excess dorsal preputial skin as it depends on the intrinsic urethral plate but is unlikely to be viable for use in circum- may be buttressed by the dartos mesentery ferential skin coverage transposed ventrally to cover the suture line. Answer: (b) The key step in preventing the 8. The most important technical step to avoid complications of a diverticulum and turbulent turbulent voiding, kinking, and diverticulum voiding is to aggressively trim the flap proxi- formation after an onlay island flap is to: mally to a wedge shape, and overall so that the combined width of the plate and flap is no (a) Leave redundancy in the island flap more than 10 mm. The neourethra may be epithelium to avoid the possibility of incorporated in the first stitch of a glansplasty stricture formation and excess dorsal preputial skin is split in the (b) Aggressively trim the proximal flap to a midline and rotated ventrally to afford adequate wedge shape to provide a consistent skin coverage in a typical hypospadias repair.

Abnormalities of the penis 72 and scrotum

Stephen Lukasewycz and Aseem R Shukla (Based on chapter written by Michael MacDonald, Julie Spencer Barthold, and Evan J Kass)

1. What percentage of males age 11–15 should mechanisms include frenular artery devascular- be able to completely retract their prepuce? ization and chronic meatitis. An upward deflec- tion of the urinary stream is pathognomonic. (a) 96% (b) 86% 3. A newborn male is noted to have a stretched (c) 76% penile length of 1.5 cm. Work-up should (d) 66% include: (e) 56% (a) Karyotype Answer: (d) Over time the physiologic (b) Alpha fetoprotein (AFP) adherence between the glans and inner prepu- (c) No work-up is necessary, penis length is tial skin decreases. By age 11–15 two-thirds of within normal limits boys are able to retract their foreskin com- (d) Inhibin level pletely and 95% of boys aged 16–17 can com- (e) No work-up is necessary, but treat- pletely retract their foreskin. ment should include intramuscular (IM) testosterone 2. Which of the following statements regarding Answer: (a) Micropenis by definition is meatal stenosis is true (select all that apply): 2–2.5 standard deviations below the mean for (a) Meatal stenosis occurs in both circum- age. For a newborn penis, a stretched length of cised and uncircumcised boys with equal <2.0 cm fits this criteria. Work-up should eva- frequency luate for gonadotropin deficiency and testicular (b) Frenular devascularization during circum- dysgenesis among other causes. AFP and cision is one proposed etiology of meatal inhibin have no part in this evaluation. Once a stenosis work-up is complete, IM testosterone can be (c) Upward deflection of the urinary stream given to assess penile growth potential. is most often caused by high-pressure voiding of childhood and is not often 4. A 16-year-old boy with sickle-cell disease pres- associated with meatal stenosis ents with a painful erection of 6 h duration. (d) Chronic meatitis from exposure to urine Initial treatment should be: in the diaper is one proposed etiology of (a) Hydration, pain control, and exchange meatal stenosis transfusion Answer: (b), (d) Meatal stenosis occurs almost (b) Winter's shunt followed by a Quackels exclusively in circumcised males. Proposed shunt if necessary 224 Clinical Pediatric Urology Study Guide

(c) Saline irrigation and intracavernosal 6. A hydrocele as a complication of varicocele papaverine injection repair is least likely via which surgical (d) Saline irrigation and intracavernosal approach? phenylephrine injection (a) Palomo mass ligation technique (e) (a) and (d) (b) Inguinal or subinguinal approach with Answer: (e) Low flow, or ischemic priapism, microscopic ligation of all venous is common in sickle-cell disease with an channels incidence of 2–35%. Initial treatment should (c) Laparoscopic mass ligation be multimodal, including transfusion and (d) Intrascrotal approach hydration along with irrigation and sympath- (e) None of the above omimetic injection. Shunt procedures should Answer: (b) Mass ligation of the testicular only be performed if this conservative therapy artery, vein, and lymphatic channels is associ- fails. Intracavernosal papaverine is likely to ated with a 10–20% risk of developing a promote erection. hydrocele postoperatively. The inguinal or subinguinal approach, where the testicular 5. All of the following regarding varioceles are artery is preserved, along with at least a single true except: lymphatic, rarely results in a hydrocele on (a) The gonadotrophin response to gonado- follow-up. trophin-releasing hormone (GnRH) stimu- lation is abnormally low in some children 7. All of the following are true regarding the with varicocele evaluation of a child with acute testicular (b) Leydig cell atrophy is the predominant torsion except: pathologic finding in teenagers undergo- ing surgery for varicocele (a) Absent cremasteric reflex (c) A varicocele raises testicular temperature (b) A high-riding testis with an abnormal and repair of the varicocele normalizes (transverse) lie testicular temperature (c) Presence of a color Doppler signal, even (d) The size of a varicocele has not been cor- in small testes, is highly accurate in ruling related with sperm count out torsion (e) All of the above are true (d) A history of scrotal trauma is common (e) All of the above are true Answer: (d) Several studies have reported that men with larger varicoceles have greater Answer: (c) Doppler signals in small testes impairment in semen parameters than men of young children must be interpreted with a small varicocele. This finding is associ- carefully. Presence of color dots does not ated with the observation that increasing vari- accurately predict the presence of normal flow cocele size may correlate with more ipsilateral and a wave form within the substance of the testis volume loss. testis must be seen. Hernia, hydroceles, testicular 73 torsion, and varicocele

Hiep T Nguyen

1. A 24-h-old male infant is noted to have an diagnosis of complete androgen insensitivity enlarged right hemiscrotum. It does not trans- syndrome. Vaginoscopy provides a reliable illuminate, is not reducible, and is not tender method to determine the presence of a cervix, to palpation. The most likely diagnosis is: which rules out the above syndrome. (a) Direct inguinal hernia 3. A 6-month-old infant underwent bilateral (b) Intravaginal testicular torsion hernia repair without complications. In (c) Extravaginal testicular torsion follow-up he was noted to have bilateral (d) Hydrocele recurrence. His testes are otherwise normal (e) Yolk sac testicular tumor but definite inguinal bulges are noted on Answer: (c) This patient is likely to have physical examination. What would be the extravaginal testicular torsion. Approximately most appropriate next step? 10% of all testicular torsion occurs in new- (a) Schedule to redo bilateral hernia repair borns. In 70% of these patients, torsion occurs (b) Observation to see if it will spontaneously prenatally. Testicular torsion in newborns resolve occurs extravaginally, where the entire sper- (c) Evaluation for possible etiology of matic cord twists en masse. Salvage of the recurrent hernia such as connective tissue torsed testis is uncommon and the role of con- disease tralateral orchidopexy is controversial. (d) Schedule for bilateral hydrocele repair through a scrotal approach 2. A 7-year-old girl presents with symptoms and findings consistent with a right hernia. She is Answer: (c) The recurrence rate following scheduled for bilateral hernia repair. Which of open hernia repair is approximately 1–3%, but the following procedures should be done in can be as high as 20% in premature infants. conjunction with her hernia repair? Recurrence may be due to technical problems such as failure to dissect a complete sac, tear- (a) Chromosomal analysis ing of the sac, or slipped sutures in ligating the (b) Fluorescence in situ hybridization (FISH) sac. However, other pathologies such as con- test nective tissue disease or abdominal ascites may (c) Vaginoscopy account for recurrence. Technical problems (d) Diagnostic laparoscopy are less likely to occur on both sides. (e) No other additional tests are required Answer: (c) In approximately 2–3% of 4. A 1-month-old boy presents with a right phenotypic girls undergoing hernia repair, a scrotal swelling that has been present testis may be found in the sac, suggesting the since birth. The rest of genital exam is normal 226 Clinical Pediatric Urology Study Guide

without evidence of hypospadias and both (d) Perform an abdominal and pelvic com- testes can be palpated in the scrotum. The puterized tomography to evaluate for parents indicated that there has not been an metastatic nodal disease inguinal swelling or significant variation in (e) Observe the patient to see if the pain the size of the scrotal swelling. The most improves with intravenous narcotics appropriate recommendation is: Answer: (a) The symptoms and findings are (a) Schedule for bilateral hernia repair as soon consistent with acute testicular torsion. The as possible patient should be taken immediate to surgery (b) Reassure parents and discharge from fur- without the need for obtaining confirmatory ther follow-up imaging study, in order to maximize the (c) Suggest subsequent follow-up to see if the chance of testicular salvage. hydrocele resolves (d) Perform inguinal ultrasonography to 6. A 3-week-old boy presents with a 2 day determine if a hernia is present history of right groin swelling. The infant has been irritable and not feeding well during Answer: (c) The symptoms and findings are the last 2 days, though there has been no most consistent with a hydrocele. Unlike her- vomiting. On examination, the right testis is nia, hydroceles may resolve spontaneously absent in the scrotum while the left testis is during the first year of life. Follow-up is rec- descended in the lower portion of the scrotum, ommend to treat the hydrocele if it does not and is of normal size and consistency. A firm resolve. Ultrasonography is neither sensitive bulge is noted in the right groin which does nor specific enough to diagnose the presence not reduce. Which of the following is the most or absence of a hernia. likely diagnosis? 5. A 14-year-old boy presents with a 4 h history (a) Congenital absence of the right testis of right testicular pain. It developed acutely (b) Torsion of the right undescended testis and was associated with nausea and vomiting. (c) A cord hydrocele The pain has been intense for most of the (d) Right testicular appendiceal torsion day and continues to be so. On physical Answer: (b) The symptoms and findings are exam, the right testis is very tender to most consistent with testicular torsion of an palpation and is high-riding. The cremasteric undescended testis. Extravaginal torsion can reflex is absent on the right and present on the occur in the undescended testis. This should left. Urinalysis was negative for esterase, be suspected in a child with an absent scrotal nitrites, and heme. The most appropriate next testis and symptoms consistent with torsion. step is: (a) Take the patient immediately to the oper- 7. A 12-year-old boy presents with several ating room for scrotal exploration and months history of intermittent left testicular possible bilateral orchidopexy pain. The pain developed acutely, usually early (b) Perform a testicular ultrasonography in the morning waking him from sleep. The to evaluate for the presence of blood pain resolved within 15–30 min. It appears to flow be more frequent over the last month. He (c) Treat the patient with anti-inflammatory denies any history of trauma, dysuria, or medication and scrotal elevation hematuria. He has been evaluated in the past Hernia, hydroceles, testicular torsion, and varicocele 227

for these episodes, usually when the pain has differential, abnormal semen parameters resolved. Urinalysis and testicular ultrasono- (when available and properly standardized), graphy were both normal. What is the most and size. In the absence of symptoms, it is appropriate management for this patient? quite controversial whether adolescent varico- cele requires immediate treatment. (a) NSAIDs and scrotal elevation (b) Repeat testicular ultrasound examination (c) Schedule for bilateral orchidopexy 9. A 16-year-old boy has a left Grade 3 varico- (d) Reassurance to parents that this likely rep- cele and a 50% discrepancy between the size resents benign orchalgia associated with of the left and right testes. After a thorough puberty discussion with his parents about the potential (e) Schedule for left hernia repair benefits of surgical correction, they elected for him to proceed with varicocelectomy Answer: (c) The patient's history and symp- using the Palomo technique. Eight months toms are worrisome for intermittent testicular later, the varicocele is no longer present but torsion and consequently orchidopexy would a left hydrocele is detected. The hydrocele be indicated. Boys with intermittent testicular was not noted at the 3 month postoperative torsion typically present with a history of visit. Which of the following statements is intermittent testicular pain which becomes true? more frequent and severe, culminating in an episode of acute testicular torsion. Physical (a) Since the hydrocele was not present at exam and imaging of the testis is typically nor- 3 months postvaricocelectomy, it is mal when the pain has resolved. unlikely that the left hydrocele is related to the varicocele repair (b) The chance of developing a hydrocele 8. A 14-year-old boy presents with an asymp- postoperatively is greater when using the tomatic left Grade 3 varicocele. He is a Tanner microscopic subinguinal technique com- Stage IV with symmetrical testes size and nor- pared to the Palomo and Ivanissevich mal consistency. What is the most appropriate techniques treatment? (c) It is not likely that the hydrocele will (a) Serial follow-up examination of testes size resolve with observation or simple scrotal and, when he is older and it is appropri- puncture ate, obtain a (d) Recurrence following varicocele repair is (b) Discharge from further follow-up care usually noted several months after surgery since it is a normal finding without any rather than several years significant clinical sequelae (e) A higher recurrence rate of the varicocele (c) Artery-sparing high ligation varicocel- is seen when performing mass ligation ectomy compared with artery-sparing surgery (d) Microscopic subinguinal varicocelectomy Answer: (e) Hydroceles are infrequently (e) Percutaneous sclerosis of the varicocele detected within 6 months of varicocele Answer: (a) Currently, the indication for repair, but more commonly at 6 months up surgical intervention of adolescent varicocele to 3 years. The microscopic subinguinal is for symptoms such as pain. More relative technique has a low risk of hydrocele (0.8% indications include marked testicular volume versus 5–10%). Approximately 80% of patients 228 Clinical Pediatric Urology Study Guide

with a hydrocele postvaricocelectomy will (b) Perform testicular ultrasonography to resolve spontaneously or following simple more accurately evaluate testicular scrotal puncture. A lower rate of recurrence is volume seen with the mass ligation technique com- (c) Obtain further imaging of the abdomen/ pared to the artery-sparing surgery. pelvis and venous system (d) Recommend varicocele repair 10. A 12-year-old boy presents with a right Grade Answer: (c) Approximately 90% of the vari- 2 varicocele. He has no testicular asymmetry cocele occurs on the left side and 2–10% and is asymptomatic. What would be the most occurs bilaterally. Isolated right-side varico- appropriate management? cele is uncommon. Patients with right-sided (a) Observation since the patient is varicocele require radiologic evaluation for a asymptomatic and there is no testicular pelvic/abdominal mass which could compress discrepancy venous return. Cryptorchidism 74 Thomas F Kolon

1. Abnormal histology of the cryptorchid testis (c) Ultrasound at 6–12 months of age includes: (d) Doppler ultrasound (e) Computerized tomography (CT) scan (a) Increased number of Leydig cells (b) Persistence of fetal gonocytes Answer: (a) Ultrasonography and CT scan (c) Increased number of Sertoli cells both show very low reliability for detecting (d) Early appearance of Ad spermatogonia intraabdominal testes. Despite the higher reli- (e) Early appearance of primary spermatocytes ability in detecting an undescended testis in a physical exam by a pediatric urologist in one Answer: (b) In the cryptorchid testis, the study, the accuracy of a physical exam is still number of Leydic cells is decreased and they only 84%. Ultimately, surgical exploration is atrophy. The total number of germ cells is still required to rule out the presence of a testis normal in cryptorchid testes; however, the when it is nonpalpable. number of spermatogonia remains low and does not increase with age, due to a lack of 4. In the first step of testicular maturation: transformation of gonocytes (fetal stem cell pool) into Ad spermatogonia (adult stem cell (a) Primordial germ cells enter the testicular pool). cords and differentiate into gonocytes (b) Gonocytes constitute the adult stem cell 2. Upon surgical exploration, what percentage of pool nonpalpable testes are present in the inguinal (c) Müllerian inhibiting substance (MIS) canal? stimulates fetal Leydig cell development (d) MIS causes regression of Wolffian (a) <10% duct and development of Müllerian (b) 10–25% structures (c) 25% (e) Gonocytes differentiate into primary (d) 50% spermatocytes (e) >50% Answer: (a) The first step of testicular matu- Answer (e) Upon surgical exploration, ration is marked by the primordial germ cells 60–80% of nonpalpable testes are present in the entering the testicular cords and differentiat- inguinal canal or intraabdominal location. The ing into gonocytes (fetal stem cell pool). MIS other 20–40% of testes are absent or a nubbin. is secreted by the Sertoli cells and does cause regression of Müllerian structures and devel- 3. Which technique is the most reliable mode of opment of the Wolffian duct. Leydig cells detecting an undescended testis? proliferate with the surge of luteinizing hor- (a) Physical exam by a pediatric urologist mone (LH) and follicle-stimulating hormone (b) Physical exam by a primary care physician (FSH), and not MIS. 230 Clinical Pediatric Urology Study Guide

5. Genes that have been associated with crypt- 7. Hormonal and laparoscopic evaluation in this orchidism in animal gene knock-out models same boy confirms bilateral anorchia. The tes- include all except: tes likely atrophied at: (a) Homeobox (HOX) (a) Conception (b) G-protein-coupled receptor affecting tes- (b) 4–6 weeks gestation tis descent (GREAT) (c) 6–8 weeks gestation (c) Cystic fibrosis transmembrane conduc- (d) 8–12 weeks gestation tance regulator (CFTR) (e) >12 weeks gestation (d) Calcitonin gene related peptide (CGRP) Answer: (e) It is generally accepted that (e) Androgen receptor (AR) since male differentiation of the genital tract Answer: (c) Homeobox (HOX) genes play has occurred, testicular tissue must have been a key role in the morphogenesis of segmental present and functioning during early fetal life structures along the anterior–posterior body up to at least the 12th week of gestation. This axis, and a knock-out in male mice is associ- is consistent with the association with a blind- ated with bilateral cryptorchidism. GREAT ending , as the presence of the gene is highly associated with the develop- spermatic cord structure supports the testis ment of the gubernaculums. It is believed that being present during early intrauterine life. calcitonin gene-related peptide (CGRP) in- directly mediates androgen-dependent inguino- 8. When considering hCG treatment to induce scrotal descent and the androgen receptor testicular descent in cryptorchidism, all of the gene (AR) is similarly involved. following are true except: (a) Distal inguinal testes in older boys are 6. A phenotypic boy is born with a normal- more likely to respond appearing penis and bilateral nonpalpable (b) Repeated courses offer little advantage cryptorchidism. A complete evaluation should (c) Side-effects of hCG include increased include: scrotal rugae and pubic hair (d) A total dose up to 25 000 IU can safely be (a) Karyotype given to avoid epiphyseal growth plate (b) Human chorionic gonadotrophin (hCG) closure stimulation test, LH, FSH (e) Long-term success rates in controlled (c) hCG stimulation test studies appear to be about 20% (d) (b) and (c) (e) (a) and (c) Answer: (d) A total hCG dose should be kept <15 000 IU, since it may induce epiphy- Answer: (d) For a neonate <3 months of seal plate fusion and retard future somatic age with bilateral nonpalpable testes, the LH, growth. FSH and testosterone levels should be checked to determine the presence of testes. In infants 9. Development of testicular histology and >3 months of age in whom LH and FSH may endocrine function begins during: have returned to prepubertal levels, an hCG stimulation test will aid in diagnosis of absent (a) The 4th gestational week testes. (b) Gestational weeks 5–6 Cryptorchidism 231

(c) The 7th gestational week (c) Low birth weight (d) Abdominal testicular descent (d) (b) and (c) (e) Inguinoscrotal testicular descent (e) (a), (b) and (c) Answer: (c) Development of testicular his- Answer: (d) Young maternal age is not tology and endocrine function begins during correlated with the risk of cryptorchidism, the 7th gestational week and continues into while low parity and low birth weight of the adulthood. It is characterized by four steps in infant increases the risk for cryptorchidism. maturation prior to the final maturational changes at puberty.

10. Obstetric risk factors for cryptorchidism include: (a) Young maternal age (b) Low parity

Surgical management of the 75 undescended testis

Israel Franco

1. The percentage of which will descend (d) Greenish-brown discoloration of the testis spontaneously after a boy is 6 months of (e) Absence of testicular tissue age is: Answer: (d) The first three answers are all (a) <10% microscopic diagnosis which are found in (b) 10–20% in utero torsion, but in the surgical field the (c) 20–30% only way to know there is hemorrhage present (d) 30–40% when the testes infarct is to see a greenish- (e) 40–50% brown discoloration of the tissue. Absence of testicular tissue can be misleading, Answer: (a) According to a study by since there may be a looping vas with an Wenzler et al only 6.9% of all testes will epididymal structure while there remains an descend spontaneously after a boy is 6 months intraabdominal testis. of age.1 4. The best place to approach the hernia sac 2. In utero torsion is more likely to occur on: during an open inguinal orchidopexy is: (a) Right side (a) Distal to the external inguinal ring (b) Left side (b) At the external inguinal ring (c) Both sides (c) Proximal to the external inguinal ring (d) Occurs infrequently (d) At the internal inguinal ring (e) Occurs only in premies (e) There is no need to take the hernia sac Answer: (b) In utero torsion tends to occur Answer: (d) The best place to take the her- more often on the left side with reported inci- nia sac is at the internal inguinal ring. Here dences of 70–90%. Bilateral in utero torsion is the sac is the narrowest, and generally leads extremely rare occurring in an estimated to the least amount of dissection on and 1:20 000 births. around the gonadal vessels and vas. The sac is easily delineated by its white color on the medial side when it is adjacent to the preperi- 3. The diagnosis of testicular in utero torsion toneal fat that is present along the cord. is best made in the operative field by the presence of: 5. The best procedure to help identify a nonpal- pable testis is: (a) Hemosiderin (b) Cartilage (a) Ultrasound of the scrotum (c) Fibrosis (b) Ultrasound of the abdomen 234 Clinical Pediatric Urology Study Guide

(c) Computerized tomography (CT) scan of 7. The most common reason for a failed the abdomen and pelvis orchidopexy is: (d) Magnetic resonance imaging (MRI) of (a) Inadequate dissection of the cord the abdomen and pelvis (b) Leaving a hernia sac behind (e) Surgical exploration (c) Lateral placement of the testis outside the Answer: (e) The standard at this time is scrotum surgical exploration, whether it is laparoscopy (d) All of the above or open surgery is the preference of the (e) None of the above surgeon. The results for radiologic procedures Answer: (c) The most common reason for a are poor and in the case of gadolinium- failed orchidopexy is the placement of the enhanced MRI the data pool is too small to testis lateral to the scrotum. The surgeon will make a definitive affirmation of its value. typically follow the path of the testis, which was ectopically placed, and try to make the 6. During an orchidopexy the surgeon finds that subdartos pouch via this lateral approach, and there is not enough length after transecting thereby be destined to have the testis migrate the gubernaculum and dissecting the cord out of the scrotum. from the floor of the canal. What additional maneuvers will give the surgeon additional 8. Testicular autotransplantation is carried out length and allow placement of the testis in the by anastomosing the gonadal vessels to the: scrotum? (a) Inferior epigastric vessels (a) Dissect the lateral spermatic fascia off the (b) Superficial circumflex iliac vessels cord (c) Superficial pudendal vessels (b) Utilize a Jones incision and gain access to (d) Femoral vessels the retroperitoneum and dissect the (e) Any of the above gonadal vessels up the posterior abdomi- nal wall Answer: (a) The inferior epigastric vessels (c) Leave the hernia sac intact and consider are mobilized for 8–9 cm beneath the rectus doing a Fowler Stephens orchidopexy in muscle towards the umbilicus. The distal end one stage is ligated and the proximal end is clipped. (d) Strip the cord of the hernia sac and do The proximal vessels are then irrigated with Fowler Stephens orchidopexy heparinized saline and the anastomosis is (e) (a), (b) and (c) completed to the gonadal vessels. Answer: (e) Any of the answers (a), (b) or 9. In a one-stage Fowler Stephens orchidopexy (c) are correct depending on the situation. the most important step in the operation is: Stripping the cord of the hernia sac and inter- rupting the blood supply between the vas and (a) High ligation of the gonadal vessels the collaterals to the epididymis will invariably (b) Distal ligation of the gonadal vessels as lead to failure in cases when a Fowler Stephens per Koff orchidopexy is performed. These collaterals (c) Complete mobilization of the cord and are essential to the preservation of blood flow stripping of the vas and vessels distally to the testis when the gonadal vessels are near the epididymis to get as much length transected. as possible Undescended testi 235

(d) Avoid manipulation of the epididymis the root of the small bowel mesentery. Only and vas, maintaining the small vessels one report of a hernia has been described to between the two date but this was at the native internal ring. (e) All of the above 11. Injury to the bladder can be avoided by: Answer: (d) Maintaining the blood supply that exists in the adventitial tissue between the (a) Making sure that a catheter is left in the vas and the epididymis is essential to a success- bladder at all times during the case ful Fowler Stephens orchidopexy. It does not (b) Making sure that the instrument making appear to make a difference if the vessels are the path is driven from the ipsilateral side ligated distally or proximally as long as the in a medial direction collateral blood supply is maintained. (c) Minimize the dissection at the pubic bone 10. A common complication during laparoscopic (d) Do not strip the peritoneum off the orchidopexy during the delivery of the testis gonadal vessels out of the abdomen is: (e) Sew the hiatus closed after the testis has been delivered outside the abdomen (a) Injury to the femoral veins (b) Injury to the bladder Answer: (a) The bladder should be left (c) Avulsion of the medial umbilical ligament decompressed the whole time to minimize the (d) Ureteral obstruction risk of bladder injury and to facilitate dissec- (e) Hernia formation at the site tion of the vas off the bladder wall. Not removing the peritoneum off the gonadal ves- Answer: (a) Injury to the bladder is by far sels will lead to insufficient length in many the most common injury other than initial tro- cases and possible ascent of the testis after the car injuries. This is most likely to occur if the orchidopexy. surgeon fails to leave the bladder decom- pressed during the whole case. Injury to the femoral vessels is more common if an approach Reference is taken that takes you lateral to the median

umbilical ligament. Ureteral obstruction has 1. Wenzler D et al. What is the rate of spontaneous occurred but it is due to inadequate dissection testicular descent in infants with cryptorchidism? of the peritoneum from the ureter at or near J Urol 2004; 171 : 849 – 51 .

Testicular tumors 76 Jonathan H Ross

1. When compared to testicular tumors (e) Providing information for local staging of occurring in adults, testicular tumors in a tumor children are: Answer: (b) Ultrasound is very accurate (a) More likely to be benign in distinguishing testicular from extratesticular (b) More likely to have a mixed pathology lesions. While ultrasound can detect cystic (c) Less likely to present as a testicular mass changes which occur more commonly in (d) More likely to have an elevated serum benign lesions, it cannot reliably make the beta-human chorionic gonadotrophin distinction between benign and malignant (hCG) level tumors. It can also be misleading with (e) When malignant, more likely to require regard to the feasibility of testis-sparing retroperitoneal lymph node dissection surgery as even large lesions may be enucle- (RPLND) and/or chemotherapy follow- ated leaving a reasonable amount of normal ing orchiectomy parenchyma. Ultrasound is not routinely used for staging or determining the vascularity of a Answer: (a) Children have a higher inci- lesion. dence of benign tumors (most often teratoma) and rarely have mixed germ cell tumors. When malignant, the large majority are managed 3. The best management for a 2-month-old boy with observation without chemotherapy and with a testicular tumor and a serum AFP level RPLND is almost never employed. The over- of 300 ng/ml is: whelming number of malignant tumors in (a) Close observation with serial ultrasounds, children are yolk sac tumors which produce physical exams and AFP levels alphafetoprotein (AFP), but not beta-hCG. (b) Inguinal orchiectomy with RPLND Most tumors present as a testicular mass. (c) Inguinal orchiectomy without RPLND (d) Tumor excision with further management 2. The most important role for ultrasound in based on frozen section analysis evaluating children with a scrotal mass is: (e) Trans-scrotal needle biopsy (a) Distinguishing benign from malignant Answer: (d) An AFP of 300 ng/ml is normal tumors in a 2-month-old boy and so the tumor is (b) Distinguishing testicular from extratestic- most likely benign. Therefore, an inguinal ular lesions exploration with excisional biopsy and frozen (c) Determining whether testis-sparing section analysis is appropriate. If a benign his- surgery is possible tology is confirmed, then nothing further need (d) Determining whether the lesion is be done. If the tumor is malignant, the orchi- hypervascular or hypovascular ectomy can be completed. 238 Clinical Pediatric Urology Study Guide

4. A 1-year-old boy undergoes orchiectomy for a half-life of 5 days. The standard therapy is yolk sac tumor. His computerized tomogra- three to four courses of platinum-based che- phy (CT) scan of the chest and abdomen is motherapy. An RPLND would be indicated normal and his serum AFP which was initially only if he had a persistent mass despite normal 5200 ng/ml decreases to 120 ng/ml at 4 weeks markers following chemotherapy – a rare postoperatively. The best management is: occurrence. (a) Close observation with frequent radio- 6. A 3-year-old boy undergoes an orchiectomy graphic and biochemical follow-up for a testicular mass. The final pathology (b) Four courses of platinum-based chemo- reveals mature teratoma. The next step in therapy management is: (c) Modified ipsilateral RPLND with further therapy based on the pathological results (a) A CT scan of the chest, abdomen, and (d) Bilateral RPLND with further therapy pelvis based on the pathological results (b) Four courses of platinum-based chemo- (e) No oncologic follow-up or treatment therapy (c) Staging RPLND Answer: (a) While the AFP is still slightly (d) (a) and (c) elevated, it is decreasing as expected based on (e) No further evaluation or therapy a half-life of 5 days. Therefore, the patient has stage 1 disease which is best managed with Answer: (e) Mature teratomas are univer- close observation followed by chemotherapy if sally benign in prepubertal patients. there is a recurrence. 7. A 7-year-old boy presents with precocious 5. A 1-year-old boy undergoes orchiectomy for a puberty and a testicular mass. Which of the yolk sac tumor. His CT scan of the chest is following is true regarding the best manage- normal. His CT scan of the abdomen reveals ment of this patient: one enlarged ipsilateral retroperitoneal lymph (a) If his serum 17-hydroxyprogesterone level node measuring 5 cm. His AFP which was is high then he should be treated with cor- initially 5600 ng/ml decreases to 3000 ng/ml ticosteroids and tumor excision at 4 weeks postoperatively. The best manage- (b) If his serum 17-hydroxyprogesterone ment is: level is high then he should be (a) Close observation with frequent radio- treated with an orchiectomy and no graphic and biochemical follow-up corticosteroids (b) Four courses of platinum-based chemo- (c) If his serum 17-hydroxyprogesterone level therapy is high then he should be treated initially (c) Modified ipsilateral RPLND with further with corticosteroids and no tumor exci- therapy based on the pathological results sion (d) Bilateral retroperitoneal lymph node (d) He will likely require chemotherapy biopsy with further therapy based on the (e) He will likely require bilateral orchiec- pathological results tomy with androgen replacement therapy (e) Four courses of chemotherapy including at puberty actinomycin and vincristine Answer: (c) The differential diagnosis is Answer: (b) This patient has metastatic dis- congenital adrenal hyperplasia (CAH) versus ease based on his CT scan and failure of his a Leydig cell tumor. If the 17-hydroxyproges- AFP to drop at a rate commensurate with a terone level is high then he likely has CAH Testicular tumors 239

and should be treated with corticosteroids. (c) Hormonally inactive and benign The testicular nodules usually regress in this (d) Hormonally inactive and multifocal setting, though excisional biopsy is occasion- (e) Malignant and multifocal ally required if they persist. If the patient has Answer: (c) The most common newborn a Leydig cell tumor it may be treated with testicular tumors are juvenile granulosa cell testis-sparing excision, particularly if bilateral. tumors and yolk sac tumors. Patients with 8. An 8-year-old boy with abnormal mucocuta- juvenile granulosa cell tumors often have neous pigmentation and hamartomatous abnormalities of the Y chromosome and intestinal polyposis undergoes an orchiectomy mosaicism. for a testicular tumor. The testis tumor histopathology typical of patients with these 10. Which of the following patients should clinical characteristics is: undergo prophylactic gonadectomy due to the (a) Yolk sac tumor risk of malignant degeneration: (b) Teratoma (a) A 3-year-old girl with Turner's syndrome (c) Large cell calcifying Sertoli cell tumor (b) A 46XX child with pure gonadal (d) Leydig cell tumor dysgenesis (e) Juvenile granulose cell tumor (c) A 46XY child with pure gonadal Answer: (c) The patient has the stigmata of dysgenesis Peutz-Jeghers syndrome which is associated (d) (b) and (c) with large cell calcifying Sertoli cell tumors of (e) None of the above the testis (as is Carney's syndrome). Answer: (c) Patients with dysgenetic gonads and a Y chromosome in their karyotype are at 9. A newborn boy with an abnormal Y chromo- high risk for gonadoblastomas (which can some is found to have a testicular tumor. This degenerate into malignant germ cell tumors). tumor is most likely: Patients with Turner's syndrome (who have (a) Hormonally active and benign 45XO karyotypes) and 46XX pure gonadal (b) Hormonally active and malignant dysgenesis are not at significant risk.

Tumors of the retroperitoneum 77 Gordon A McLorie (Based on chapter written by Gordon A McLorie and Darius J Bagli)

1. The best prognostic feature for children (d) Should be investigated for distant meta- presenting with neuroblastoma is: stases Answer: (a) Sacrococcygeal teratomas, the (a) n-myc amplifications most common neoplasm in the newborn, are (b) Less than 1 year at presentation now commonly diagnosed antenatally, with (c) Presence of Homer-Wright pseudo- the majority of those diagnosed prior to rosettes in histology 1 year of age being benign. Complete en bloc (d) Alterations in chromosome 3 or 11 removal of the tumor with the involved por- Answer: (b) Patients diagnosed at 1 year of tion of the coccyx is the treatment of choice. age appear to show consistently better survival 3. With pheochromocytoma in children, which outcomes than older children, even in the of the following statements are true? presence of advanced disease at presentation. Abnormal rearrangements of chromosomes (a) Ratio of norepinephrine to epinephrine is 1 and 17, and amplification of n-myc proto- helpful in distinguishing origin oncogene, have been implicated in poor (b) Metaiodobenzylguanidine (MIBG) scan survival. Homer-Wright pseudorossettes are is helpful in distinguishing origin histologically diagnostic of neuroblastoma, (c) Alpha-adrenergic receptor blockade is characterized by eosinophilic neutrophils important in surgical therapy surrounded by neuroblasts (occur in 50% of (d) All of the above cases). Answer: (d) Pheochromocytomas, particu- larly of adrenal origin, catalyze the conversion 2. Infants with sacrococcygeal tumors presenting of norepinephrine to epinephrine, so the ratio in infancy: distinguishes whether tumors are of adrenal or extraadrenal origin. MIBG is the most specific (a) Are best treated by surgical excision in diagnostic imaging modality to diagnose infancy pheochromocytoma. Alpha-adrenergic block- (b) Are best treated by multimodality ade often combined with cardiac beta block- treatment regimens ade to control reflex tachycardia remains a (c) Often have hormonal disturbances mainstay of preoperative preparation.

Tissue engineering applications 78 in pediatric urology

Anthony Atala

1. Challenges for the engineering of normal 3. Inert biomaterials without cells can regenerate pediatric urologic tissues include: normal tissue when addressing:

(a) An adequate cell source (a) Defects up to 0.5 cm in diameter (b) Adequate biomaterials (b) Defects up to 1.0 cm in diameter (c) Adequate vascularization (c) Defects up to 2.0 cm in diameter (d) All of the above (d) Defects up to 5.0 cm in diameter (e) None of the above (e) Defects of all sizes Answer: (b) The current limit of normal tis- Answer: (d) This question underlies some of sue regeneration using biomaterials alone is the major challenges in the field of tissue engi- 1 cm. Defects >1 cm usually require the use of neering. Adequate cell sources, biomaterials, biomaterials and cells for normal tissue devel- and vascularization are all required for the suc- opment without fibrosis or scar formation. cessful engineering of tissues. 4. When inert biomaterials are used without cells 2. The use of biomaterials: it could lead to:

(a) Facilitates the localization and delivery of (a) Abnormal collagen deposition cells (b) Abnormal fibroblast deposition (b) Facilitates the localization and delivery of (c) Scar formation bioactive factors (d) Graft contracture (c) Defines a three-dimensional space for the (e) All of the above formation of new tissues Answer: (e) Several pathologic events may (d) Guides the development of new tissues result when biomaterials are used without cells with appropriate function to cover defects >1 cm in diameter. These (e) All of the above findings may include extensive collagen and Answer: (e) A biomaterial is essential for fibroblast deposition, scar formation, and most applications in the field of tissue engi- eventual graft contracture. neering. Biomaterials can be used for cell 5. Which approach has been used for the vascu- attachment, to define the three-dimensional larization of engineered tissue? space of the new tissue or to guide the devel- opment of the new tissues. Biomaterials can (a) Incorporation of angiogenic factors in the be either implanted or injected. bioengineered tissue 244 Clinical Pediatric Urology Study Guide

(b) Seeding extracellular matrix (ECM) with (b) Normal cells, regardless of whether the various cell types in the bioengineered niche resides in either normal or diseased tissue tissues (c) Prevascularization of the matrix prior to (c) Other progenitor cells, but only if the cell seeding niche resides in normal tissue (d) All of the above (d) Normal cells, but only if the niche resides (e) None of the above in normal tissue (e) None of the above Answer: (d) One of the restrictions of the engineering of tissues is that cells cannot be Answer: (b) Most tissue types in the body implanted in volumes exceeding 3 mm3 have been shown to have progenitor cell because of the limitations of nutrition and gas niches. The progenitor cell niche gives rise to exchange. To achieve the goals of engineering normal cells which are expanded during nor- large complex tissues and possibly internal mal regeneration in response to injury or dis- organs, vascularization of the regenerating ease. Although epigenetic changes may occur, tissue is essential. All of the approaches the cells are genetically normal and usually listed have been used for vascularization of retain their function. bioengineered tissue. 8. Therapeutic cloning: 6. The preferred cell source for tissue engineer- (a) Can be used to generate embryonic stem ing currently involves: cell lines which will not be rejected (a) Human embryonic stem cells (b) Has been banned in most countries (b) Heterologous adult cells (c) Was used to clone the first mammal, a (c) Autologous native cells sheep named Dolly (d) Bone marrow stem cells (d) Is a technique used to generate genetically (e) All of the above unmatched material different from its source Answer: (c) There are many sources of cells (e) All of the above which can be used for tissue engineering. However, the preferred source still remains Answer: (a) Two types of nuclear cloning – autologous native cells. Native cells do not reproductive cloning and therapeutic cloning – elicit an immune rejection response and they have been described. Banned in most countries need minimal manipulation for terminal dif- for human applications, reproductive cloning ferentiation. is used to generate an embryo that has the identical genetic material to its cell source. This embryo can then be implanted into the 7. Genetically normal progenitor cells, which are uterus of a female to give rise to an infant that the reservoirs for new cell formation, are pro- is a clone of the donor. On the other hand, grammed to give rise to: therapeutic cloning is used to generate early (a) Other progenitor cells, regardless of stage embryos that are explanted in culture to whether the niche resides in either normal produce embryonic stem cell lines that are or diseased tissues genetically identical to the source. Index

abdominal wall development 147 anticholinergic therapy 132–3, 138 postvoid scan 153 abuse 107–8 antireflux therapy 112, 167 in prune belly syndrome 200 accessory renal arteries 42 antiserum immunopositivity in remodeling 142 acetylcholine 155 RMS 183 resistance 163 acid–base balance 61–2 anus, imperforate see imperforate anus smooth muscle 141, 157 acute interstitial nephritis (AIN) 67 anxiolysis 131–2 spatial imaging 149 acute renal failure (ARF) 67–9 appendicovesicostomy 168 trabeculated 151 associated disorders 67 umbilical access 180 volume calculation 150 management 68 aquaporin 197 bladder agenesis 189 metabolic complications 68 arterial vascularization of bladder augmentation nonoliguric 67 kidneys 101–2 techniques 167–9 survival factors 67 arteriovenous fistula 107 bladder autoaugmentation 179 urine studies 68 artificial urinary sphincter (AUS) 165 bowel segment preservation 167–8 acute tubular necrosis (ATN) 67 AUA Pediatric Reflux Guidelines 132 choice 172 risk factors 68 AUA Practice Guidelines 133 intravesical pressures 172 toxin-mediated 68 augmentation cystoplasty 168 laparoscopically-assisted 180 adenoma, aldosterone-producing autosomic recessive polycystic kidney minimally-invasive (APA) 49 disease (ARPKD) 41 contraindications 179 adenosine triphosfate (ATP) 155 vs open 179 adenovirus 24 bacterial virulence factors: 18 after spinal cord adrenal cortical development 41–2, bacteriuria 17–18 neuroblastoma 176–7 43–4 balanoposthitis 29 success factors 169 adrenal glucocorticoids 44 balantis xerotica obliterans 219 umbilical access 180 adrenal hemorrhage 47 ballistic lithotripsy 80 bladder diverticula, congenital 190 adrenal masses 47–8 Beckwith–Wiedemann syndrome 50, bladder dysfunction/disorders adrenal tumors 49–51 104 imperforate anus 211 adrenal vein sampling 49–50 associated disorders 181 postoperative 126 adrenalectomy 53–4 benign fibroepithelial polyps 117 radiologic assessment 149–50 aldosterone bicarbonate levels 190 bladder exstrophy 13, 185–7 adrenal vein sampling 49–50 bio-feedback therapy 133 complications 187 secretion 49 biomaterials 243 repair 185–6 aldosterone-producing adenoma bladder bladder neck sling procedure 165 (APA) 49 basic science 141–2 bladder outlet obstruction 5, 159 allograft dysfunction, early 73–4 capacity 8, 152 partial 193 alpha blockers 54 chilled water test 157 bladder outlet weakness 165 alphafetoprotein (AFP) 3, 237, 238 development 44, 141, 142, 148, bladder trauma 189 alveolar RMS (ARMS) 181–2 189 imaging 190 ambiguous genitalia 4, 13, 15, 207 epithelium 144 in orchidopexy 235 Amh/AMH 205–6 fill rate 152 perforation 168 amitriptyline 27 future studies 142 rupture 189 amniotic fluid 5 guarding reflex 156 bladder wall thickening 24–5 amphoteracin B 23–4 innervation 155, 157 blind ectopic ureteroceles 124 aneurysms, intracranial 64 mechanotransduction 141–2 blood volume 50 angiomyolipomas 65 neonatal 156 bowel reconfiguration 167, 172–3 anorectal malformations 14–15, neurogenic control 159–61 bowel reconstruction 176 211–13 neurogenic dysfunction 161 bowel vaginoplasty 216 antegrade pyelogram 108 neuropathic 14–15, 114 boytroid embryonal RMS anti-Müllerian hormone 205–6 injection therapy 138 (BRMS) 182 antibiotics 191 neurotransmitters 142, 155 brain metastases 103 administrative route 21 normal function 141 buccal graft urethroplasty, for posterior urethral valves 195 outpouching 191 two-stage 219 prophylactic 21, 107, 131 poor compliance 160 bupivicaine 27–8 246 Index c-fibers 155, 157 bladder trauma 189 dextranomer/hyaluronic acid 137–8 calcineurin inhibitor 73 genitourinary trauma 105 dextrose therapy 39 calcium-channel blockers 54 inferior vena cava thrombus diagnostic tools 1 calculus disease 75–8 invasion 99 dialysis indications 68 endourology 79–81 metastases, screening for 103 diclofenac 27 after enterocystoplasty 168 renal injury follow-up 108 99mTc-diethylenetriaminepentaacetic imaging 115 renal parenchymal imaging 83–4 acid (DTPA) 11 stone extraction, ureteroscopic 80 urolithiasis 79 gadolinium-enhanced 87–8 Candida albicans infections 30 confidentiality 1 digital rectal examinations 2 candidiasis, systemic 23 congenital adrenal hyperplasia dihydrotestosterone (DHT) 143, candiduria 23 (CAH) 4, 13, 208–9, 238 207, 209 Carney’s syndrome 239 salt wasting 15 99mTc-dimercaptosuccinic acid castration anxiety 2 treatment 14 (DMSA) 11, 20–1 catheterization infection 23 congenital bladder diverticula 190 pyelonephritis diagnosis 83 cecoureteroceles 124, 125 congenital dislocation of hips 201 distraction techniques 1 cerebral palsy 161 congenital mesoblastic nephroma 14, diuresis chemotherapy 100 postnatal 61 complications 96 congenital renal anomalies 105 postobstructive 196–7 preoperative 99 constipation and MACE 175 postoperative 99 child abuse 107–8 continent urinary reservoirs 173 diuretic renography 90 Children’s Oncology Group (COG) conus medullaris 160 diurnal enuresis 163 renal tumor study 96 corpora cavernosa 220 diverticulum formation, avoiding 221 chilled water test 157 corpus spongiosum 220 99mTc-DMSA 11, 20–1, 83 chordee 220 corticosteroids, prenatal 14 Doppler ultrasonography cilium of tubular cells 63 Costello syndrome 181 resistive index 88 circulating blood volume 50 CREAT 206 testicular 224 circulation, placental 59 creatinine clearance, neonatal 60 doxorubicin 96 circumcision 195 Credé voiding 159–60 drug exposure, maternal 161 meatal stenosis 223 cross-trigonal repair 135–6 99mTc-DTPA 11 neonatal 13–14 crossed renal ectopia 57 gadolinium-enhanced 87–8 non-neonatal 29 cryptorchidism 4, 228 duplication anomalies 123 clean intermittent catheterization genes associated 230 duplex system kidneys 9, 115 (CIC) 152, 194–5 histology, abnormal 229 laparoscopy 127–8 clear cell sarcoma 84 non-palpable 230 dysfunctional voiding 19, 157 clitoroplasty 215 obstetric risk factors 231 neonatal 159 cloaca treatment 230 neurogenic 159–61 development 147, 148 Cushing’s syndrome 49 dysfunctional voiding/elimination persistent 212–13 cutaneous ureterostomy 120 syndrome 152 cloacal exstrophy 186 cutback vaginoplasty 215–16 dysfunctional voiding scoring system cloacal membrane development 186 cystic kidney diseases 63–5 (DVSS) 131 cloning, therapeutic 244 autosomal dominant 63–4 coccydiomycosis, renal 24 autosomal recessive 63, 84–5 early allograft dysfunction 73–4 Cohen’s cross-trigonal repair 135–6 neonatal 63 echinococcal infections 24 collagen cystinuria 75 ectopic ureters 123 cross-linked bovine 137 cystitis, viral 24 imaging 124 pathological bladder states 157 cystometrogram 153 management 124–5 type III 113, 119 cystoplasty, augmentation 168 Ehlers–Danlos syndrome 190 colon conduit diversion 172 cystoscopy 168 electrohydraulic lithotripsy (EHL) 80 complete androgen insensitivity cystostomy tube, open 171 electrolyte requirements 38 syndrom(e) 208 electrolytes, fetal 193–4 complete primary exstrophy repair daytime frequency syndrome 19 embolization therapy 107 (CPRE) 185 desmopressin therapy 163 embryology 41–5, 57 complications 186 resistance 163 abdominal wall 147 compliance detrusor muscle bladder 141, 142, 148, 189 bladder 160, 193 innervation 142, 157 cloaca 147, 148 patient instability 129 cloacal membrane 186 bladder emptying schedule 179 detrusor–sphincter dyssynergy 114, genitalia 220 renal transplantation 74 159, 161 prostate 143–5 computed tomography (CT) detubularization 167, 172–3 ureters 111, 123 Index 247

urinary/fecal tracts 147 gamma-aminobutyric acid hernia 225 vagina 215 (GABA) 155 recurrent 225 embryonal RMS (ERMS) 182 ganglioneuroblastomas 51 hilar dissection 100, 102–3 classification 183–4 Gartner’ s duct cysts 30 hip dysplasia, congenital 201 emergencies 13–15 gastrocystoplasty 167, 168–9 Hirshspung’s disease 175, 176 end-stage renal disease (ESRD) 72 genetic imprinting 75 history, medical 1 endopyelotomy 91 genital reconstruction 216–17 homeobox (HOX) genes 230 engineered tissue genitalia hormonal evaluation 230 cell source 244 ambiguous 4, 13, 15, 207 horseshoe kidneys 43, 57, 58 normal 243 basic science 203–4 HPRT deficiency 76 progenitor cells 244 development 215, 220 human chorionic gonadotrophin vascularization 243–4 genitourinary trauma 105 (hCG) enterocystoplasty 167, 168 germ cell derivation 203 stimulation test 230 enuresis see diurnal enuresis; nocturnal Gil–Vernet advancement repair 136 treatment 230 enuresis Glenn–Anderson repair 136 human immunodeficiency virus epididymitis 25 glomerular filtration rate (GFR) (HIV) 24–5 examination, physical 1–2 in acute renal failure 67 hydroceles 224, 226 extracorporeal shock-wave lithotripsy fetal 60 neonatal 3 (ESWL) 3–4, 80–1 neonatal 5, 60 after varicocele treatment 227–8 extravaginal testicular torsion 225 nuclear evaluation 11 hydronephrosis glomeruli 59 imaging 87, 88, 115 familial pheochromocytomas 50 glucocorticoids 44 prenatal 6 fears 2 glucose requirements 38 resistance calculation 89 fertility see infertility/fertility gonadal dysgenesis 205, 239 ultrasonography 7–9 fetal bladder shunting 194 gonadectomy, prophylactic 239 hydroureteronephrosis, severe 195 fetal electrolytes 193–4 gonadoblastoma risk 239 bilateral 194 fetal interventions 194 gonads hydroxylase deficiencies 13 5α-reductase deficiency determination 205 17-hydroxyprogesterone test 4, 13, syndrome 143 fetal 44 238 5α-reductase expression 144 impalpable 207 3β-hydroxysteroid dehydrogenase fluid compartments 38, 61 malignancy and intersex 208 deficiency 13 fluid management 37–9 streak gonad 209 hyperaldosteronism 49 daily/hourly requirements 37 Great/Lgr8 gene 220 hypercalcemia 39 electrolyte/glucose requirements 38 growth disturbances 96 hypercalciuria 75, 77 intravenous fluids 38–9 guarding reflex 156 hyperchloremic metabolic sodium reabsorption 37–8 guidewires, hydrophilic-coated 79–80 acidosis 211–13 fluid therapy hyperkalemia 39, 173 intraoperative 39 H2 receptor blockers 169 hyperoxaluria 75 postoperative 39 Health Insurance Portability and hypertension 209 fluoroscopy 90 Accountability Act (HIPAA) hypochloremic metabolic alkalosis 173 bladder disorder assessment 149 guidelines 1 hypocitriuria 76 fluoxetine 27 hematoma, perinephric 79 hypogastric nerves 155 Foley Y–V plasty 91 hematuria 3 hypokalemia 173 folic acid 159 glomerular 30 hyponatremia 68, 190 foreskin retraction 223 gross 15, 63, 105 hypospadias 219–20 Fowler Stephens orchidopexy, delayed 106 distal repair 219 one-stage 235 persistent 30 fistula formation 219–20 fractional excretion of sodium hematuria–dysuria syndrome etiology 220 (FENa) 61 (HDS) 169 severe penoscrotal 143 fresh frozen plasma 39 heminephrectomy 126 hypothalamic–pituitary–adrenal axis, fungal infections 23–4, 30 hemiscrotum 225 fetal 44 hemodynamics, fetal 59 G-protein coupled receptor affecting hemorrhage, delayed 106 idiopathic scrotal edema 25 testicular descent hepatic cysts 64 idiopathic urethrorrhagia 30–1 (GREAT) 206 hepatic fibrosis 63 ileal conduit diversion 172 G-tip proteins 18 hepatic peroxisomal alanine-glyoxylate ileocystoplasty, laparoscopic 179–80 gabapentin 27 deficiency 76 ileum gadolinium-DTPA 87–8 heptectomy 108 reconfigured 173 gadolinium-enhanced MRI 87 hermaphroditism 207 resection 173 248 Index immunity, depressed 30 duplex system 9, 115, 123 macroglossia 50 immunocompromise 24–5 edematous 15 macrosomia 50 immunosuppression fetal 43, 57, 59 99mTc-MAG3 11 compliance 74 multicystic dysplastic 5–6, 9, 14 MAG3 studies 115, 120 enhanced 73 normal 9 magnesium ions 76–7 renal transplantation 72–3 pelvic 57, 58 magnetic resonance imaging (MRI) imperforate anus 161, 211–13 pelvic pressures 89, 90 adrenal masses 47 bladder dysfunction 211 renal ascent 57–8 bladder 149 evaluation, initial 211 solitary 88 gadolinium-enhanced 87 fecal diversion 212 see also renal entries inferior vena cava thrombosis 97 lesion level 211 kinky hair syndrome 190 inferior vena cava thrombus management, neonatal 211 invasion 99 in-office procedures 29–31 labial adherence 29–30 metastases, screening for 103 ultrasonography 7–9 laboratory assessments 3–4 renal parenchymal imaging 83, 84 in utero torsion 233 lactated Ringer’s solution 39 magnetic resonance urography incontinence 8 laparoscopic ileocystoplasty 179–80 advantages 88 fecal 151 laparoscopy disadvantages 88, 116 urinary 19, 151 adrenalectomy 53 and ferromagnetic implants 88 indifferent gondal ridge 205 approaches malacoplakia 25 inferior vena cava 42 retroperitoneal 34–5, 127 malignancy 14 thrombosis 96–7 transperitoneal 127 imaging 84 thrombus invasion 99–100, 103 contraindications 33 intersex 208 infertility/fertility 200 duplication anomalies 127–8 Malone Antegrade Continence Enema intersex 208 nephrectomy 92, 93 (MACE) procedure 173, 175–7 inguinal hernias 64 partial 127–8 appendix, alternatives to 176 inguinal nerve block 27–8 patient positioning 33, 34 channel establishment 176 injection therapy 137–40 pyeloplasty 93 complications 175–6 complications 138 robotic vs normal 128 irrigation 175 endoscopic 138 UPJ obstruction 91 left-sided 175 failure, reasons for 139 urine output 33 patient, ideal 176 skin testing, preinjection 137 vascular injury 34 quality of life 175 technique 137 Wilms’ tumor surgery 102 right-sided 176 Insl3 206 large cell calcifying Sertoli cell tumors washout times 176 insulin dependency, maternal 161 of testis 239 mannitol 99 intermittent catheterization 138 Law of Laplace 113 masculinization 208–9 International Reflux Study 132 Lesch–Nyhan syndrome 76 mature teratoma 238 intersex 207–9 leukemia 96 meatal stenosis 223 fertility 208 Leydig cell tumor 238 mechanotransduction 141–2 surgery 215–18 Leydig cells 229 megalourethra 201 see also ambiguous genitalia ligation of testicular vessels 224 megaureters 119–21 intestinal conduit diversion 172 lithogenesis definition 119 intraoperative fluid therapy 39 inhibition 76–7 functional evaluation 119–20 intraperitoneal adrenalectomy 53 onset 77 incidence 119 intravenous fluids 38–9 lithotripsy obstructed 113 intravesical ureteroceles 123, 124 intracorporeal 80 radiotracer of choice 120 ischemia, protection against 102–3 percutaneous nephrolithotripsy refluxing 113, 119 island onlay flap 220–1 (PCNL) 81 surgery 120 avoiding complications 221 shock-wave (SWL) 79, 80–1 MEN 2 50 isotonic solutions 38–9 liver transplantation 75–6 Menkes syndrome 190 loop ureterostomy 171 menses 2 jejunum bladder augmentation 172, lower urinary tract 99mTc-mercaptoacetyl triglycine 173 in cerebral palsy 161 (MAG3) 11 juvenile granulosa cell tumors 239 pathology 5 Tc-mercaptoacetyltriglycine trauma 189 (MAG3) 87 karyotype testing 4, 230 urodynamics 151–3 mesodermal arrest theory 200 kidneys lower urinary tract reconstruction 173 mesonephros 41–43 arterial vascularization 101–2 minimally-invasive surgery 179–80 metabolic acidosis 167 basic science 55–6 lumbrosacral imaging 151 hyperchloremic 211–13 congenital anomalies 105 lung development, fetal 5 metabolic alkalosis, hypochloremic 173 Index 249 metanephric mass 41–43, 57 nerve blocks 27–8 PAX3–FKHR fusion 181–2 metastases Nesbit dorsal plication 220 pelvic kidney 57, 58 brain 103 neural tube defects 159 pelvic nerves 155 peritoneal 104 see also spina bifida pelvic RMS 182 from testicular tumors 238 neuroblastoma 14, 47 pelvic spatial imaging 149 Michaelis–Gutman bodies 25 fetal 51 Peña procedure 161 micropenis 223 prognostic factors 241 penile nerve block 28 microscopic urinalysis 3 spinal cord 176–7 penis, non-appearing 230 micturition 156 neurofibromatosis 181 percutaneous nephrolithotripsy see also voiding type 1 50 (PCNL) 81 midazolam 131–2 neurogenic bladder see bladder, percutaneous nephrostomy 107, 171–2 mineralocorticoid deficit 50 neurogenic dysfunction perineal fistula 211 minimally-invasive surgery 33–5 neuropathic bladder see bladder, perinephric hematoma 79 lower urinary tract neuropathic perinephric urinoma 107 reconstruction 179–80 neuropeptides 155 peritoneal metastases 104 mitral valve prolapse 64 neurotransmitter receptors 155 periureteral adventitia removal 117 modified Politano–Leadbetter neurotransmitters 142, 155, 241 peroxisomal alanine-glyoxylate repair 135 nitrofurantoin side-effects 132 deficiency 76 monitoring, intra-operative 33 nitrous oxide 155 persistent cloaca 212–13 Monti-Yang tube 173 nocturnal enuresis 64, 163–4 persistent Müllerian duct mound shifting 139 pathophysiology 163 syndrome 208 Müllerian ducts 44, 45 treatment 164 Peutz–Jeghers syndrome 238–9 differentiation 204 nonspecific vulvovaginitis 25 pheochromocytomas 50–1, 241 formation 148 norepinephrine 155 adrenalectomy 54 regression 203 nuclear medicine 11 familial 50 Müllerian inhibiting substance 209, 229 phimosis, physiologic 29 multicystic dysplastic kidney obstructive uropathy 194 photosensitivity 132 (MCDK) 5–6, 14, 84 office procedures 29–31 placental circulation 59 associated disorders 57, 65 ultrasonography 7–9 plasma, fresh frozen 39 bilateral 64 omphalocele 50 pneumatic lithotripsy 80 causes 41 one-stage Fowler Stephens Politano–Leadbetter repair 135 inheritance 64 orchidopexy 235 polyclonal antidesmin (P-DES) non-involuting 92 Onuf’s nucleus 155 antibody 183 surgical management 92 open cystostomy tube 171 polycystic kidney disease ultrasonography 8, 64 orchidopexy 227 autosomal dominant 85 multilocular cystic nephroma 85 cord lengthening techniques 234 autosomal recessive 84–5 myelodysplasia 88 failure 234 polydipsia 64 myelolipoma 48 laparoscopic, complications in 235 pontine micturition center 156 myelomeningocele one-stage Fowler Stephens 235 postcardinal/posterior cardinal veins 42 antireflux operation 160 open inguinal 233 postembolization syndrome 107 thoracic level 159 ‘owl-eye’ cytoplasmic inclusions 25 posterior urethral valves 114, 124, oxalate deposition 75 193–7 National Wilms’ Tumor study imaging 194 (NWTS) 96 P fimbria 18 prognosis, long-term 194 nephrectomy pacemaker cells 111, 113 videourodynamic studies 195–6 approaches 93 packed red blood cells (PRBCs) 39 postnatal diuresis 61 congenital mesoblastic pain intensity assessment 27 postobstructive diuresis 196–7 nephroma 100 pain management 27–8 postoperative fluid therapy 39 landmarks 93 medication 27 postvoid bladder scan 153 laparoscopic 92, 93 nerve blocks 27–8 Potter syndrome 64 partial 100, 102–3, 127 papillary duct 77 pre-Sertoli cells 205 indications 103–4 para-aminohippurate (PAH) precocious puberty 238 laparoscopic 127–8 clearance 59 prednisone 27 pretransplant 72 parasitic diseases 24–5 prenatal diagnosis 5–6 nephrolithiasis 3–4, 77 parasympathetic postganglionic prepuce retraction 223 nephron-sparing surgery 103–4 neurons of bladder 155 priapism, low-flow/ischemic 223–4 nephronophthisis 64 patient positioning primary exstrophy repair (PRE) 185–6 nephrostomy tube, laparoscopy 33, 34 safety 185–6 percutaneous 171–2 ultrasonography 7 success factors 185 250 Index privacy 1, 2 surgery 160 tumor rupture/spillage 101 progesterone 13 reflux nephropathy 19 renal vein thrombosis 15, 83 progesterone testing 13 renal renal veins pronephros 41–43, 45 see also kidneys anatomic relationships 53 formation 148 renal agenesis 57 short, in transplantation 71 prophylactic antibiotics unilateral 57 renin–angiotensin system 49 embolization therapy 107 renal arteries resistive index (RI) 88 and pyelonephritis 131 accessory 42 retrograde pyelogram 108 upper tract pressure/flow studies 90 multiple 102 retrograde urine flow see reflux prophylactic gonadectomy 239 reconstruction 109 retroperitoneal renal exploration 108–9 prostate development 143–5 trauma 109 retroperitoneal space 34 prostatic duct development 144 renal ascent 57–8 retroperitoneal tumors 241 proteinuria 68 renal blood flow, neonatal 59 retroperitoneoscopy 34–5, 53 proton pump blockers 169 renal cell carcinoma 14, 84, 100 rhabdoid tumor 84 prune belly syndrome 199–201 metastases 104 metastases imaging 103 associated disorders 200–1 renal cysts 65 rhabdomyosarcoma (RMS) definition/classification 199 renal deterioration, early-onset 5 alveolar (ARMS) 181, 182 evaluation 199–200 renal dysplasia 199, 200 antiserum immunopositivity 183 mesodermal arrest theory 200 renal ectopia, crossed 57 associated disorders 181 pseudohermaphroditism 143, 207 renal failure 167 boytroid embryonal (BRMS) 182 PTFE particle migration 139 renal function chromosomal anomalies 181 puberty fetal 59 embryonal (ERMS) 182, 183–4 boys 1–2 neonatal 59–62 pelvic 182 girls 2 renal injury prognosis by type 182 precocious 238 follow-up radiology 108 residual rhabdomyoblasts 182–3 pudendal nerve nucleus 155 grading 105–6 survival factors 183 pull-through vaginoplasty 215–16 hemorrhage, delayed 106 rhabdosphincter 156 pyelography 108 management guidelines 108–9 RMS see rhabdomyosarcoma pyelonephritis 68–9, 168 renal laceration 105–6, 107 robotic laparoscopy 128 acute 20, 21 renal medullary carcinoma 84 diagnosis 83 renal obstructive disorders sacral agenesis 160 and VUR 131 imaging 87–8 sacrococcygeal tumors 241 pyeloplasty urodynamics of upper tract 89–90 salt craving 64 infant 92 renal parenchymal imaging 83–5 Scardino–Prince vertical flap 91 laparoscopic 93 renal pelvic fungal ball 23–4 schistosomiasis infection 24 redo 92 renal pelvic pressures 89, 90 scintigraphy, renal 119–20 renal pelvic stones 79 scrotal edema 25 radiology renal pelvis reduction 91–2 scrotal masses 237 adrenal masses 47–8 renal scarring 19–20, 132 scrotal pain 25 bladder disorder assessment 149–50 renal transplantation 71–4 sedation 131–2 renal injury follow-up 108 adult kidneys 73 separation anxiety 2 radionuclide cystogram (RNC) 115 age and outcome 74 sequential multi-channel analysis radionuclide studies 124 compliance 74 (SMA)-7 15 radiotherapy and growth contraindications 72 Sertoli cells 229 disturbances 96 immunosuppressants 72–3 serum creatinine, neonatal 60 red blood cell abnormalities 3 intravascular volume 73 severe fetal obstructive uropathy 193 reflux 111–12 management challenges 74 sexual differentiation 203 antireflux therapy 112, 167 organ preservation 71 shock-wave lithotripsy (SWL) 79 clean intermittent primary hyperoxaluria 76 short gut 186 catheterization 194–5 renal tubular acidosis 76, 77–8 sickle-cell disease 223–4 and Credé voiding 159–60 renal tumors SMA-7 15 ectopic ureters 125 cytoreductive surgery 100 Sober Y ureterostomy 171 following ureteral reimplant 138 inferior vena cava sodium citrate 39 following UTI 133 involvement 99–100 sodium handling 37–8, 39 imaging 115 intravascular tumor extension 103 neonatal 61, 62 persistent malignant 84 sonographically evident renal pelvis after medical management 132 surgery 99–104 (SERP) 8–9 postoperative 121 approaches 102 sperm count 224 secondary 114 danger points 101 sphincteric ureteroceles 124, 125 Index 251 spina bifida 138, 152, 159, fetal 143 grading 105–6 167–8 tethered spinal cord 15, 153 hematuria, disproportionate 105 associated disorders 186 tip adhesins 18 hemorrhage, delayed 106 MACE in 175 tissue engineering applications urodynamics 89–90, 151–3 spinal cord, tethered 15, 153 243–4 urachal cyst 191 spinal dysraphism, occult 160 biomaterials 243 urachal diverticulum 149–50 spinal irradiation 96 cloning, therapeutic 244 ureters 111–14 splenectomy 108 normal tissue engineering 243 anomalies 117, 123–6 SRY gene 203, 205, 215 tissue recombinant experiments 144 blood supply 119 STAR protein 203 total body water 38, 61 development 111, 123 stents/stenting 107 transitional cell carcinoma 183–4 distal dilation 149–50 inadvertent removal 171 transverse colon conduit 171 duplication anomalies 123–6 Stephens ‘ classification 124 transversis abdominis muscle 199 ectopic 123 steroidogenesis acute regulatory Trendelenburg position 33, 34 imaging 124 (STAR) protein 203 tuberculosis, renal 24 management 124–5 stoma stenosis 175–6 tuberous sclerosis 65 peristalsis 111, 112 stone disease see calculus disease tubular reabsorption 61–2 physiology 112 stone extraction, ureteroscopic 80 tubularized incised plate (TIP) prune belly syndrome 200 streak gonad 209 urethroplasty 219, 220–1 in renal agenesis 57 submucosal tunneled ureterointestinal tumor rupture/spillage 101 resting pressure 112 anastomosis 172 Turner’s syndrome 57, 239 retrocaval 117 subureteric transurethral injection horseshoe kidneys 57 Whittaker test 152–3 (STING) injection ureteral access 80 therapy 137 ultrasonic lithotripsy 80 ureteral disease 115–16 superior mesenteric artery ultrasonography ureteral obstruction 135 dissection 101 adrenal masses 47–8 ureteral opening pressure 90 supersaturation theory 76–7 approach 7–8 ureteral reconstruction 71–2 suprapubic tap 23 bladder 150 ureteral reimplant 139 disorder assessment 149 approaches 71 Tamm-Horsfall protein 8 neonatal 8 open 135 Tanner stages 2 trauma 189 ureteral stents 71 temporary vesicostomy 171 in-office 7–9 ureteral strictures 117 teratoma, mature 238 kidney 30 ureteral stumps 125 testes normal 9 ureterectomy 126 basic science 205–6 lower urinary tract pathology 5 ureteric bud 43, 57, 123 cryptorchid, histology of 229 multicystic dysplastic kidney 64 ureterocalicostomy 92 non-palpable 229, 230 patient positioning 7 ureteroceles 115, 123, 149 identification 233–4 probe sizes 7 classification 124 spontaneous descent 233 renal vein thrombosis 15 cystoscopic incision 125 undescended 200–1 screening 13, 87 imaging 124 detection 229 scrotal masses 237 postoperative management 126 surgery 233–4 ureters 115, 149–50 sphincteric 124, 125 testicular autotransplantation 234 urolithiasis 79 ureteroneocystotomy 72 testicular descent 44–5 Wilms’ tumor 83–4 ureteropelvic junction (UPJ) testicular development 230–1 see also Doppler ultrasonography disruption 105 testicular maturation 229 umbilical skin flap 180 surgical management 108 testicular torsion 15 University of Wisconsin (UW) traumatic 108 acute 224, 226 solution 71 obstruction 14 extravaginal 225, 226 UPJ obstruction see ureteropelvic imaging 89–90 intermittent 227 junction, obstruction in pelvic kidney 58 of undescended testis 226 upper urinary tract surgical management 91–2 in utero 233 infection 112 ultrasonography 7, 8 testicular tumors 237–40 in myelodysplasia 152 ureteroscopic stone extraction 80 adults vs children 237 obstruction 112 ureterosigmoidostomy 173 benign 237 physiology 112 ureterostomy metastatic disease 238 pressure/flow studies 89 cutaneous 120 treatment 237 prophylactic antibiotics 90 loop 171 testicular vessel ligation 224 pressures 152–3 Sober 171 testosterone production 209 trauma 105–9 ureterotomy, intubated 91 252 Index ureterovesical junction (UVJ) pyelonephritis 69 renal scarring 19 anatomy/function 129 urinoma, perinephric 107 ultrasonography 7 antireflux system 113 urodynamics voiding cystourethrogram in 58 antireflux therapy 112 catheter size 151–2 voiding dysfunction 129 ‘flap-valve’ mechanism 129 fill rate/bladder capacity 152 videourodynamic studies 195–6 urethral anomalies 115 lower tract 151–3 vitamin B12 deficiency 173 urethral catheterization 135 upper tract 89–90, 151–3 voiding urethral plate 220 urogenital mesenchyme 143, 144 control 156 urethral prolapse 29 urogenital mobilization, total 216, 217 dysfunctional 19, 157 urethral sling procedure 165 urogenital sinus 141, 143, 215, 217 neonatal 159 urethral valves, posterior see posterior urolithiasis neurogenic 159–61 urethral valves imaging 79 history/diary 151 urethrocutaneous fistula 186 pain 79 neurologic control 155–7 urethroplasty 219, 220–1 posttransplantation 72 patterns, therapeutic 163 urethrorrhagia, idiopathic 30–1 predisposing factors 77 turbulent, avoiding 221 urgency/urge incontinence 19 presentation 79 voiding cystourethrogram urinalysis uric acid 76 (VCUG) 29, 57–8 acute renal failure 68 see also calculus disease hydronephrosis 87 microscopic 3 urologic emergencies 13–15 megaureters 120, 121 urinary diversion 171–3 urology, fetal 5–6 and midazolam 131–2 anastomotic leak 171 urorectal septum 148 reflux 115 urinary/fecal tracts development 147 UTIs see urinary tract infections ureterocele assessment 149 urinary fistulae 125 von Hippel-Lindau syndrome 50 urinary frequency 19 vagina vulvovaginitis, nonspecific 25 dysfunctional voiding 19 development 215 VUR see vesicoureteral reflux urinary tract deterioration, early 159 low/high vaginal confluence 215 urinary tract infections (UTIs) 17–21 vaginal replacement 216 Waldeyer’s sheath 111 acute 21 vaginoplasty 215–16 Whittaker test 152–3 bacterial virulence factors: 18 vaginoscopy 225 Williams syndrome 190 and circumcision 29 varioceles 224 Wilms’/aniridia/genital anomaly/mental febrile 20, 87, 132 recurrent 227–8 retardation (WAGR) host risk factors 18 repair complications 224 syndrome 95, 104 incidence 17 right-sided 228 Wilms’ tumor 14, 95–7 propensity 17 treatment 227 associated disorders 95 radiography following 20 vascular spasm 125–6 chemotherapy complications 96 recurrent 8, 151 ventral penile curvature 220 histology, unfavorable 95–6 transient urodynamic abnormalities vesicostomy, temporary 171 in horseshoe kidneys 58 have 113 vesicoureteral reflux (VUR) 6, 18 imaging 83–4 ultrasonography 87 demographics 129 inoperable 102 upper urinary tract 112 ectopic ureters 125 intraoperative finding 104 urine etiology 129–30 laparoscopy 102 as amniotic fluid component 5 gender differences 113, 129 staging 96 coffee colored 168–9 grading/outcome 133 suppressor gene WT1 43 cola colored 30 hypercalciuria 75 surgical approach 102 concentrating ability 112 management surgical management 100, 103–4 electrolytes in trauma 190 bio-feedback therapy 133 WT1 gene 95 flow patterns 151 clinical 130 Wolffian duct formation 148 flow, retrograde 111–12 extravesical approach 136 WT1 gene 43, 95 neurologic control of storage/ injection therapy 137–40 XY gonadal dysgenesis 205 voiding 155–7 non-surgical 131–3 residual 153 surgical 135–6 Y chromosome abnormality 239 urine output neonatal 113 yeast infections 23, 30 during laparoscopy 33 persistent postoperative 121 yolk sac tumor 237–8, 239 neonatal 61 primary vs secondary 130 nephronophthisis 64 prophylactic antibiotics 21 zipper trauma 29